JAC Class 12 History Important Questions Chapter 15 संविधान का निर्माण : एक नए युग की शुरुआत

Jharkhand Board JAC Class 12 History Important Questions Chapter 15 संविधान का निर्माण : एक नए युग की शुरुआत Important Questions and Answers.

JAC Board Class 12 History Important Questions Chapter 15 संविधान का निर्माण : एक नए युग की शुरुआत

बहुविकल्पीय प्रश्न (Multiple Choice Questions)

1. भारतीय संविधान कब अस्तित्व में आया था?
(क) 26 जनवरी, 1949
(ख) 26 जनवरी, 1930
(ग) 26 जनवरी, 1950
(घ) 26 जनवरी, 1948
उत्तर:
(ग) 26 जनवरी, 1950

2. संविधान सभा के अध्यक्ष थे –
(क) डॉ. भीमराव अम्बेडकर
(ख) डॉ. राजेन्द्र प्रसाद
(ग) जवाहर लाल नेहरू
(घ) पट्टाभि सीतारमैया
उत्तर:
(ख) डॉ. राजेन्द्र प्रसाद

3. संविधान सभा के कुल कितने सत्र हुए थे –
(क) 11
(ग) 21
(ख) 19
(घ) 17
उत्तर:
(क) 11

JAC Class 12 History Important Questions Chapter 15 संविधान का निर्माण : एक नए युग की शुरुआत

4. संविधान सभा के कितने प्रतिशत सदस्य काँग्रेस के भी सदस्य थे –
(क) 95 प्रतिशत
(ख) 75 प्रतिशत
(ग) 82 प्रतिशत
(घ) 62 प्रतिशत
उत्तर:
(ग) 82 प्रतिशत

5. भारतीय संविधान सभा को कब से कब तक के मध्य सूत्रबद्ध किया गया –
(क) दिसम्बर, 1946 से दिसम्बर, 1949 के बीच
(ख) जनवरी 1947 से अक्टूबर 1949 के बीच
(ग) दिसम्बर, 1945 से दिसम्बर, 1948 के बीच
(घ) सितम्बर, 1946 से दिसम्बर, 1949 के बीच
उत्तर:
(क) दिसम्बर, 1946 से दिसम्बर, 1949 के बीच

6. संविधान पर तीन साल तक चली बहस के बाद हस्ताक्षर किए गए –
(क) दिसम्बर, 1949 में
(ख) दिसम्बर, 1948 में
(ग) अक्टूबर, 1949 में
(घ) इनमें से कोई नहीं
उत्तर:
(क) दिसम्बर, 1949 में

7. संविधान सभा के कितने दिन बैठकों में गए –
(क) 169 दिन
(ग) 175 दिन
(ख) 165 दिन
(घ) 140 दिन
उत्तर:
(ख) 165 दिन

8. रॉयल इण्डियन नेवी के सिपाहियों ने विद्रोह किया था –
(क) 1946 के बसंत में
(ख) 1942 के बसंत में
(ग) 1947 के बसंत में
(घ) 1944 के बसंत में
उत्तर:
(क) 1946 के बसंत में

9. संविधान सभा की प्रारूप समिति के अध्यक्ष थे –
(क) डॉ. राजेन्द्र प्रसाद
(ख) डॉ. भीमराव अम्बेडकर
(ग) महात्मा गाँधी
(घ) सरदार पटेल
उत्तर:
(ख) डॉ. भीमराव अम्बेडकर

JAC Class 12 History Important Questions Chapter 15 संविधान का निर्माण : एक नए युग की शुरुआत

10. विश्व का सबसे बड़ा संविधान निम्न में से किस देश का है?
(क) चीन
(ग) रूस
(ख) संयुक्त राज्य अमेरिका
(घ) भारत
उत्तर:
(घ) भारत

11. संविधान सभा का उद्देश्य प्रस्ताव किसने पढ़ा था?
(क) डॉ. राजेन्द्र प्रसाद
(ख) जवाहर लाल नेहरू
(ग) गोविन्द वल्लभ पंत
(घ) महात्मा गाँधी
उत्तर:
(ख) जवाहर लाल नेहरू

12. “हम सिर्फ नकल करने वाले नहीं हैं।” यह कथन किसका है?
(क) जवाहरलाल नेहरू
(ख) महात्मा गाँधी
(ग) वल्लभ भाई पटेल
(घ) भीमराव अम्बेडकर
उत्तर:
(क) जवाहरलाल नेहरू

13. ब्रिटिश औपनिवेशिक शासन में गवर्नमेंट ऑफ इण्डिया एक्ट के तहत निम्न में से किस वर्ष में प्रान्तीय संसद के चुनाव हुए थे?
(क) 1935
(ग) 1946
(ख) 1937
(घ) 1947
उत्तर:
(क) 1935

14. ” अंग्रेज तो चले गए, लेकिन जाते-जाते शरारत क बीज बो गए।” संविधान सभा में यह किसने कहा था?
(क) नेहरू ने
(ख) सरदार पटेल ने
(ग) जी.बी. पंत ने
(घ) जिन्ना ने
उत्तर:
(ख) सरदार पटेल ने

15. निम्न में से संविधान सभा के किस सदस्य को लगता था कि पृथक निर्वाचिका आत्मघाती साबित होगी?
(क) बेगम ऐजाज रसूल
(ख) भीमराव अम्बेडकर
(ग) एन. जी. रंगा
(घ) जे.बी. पन्त
उत्तर:
(क) बेगम ऐजाज रसूल

16. राष्ट्रीय आन्दोलन के दौरान किस राजनेता ने दमित जातियों के लिए पृथक् निर्वाचिकाओं की माँग की थी ?
(क) भीमराव अम्बेडकर
(ख) महात्मा गाँधी
(ग) जवाहरलाल नेहरू
(च) सुभाषचन्द्र बोस
उत्तर:
(क) भीमराव अम्बेडकर

JAC Class 12 History Important Questions Chapter 15 संविधान का निर्माण : एक नए युग की शुरुआत

17. महात्मा गाँधी समर्थक थे-
(क) हिन्दुस्तानी भाषा के
(ख) हिन्दी के
(ग) उर्दू के
(घ) संस्कृत के
उत्तर:
(क) हिन्दुस्तानी भाषा के

18. “दक्षिण में हिन्दी का विरोध बहुत अधिक है।” यह कथन किसका है?
(क) दुर्गाबाई
(ख) बेगम एजाज रसूल
(घ) सन्तनम
(ग) पं. नेहरू
उत्तर:
(क) दुर्गाबाई

रिक्त स्थानों की पूर्ति कीजिए:

1. संविधान सभा में कुल ……………. सदस्य थे।
2. ……………. को मुस्लिम लीग द्वारा ‘प्रत्यक्ष कार्यवाही दिवस ‘का ऐलान किया।
3. संविधान सभा का अधिवेशन ……………. को शुरू हुआ।
4. ……………. के दिन पाकिस्तान स्वतन्त्र हुआ तथा ……………. में जश्न हुआ।
5. भारत के संविधान को दिसम्बर नवम्बर ……………. के बीच सूत्रबद्ध किया गया।
6. बिर्रिश राज के दौरान उपमहाद्वीप का लगभग-क. ……………. भू-भाग नवाबों और रजवाड़ों के नियन्न्नण में था।
7. 13 दिसम्बर, 1946 को ……………. ने संविधान सभा के सामने उद्देश्र प्रस्ताव पेश किया।
8. गणराज्य शब्द में ……………. शब्द पहले से ही निहित होता है।
9. गवर्नमेंट ऑफ इण्डिया एक्ट के अन्तर्गत 1937 में चुनाव हुए तो कांगेस की सरकार 11 में से ……………. प्रान्तों में बनी।
10. ……………. के अनुसार जनसंख्या की दृष्टि से हरिजन अल्पसंख्यक नहीं है।
11. अनुच्छेद ……………. पर केन्द्र सरकार को राज्य सरकार के सारे अधिकार अपने हाथ में लेने का अधिकार है।
12. ……………. द्वारा सदन को यह बताया गया कि दक्षिण में हिन्दी का विरोध बहुत ज्यादा है।
उत्तर:
1. 300
2. 16 अगस्त, 1946
3. 9 दिसम्बर, 1946
4. 14 अगस्त, 1947 कराची
5. 1946, 19496. एक तिहाई
7. जवाहरलाल नेहरू
8. लोकतान्त्रिक
9. 8
10. नागप्पा
11. 356
12. दुर्गाबाई

अतिलघूत्तरात्मक प्रश्न

प्रश्न 1.
भारतीय संविधान कब अस्तित्व में आया?
उत्तर:
26 जनवरी, 1950 को

प्रश्न 2.
भारत को स्वतंत्रता कब मिली?
उत्तर:
15 अगस्त, 1947 को

JAC Class 12 History Important Questions Chapter 15 संविधान का निर्माण : एक नए युग की शुरुआत

प्रश्न 3.
रॉयल इंडियन नेवी के सिपाहियों ने विद्रोह कब और कहाँ किया ?
उत्तर:
1946 में बम्बई तथा अन्य शहरों में।

प्रश्न 4.
संविधान सभा में कितने सदस्य थे?
उत्तर:
तीन सौ।

प्रश्न 5.
संविधान सभा के तीन प्रमुख सदस्यों के नाम लिखिए जिनकी भूमिका महत्त्वपूर्ण रही।
उत्तर:
(1) पं. जवाहरलाल नेहरू
(2) बॅ. राजेन्द्र प्रसाद
(3) वल्लभभाई पटेल।

प्रश्न 6.
सरकारें सरकारी कागजों से नहीं बनतीं। सरकार जनता की इच्छा की अभिव्यक्ति होती है? यह कथन किसका था?
उत्तर:
पं. जवाहरलाल नेहरू का।

प्रश्न 7.
पृथक निर्वाचिका का समर्थन करने वाले एक सदस्य का नाम लिखिए।
उत्तर:
समर्थन करने वाले सदस्य मद्रास के बी. पोकर बहादुर थे।

प्रश्न 8.
पृथक् निर्वाचिका का विरोध करने वाले एक सदस्य का नाम लिखिए।
उत्तर:
आर.वी. धुलेकर।

प्रश्न 9.
“अंग्रेज तो चले गए, मगर जाते-जाते शरारत के बीज बो गए।” यह कथन किसका था?
उत्तर:
सरदार वल्लभ भाई पटेल का ।

प्रश्न 10.
एन. जी. रंगा के अनुसार कौन लोग असली अल्पसंख्यक थे?
उत्तर:
आदिवासी ।

प्रश्न 11.
किस अनुच्छेद के अनुसार गवर्नर की सिफारिश पर केन्द्र सरकार को राज्य सरकार के समस्त अधिकार अपने हाथ में लेने का अधिकार था ?
उत्तर:
अनुच्छेद 356

JAC Class 12 History Important Questions Chapter 15 संविधान का निर्माण : एक नए युग की शुरुआत

प्रश्न 12.
महात्मा गाँधी किस भाषा को राष्ट्रीय भाषा बनाना चाहते थे?
उत्तर:
हिन्दुस्तानी भाषा को।

प्रश्न 13.
दो सदस्यों के नाम लिखिए जिन्होंने शक्तिशाली केन्द्र की हिमायत की थी?
उत्तर:
(1) डॉ. भीमराव अम्बेडकर तथा
(2) बालकृष्ण शर्मा।

प्रश्न 14.
किस आदिवासी नेता ने विधायिका में आदिवासियों को पृथक् निर्वाचिका का अधिकार दिए जाने की माँग की थी?
उत्तर:
जयपाल सिंह ने।

प्रश्न 15.
पृथक् निर्वाचिका का विरोध करने वाले दो सदस्यों का उल्लेख कीजिये।
उत्तर:
(1) सरदार वल्लभ भाई पटेल
(2) गोविन्द वल्लभ पंत ।

प्रश्न 16.
स्वतन्त्रता प्राप्ति के बाद किसकी सलाह पर डॉ. अम्बेडकर को केन्द्रीय विधिमन्त्री बनाया गया था ?
उत्तर:
महात्मा गाँधी की सलाह पर

प्रश्न 17.
इस भूमिका में डॉ. अम्बेडकर ने किसके रूप में काम किया?
उत्तर:
संविधान की प्रारूप समिति के अध्यक्ष के रूप में।

प्रश्न 18.
भारत के राष्ट्रीय ध्वज में किन रंगों की तीन बराबर चौड़ाई वाली पट्टियाँ हैं ?
उत्तर:
केसरिया, सफेद तथा गहरे हरे रंग की । प्रश्न 19. संविधान सभा के अध्यक्ष कौन थे? उत्तर- डॉ. राजेन्द्र प्रसाद ।

प्रश्न 20.
संविधान सभा के प्रारूप समिति के अध्यक्ष कौन थे?
उत्तर:
डॉ. बी. आर. अम्बेडकर।

प्रश्न 21.
कौनसी सूची के विषय केवल राज्य सरकार के अन्तर्गत आते हैं?
उत्तर:
राज्य सूची के।

प्रश्न 22.
भारत का संविधान 26 जनवरी, 1950 को क्यों लागू किया गया?
उत्तर:
26 जनवरी, 1930 को भारत में पहली बार स्वतन्त्रता दिवस मनाया गया था। अतः 26 जनवरी, 1950 को भारतीय संविधान लागू किया गया।

JAC Class 12 History Important Questions Chapter 15 संविधान का निर्माण : एक नए युग की शुरुआत

प्रश्न 23.
भारतीय संविधान की प्रस्तावना के मुख्य आदर्श क्या हैं ?
उत्तर:
(1) न्याय
(2) स्वतन्त्रता
(3) समानता तथा
(4) अल्पसंख्यकों, पिछड़े तथा जनजातीय लोगों के लिए रक्षात्मक प्रावधान।

प्रश्न 24
विभाजित नवजात भारत राष्ट्र के सामने दो गंभीर समस्याएँ क्या थीं?
उत्तर:
(1) बर्बर हिंसा को समाप्त कर सांप्रदायिक सौहार्द स्थापित करना।
(2) देशी रियासतों के एकीकरण की समस्या।

प्रश्न 25.
भारत में हुए विभिन्न आन्दोलनों का एक अहम पहलू कौनसा था ?
उत्तर:
हिन्दू-मुस्लिम एकता।

प्रश्न 26.
संविधान सभा के सदस्यों को कैसे चुना गया?
उत्तर:
प्रान्तीय संसदों से संविधान सभा के सदस्यों को चुना गया।

प्रश्न 27.
नयी संविधान सभा में कौनसा दल प्रभावशाली था?
उत्तर:
कांग्रेस।

प्रश्न 28.
भारतीय संविधान के संवैधानिक सलाहकार कौन थे?
उत्तर:
बी.एन. राव ।

प्रश्न 29.
ब्रिटिश शासन में किन सुधारों के तहत प्रान्तीय विधायिकाओं में सीमित प्रतिनिधित्व की व्यवस्था लागू की गयी थी?
उत्तर:
मोंटेग्यू चेम्सफोर्ड सुधारों के तहत।

प्रश्न 30.
मद्रास की दक्षायणी वेलायुधान देश के कमजोर वर्ग के लिए क्या चाहती थी?
उत्तर:
मद्रास की दक्षायणी वेलायुधान देश के कमजोर वर्गों हेतु नैतिक सुरक्षा का आवरण चाहती थी।

JAC Class 12 History Important Questions Chapter 15 संविधान का निर्माण : एक नए युग की शुरुआत

प्रश्न 31.
क्या संविधान सभा के सदस्यों का चुनाव सार्वभौमिक मताधिकार के आधार पर हुआ था ?
उत्तर:
नहीं, संविधान सभा के सदस्यों का चुनाव सार्वभौमिक मताधिकार के आधार पर नहीं हुआ था।

प्रश्न 32.
संविधान सभा के सदस्यों को किसने चुना
उत्तर:
1945-46 में भारत के प्रांतों के निर्वाचित सांसदों ने संविधान सभा के सदस्यों को चुना ।

प्रश्न 33.
विभाजन के बाद बनी नई संविधान सभा में कौनसा दल सर्वाधिक प्रभावशाली था?
उत्तर:
भारतीय राष्ट्रीय काँग्रेस नामक राजनैतिक दल

प्रश्न 34.
संविधान सभा में किन छह सदस्यों की भूमिका महत्त्वपूर्ण रही?
उत्तर:
(1) जवाहर लाल नेहरू
(2) वल्लभ भाई पटेल
(3) डॉ. राजेन्द्र प्रसाद
(4) डॉ. बी. आर. अम्बेडकर
(5) के. एम. मुंशी और
(6) अल्लादि कृष्णा स्वामी अय्यर।

प्रश्न 35.
संविधान सभा में उद्देश्य प्रस्ताव कब और किसने प्रस्तुत किया?
उत्तर:
पं. जवाहरलाल नेहरू ने 13 दिसम्बर, 1946 को सभा के सामने उद्देश्य प्रस्ताव प्रस्तुत किया।

प्रश्न 36.
संविधान के मसविदे में कितनी सूचियाँ बनायी गई थीं, उनके नाम लिखिये।
उत्तर:
संविधान के मसविदे में तीन सूचियाँ –
(1) केन्द्रीय सूची
(2) राज्य सूची और
(3) समवर्ती सूची बनाई गई थीं।

प्रश्न 37.
केन्द्रीय सूची के विषय किस सरकार के अधीन हैं?
उत्तर:
केन्द्रीय सूची के विषय केवल केन्द्र सरकार के अधीन हैं।

JAC Class 12 History Important Questions Chapter 15 संविधान का निर्माण : एक नए युग की शुरुआत

प्रश्न 38.
संविधान के कोई दो केन्द्रीय अभिलक्षण लिखिये।
उत्तर:
(1) वयस्क मताधिकार
(2) धर्मनिरपेक्षता पर बल

प्रश्न 39.
संविधान क्या है?
उत्तर:
संविधान एक कानूनी दस्तावेज है, जिसके माध्यम से किसी भी देश का शासन चलाया जाता है।

प्रश्न 40.
भारत का संविधान कब बन कर तैयार हुआ?
उत्तर:
भारत का संविधान 26 नवम्बर 1949 को बनकर तैयार हुआ।

प्रश्न 41.
संविधान की प्रस्तावना का अर्थ समझाते हुए उसका महत्त्व बताइये।
उत्तर:
संविधान की प्रस्तावना के द्वारा संविधान का परिचय कराया जाता है, प्रस्तावना इसे सरकार को मार्ग- दर्शन प्राप्त होता है।

प्रश्न 42.
भारतीय संविधान में धर्मनिरपेक्षता शब्द कब जोड़ा गया ?
उत्तर:
भारतीय संविधान में धर्मनिरपेक्षता शब्द 1976 में 42वें संशोधन द्वारा जोड़ा गया।

प्रश्न 43.
धर्मनिरपेक्षता का अर्थ स्पष्ट कीजिये।
उत्तर:
धर्मनिरपेक्षता का अर्थ है कि राज्य की दृष्टि में सभी धर्म समान हैं, वह किसी भी धर्म को राजधर्म घोषित नहीं करेगा।

प्रश्न 44.
महिलाओं के लिए न्याय की माँग किसने की थी?
उत्तर:
बम्बई की हंसा मेहता

प्रश्न 45.
हंसा मेहता ने महिलाओं के लिए किस प्रकार के न्याय की मांग की थी?
उत्तर:
हंसा मेहता ने महिलाओं के लिए सामाजिक, आर्थिक एवं राजनैतिक न्याय की मांग की थी।

प्रश्न 46.
हिन्दुस्तानी भाषा के प्रश्न पर महात्मा गाँधी को क्या लगता था?
उत्तर:
महात्मा गाँधी को लगता था कि हिन्दुस्तानी भाषा विविध समुदायों के मध्य संचार की आदर्श भाषा हो सकती है।

प्रश्न 47.
हमारे संविधान ने सभी नागरिकों को धार्मिक स्वतंत्रता प्रदान कर रखी है। इस धार्मिक स्वतंत्रता को स्पष्ट कीजिए।
उत्तर:
इसका अर्थ है कि व्यक्ति स्वेच्छा से किसी भी धर्म को अपना सकता है और उसका प्रचार-प्रसार कर सकता है।

JAC Class 12 History Important Questions Chapter 15 संविधान का निर्माण : एक नए युग की शुरुआत

प्रश्न 48.
देशी रियासतों का एकीकरण किसके नेतृत्व में हुआ?
उत्तर:
देशी रियासतों का एकीकरण लौह पुरुष सरदार वल्लभ भाई पटेल के कुशल नेतृत्व में सम्पन्न हुआ। प्रश्न 49 समानता का अधिकार क्या है ? उत्तर- इस अधिकार के तहत कानून की दृष्टि में सभी समान होंगे। सरकारी नौकरी पाने का सभी को समान अवसर मिलेगा।

प्रश्न 50.
भारतीय संविधान के आधारभूत सिद्धान्त व मान्यताएँ क्या थीं?
उत्तर:
संविधान के अनुसार भारत एक धर्मनिरपेक्ष, समाजवादी लोकतांत्रिक गणराज्य होगा, जिसमें वयस्क मताधिकार के आधार पर एक संसदीय प्रणाली होगी।

प्रश्न 51.
संविधान सभा में राष्ट्रीय ध्वज का प्रस्ताव किसने पेश किया था?
उत्तर:
संविधान सभा में पं. जवाहरलाल नेहरू ने उद्देश्य प्रस्ताव के साथ-साथ राष्ट्रीय ध्वज का प्रस्ताव भी पेश किया था।

प्रश्न 52.
शाही भारतीय सेना के सिपाहियों ने कब विद्रोह किया?
उत्तर:
1946 ई. के बसंत में

प्रश्न 53.
पृथक् निर्वाचिका के सवाल पर गोविन्द बल्लभ पंत ने क्या कहा था?
उत्तर:
गोविन्द वल्लभ पंत के अनुसार पृथक् निर्वाचिका अल्पसंख्यकों के लिए आत्मघाती होगी।

प्रश्न 54.
आप कैसे कह सकते हैं कि सारे मुसलमान पृथक् निर्वाचिका के पक्ष में नहीं थे?
उत्तर:
बेगम एजाज रसूल के अनुसार पृथक् निर्वाचिका आत्मघाती साबित होगी क्योंकि इससे अल्पसंख्यक बहुसंख्यकों से कट जाएँगे।

प्रश्न 55.
संविधान सभा की भाषा समिति ने राष्ट्रभाषा के सवाल पर क्या सुझाव दिया?
उत्तर;
भाषा समिति ने सुझाव दिया कि देवनागरी लिपि में लिखी हिन्दी भारत की राजकीय भाषा होगी।

प्रश्न 56.
संविधान निर्माण सभा के प्रमुख चार सदस्यों के नाम लिखिए।
उत्तर:

  • डॉ. राजेन्द्र प्रसाद
  • जवाहरलाल नेहरू
  • सरदार पटेल
  • मौलाना अबुल कलाम आजाद।

प्रश्न 57.
संविधान सभा की प्रथम बैठक कब आयोजित की गई ?
उत्तर:
संविधान सभा की प्रथम बैठक 9 दिसम्बर, 1946 को आयोजित की गई।

प्रश्न 58.
भारत को कैब स्वतन्त्रता प्राप्त हुई ?
उत्तर:
15 अगस्त, 1947 को

लघुत्तरात्मक प्रश्न

प्रश्न 1.
भारतीय संविधान के निर्माण में महत्त्वपूर्ण भूमिका निभाने वाले त्रिगुट का वर्णन कीजिए।
उत्तर:
(1) पं. जवाहरलाल नेहरू ने उद्देश्य प्रस्ताव को प्रस्तुत करते हुए स्वतन्त्र भारत के संविधान के मूल आदर्शों की रूपरेखा प्रस्तुत की।
(2) सरदार वल्लभ भाई पटेल ने कई महत्त्वपूर्ण रिपोर्टों के प्रारूप लिखने में विशेष सहायता की।
(3) डॉ. राजेन्द्र प्रसाद ने संविधान सभा के अध्यक्ष के रूप में कार्य किया और संविधान सभा में चर्चा को रचनात्मक बनाए रखा।

JAC Class 12 History Important Questions Chapter 15 संविधान का निर्माण : एक नए युग की शुरुआत

प्रश्न 2.
हमारा संविधान 26 जनवरी, 1950 को क्यों लागू किया गया? सकारण उत्तर दीजिये।
उत्तर:
हमारा संविधान नवम्बर, 1949 को बनकर तैयार हो गया था लेकिन उसे 2 महीने बाद 26 जनवरी, 1950 को लागू किया गया था। इसके पीछे यह कारण निहित है कि काँग्रेस के 1929 के दिसम्बर में लाहौर में आयोजित अधिवेशन में जवाहर लाल नेहरू ने पूर्ण स्वतंत्रता की घोषणा की तथा 26 जनवरी, 1930 को स्वतन्त्रता दिवस मनाने की घोषणा की। 26 जनवरी 1930 को औपनिवेशिक भारत में प्रथम स्वतंत्रता दिवस मनाया गया।

प्रश्न 3.
राज्य के नीति-निर्देशक तत्व न्यायिक अयोग्यता रखते हैं, क्यों?
उत्तर:
भारत के संविधान के भाग संख्या 4 में नागरिकों के लिए कुछ गारंटी दी गई हैं लेकिन ये न्याय योग्य नहीं हैं। इन्हें राज्य के नीति-निर्देशक तत्व कहा जाता है। इन्हें लागू करना पूर्णतः राज्य की इच्छा पर निर्भर करता है। इन्हें लागू करने के लिए सरकार को बाध्य नहीं किया जा सकता और न ही नागरिक उन्हें लागू करवाने हेतु न्यायालय की शरण में जा सकता है।

प्रश्न 4.
भारतीय संविधान की रूपरेखा पर एक संक्षिप्त टिप्पणी लिखिए।
उत्तर:
भारत की संविधान सभा का गठन 1946 की कैबिनेट मिशन योजना के अन्तर्गत हुआ। डॉ. राजेन्द्र प्रसाद इस सभा के अध्यक्ष बनाए गए। उद्देश्य स्वतंत्र भारत के लिए एक था भारतीय संविधान सभा के संविधान सभा का प्रमुख संविधान का निर्माण करना अधिवेशन के 9 दिसम्बर, 1946 से 26 नवम्बर, 1949 तक कुल 11 सत्र हुए। मूल संविधान 395 धाराओं 22 भागों और आठ अनुसूचियों में बँटा हुआ है, जिसमें 90 हजार शब्द हैं।

प्रश्न 5.
संविधान सभा ने सम्पूर्ण देश का प्रतिनिधित्व किया तो भी यह एक ही पार्टी का समूह बनकर क्यों रह गयी ?
उत्तर:
संविधान सभा के सदस्यों का चुनाव 1946 ई. के प्रान्तीय चुनावों के आधार पर किया गया था। संविधान सभा में भारत के ब्रिटिश प्रान्तों द्वारा भेजे गये सदस्यों के अतिरिक्त रियासतों के प्रतिनिधि भी सम्मिलित थे। मुस्लिम लीग ने स्वतन्त्रता के पूर्व की संविधान सभा की बैठकों का बहिष्कार किया जिसके कारण इस दौर में संविधान सभा एक ही पार्टी का समूह बनकर रह गई थी। संविधान सभा के 82 प्रतिशत सदस्य कांग्रेस पार्टी के सदस्य थे।

प्रश्न 6.
आप कैसे कह सकते हैं कि समस्त मुसलमान पृथक निर्वाचिका की माँग के समर्थन में नहीं थे?
उत्तर:
बेगम ऐजाज रसूल के संविधान सभा में दिए गए भाषण के आधार पर हम कह सकते हैं कि समस्त मुसलमान पृथक् निर्वाचिका के समर्थन में नहीं थे बेगम ऐजाज रसूल को लगता था कि पृथक् निर्वाचिका आत्मघाती सिद्ध होगी। क्योंकि इससे अल्पसंख्यक बहुसंख्यकों से कट जायेंगे।

प्रश्न 7.
संविधान में केन्द्र को अधिक शक्तिशाली बनाने के लिए किए गए किन्हीं तीन प्रावधानों का उल्लेख कीजिए।
उत्तर;

  • केन्द्रीय सूची में बहुत अधिक विषय रखे गये।
  • खनिज पदार्थों एवं आधारभूत उद्योगों पर केन्द्र सरकार का ही नियन्त्रण रखा गया।
  • अनुच्छेद 356 के तहत राज्यपाल की सिफारिश पर केन्द्र सरकार को राज्य सरकार के समस्त अधिकार अपने हाथ में लेने का अधिकार दिया गया।

प्रश्न 8.
संविधान सभा में हुई चर्चाएँ जनमत से कैसे प्रभावित होती थी? स्पष्ट कीजिए।
उत्तर:

  • जब संविधान सभा में बहस होती थी तो विभिन्न पक्षों के तर्क समाचार-पत्रों में छपते थे तथा समस्त प्रस्तावों पर सार्वजनिक रूप से बहस चलती थी। इस तरह प्रेस में होने वाली इस आलोचना तथा जवाबी आलोचना से किसी मुद्दे पर बनने वाली सहमति या असहमति पर गहरा प्रभाव पड़ता था।
  • सामूहिक सहभागिता बनाने के लिए देश की जनता के सुझाव भी आमन्त्रित किये जाते थे।
  • कई भाषायी अल्पसंख्यक अपनी मातृभाषा की रक्षा की माँग करते थे।
  • धार्मिक अल्पसंख्यक अपने विशेष हित सुरक्षित करवाना चाहते थे और दलित जाति के लोग शोषण के अन्त की माँग करते हुए राजकीय संस्थाओं में आरक्षण चाहते थे।

प्रश्न 9.
भारतीय संविधान निर्माण से पहले के वर्ष काफी उथल-पुथल वाले थे क्यों ?
अथवा
“संविधान निर्माण के पूर्व के वर्ष उथल-पुथल के दौर से गुजर रहे थे।” उदाहरण देकर इस कथन को स्पष्ट कीजिए।
उत्तर:
15 अगस्त, 1947 को मिली आजादी के साथ ही देश को दो टुकड़ों में बाँट दिया गया। लोगों के मस्तिष्क में 1942 के भारत छोड़ो आन्दोलन की यादें अभी भी जीवित थीं, जो ब्रिटिश शासन के विरुद्ध सबसे बड़ा जन आन्दोलन था। नेताजी सुभाष चन्द्र बोस द्वारा विदेशी सहायता से देश को स्वतंत्र कराने के प्रयास लोगों को बखूबी याद थे। 1946 के बसंत में बम्बई तथा अन्य शहरों में रॉयल इण्डियन नेवी (शाही भारतीय नौसेना) के सिपाहियों द्वारा किया जाने वाला विद्रोह भी उल्लेखनीय था।

JAC Class 12 History Important Questions Chapter 15 संविधान का निर्माण : एक नए युग की शुरुआत

प्रश्न 10.
स्वतन्त्रता के समय देशी रियासतों की समस्याओं का वर्णन कीजिये।
अथवा
नवजात राष्ट्र के सामने देशी रजवाड़ों के एकीकरण की समस्या बहुत ही गंभीर थी। विवेचना कीजिये।
उत्तर:
ब्रिटिश शासन के दौरान भारत का लगभग एक-तिहाई भू-भाग ऐसे नवाबों और रजवाड़ों के नियन्त्रण में था, जो ब्रिटिश ताज की अधीनता स्वीकार कर चुके थे। उन्हें काफी स्वतंत्रता प्राप्त थी अंग्रेजों के भारत से चले जाने के बाद इन राजाओं और नवाबों की संवैधानिक स्थिति बहुत विचित्र हो गई थी। एक प्रेक्षक ने कहा था कि कुछ शासक तो अनेक टुकड़ों में बटे भारत में स्वतन्त्र सत्ता का सपना देख रहे थे।

प्रश्न 11.
एक शक्तिशाली केन्द्र सरकार के पक्ष में नेहरूजी ने जो बयान दिया था उसे लिखिए।
उत्तर:
नेहरूजी शक्तिशाली केन्द्र के पक्ष में थे। उन्होंने संविधान सभा के अध्यक्ष डॉ. राजेन्द्र प्रसाद को लिखे पत्र में कहा था, “अब जबकि विभाजन एक हकीकत बन चुका है…… एक दुर्बल केन्द्रीय शासन व्यवस्था देश के लिए हानिकारक सिद्ध होगी क्योंकि ऐसा केन्द्र शान्ति स्थापित करने में आम सरोकारों के बीच समन्वय स्थापित करने में और अंतर्राष्ट्रीय स्तर पर पूरे देश के लिए आवाज उठाने में सक्षम नहीं होगा।”

प्रश्न 12.
संविधान निर्माण से पूर्व के कुछ आन्दोलनों का महत्त्वपूर्ण पहलू किस तरह से व्यापक हिन्दू-मुस्लिम एकता को धारण किए हुए था? संक्षेप में स्पष्ट कीजिए।
उत्तर:
संविधान निर्माण से पूर्व के कुछ आन्दोलनों का एक अहम पहलू व्यापक हिन्दू-मुस्लिम एकता इन जन आन्दोलनों का एक अहम पहलू था। इसके विपरीत काँग्रेस और मुस्लिम लीग दोनों प्रमुख राजनैतिक दल धार्मिक सौहार्द और सामाजिक सामंजस्य स्थापित करने के लिए सुलह-सफाई की कोशिशों में असफल होते जा रहे थे। अगस्त, 1946 में कलकत्ता में शुरू हुई हिंसा के साथ उत्तरी और पूर्वी भारत में लगभग साल भर चलने वाले दंगा- फसाद भड़क उठे ।

प्रश्न 13.
एक शक्तिशाली केन्द्र के विषय में के. सन्तनम के विचार लिखिए।
उत्तर:
एक शक्तिशाली केन्द्र के विषय में के. सन्तनम ने कहा कि न केवल राज्यों को बल्कि केन्द्र को मजबूत बनाने के लिए भी शक्तियों का पुनर्वितरण आवश्यक है। यदि केन्द्र के पास आवश्यकता से अधिक जिम्मेदारियाँ होंगी तो वह प्रभावी ढंग से कार्य नहीं कर पाएगा। उसके कुछ दायित्वों को राज्यों को सौंपने से केन्द्र अधिक मजबूत हो सकता है।

प्रश्न 14.
भारतीय संविधान सभा की भाषा समिति राष्ट्रभाषा के प्रश्न पर क्या सुझाव दिया?
उत्तर:
राष्ट्रभाषा के सवाल पर संविधान सभा की भाषा समिति ने सुझाव दिया कि देवनागरी लिपि में लिखी हिन्दी भारत की राजकीय भाषा होगी। परन्तु इस फार्मूले को समिति ने घोषित नहीं किया था। हिन्दी को राष्ट्रभाषा बनाने के लिए धीरे-धीरे आगे बढ़ना चाहिए। पहले 15 वर्षों तक राजकीय कार्यों में अंग्रेजी का प्रयोग जारी रहेगा।

प्रश्न 15.
अगस्त 1947 का अवसर अनेक मुसलमानों, हिन्दुओं और सिक्खों के लिए निर्मम चुनाव का क्षण किस तरह से था? स्पष्ट कीजिए।
उत्तर;
15 अगस्त, 1947 को स्वतन्त्रता दिवस पर आनन्द व उम्मीद का वातावरण था परन्तु भारत के बहुत से मुसलमानों और पाकिस्तान में रहने वाले हिन्दुओं तथा सिखों के लिए यह एक निर्मम क्षण था उन्हें मृत्यु अथवा अपनी पीढ़ियों की पुरानी जगह छोड़ने के बीच चुनाव करना था। करोड़ों की संख्या में शरणार्थी इधर से उधर जा रहे थे। मुसलमान पूर्वी व पश्चिमी पाकिस्तान की ओर तो हिन्दू व सिख पश्चिमी बंगाल एवं पूर्वी पंजाब की ओर बड़े जा रहे थे। उन लोगों में अनेक कभी मंजिल तक ही नहीं पहुँच सके और बीच रास्ते में ही दम तोड़ दिया।

JAC Class 12 History Important Questions Chapter 15 संविधान का निर्माण : एक नए युग की शुरुआत

प्रश्न 16.
एक शक्तिशाली केन्द्र सरकार के पक्ष में नेहरूजी ने संविधान सभा में क्या बयान दिया? टिप्पणी लिखिए।
उत्तर:
संविधान सभा में केन्द्र व राज्य सरकारों के अधिकारों को लेकर काफी बहस हुई शक्तिशाली केन्द्र के पक्ष में नेहरूजी ने अपने विचार प्रकट किये थे। उन्होंने संविधान सभा के अध्यक्ष डॉ. राजेन्द्र प्रसाद को लिखे पत्र में कहा था- ” अब जबकि विभाजन एक हकीकत बन चुका है। एक कमजोर केन्द्रीय शासन व्यवस्था देश के लिए हानिकारक सिद्ध होगी क्योंकि ऐसा केन्द्र शान्ति स्थापित करने में आम सरोकारों के मध्य समन्वय स्थापित करने में और अन्तर्राष्ट्रीय स्तर पर पूरे देश के लिए आवाज उठाने में सक्षम नहीं होगा। इसलिए राज्यों से केन्द्र को अधिक ताकतवर बनाना ही ठीक होगा।”

प्रश्न 17.
17 अगस्त, 1947 की मध्य रात्रि को नेहरूजी ने संविधान सभा में जो भाषण दिया था उसके एक महत्त्वपूर्ण अंश की संक्षिप्त व्याख्या अपने शब्दों में कीजिए।
उत्तर:
14 अगस्त, 1947 को मध्य रात्रि में नेहरूजी ने संविधान सभा में भाषण देते हुए कहा था –
“बहुत समय पहले हमने नियति से साक्षात्कार किया था और अब समय आ चुका है कि हम अपने उस संकल्प को न केवल पूर्ण रूप से या समग्रता में बल्कि उल्लेखनीय रूप से साकार करें अर्द्धरात्रि के इस क्षण में जब दुनिया सो रही है, भारत जीवन और स्वतंत्रता की ओर जाग रहा है।”

प्रश्न 18.
उद्देश्य प्रस्ताव का स्वागत करते हुए आदिवासी प्रतिनिधि जयपाल सिंह ने जो विचार प्रकट किए उन्हें संक्षेप में लिखिए।
उत्तर:
अगर भारतीय समाज में कोई ऐसा समूह है –
जिसके साथ सही व्यवहार नहीं किया गया है तो वह मेरा समूह है जिसे पिछले 6000 वर्षों से अपमानित किया जा रहा है और उपेक्षा का शिकार हो रहा है। आदिवासी कबीले संख्या की दृष्टि से अल्पसंख्यक नहीं हैं लेकिन उन्हें संरक्षण की आवश्यकता है। उन्हें उनके चरागाहों व जंगलों से बचत कर दिया गया है। हम आपके साथ मेलजोल चाहते हैं। आदिवासियों को प्रतिनिधित्व प्रदान करने के लिए सीटों के आरक्षण की व्यवस्था जरूरी है।

प्रश्न 19.
नई संविधान सभा में काँग्रेस प्रभावशाली क्यों थी?
उत्तर:

  1. प्रांतीय चुनावों में काँग्रेस ने सामान्य चुनाव क्षेत्रों में भारी जीत प्राप्त की।
  2. यद्यपि मुस्लिम लीग को अधिकांश आरक्षित मुस्लिम सीटें मिल गई थीं, लेकिन लीग ने संविधान सभा का बहिष्कार कर दिया था और वह पाकिस्तान की माँग जारी रखे हुए थी।
  3. प्रारम्भ में समाजवादी भी संविधान सभा से परे रहे क्योंकि वे उसे अँग्रेजों की बनाई संस्था मानते थे। वे मानते थे कि इस सभा का वाकई स्वायत्त होना असम्भव है।
  4. संविधान सभा के 82 प्रतिशत सदस्य काँग्रेस पार्टी के ही सदस्य थे।

प्रश्न 20.
भारतीय संविधान की रूपरेखा को संक्षेप में बताइए।
उत्तर:
भारत के संविधान हेतु संविधान सभा का गठन त 1946 ई. के कैबिनेट मिशन योजना के अन्तर्गत हुआ। इस त संविधान सभा में 300 सदस्य थे डॉ. राजेन्द्र प्रसाद को इस न संविधान सभा का अध्यक्ष बनाया गया था। भारतीय संविधान को 9 दिसम्बर, 1946 से 26 नवम्बर, 1949 के मध्य सूत्रबद्ध किया गया। संविधान सभा के कुल 11 सत्र हुए जिनमें 165 दिन बैठकों में गए मूल संविधान में 22 भाग, 395 अनुच्छेद एवं 8 अनुसूचियाँ थीं जिनके बाद में कई संशोधन हो चुके हैं। यह विश्व का सबसे लम्बा संविधान है जो 26 जनवरी, 1950 को अस्तित्व में आया।

प्रश्न 21.
नेहरूजी ने अपने उद्देश्य प्रस्ताव में अमेरिकी व फ्रांसीसी संविधान सभाओं से हमको प्रेरणा लेने की बात कही है। क्यों?
उत्तर:
नेहरूजी ने अपने उद्देश्य प्रस्ताव में अमेरिकी व फ्रांसीसी संविधान सभाओं का उल्लेख करते हुए कहा ” कि जिस प्रकार उन्होंने अनेक कठिनाइयों का सामना करते हुए संविधान के निर्माण का कार्य पूर्ण किया है उसी प्रकार हम भी उन संविधान सभाओं की तरह ही अपना संविधान बनाकर ही दम लेंगे चाहे इसके मार्ग में कितनी ही परेशानियाँ एवं रुकावटें आएँ हम भी उनकी ही तरह एक कालजयी संविधान का निर्माण करेंगे जो हमारी जनता के स्वभाव के अनुकूल होकर उनकी समस्याओं का समाधान प्रस्तुत करेगा। हमारा संविधान भी उन संविधानों की तरह ही लोकतन्त्रात्मक, धर्मनिरपेक्ष एवं आर्थिक-सामाजिक न्याय को स्थापना करने वाला होगा।

JAC Class 12 History Important Questions Chapter 15 संविधान का निर्माण : एक नए युग की शुरुआत

प्रश्न 22.
नेहरू ने अमेरिकी संविधान निर्माताओं और फ्रांसीसी संविधान निर्माताओं का उल्लेख उद्देश्य प्रस्ताव में किसलिए किया?
उत्तर:
नेहरू ने अमेरिका संविधान निर्माण की प्रक्रिया के सम्बन्ध में कहा कि अमेरिकी राष्ट्र निर्माताओं ने एक ऐसा संविधान रचा जो डेढ़ सदी से भी ज्यादा समय से कसौटी पर खरा उतर रहा है। उन्होंने संविधान पर आधारित एक महान् राष्ट्र गढ़ा। दूसरे, नेहरू ने उस संविधान सभा का उल्लेख किया जो स्वतंत्रता के इतने सारे संघर्ष लड़ने वाले पेरिस के भव्य एवं खूबसूरत शहर में जुटी थी उस संविधान सभा ने अनेक मुश्किलों का सामना किया।

प्रश्न 23.
” संविधान सभा अंग्रेजों की बनाई हुई है और वह अंग्रेजों की योजना को साकार करने का काम कर रही है।” सोमनाथ लाहिड़ी के इस कथन पर अपने विचार व्यक्त कीजिए।
उत्तर:
संविधान सभा के कम्युनिस्ट सदस्य सोमनाथ लाहिड़ी का कहना था कि संविधान सभा अंग्रेजों की बनाई हुई है और वह अंग्रेजों की योजना को साकार करने का काम कर रही है न केवल ब्रिटिश योजना ने भावी संविधान बना दिया है, बल्कि इससे यह भी संकेत मिलता है कि मामूली से मामूली मतभेद के लिए भी संघीय न्यायालय जाना होगा।

प्रश्न 24.
भारतीय संविधान में विषयों की तीन सूचियों और अनुच्छेद 356 का उल्लेख करें।
अथवा
संविधान के अनुसार केन्द्र और राज्य सरकारों के बीच शक्तियों का बँटवारा किस प्रकार किया गया?
अथवा
संघ सूची, राज्य सूची तथा समवर्ती सूची का संक्षिप्त वर्णन करें।
अथवा
राज्य सूची के विषय में आप क्या जानते हैं ? समवर्ती सूची पर प्रकाश डालिए।
उत्तर:
संविधान के मसविदे में समस्त विषयों की तीन सूचियाँ बनाई गई हैं। ये हैं –
(1) केन्द्रीय या संघ सूची
(2) राज्य सूची और
(3) समवर्ती सूची यथा
(1) केन्द्रीय सूची में दिए गए विषय केवल केन्द्र सरकार के अधीन रखे गए हैं। राज्य सूची
(2) अन्तर्गत रखे गये हैं। के विषय केवल राज्य सरकारों के
(3) समवर्ती सूची में दिए गए केन्द्र और राज्य दोनों की साझा जिम्मेदारी है।
अनुच्छेद 356 में गवर्नर की सिफारिश पर केन्द्र सरकार को राज्य सरकार के समस्त अधिकार अपने हाथ में लेने का अधिकार दिया गया है।

प्रश्न 25.
केन्द्र को अधिक शक्तिशाली बनाने वाले प्रावधानों का उल्लेख कीजिये ।
उत्तर:
(1) अन्य संघों की तुलना में केन्द्रीय सूची में बहुत ज्यादा विषयों को केवल केन्द्रीय नियन्त्रण में रखा गया है।
(2) समवर्ती सूची में भी प्रान्तों की इच्छाओं की उपेक्षा करते हुए बहुत ज्यादा विषय रखे गये हैं तथा राज्य की तुलना में केन्द्र को वरीयता दी गई है।
(3) खनिज पदार्थों तथा प्रमुख उद्योगों पर भी केन्द्र सरकार को ही नियन्त्रण दिया गया है।
(4) अनुच्छेद 356 में गवर्नर की सिफारिश पर केन्द्र सरकार को राज्य सरकार के सारे अधिकार अपने हाथ में लेने का अधिकार दिया गया है।

JAC Class 12 History Important Questions Chapter 15 संविधान का निर्माण : एक नए युग की शुरुआत

प्रश्न 26.
संविधान में राजकोषीय संघवाद की क्या व्यवस्था की गई है?
उत्तर:
(1) कुछ करों (जैसे सीमा शुल्क और कम्पनी कर) से होने वाली सारी आय केन्द्र सरकार के पास रखी गई है।
(2) कुछ अन्य मामलों में (जैसे- आय कर और आबकारी शुल्क) में होने वाली आय राज्य और केन्द्र सरकार के बीच बाँट दी गई है।
(3) कुछ अन्य मामलों (जैसे- राज्य स्तरीय शुल्क) से होने वाली आय पूरी तरह राज्यों को सौंप दी गई है। (4) राज्य सरकारों को अपने स्तर पर भी कुछ अधिभार और कर वसूलने का अधिकार दिया गया है।

निबन्धात्मक प्रश्न

प्रश्न 1.
संविधान सभा में पृथक् निर्वाचिका की माँग किसके लिए की गई थी? यह माँग किसने उठाई तथा इसका क्या परिणाम रहा?
उत्तर:
पृथक निर्वाचिका की माँग अल्पसंख्यकों के लिए विभिन्न नेताओं ने समय-समय पर संविधान सभा में अपने तर्क देते हुए उठाई थी, उनकी माँग का विरोध भी किया गया।

(1) बी. पोकर बहादुर की माँग-27 अगस्त, 1947 को पास के थी. पोकर बहादुर ने पृथक् निर्वाचिका बनाए रखने के पक्ष में एक प्रभावशाली भाषण दिया। बहादुर कहा कि अल्पसंख्यक सब जगह होते हैं, हम उन्हें चाहकर भी हटा नहीं सकते। हमें जरूरत एक ऐसे ढाँचे की है जिसके भीतर अल्पसंख्यक भी औरों के साथ सद्भाव से रह सकें और समुदायों के बीच मतभेद कम से कम हों।

(2) एन. जी. रंगा का बयान-रंगा ने कहा था कि तथाकथित पाकिस्तानी प्रान्तों में रहने वाले हिन्दू, सिख यहाँ तक कि मुसलमान भी अल्पसंख्यक नहीं हैं। असली अल्पसंख्यक तो यहाँ की जनता है।

(3) जयपाल सिंह का बयान-जयपाल सिंह आदिवासी नेता थे। उन्होंने कहा कि आदिवासी कबीले संख्या की दृष्टि से अल्पसंख्यक नहीं हैं लेकिन उन्हें संरक्षण की जरूरत है। वे विधायिका में आदिवासियों के प्रतिनिधित्व के लिए आरक्षण की व्यवस्था चाहते थे।

(4) डॉ. अम्बेडकर की माँग डॉ. अम्बेडकर ने राष्ट्रीय आन्दोलन के दौरान पृथक् निर्वाचिका की माँग की थी।

(5) नागप्पा का बयान मद्रास के सदस्य जे. नागप्पा ने पृथक निर्वाचिका की माँग उठाई।

(6) के. जे. खाण्डेलकर का बयान-खाण्डेलकर ने कहा था—“हमें हजारों साल तक दबाया गया है। इस हद तक दबाया गया कि हमारे दिमाग, हमारी देह काम नहीं करती और अब हमारा हृदय भी भावशून्य हो चुका है।

” पृथक् निर्वाचिका का विरोध –
(1) पं. गोविन्द वल्लभ पंत के विचार-यह प्रस्ताव न केवल राष्ट्र के लिए बल्कि अल्पसंख्यकों के लिए भी खतरनाक है। उनका मानना था कि पृथक निर्वाचिका अल्पसंख्यकों के लिए आत्मघाती साबित होगी, जो उन्हें कमजोर बना देगी और शासन में उन्हें प्रभावी हिस्सेदारी नहीं मिल पायेगी।

(2) बेगम एजाज रसूल के विचार बेगम एजज रसूल को लगता था कि पृथक निर्वाचिका आत्मघाती साबित होगी क्योंकि इससे अल्पसंख्यक बहुसंख्यकों से कट जाएँगे।

(3) महात्मा गाँधी द्वारा विरोध-गाँधीजी ने यह कहते हुए अपना विरोध प्रकट किया था कि पृथक् निर्वाचिका की माँग करने से ये समुदाय शेष समुदायों से हमेशा के लिए कट जाएँगे। संविधान सभा का सुझाव-संविधान सभा ने अंततः यह सुझाव दिया कि अस्पृश्यता का उन्मूलन किया जाए, हिन्दू मन्दिरों के द्वार सभी जातियों के लिए खोल दिए जाएँ और निचली जातियों को विधायिकाओं और सरकारी नौकरियाँ में आरक्षण दिया जाए।

प्रश्न 2.
संविधान निर्माण से पूर्व के वर्ष भारत के लिए बहुत उथल-पुथल बाल थे।” उपर्युक्त कथन के समर्थन में उदाहरण सहित अपना तर्क लिखिए।
उत्तर:
इसमें कोई सन्देह नहीं है कि संविधान निर्माण से पूर्व के वर्ष भारत के लिए बहुत उथल-पुथल वाले थे। इस सम्बन्ध में निम्नलिखित तर्क दिए जा सकते हैं –
(1) 15 अगस्त, 1947 को भारत स्वतन्त्र तो हो गया, परन्तु इसके साथ ही इसे दो भागों भारत व पाकिस्तान के रूप में विभाजित भी कर दिया गया।

(2) लोगों की याद में 1942 भारत छोड़ो आन्दोलन अभी भी जीवित था जो ब्रिटिश औपनिवेशिक राज्य के विरुद्ध सम्भवतः सबसे व्यापक जनान्दोलन था

(3) विदेशी सहायता से सशस्त्र संघर्ष द्वारा के ता पाने के लिए सुभाष चन्द्र बोस द्वारा किए गए प्रयत्न भी लोगों को याद थे।

(4) सन् 1946 में बम्बई व देश के अन्य शहरों में रॉयल्स इण्डिया नेवी (शाही भारतीय नौसेना) के सिपाहियों का विद्रोह भी लोगों को बार-बार आन्दोलित कर रहा था। लोगों की सहानुभूति इन सिपाहियों के साथ थी।

(5) 1940 के दशक के अन्तिम वर्षों में देश के विभिन्न भागों में किसानों व मजदूरों के आन्दोलन भी हो रहे थे।

(6) हिन्दू मुस्लिम एकता विभिन्न जनान्दोलनों का एक महत्त्वपूर्ण पहलू था। इसके विपरीत कांग्रेस व मुस्लिम लीग दोनों ही मुख्य राजनीतिक दल धार्मिक सद्भावना और सामाजिक तालमेल स्थापित करने में सफल नहीं हो पा रहे थे।

(7) 16 अगस्त, 1946 में मुस्लिम लीग द्वारा प्रत्यक्ष कार्यवाही दिवस मनाने की घोषणा से कलकत्ता में हिंसा भड़क उठी।

(8) देश के भारत व पाकिस्तान के रूप में विभाजन की घोषणा के पश्चात् असंख्य लोग एक स्थान से दूसरे स्थान पर जाने लगे। जिससे शरणार्थियों की समस्या खड़ी हो गई थी।

(9) 15 अगस्त, 1947 को स्वतन्त्रता दिवस पर आनन्द और उम्मीद का वातावरण था। लेकिन भारत के मुसलमानों व पाकिस्तान में रहने वाले हिन्दुओं व सिखों के लिए यह एक निर्मम क्षण था। मुसलमान पूर्वी व पश्चिमी पाकिस्तान की ओर तो हिन्दू और सिख पश्चिमी बंगाल तथा पूर्वी पंजाब की ओर बढ़ रहे थे।

(10) नवजात राष्ट्र के समक्ष एक और समस्या देशी रियासतों को लेकर थी। ब्रिटिश औपनिवेशिक सरकार के शासन काल के दौरान भारतीय उपमहाद्वीप का लगभग एक-तिहाई भू-भाग ऐसे नवाबों और रजवाड़ों के नियन्त्रण में था जो ब्रिटिश ताज की अधीनता स्वीकार कर चुके थे

JAC Class 12 History Important Questions Chapter 15 संविधान का निर्माण : एक नए युग की शुरुआत

प्रश्न 3.
संविधान सभा में पृथक् निर्वाचिकाओं की माँग के जवाब में सरदार पटेल, धुलेकर तथा गोविन्द वल्लभ पंत आदि प्रमुख कांग्रेसी सदस्यों ने अनेक दलीलें प्रस्तुत कीं। इन दलीलों के पीछे कौनसी चिन्ता काम कर रही थी? अन्त में क्या सहमति बनी?
उत्तर:
पृथक् निर्वाचिकाओं की मांग के विरोध में दी गई समस्त दलीलों के पीछे एक एकीकृत राज्य के निर्माण की चिन्ता काम कर रही थी। वह इस प्रकार थी –
(1) व्यक्ति को नागरिक बनाना तथा प्रत्येक समूह को राष्ट्र का अंग बनाना-राजनीतिक एकता और राष्ट्र की स्थापना करने के लिए प्रत्येक व्यक्ति को राज्य के नागरिक के सांचे में ढालना था, हर समूह को राष्ट्र भीतर समाहित किया जाना था।

(2) नागरिकों में राज्य के प्रति निष्ठा का होना- संविधान नागरिकों को अधिकार देगा परन्तु नागरिकों को भी राज्य के प्रति अपनी निष्ठा का वचन लेना होगा।

(3) सभी समुदायों के सदस्यों को राज्य के सामान्य सदस्यों के रूप में काम करना-समुदायों को सांस्कृतिक इकाइयों के रूप में मान्यता दी जा सकती थी और उन्हें सांस्कृतिक अधिकारों का आश्वासन दिया जा सकता था मगर राजनीतिक रूप से सभी समुदायों के सदस्यों को राज्य के सामान्य सदस्य के रूप में काम करना था अन्यथा उनकी निष्ठाएँ विभाजित होतीं। पंत ने इसे स्पष्ट करते हुए कहा कि, “हमारे भीतर यह आत्मघाती और अपमानजनक आदत बनी हुई है कि हम कभी नागरिक के रूप में नहीं सोचते बल्कि समुदाय के रूप में ही सोच पाते हैं…….। हमें याद रखना चाहिए कि महत्व केवल नागरिक का होता है। सामाजिक पिरामिड का आधार भी और उसकी चोटी भी नागरिक ही होता है।”

(4) एक शक्तिशाली राष्ट्र व शक्तिशाली राज्य की स्थापना-जब सामुदायिक अधिकारों का महत्त्व रेखांकित किया जा रहा था, उस समय भी बहुत सारे राष्ट्रवादियों में यह भय सिर उठाने लगा था कि इससे निष्ठाएँ खण्डित होंगी और एक शक्तिशाली राष्ट्र व शक्तिशाली राज्य की स्थापना नहीं हो पायेगी।

(5) कुछ मुसलमान भी पृथक् निर्वाचिका की मांग के समर्थन में नहीं-सारे मुसलमान भी पृथक् निर्वाचिका की मांग के समर्थन में नहीं थे। उदाहरण के लिए बेगम एजाज रसूल को लगता था कि पृथक निर्वाचिका आत्मघाती साबित होगी क्योंकि इससे अल्पसंख्यक बहुसंख्यकों से कट जायेंगे।

पृथक् निर्वाचिका की माँग पर निर्णय सन् 1949 तक संविधान सभा के ज्यादातर सदस्य इस बात पर सहमत हो गए थे कि पृथक् निर्वाचिका का प्रस्ताव अल्पसंख्यकों के हितों के खिलाफ जाता है। इसकी बजाय मुसलमानों को लोकतांत्रिक प्रक्रिया में सक्रिय भूमिका निभानी चाहिए ताकि राजनीतिक व्यवस्था में उनको एक निर्णायक आवाज मिल सके।

प्रश्न 4.
संविधान सभा के ऐसे दो महत्त्वपूर्ण अभिलक्षणों का उल्लेख कीजिए जिन पर संविधान सभा में काफी हद तक सहमति थी।
उत्तर:
व्यापक सहमति वाले महत्त्वपूर्ण अभिलक्षण – भारतीय संविधान गहन विवादों और परिचर्चाओं से गुजरते हुए बना उसके कई प्रावधान लेन-देन की प्रक्रिया के जरिए बनाए गए थे उन पर सहमति तब बन पाई जब सदस्यों ने दो विरोधी विचारों के बीच की जमीन तैयार कर ली लेकिन संविधान के कुछ ऐसे महत्त्वपूर्ण अभिलक्षण भी हैं, जिन पर संविधान सभा में काफी हद तक सहमति थी। यथा –

(1) वयस्क मताधिकार संविधान का एक केन्द्रीय अभिलक्षण वयस्क मताधिकार है। इस पर संविधान सभा में प्रायः आम सहमति थी यह सहमति प्रत्येक वयस्क भारतीय को मताधिकार देने पर थी। इसके पीछे एक खास किस्म का भरोसा था जिसके पूर्व उदाहरण अन्य देश के इतिहास में नहीं थे। दूसरे लोकतंत्रों में पूर्ण वयस्क मताधिकार धीरे-धीरे कई चरणों से गुजरते हुए, लोगों को मिला। संयुक्त राज्य अमेरिका और यूनाइटेड किंगडम जैसे देशों में शुरू-शुरू में मताधिकार केवल सम्पत्ति रखने वाले पुरुषों को ही दिया गया, फिर पढ़े-लिखे पुरुषों को इस विशेष वर्ग में शामिल किया गया। लम्बे व कटु संघर्षो के बाद श्रमिक व किसान वर्ग के पुरुषों को मताधिकार मिल पाया। ऐसा अधिकार पाने के लिए महिलाओं को और भी लम्बा संघर्ष करना पड़ा।

(2) धर्मनिरपेक्षता पर बल – हमारे संविधान का दूसरा महत्त्वपूर्ण अभिलक्षण था-
धर्मनिरपेक्षता पर बल। संविधान की प्रस्तावना में धर्मनिरपेक्षता के गुण तो नहीं गाए गए थे परन्तु संविधान व समाज को चलाने के लिए भारतीय सन्दर्भों में उसके मुख्य अभिलक्षणों का जिक्र आदर्श रूप में किया गया था। ऐसा मूल अधिकारों की श्रृंखला को रचने के जरिये किया गया, विशेषकर ‘धार्मिक स्वतंत्रता का अधिकार’ (अनुच्छेद 25-28), ‘सांस्कृतिक एवं शैक्षिक अधिकार’ (अनुच्छेद 29-30) एवं ‘समानता का अधिकार’ (अनुच्छेद 14, 16, 17)।

JAC Class 12 History Important Questions Chapter 15 संविधान का निर्माण : एक नए युग की शुरुआत

यथा –
(1) राज्य ने सभी धर्मों के प्रति समान व्यवहार की गारन्टी दी और उन्हें हितैषी संस्थाएँ बनाने का अधिकार भी दिया। और सरकारी स्कूलों व
(2) राज्य ने अपने आपको विभिन्न धार्मिक समुदायों से दूर रखने की कोशिश की कॉलेजों में अनिवार्य धार्मिक शिक्षा पर रोक लगा दी।
(3) सरकार ने रोजगार में धार्मिक भेदभाव को अवैध ठहराया।
(4) राज्य धार्मिक समुदायों से जुड़े सामाजिक सुधार मुं कार्यक्रमों के लिए अवश्य कुछ कानूनी गुंजाइश अर्थात् राज्य ने उसमें दखल देने की गुंजाइश रखी। ऐसा करके ही अस्पृश्यता पर कानूनी रोक लग पायी और इसी कारण इ व्यक्तिगत एवं पारिवारिक कानूनों में परिवर्तन हो पाये।
(5) भारतीय राजनीतिक धर्मनिरपेक्षता में राज्य व धर्म के बीच पूर्ण विच्छेद नहीं रहा। संविधान सभा ने इन दोनों के बीच एक विवेकपूर्ण फासला बनाने की कोशिश की है।

प्रश्न 5.
संविधान सभा के क्रियाकलापों में किन- किन सदस्यों की भूमिका सर्वाधिक महत्त्वपूर्ण थी? उनकी भूमिका पर विस्तार से प्रकाश डालिए।
उत्तर:
संविधान सभा के कुल 300 सदस्यों में से 6 सदस्यों की भूमिका सर्वाधिक महत्त्वपूर्ण थी। इनके नाम है –

  1. पं. जवाहरलाल नेहरू
  2. सरदार वल्लभ भाई पटेल
  3. डॉ. राजेन्द्र प्रसाद
  4. डॉ. भीमराव रामजी अम्बेडकर
  5. के.एम. मुंशी
  6. अल्लादि कृष्णास्वामी अय्यर।

(1) पं. जवाहरलाल नेहरू ने संविधान सभा में 13 दिसम्बर, 1946 को एक निर्णायक प्रस्ताव ‘उद्देश्य प्रस्ताव’ प्रस्तुत किया था। यह एक ऐतिहासिक प्रस्ताव था जिसमें स्वतन्त्र भारत के संविधान के मूल आदर्शों की रूपरेखा प्रस्तुत की गयी थी तथा यह फ्रेमवर्क सुझाया गया था जिसके तहत संविधान का कार्य आगे बढ़ना था। पं. नेहरू ने संविधान सभा में झण्डा प्रस्ताव भी पेश किया था। नेहरू ने कहा था कि भारत का राष्ट्रीय ध्वज केसरिया, सफेद एवं गहरे हरे रंग की तीन बराबर पट्टियों वाला तिरंगा झण्डा होगा जिसके मध्य में गहरे नीले रंग का चक्र होगा।

(2) जवाहरलाल नेहरू के विपरीत वल्लभ भाई पटेल की भूमिका परदे के पीछे की थी उन्होंने अनेक प्रतिवेदनों के प्रारूप लिखे।

(3) भारत के प्रथम राष्ट्रपति तथा संविधान सभा के अध्यक्ष डॉ. राजेन्द्र प्रसाद सभा की चर्चाओं को रचनात्मक दिशा की ओर ले जाते थे। वे इस बात का भी ध्यान रखते थे कि सभी सदस्यों को अपनी बात रखने का अवसर मिले।

(4) कांग्रेस के इस त्रिगुट के अतिरिक्त प्रसिद्ध विधिवेत्ता एवं अर्थशास्त्री डॉ. भीमराव रामजी अम्बेडकर भी संविधान सभा के सबसे महत्वपूर्ण सदस्यों में से एक थे भीमराव रामजी अम्बेडकर पर संविधान में संविधान के प्रारूप को पारित करवाने की जिम्मेदारी थी।

(5) संविधान की प्रारूप समिति के अध्यक्ष डॉ. भीमराव रामजी अम्बेडकर के साथ-साथ दो अन्य प्रसिद्ध वकील कार्य कर रहे थे, इनमें से एक गुजरात के के.एम. मुंशी तो द्वितीय मद्रास के अल्लादि कृष्णास्वामी अय्यर थे।

(6) संविधान सभा में इन छः सदस्यों के अतिरिक्त दो प्रशासनिक अधिकारी भी महत्त्वपूर्ण योगदान दे रहे थे। इनमें से एक बी. एन. राव संविधान सभा अथवा भारत के संवैधानिक सलाहकार थे। संविधान सभा में एस.एन. मुखर्जी की स्थिति भी अत्यधिक महत्त्वपूर्ण थी। वे संविधान सभा में मुख्य योजनाकार की भूमिका निभा रहे थे।

JAC Class 12 History Important Questions Chapter 15 संविधान का निर्माण : एक नए युग की शुरुआत

प्रश्न 6.
संविधान सभा के गठन का विवेचन कीजिये।
अथवा
संविधान सभा के निर्माण और कार्यप्रणाली की म चर्चा कीजिये।
अथवा
संविधान सभा कैसे घटित हुई थी?
उत्तर:
संविधान सभा का गठन संविधान सभा के सदस्यों का चुनाव सार्वभौमिक 3 मताधिकार के आधार पर नहीं हुआ था। 1945-46 की सर्दियों में भारत के प्रान्तों में चुनाव हुए थे। इसके पश्चात् में प्रान्तीय संसदों ने संविधान सभा के सदस्यों को चुना।

(1) नई संविधान सभा में कांग्रेस का प्रभावशाली या होना नई संविधान सभा में कांग्रेस प्रभावशाली थी। रू प्रान्तीय चुनावों में कांग्रेस ने सामान्य चुनाव क्षेत्रों में भारी क विजय प्राप्त की और मुस्लिम लीग को अधिकांश आरक्षित मुस्लिम सीटें मिल गई। परन्तु मुस्लिम लीग ने संविधान गा सभा का बहिष्कार उचित समझा और एक अन्य संविधान बनाकर उसने पाकिस्तान के निर्माण की माँग जारी रखी। प्रारम्भ में समाजवादी भी संविधान सभा से दूर रहे क्योंकि वे उसे अंग्रेजों के द्वारा बनाई हुई संस्था मानते थे। इन सभी कारणों से संविधान सभा के 82 प्रतिशत सदस्य के कांग्रेस पार्टी के ही सदस्य थे।

(2) कांग्रेस में मतभेद – सभी कांग्रेस सदस्य एकमत ते नहीं थे कई निर्णायक मुद्दों पर उनके भिन्न-भिन्न मत थे। सर कई कांग्रेसी समाजवाद से प्रेरित थे तो कई जमींदारी के समर्थक थे। कई कांग्रेसी समाजवाद से प्रेरित थे तो कई अन्य जमींदारी के समर्थक थे कुछ साम्प्रदायिक दलों के निकट थे, तो कुछ पक्के धर्मनिरपेक्ष थे राष्ट्रीय आन्दोलन के कारण कांग्रेसी बाद-विवाद करना और मतभेदों पर बातचीत कर समझौतों की खोज करना सीख गए थे। संविधान सभा में भी कांग्रेस सदस्यों ने इसी प्रकार का दृष्टिकोण अपनाया।

(3) संविधान सभा में हुई चर्चाओं का जनमत से प्रभावित होना-संविधान सभा में हुई चर्चाएँ जनमत से भी प्रभावित होती थीं। जब संविधान सभा में बहस होती थी, तो विभिन्न पक्षों के तर्क समाचार-पत्रों में भी प्रकाशित होते थे और समस्त प्रस्तावों पर सार्वजनिक रूप से बहस चलती थी। इस प्रकार की आलोचना और जवाबी आलोचना में किसी मुद्दे पर बनने वाली सहमति या असहमति पर गहरा प्रभाव पड़ता था।

(4) सामूहिक सहभागिता-सामूहिक सहभागिता बनाने के लिए जनता के सुझाव भी मांगे जाते थे। कई भाषाई अल्पसंख्यक अपनी मातृभाषा की रक्षा की मांग करते थे। धार्मिक अल्पसंख्यक अपने विशेष हित सुरक्षित करवाना चाहते थे और दलित वर्गों के लोग शोषण के अन्त की माँग करते हुए सरकारी संस्थाओं में आरक्षण चाहते थे।

प्रश्न 7.
संविधान सभा में विभिन्न सदस्यों की महत्त्वपूर्ण भूमिकाओं की विवेचना कीजिये।
उत्तर:
संविधान सभा में विभिन्न सदस्यों की भूमिका संविधान सभा में तीन सौ सदस्य थे। इनमें निम्नलिखित 6 सदस्यों की भूमिका बड़ी महत्त्वपूर्ण रही –
(1) पं. जवाहरलाल नेहरू-पं. जवाहरलाल नेहरू ने सविधान सभा में 13 दिसम्बर, 1946 को एक निर्णायक प्रस्ताव ‘उद्देश्य प्रस्ताव’ को प्रस्तुत किया था। इसमें उन्होंने स्वतन्त्र भारत के संविधान के मूल आदर्शों की रूपरेखा प्रस्तुत की थी। उन्होंने यह प्रस्ताव भी प्रस्तुत किया था कि भारत का राष्ट्रीय ध्वज केसरिया, सफेद और गहरे रंग की तीन बराबर चौड़ाई वाली पट्टियों का तिरंगा झंडा होगा जिसके बीच में गहरे नीले रंग का चक्र होगा।

(2) सरदार वल्लभ भाई पटेल- सरदार वल्लभ भाई पटेल ने मुख्य रूप से परदे के पीछे कई महत्त्वपूर्ण कार्य किये। उन्होंने कई महत्त्वपूर्ण रिपोर्टों के प्रारूप लिखने में विशेष सहायता की और कई परस्पर विरोधी विचारों के बीच सहमति उत्पन्न करने में महत्त्वपूर्ण भूमिका निभाई।

(3) डॉ. राजेन्द्र प्रसाद-डॉ. राजेन्द्र प्रसाद संविधान सभा के अध्यक्ष थे। उनकी ये जिम्मेदारियाँ थीं कि संविधान सभा में चर्चा रचनात्मक दिशा ले और सभी सदस्यों को अपनी बात कहने का अवसर मिले।

(4) डॉ. भीमराव अम्बेडकर डॉ. भीमराव अम्बेडकर एक प्रख्यात विधिवेत्ता तथा अर्थशास्त्री थे। वह संविधान सभा के सबसे महत्त्वपूर्ण सदस्यों में से एक थे। स्वतन्त्रता प्राप्त करने के बाद महात्मा गांधी की सलाह पर डॉ. अम्बेडकर को केन्द्रीय विधिमंत्री के पद पर नियुक्त किया गया था। इस भूमिका में उन्होंने संविधान की प्रारूप समिति के अध्यक्ष के रूप में कार्य किया। उनके पास सभा में संविधान के प्रारूप को पारित करवाने की जिम्मेदारी थी।

JAC Class 12 History Important Questions Chapter 15 संविधान का निर्माण : एक नए युग की शुरुआत

(5) के. एम. मुंशी डॉ. अम्बेडकर के साथ दो अन्य वकील भी कार्य कर रहे थे। एक गुजरात के के. एम. मुंशी थे तथा दूसरे मद्रास के अल्लादि कृष्णास्वामी अय्यर। इन- दोनों ने संविधान के प्रारूप पर महत्त्वपूर्ण सुझाव दिए।

(6) दो प्रशासनिक अधिकारी-उपरोक्त 6 सदस्यों को दो प्रशासनिक अधिकारियों ने महत्त्वपूर्ण सहायता दी। इनमें से एक बी. एन. राव थे। वह भारत सरकार के संवैधानिक सलाहकार थे और उन्होंने अन्य देशों की राजनीतिक व्यवस्थाओं का गहन अध्ययन करके कई चर्चा- पत्र तैयार किए थे। दूसरे अधिकारी एस.एन. मुखर्जी थे। इनकी भूमिका मुख्य योजनाकार की थी। मुखर्जी जटिल प्रस्तावों को स्पष्ट वैधिक भाषा में व्यक्त करने की क्षमता रखते थे।

JAC Class 12 History Important Questions Chapter 14 विभाजन को समझना : राजनीति, स्मृति, अनुभव

Jharkhand Board JAC Class 12 History Important Questions Chapter 14 विभाजन को समझना : राजनीति, स्मृति, अनुभव Important Questions and Answers.

JAC Board Class 12 History Important Questions Chapter 14 विभाजन को समझना : राजनीति, स्मृति, अनुभव

बहुविकल्पीय प्रश्न (Multiple Choice Questions)

1. मुस्लिम लीग ने प्रत्यक्ष कार्यवाही दिवस मनाने की घोषणा की थी –
(क) 16 अगस्त, 1948
(ख) 16 अगस्त, 1946
(ग) 12 अगस्त, 1942
(घ) 15 अगस्त, 1944
उत्तर:
(ख) 16 अगस्त, 1946

2. ‘पाकिस्तान’ नाम के प्रस्ताव को सर्वप्रथम जिसने प्रस्तुत किया था, वह था-
(क) चौधरी रहमत अली
(ख) चौधरी मोहम्मद अली
(ग) चौधरी इनायत अली
(घ) चौधरी लियाकत अली
उत्तर:
(क) चौधरी रहमत अली

3. शुद्धि आन्दोलन चलाने वाली संस्था थी –
(क) ब्रह्म समाज
(ग) हिन्दू महासभा
(ख) आर्य समाज
(घ) कॉंग्रेस पार्टी
उत्तर:
(ख) आर्य समाज

4. भारत विभाजन से लगभग कितने लोगों को उजड़ कर दूसरी जगह जाने को मजबूर होना पड़ा –
(क) लगभग दो करोड़ से ज्यादा
(ख) लगभग डेढ़ करोड़
(ग) चार करोड़ से ज्यादा
(घ) पचास लाख से ज्यादा
उत्तर:
(ख) लगभग डेढ़ करोड़

5. भारत विभाजन के समय जो नस्ली सफाया हुआ, वह कारगुजारी थी –
(क) धार्मिक समुदायों के स्वयंभू प्रतिनिधियों की
(ख) सरकारी निकार्यों की
(ग) अंग्रेजों की
(घ) सेना की
उत्तर:
(क) धार्मिक समुदायों के स्वयंभू प्रतिनिधियों की

JAC Class 12 History Important Questions Chapter 14 विभाजन को समझना : राजनीति, स्मृति, अनुभव

6. “मैं तो सिर्फ अपने अब्बा पर चढ़ा हुआ कर्ज चुका रहा हूँ।” यह किसने कहा था –
(क) अब्दुल रज्जाक ने
(ख) अब्दुल लतीफ ने
(ग) मोहम्मद अली ने
(घ) शौकत अली ने
उत्तर:
(ख) अब्दुल लतीफ ने

7. अब्दुल लतीफ के अब्बा की मदद की थी –
(क) एक हिन्दू ने
(ख) एक सिख ने
(ग) एक हिन्दू माई ने
(घ) इनमें से कोई नहीं
उत्तर:
(ग) एक हिन्दू माई ने

8. औपनिवेशिक भारत में हिन्दू और मुसलमान दो पृथक् राष्ट्र थे। यह सोच थी-
(क) रहमत अली की
(ख) मोहम्मद अली जिन्ना की
(ग) लियाकत अली की
(घ) मौलाना आजाद की
उत्तर:
(ख) मोहम्मद अली जिन्ना की

9. काँग्रेस और मुस्लिम लीग का लखनऊ समझौता कब हुआ था?
(क) 1915 में
(ख) 1916 में
(ग) 1919 में
(घ) 1917 में
उत्तर:
(ख) 1916 में

10. होलोकॉस्ट क्या है?
(क) बँटवारा
(ख) संघर्ष
(ग) मित्रता
(घ) संगठन
उत्तर:
(क) बँटवारा

JAC Class 12 History Important Questions Chapter 14 विभाजन को समझना : राजनीति, स्मृति, अनुभव

11. मुस्लिम लीग की स्थापना कब हुई थी?
(क) 1904 में
(ख) 1906 में
(ग) 1912 में.
(घ) 1984 में
उत्तर:
(ख) 1906 में

12. 1937 में के प्रान्तीय चुनावों में कांग्रेस को निम्न में से कितने प्रान्तों में पूर्ण बहुमत प्राप्त हुआ था ?
(क) 4
(ग) 6
(ख) 5
(घ) 7
उत्तर:
(ख) 5

13. पाकिस्तान नाम सर्वप्रथम दिया था
(क) मोहम्मद अली जिन्ना ने
(ख) मोहम्मद इकबाल ने
(ग) खान अब्दुल गफ्फार खान ने
(घ) चौधरी रहमत अली ने
उत्तर:
(घ) चौधरी रहमत अली ने

14. पंजाब में हिन्दू मुस्लिम एवं सिक्ख भू-स्वामियों के हितों का प्रतिनिधित्व करने वाली राजनैतिक पार्टी थी –
(क) हिन्दू महासभा
(ग) मुस्लिम लीग
(ख) कांग्रेस
(घ) यूनियनिस्ट पार्टी
उत्तर:
(घ) यूनियनिस्ट पार्टी

15. सीमान्त गांधी कहा जाता था –
(क) जिन्ना को
(ख) महात्मा गाँधी को
(ग) खान अब्दुल गफ्फार खान को
(घ) सुशीला नायर को
उत्तर:
(ग) खान अब्दुल गफ्फार खान को

16. गाँधीजी के दिल्ली आगमन को “बड़ी लम्बी और कठोर गर्मी के बाद बरसात की फुहारों के आने” जैसा महसूस किया था –
(क) जवाहर लाल नेहरू
(ख) शाहिद अहमद देहलवी ने
(ग) जिन्ना से
(घ) खान अब्दुल गफ्फार खान ने
उत्तर:
(ख) शाहिद अहमद देहलवी ने

17. द्वि-राष्ट्र सिद्धान्त किसने दिया था?
(क) रहमत अली
(ग) महात्मा गाँधी
(ख) मोहम्मद अली जिन्ना
(घ) उपर्युक्त सभी
उत्तर:
(ख) मोहम्मद अली जिन्ना

JAC Class 12 History Important Questions Chapter 14 विभाजन को समझना : राजनीति, स्मृति, अनुभव

18. “लव इज स्ट्रांगर देन हेट, ए रिमेम्बरेंस ऑफ 1947” नामक संस्मरण के लेखक हैं –
(क) डॉ. सुखदेव सिंह
(ख) डॉ. रवीन्द्र
(घ) महात्मा गाँधी
(ग) जिन्ना
उत्तर:
(क) डॉ. सुखदेव सिंह

रिक्त स्थानों की पूर्ति कीजिए:

1. विभाजन की वजह से लाखों लोग …………… खनकर रह गए।
2. पत्रकार आर. एम. मर्फी के अनुसार हिन्दू काले, कायर ……………. तथा शाकाहारी होते हैं।
3. …………….. में नात्सी जर्मनी में लोगों को मारने के लिए सरकारी मुहिम चली थी।
4. ……………… समझौता दिसम्बर 1916 में हुआ था।
5. ……………….. लखनक समझौता ……………… और ……………… के बीच हुआ था।
6. कुछ विद्वानों के अनुसार देश का बँटवारा एक ऐसी साम्प्रदायिक राजनीति का आखिरी बिन्दु था जो ……………… वीं शताब्दी के प्रारस्भिक दशकों में शुरू हुई।
7. ………………. का तात्पर्य है वह राजनीति जो धार्मिकसमुदायों के बीच विरोध और झगड़े पैदा करती है।
8. बहु-धार्मिक देश में ‘धार्मिक राष्ट्रवाद’ शब्दों का अर्थ भी ………………. के करीब-करीब हो सकता है।
9. मुस्लिम लीग की स्थापना 1906 में ………………. में हुआ था।
उत्तर:
1. शरणार्थी
2. बहु-ईश्वरवादी
3. 1947- 48
4. लखनऊ
5. कांग्रेस, मुस्लिम लीग
6. 20
7. साम्प्रदायिकता
8 साम्प्रदायिकता
9. ढाका
10. 1915
11. 1937
12. यूनियनिस्ट
13. 1942
14. प्रत्यक्ष कार्यवाही।

अतिलघूत्तरात्मक प्रश्न

प्रश्न 1.
सीमान्त गाँधी किसे कहा जाता है ?
उत्तर:
खान अब्दुल गफ्फार खान।

प्रश्न 2.
सर्वप्रथम उत्तर-पश्चिम क्षेत्र में मुस्लिम राज्य की माँग किसने की थी?
उत्तर:
मुहम्मद इकबाल ने।

प्रश्न 3.
1971 में बंगाली मुसलमानों द्वारा पाकिस्तान से अलग होने का फैसला लेकर जिला के किस सिद्धान्त को नकार दिया था?
उत्तर:
द्विराष्ट्र सिद्धान्त को।

प्रश्न 4.
नोआखली वर्तमान में किस देश में स्थित है?
उत्तर:
बांग्लादेश में।

JAC Class 12 History Important Questions Chapter 14 विभाजन को समझना : राजनीति, स्मृति, अनुभव

प्रश्न 5.
उन दो नेताओं के नाम लिखिये जो अन्त तक भारत के विभाजन का विरोध करते रहे।
उत्तर:
(1) महात्मा गाँधी
(2) खान अब्दुल गफ्फार खान।

प्रश्न 6.
स्वतन्त्रतापूर्व का ‘संयुक्त प्रान्त’ वर्तमान में कौन-से राज्य के नाम से जाना जाता है?
उत्तर:
उत्तर प्रदेश।

प्रश्न 7.
जर्मन होलोकास्ट और भारत के विभाजन में क्या अन्तर था?
उत्तर:
जर्मनी में भीषण विनाशलीला हेतु नाजी सरकार उत्तरदायी थी, भारत के विभाजन के लिए धार्मिक नेता उत्तरदायी थे।

प्रश्न 8.
आर्य समाज का मुख्य नारा क्या था?
उत्तर:
शुद्धि आन्दोलन।

प्रश्न 9.
“मैं तो सिर्फ अपने अब्बा पर चढ़ा हुआ कर्ज चुका रहा हूँ।” यह किसने कहा था और किससे कहा था?
उत्तर:
(1) अब्दुल लतीफ ने
(2) शोधकर्ता से।

प्रश्न 10.
1947 के बँटवारे को लोग किन नामों से पुकारते हैं?
उत्तर:
‘मार्शल लॉ’, ‘मारामारी’, ‘रौला’ एवं ‘हुल्लड़’।

प्रश्न 11.
जर्मन होलोकास्ट’ से क्या अभिप्राय है?
उत्तर:
नाजी शासन के दौरान जर्मनी में गैर-जर्मन लोगों का संहार।

प्रश्न 12.
किस अधिनियम में सर्वप्रथम मुसलमानों के लिए पृथक् निर्वाचन क्षेत्रों की व्यवस्था की गई ?
उत्तर:
1909 के मिण्टये मार्ले सुधारों में।

प्रश्न 13.
कांग्रेस और मुस्लिम लीग में समझौता कब हुआ?
उत्तर:
दिसम्बर, 1916 में।

JAC Class 12 History Important Questions Chapter 14 विभाजन को समझना : राजनीति, स्मृति, अनुभव

प्रश्न 14.
लखनऊ समझौता किस-किसके बीच हुआ?
उत्तर:
कांग्रेस और मुस्लिम लीग के बीच

प्रश्न 15.
हिन्दुओं और मुसलमानों के बीच किन बातों से साम्प्रदायिक तनाव उत्पन्न हुआ?
उत्तर:

  • मस्जिद के सामने संगीत
  • गोरक्षा आन्दोलन
  • शुद्धि आन्दोलन।

प्रश्न 16.
हिन्दुओं और मुसलमानों के बीच मुसलमानों की किन गतिविधियों से दोनों सम्प्रदायों में तनाव उत्पन्न हुआ?
उत्तर:
(1) तबलीग (प्रचार) और
(2) तंजीम (संगठन) के विस्तार से

प्रश्न 17.
मुस्लिम लीग की स्थापना कब हुई ?
उत्तर:
1906 ई. में।

प्रश्न 18.
मुस्लिम लीग की स्थापना कहाँ हुई?
उत्तर:
ढाका में।

प्रश्न 19.
हिन्दू महासभा की स्थापना कब हुई ?
उत्तर:
1915

प्रश्न 20.
मुस्लिम लीग ने ‘पाकिस्तान’ का प्रस्ताव कब पास किया था?
उत्तर:
23 मार्च, 1940 को

प्रश्न 21.
‘सारे जहाँ से अच्छा हिन्दोस्ताँ हमारा’ इस गीत की रचना किसने की थी?
उत्तर:
मुहम्मद इकबाल ने।

प्रश्न 22.
सर्वप्रथम ‘पाकिस्तान’ का उल्लेख किसने किया था?
उत्तर:
केम्ब्रिज के एक मुस्लिम छात्र चौधरी रहमत अली ने

प्रश्न 23.
1930 के मुस्लिम लीग के अधिवेशन में किसने ‘उत्तर-पश्चिमी भारतीय मुस्लिम राज्य’ की स्थापना पर जोर दिया था?
उत्तर:
मुहम्मद इकबाल ने।

JAC Class 12 History Important Questions Chapter 14 विभाजन को समझना : राजनीति, स्मृति, अनुभव

प्रश्न 24.
ब्रिटिश सरकार ने केबिनेट मिशन दिल्ली कब भेजा ? इसमें कितने सदस्य थे?
उत्तर:
(1) मार्च 1946 में
(2) तीन सदस्य।

प्रश्न 25.
सीमान्त गाँधी’ या ‘फ्रंटियर गाँधी’ किन्हें कहा जाता था?
उत्तर:
खान अब्दुल गफ्फार खान

‘प्रश्न 26.
मुस्लिम लीग द्वारा ‘प्रत्यक्ष कार्यवाही दिवस’ कब मनाने की घोषणा की गई थी?
उत्तर:
16 अगस्त, 1946 को

प्रश्न 27.
गाँधीजी द्वारा दिल्ली में दंगाग्रस्त क्षेत्रों का दौरा करने पर किसने कहा था कि ” अब दिल्ली बच जायेगी।”
उत्तर:
शाहिद अहमद देहलवी ने

प्रश्न 28.
‘पंजाबी सेंचुरी’ के रचयिता कौन थे ?
उत्तर:
प्रकाश टण्डन।

प्रश्न 29.
‘द अदर साइड ऑफ वाइलेंस’ की रचना किसने की थी?
उत्तर:
उर्वशी बुटालिया ने।

प्रश्न 30.
‘मुहब्बत नफरत से ज्यादा ताकतवर होती है 1947 की यादें’ के रचयिता कौन थे?
उत्तर:
डॉ. खुशदेवसिंह।

प्रश्न 31.
भारत के बँटवारे के दौरान के महाध्वंस और यूरोप के नात्सी महाध्वंस में प्रमुख अन्तर क्या है?
उत्तर:
भारत के बँटवारे का महाध्वंस धार्मिक समुदायों के स्वयंभू प्रतिनिधियों की कारगुजारी था जबकि यूरोपीय महाध्वंस सरकार की कारगुजारी था।

प्रश्न 32.
विभाजन की स्मृतियों, घृणाओं और छवियों की आज क्या भूमिका है?
उत्तर:
विभाजन की स्मृतियाँ छवियाँ आज भी सरहद के दोनों तरफ के लोगों के इतिहास व सम्बन्धों को तय करती हैं।

प्रश्न 33.
किसी धार्मिक जुलूस के द्वारा नमाज के समय मस्जिद के बाहर संगीत बजाए जाने से हिन्दू- मुस्लिम हिंसा क्यों हो सकती थी?
उत्तर:
नमाज के समय मस्जिद के सामने संगीत बजाने को रूढ़िवादी मुसलमान अपनी नमाज या इबादत में खलल मानते हैं।

प्रश्न 34.
साम्प्रदायिकता से क्या अभिप्राय है?
उत्तर:
सांप्रदायिकता धार्मिक अस्मिता का विशेष तरह से राजनीतिकरण है जो धार्मिक समुदायों में झगड़े पैदा करवाने की कोशिश करती है।

प्रश्न 35.
1937 के चुनावों के बाद जिन्ना की प्रमुख जिद क्या थी?
उत्तर:
जिला की जिद थी कि मुस्लिम लीग को ही मुसलमानों का एकमात्र प्रवक्ता माना जाये।

JAC Class 12 History Important Questions Chapter 14 विभाजन को समझना : राजनीति, स्मृति, अनुभव

प्रश्न 36.
1937 के चुनावों के बाद संयुक्त प्रान्त में काँग्रेस पार्टी ने गठबंधन सरकार बनाने के बारे में मुस्लिम लीग के प्रस्ताव को खारिज क्यों कर दिया था?
उत्तर:
संयुक्त प्रान्त में कॉंग्रेस को पूर्ण बहुमत प्राप्त था तथा जमींदारी प्रथा के सम्बन्ध में दोनों के विचारों में अन्तर था।

प्रश्न 37.
मुस्लिम लीग ने उपमहाद्वीप में मुस्लिम- बहुल इलाकों के लिए कुछ स्वायत्तता की मांग का प्रस्ताव कब पेश किया?
उत्तर:
23 मार्च, 1940 को

प्रश्न 38.
1945 में अँग्रेजों के एक केन्द्रीय कार्यकारिणी सभा बनाने के प्रस्ताव पर वार्ता जिन्ना की किस जिद के कारण टूट गई?
उत्तर:
जिन्ना इस बात पर अड़े रहे कि कार्यकारिणी सभा के मुस्लिम सदस्यों का चुनाव करने का अधिकार मुस्लिम लीग का होगा।

प्रश्न 39.
किस मिशन के प्रस्ताव को लीग और काँग्रेस द्वारा स्वीकार नहीं करने पर विभाजन कमोबेश अनिवार्य हो गया था?
उत्तर:
कैबिनेट मिशन प्रस्ताव को स्वीकार नहीं करने पर विभाजन कमोबेश अनिवार्य हो गया था।

प्रश्न 40.
कैबिनेट मिशन योजना की असफलता के बाद मुस्लिम लीग ने क्या कार्यवाही की ?
उत्तर:
लीग ने पाकिस्तान की अपनी माँग को मनवाने हेतु 16 अगस्त, 1946 को ‘प्रत्यक्ष कार्यवाही दिवस’ मनाने का फैसला किया।

प्रश्न 41.
सन् 1947 में भारत विभाजन के दो कारण लिखिये।
उत्तर:
(1) अंग्रेजों की ‘फूट डालो और राज करो’
(2) कैबिनेट मिशन की विफलता।

प्रश्न 42.
द्विराष्ट्र सिद्धान्त का क्या अर्थ है?
उत्तर:
द्विराष्ट्र सिद्धान्त का अर्थ है कि हिन्दू और मुसलमानों के दो अलग-अलग राष्ट्र (देश) हैं वे एक साथ नहीं रह सकते।

प्रश्न 43.
मुस्लिम लीग ने क्रिप्स प्रस्ताव को क्यों अस्वीकार किया?
उत्तर:
मुस्लिम लीग के अनुसार इस प्रस्ताव में उसकी प्रमुख माँग पाकिस्तान के निर्माण का कहीं भी जिक्र नहीं था।

प्रश्न 44.
अब्दुल लतीफ खाँ शोधार्थी की मदद क्यों करते थे?
उत्तर:
एक हिन्दू बूढ़ी माई ने अब्दुल लतीफ खाँ के वालिद की दंगाइयों से जान बचाई थी।

प्रश्न 45.
हिन्दू महासभा के बारे में आप क्या जानते हैं? लिखिए।
उत्तर:
हिन्दू महासभा की स्थापना 1915 में हिन्दू समाज में एक ता पैदा करने के उद्देश्य से की गई।

प्रश्न 46.
यूनियनिस्ट पार्टी क्या थी?
उत्तर:
यह पंजाब में हिन्दू-मुस्लिम और सिख भू- स्वामियों के हितों का प्रतिनिधित्व करने वाली एक राजनीतिक पार्टी थी।

JAC Class 12 History Important Questions Chapter 14 विभाजन को समझना : राजनीति, स्मृति, अनुभव

प्रश्न 47.
मुस्लिम लीग के 1930 में अधिवेशन के अध्यक्षीय भाषण में मोहम्मद इकबाल ने क्या माँग की थी?
उत्तर:
पश्चिमोत्तर भारत में मुस्लिम बहुल इलाकों को एकीकृत, शिथिल भारतीय संघ के अन्दर एक स्वायत्त इकाई की स्थापना करना।

प्रश्न 48.
1942 के भारत छोड़ो आन्दोलन का अंग्रेजों पर क्या प्रभाव पड़ा ?
उत्तर:
अंग्रेजों को संभावित सत्ता हस्तान्तरण के बारे में भारतीय पक्षों के साथ बातचीत के लिए तैयार होना पड़ा।

प्रश्न 49.
अंग्रेजी शिक्षा भारतीयों के लिए किस प्रकार लाभदायक सिद्ध हुई?
उत्तर:
(1) भारतीय अंग्रेजी साहित्य, विज्ञान, गणित तथा तकनीकी विषयों के ज्ञान से परिचित हुए। (2) इसने राष्ट्रीय चेतना का प्रसार किया।

प्रश्न 50.
मार्च, 1947 में काँग्रेस हाईकमान ने किस प्रस्ताव पर मंजूरी दे दी थी?
उत्तर:
पंजाब को मुस्लिम बहुल और हिन्दू/ सिख बहुल दो हिस्सों में बाँटने के प्रस्ताव पर अपनी मंजूरी देना।

प्रश्न 51.
भारत विभाजन के समय साम्प्रदायिक दंगों के लिए किन-किन शब्दों का प्रयोग होता है?
उत्तर:
मॉर्शल लॉ, मारामारी, रौला या हुल्लड़ आदि।

प्रश्न 52.
दिल्ली में गाँधीजी के अनशन में आश्चर्यजनक बात क्या थी?
उत्तर:
दिल्ली अनशन में आश्चर्यजनक बात यह थी कि पाकिस्तान से आए शरणार्थी चाहे वे हिन्दू हों या सिख, अनशन में साथ बैठते थे।

प्रश्न 53.
समकालीन प्रेक्षकों और विद्वानों ने 1947 के दंगों में हुई विनाशलीला को देखते हुए इसे महाध्वंस (होलोकास्ट) क्यों कहा है?
उत्तर:
वे इस सामूहिक जनसंहार की भयानकता को उजागर करना चाहते हैं।

प्रश्न 54.
थुआ गाँव का हादसा क्या था?
उत्तर:
थुआ गाँव की 90 स्वियों ने शत्रुओं के हाथों में पड़ने की बजाय कुएं में कूदकर अपनी जान दे दी थी।

प्रश्न 55.
डॉ. खुशदेवसिंह क्यों प्रसिद्ध थे?
उत्तर:
डॉ. खुशदेवसिंह ने धर्मपुर (हिमाचल प्रदेश) में रहते हुए हिन्दुओं, सिक्खों तथा मुसलमानों को भोजन और आश्रय प्रदान किया।

प्रश्न 56.
आर्य समाज का क्या उद्देश्य था ?
उत्तर:
आर्य समाज वैदिक ज्ञान का पुनरुत्थान कर उसको विज्ञान की आधुनिक शिक्षा से जोड़ना चाहता था।

प्रश्न 57.
अविभाजित भारत में दूसरी बार प्रान्तीय चुनाव कब हुए?
उत्तर:
1946 ई. में।

प्रश्न 58.
गुरुद्वारा शीशगंज में गाँधीजी ने किस बात को शर्मनाक बताया?
उत्तर:
गाँधीजी ने इस बात को शर्मनाक बताया कि दिल्ली का दिल कहलाने वाले चाँदनी चौक में उन्हें एक भी मुसलमान दिखाई नहीं दिया।

JAC Class 12 History Important Questions Chapter 14 विभाजन को समझना : राजनीति, स्मृति, अनुभव

प्रश्न 59.
किस मिशन के प्रस्ताव को कांग्रेस व मुस्लिम लीग द्वारा स्वीकार न किये जाने के कारण विभाजन अनिवार्य- सा हो गया था ?
उत्तर:
कैबिनेट मिशन प्रस्ताव।

प्रश्न 60.
उत्तर-पश्चिमी भारतीय मुस्लिम राज्य की स्थापना की माँग किसने की थी और कब की थी?
उत्तर:
1930 में मोहम्मद इकबाल ने।

प्रश्न 61.
केबिनेट मिशन की दो सिफारिशों का उल्लेख कीजिये।
उत्तर:
(1) एक शिथिल त्रिस्तरीय महासंघ का निर्माण करना।
(2) संविधान सभा का चुनाव करना।

प्रश्न 62.
साम्प्रदायिक दंगों से पीड़ित लोगों को सान्त्वना देने के लिए गाँधीजी ने किन स्थानों की यात्रा की?
उत्तर:
गांधीजी ने नोआखली (वर्तमान बांग्लादेश), बिहार, कोलकाता तथा दिल्ली की यात्राएं कीं।

प्रश्न 63.
“हमारे लिए इससे ज्यादा शर्म की बात और क्या हो सकती है कि चाँदनी चौक में एक भी मुसलमान नहीं है।” यह किसका कथन था?
उत्तर:
यह कथन गाँधीजी का था।

प्रश्न 64.
दिल्ली में गाँधीजी के अनशन का असर “आसमान की बिजली जैसा रहा”, यह कथन किसका था?
उत्तर:
यह कथन मौलाना आजाद का था।

प्रश्न 65.
‘प्रेम घृणा से अधिक शक्तिशाली होता है : 1947 की यादें’ नामक पुस्तक में किसके संस्मरण संकलित है?
उत्तर:
डॉ. खुशदेवसिंह के।

प्रश्न 66.
कांग्रेस ने कैबिनेट मिशन की सिफारिशों को क्यों स्वीकार नहीं किया?
उत्तर:
कांग्रेस की माँग थी कि प्रान्तों को अपनी इच्छा का समूह चुनने का अधिकार मिलना चाहिए।

JAC Class 12 History Important Questions Chapter 14 विभाजन को समझना : राजनीति, स्मृति, अनुभव

प्रश्न 67.
मुस्लिम लीग ने कैबिनेट मिशन की सिफारिशों को क्यों नहीं माना?
उत्तर:
मुस्लिम लीग की माँग थी कि प्रान्तों की समूहबद्धता अनिवार्य होनी चाहिए।

लघुत्तरात्मक प्रश्न

प्रश्न 1.
1947 में विभाजन के दौरान हुई भीषण विनाशलीला को ‘महाध्वंस’ (होलोकॉस्ट) क्यों कहा कि जाता है?
उत्तर:
कुछ प्रेक्षकों ने विभाजन के दौरान हुई हत्याओं, बलात्कार, आगजनी तथा लूटपाट को ‘महाध्वंस’ लम (होलोकास्ट) की संज्ञा दी है। वे इस शब्द के द्वारा – सामूहिक जनसंहार की भयानकता को उजागर करना चाहते हैं। यह हादसा इतना भीषण था कि ‘विभाजन’ या ‘बँटवारे’ से उसके समस्त पहलू सामने नहीं आते। जहाँ नाजी जर्मनी में महाध्वंस सरकार के द्वारा किया गया, वहाँ भारत में यह महाध्वंस धार्मिक समुदायों के स्वयंभू प्रतिनिधियों की कारगुजारी थी।

प्रश्न 2.
हिन्दू महासभा के बारे में आप क्या जानते हैं?
उत्तर:
हिन्दू महासभा की स्थापना 1915 में हुई। यह एक हिन्दू पार्टी थी जो कमोबेश उत्तर भारत तक सीमित रही यह पार्टी हिन्दुओं के बीच जाति एवं सम्प्रदाय के फर्कों को खत्म कर पैदा करने की कोशिश करती थी अस्मिता को मुस्लिम अस्मिता के करने का प्रयास करती थी। हिन्दू समाज में एकता हिन्दू महासभा, हिन्दू विरोध में परिभाषित

प्रश्न 3.
कैबिनेट मिशन के प्रमुख सुझाव क्या थे?
उत्तर:
कैबिनेट मिशन के प्रमुख सुझाव निम्नलिखित –
(1) इस कैबिनेट मिशन ने तीन महीने तक भारत का दौरा किया और एक ढीले-ढाले त्रिस्तरीय महासंघ का सुझाव दिया। इसमें भारत एकीकृत ही रहने वाला था जिसकी केन्द्रीय सरकार काफी कमजोर होती और उसके पास केवल विदेश, रक्षा और संचार का जिम्मा होता।

(2) संविधान सभा का चुनाव करते हुए मौजूदा प्रान्तीय सभाओं को तीन हिस्सों में समूहबद्ध किया जाना था हिन्दू बहुल प्रान्तों को समूह ‘क’ पश्चिमोत्तर मुस्लिम बहुल प्रान्तों को समूह ‘ख’ और पूर्वोत्तर (असम सहित ) के मुस्लिम बहुल प्रान्तों को समूह ‘ग’ में रखा गया था।

(3) प्रान्तों के इन खण्डों या समूहों को मिला कर क्षेत्रीय इकाइयों का गठन किया जाना था माध्यमिक स्तर की कार्यकारी और विधायी शक्तियाँ उनके पास ही रहने वाली थी।

प्रश्न 4.
साम्प्रदायिकता से क्या अभिप्राय है?
उत्तर:
साम्प्रदायिकता उस राजनीति को कहा जाता है, जो धार्मिक समुदायों के बीच विरोध और झगड़े पैदा करती है। ऐसी राजनीति धार्मिक पहचान को बुनियादी और अटल मानती है। साम्प्रदायिकता किसी चिह्नित ‘गैर’ के विरुद्ध घृणा की राजनीति को पोषित करती है। मुस्लिम साम्प्रदायिकता हिन्दुओं को ‘गैर’ बताकर उनका विरोध करती है तथा हिन्दू साम्प्रदायिकता मुसलमानों को गैर बताकर उनका विरोध करती है। साम्प्रदायिकता धार्मिक अस्मिता का विशेष प्रकार से राजनीतिकरण है।

प्रश्न 5.
बँटवारे में औरतों की बरामदगी पर एक संक्षिप्त टिप्पणी लिखिए।
उत्तर:
घंटवारे के दौरान स्त्रियों के साथ बलात्कार हुए, उनका अपहरण किया गया, उन्हें बार-बार खरीदा- बेचा गया तथा अनजान परिस्थितियों में अजनबियों के साथ एक नया जीवन बसर करने के लिए विवश किया गया। बहुत सी स्त्रियों को जबरदस्ती घर बिठा ली गई तथा उन्हें उनके नये परिवारों से छीनकर पुनः पुराने परिवारों या स्थानों पर भेज दिया गया। इस अभियान में लगभग 30,000 स्त्रियों को बरामद किया गया, इनमें से 22,000 मुस्लिम स्त्रियों को भारत से तथा 8,000 हिन्दू स्त्रियों को पाकिस्तान से निकाला गया।

JAC Class 12 History Important Questions Chapter 14 विभाजन को समझना : राजनीति, स्मृति, अनुभव

प्रश्न 6.
डॉ. खुशदेवसिंह के बारे में आप क्या जानते हैं?
उत्तर:
डॉ. खुशदेवसिंह एक सिक्ख डॉक्टर थे तथा तपेदिक के विशेषज्ञ थे वे विभाजन के दौरान धर्मपुर (हिमाचल प्रदेश) में नियुक्त थे। उन्होंने दिन-रात लगकर असंख्य प्रवासी मुसलमानों सिक्खों हिन्दुओं को बिना किसी भेदभाव के भोजन, आश्रय और सुरक्षा प्रदान की। उन पर लोगों का ऐसा ही विश्वास था जैसा दिल्ली और कई जगह के मुसलमानों को गाँधीजी पर था।

प्रश्न 7.
” बँटवारे के समय हुई हिंसा से पीड़ित लोग तिनकों में अपनी जिन्दगी दोबारा खड़ी करने के लिए मजबूर हो गये।” इस कथन के सन्दर्भ में एक मार्मिक चित्रण प्रस्तुत कीजिए।
उत्तर:
उपर्युक्त कथन के सन्दर्भ की मार्मिक चित्रण को हम निम्न उदाहरणों द्वारा समझ सकते हैं-

  • भारत विभाजन में कई लाख लोग मारे गये और न जाने कितनी महिलाओं का बलात्कार एवं अपहरण हुआ।
  • करोड़ों लोग उजड़ गये। कुछ रातों-रात अजनबी जमीन पर शरणार्थी बनकर रह गए।
  • लगभग डेढ़ करोड़ लोगों को भारत और पाकिस्तान के मध्य रातों-रात खड़ी कर दी गई सीमा के इस या उस पार जाना पड़ा। जैसे ही उन्होंने इस ‘छाया सीमा’ से ठोकर खाई, वे बेघर बार हो गये।
  • पलक झपकते ही उनकी धन सम्पत्ति हाथ से जाती रही।
  • उनके मित्र तथा रिश्तेदार बिछड़ गए। वे अपनी मकानों, खेतों तथा कारोबार से वंचित हो गए।

प्रश्न 8.
1920 व 1930 ई. के दशकों में कौन- कौनसे मुद्दे हिन्दू व मुसलमानों के मध्य तनाव का कारण बने?
अथवा
20 वीं शताब्दी के प्रारम्भिक दशकों में साम्प्रदायिक अस्मिता के पक्की होने के अन्य कारण क्या थे?
उत्तर:
20वीं शताब्दी के प्रारम्भिक दशकों में साम्प्रदायिक अस्मिताएँ कई अन्य कारणों से भी ज्यादा पक्की हुई. 1920 और 1930 के दशकों में कई घटनाओं की वजह से तनाव उभरे मुसलमानों को ‘मस्जिद के सामने संगीत’, गो-रक्षा आन्दोलन, और आर्य समाज की शुद्धि की कोशिशें (यानी कि नव-मुसलमानों को फिर से हिन्दू बनाना) जैसे मुद्दों पर गुस्सा आया। दूसरी ओर हिन्दू 1923 के बाद तबलीग (प्रचार) और तंजीम के विस्तार से उत्तेजित हुए। जैसे-जैसे मध्यमवर्गीय प्रचारक और साम्प्रदायिक कार्यकर्ता अपने समुदायों में लोगों को दूसरे समुदायों के खिलाफ लामबंद करते हुए, ज्यादा एकजुटता बनाने लगे, देश के विभिन्न भागों में दंगे फैलते हुए।

JAC Class 12 History Important Questions Chapter 14 विभाजन को समझना : राजनीति, स्मृति, अनुभव

प्रश्न 9.
संयुक्त प्रान्त (वर्तमान उत्तर प्रदेश) में काँग्रेस ने मुस्लिम लीग के गठबंधन प्रस्ताव को क्यों खारिज कर दिया?
उत्तर:
प्रथमतः, संयुक्त प्रांत में काँग्रेस को पूर्ण बहुमत प्राप्त हुआ था। लीग वहाँ गठबंधन सरकार बनाना चाहती थी जिसे काँग्रेस ने स्वीकार नहीं किया, क्योंकि काँग्रेस को बहुमत के लिए दूसरे दल के साथ गठबंधन की आवश्यकता नहीं थी। दूसरे, इसके पीछे मुख्य कारण यह था कि काँग्रेस पार्टी जमींदारी प्रथा को खत्म करना चाहती थी और मुस्लिम लीग जमींदारी प्रथा का समर्थन करती हुई प्रतीत होती थी। तीसरे, मुस्लिम लीग मुसलमानों की एकमात्र प्रवक्ता होने पर बल दे रही थी।

प्रश्न 10.
पाकिस्तान का नाम का प्रस्ताव सर्वप्रथम किसने रखा था? लिखिए।
उत्तर”
पाकिस्तान अथवा पाकस्तान (पंजाब, अफगानिस्तान, कश्मीर, सिन्ध और बिलोचिस्तान) नाम सबसे पहले कैम्ब्रिज में पढ़ने वाले पंजाबी मुसलमान छात्र चौधरी रहमत अली ने 1933 और 1935 में लिखित अपने दो पचों में गढ़ा। रहमत अली इस नई इकाई के लिए अलग राष्ट्रीय हैसियत चाहता था। 1930 के दशक में किसी ने भी उसकी बात को गंभीरता से नहीं लिया। यहाँ तक कि मुस्लिम लीग तथा अन्य मुस्लिम नेताओं ने भी उसके इस विचार को खारिज कर दिया था।

प्रश्न 11.
पाकिस्तान का प्रस्ताव कब रखा गया? क्या सभी मुस्लिम नेता पाकिस्तान निर्माण या विभाजन के पक्ष में थे?
उत्तर:
23 मार्च 1940 को मुस्लिम लीग ने उपमहाद्वीप के मुस्लिम बहुल इलाकों के लिए कुछ स्वायत्तता की माँग का प्रस्ताव पेश किया। इस अस्पष्ट से प्रस्ताव में कहीं भी विभाजन या पाकिस्तान का जिक्र नहीं था बल्कि इस प्रस्ताव को लिखने वाले पंजाब के प्रधानमंत्री सिकन्दर हयात खान ने 1 मार्च, 1941 को पंजाब असेम्बली में अपने भाषण में कहा था कि “वह ऐसे पाकिस्तान की अवधारणा का विरोध करते हैं जिसमें यहाँ मुस्लिम राज और बाकी जंगह हिन्दू राज होगा।”

प्रश्न 12.
क्या ‘सारे जहाँ से अच्छा हिन्दोस्तां हमारा’ गीत के लेखक मो. इकबाल अलग पाकिस्तान निर्माण के पक्ष में थे? स्पष्ट कीजिए।
उत्तर:
1930 में मुस्लिम लीग के अध्यक्षीय भाषण में मो. इकबाल ने ‘उत्तर-पश्चिमी भारतीय मुस्लिम राज्य’ की आवश्यकता पर जोर दिया था। अपने भाषण में इकबाल एक नए देश के उदय पर नहीं बल्कि पश्चिमोत्तर भारत में मुस्लिम बहुल इलाकों को एकीकृत शिथिल भारतीय संघ के भीतर एक स्वायत्त इकाई की स्थापना पर जोर दे रहे थे। इससे स्पष्ट होता है कि मो. इकबाल विभाजन के पक्ष में नहीं थे।

प्रश्न 13.
क्या मुस्लिम नेताओं और स्वयं जिन्ना ने पाकिस्तान की माँग को गंभीरता से उठाया था? यदि नहीं तो क्यों?
उत्तर:
प्रारम्भ में मुस्लिम नेताओं तथा स्वयं मोहम्मद अली जिन्ना ने पाकिस्तान की माँग को गंभीरता से नहीं उठाया था क्योंकि जिना इस माँग को एक सौदेबाजी के पैंतरे के रूप में प्रयोग कर रहे थे। उनका उद्देश्य ब्रिटिश सरकार द्वारा काँग्रेस को मिलने वाली रिवायतों पर रोक लगाने तथा मुसलमानों के लिए और रियायतें हासिल करना था। द्वितीय विश्वयुद्ध के कारण अंग्रेजों को स्वतन्त्रता के बारे में औपचारिक वार्ताएँ कुछ समय के लिए टालनी पड़ीं।

प्रश्न 14.
1945 में दोबारा शुरू हुई वार्ताएँ क्यों टूट गई?
उत्तर:
(1) अँग्रेज इस बात पर सहमत हुए कि एक केन्द्रीय कार्यकारिणी सभा बनाई जाएगी, जिसके सभी सदस्य भारतीय होंगे सिवाय वायसराव और सशस्त्र सेनाओं के सेनापति के।
(2) लेकिन यह वार्ता टूट गई। जिना इस बात पर अड़े हुए थे कि कार्यकारिणी सभा के मुस्लिम सदस्यों का चुनाव

JAC Class 12 History Important Questions Chapter 14 विभाजन को समझना : राजनीति, स्मृति, अनुभव

प्रश्न 8.
1920 व 1930 ई. के दशकों में कौन- कौनसे मुद्दे हिन्दू व मुसलमानों के मध्य तनाव का कारण बने?
अथवा
20वीं शताब्दी के प्रारम्भिक दशकों में साम्प्रदायिक अस्मिता के पक्की होने के अन्य कारण क्या थे?
उत्तर:
20वीं शताब्दी के प्रारम्भिक दशकों में साम्प्रदायिक अस्मिताएँ कई अन्य कारणों से भी ज्यादा पक्की हुई. 1920 और 1930 के दशकों में कई घटनाओं की वजह से तनाव उभरे मुसलमानों को ‘मस्जिद के सामने संगीत’, गो-रक्षा आन्दोलन, और आर्य समाज की शुद्धि की कोशिशें (यानी कि नव-मुसलमानों को फिर से हिन्दू बनाना) जैसे मुद्दों पर गुस्सा आया। दूसरी ओर हिन्दू 1923 के बाद तबलीग (प्रचार) और तंजीम के विस्तार से उत्तेजित हुए। जैसे-जैसे मध्यमवर्गीय प्रचारक और साम्प्रदायिक कार्यकर्ता अपने समुदायों में लोगों को दूसरे समुदायों के खिलाफ लामबंद करते हुए ज्यादा एकजुटता बनाने लगे, देश के विभिन्न भागों में दंगे फैलते हुए।

प्रश्न 9.
संयुक्त प्रान्त (वर्तमान उत्तर प्रदेश) में काँग्रेस ने मुस्लिम लीग के गठबंधन प्रस्ताव को क्यों खारिज कर दिया ?
उत्त:
प्रथमत:, संयुक्त प्रांत में काँग्रेस को पूर्ण बहुमत प्राप्त हुआ था। लीग वहाँ गठबंधन सरकार बनाना चाहती थी जिसे काँग्रेस ने स्वीकार नहीं किया, क्योंकि काँग्रेस को बहुमत के लिए दूसरे दल के साथ गठबंधन की आवश्यकता नहीं थी। दूसरे, इसके पीछे मुख्य कारण यह था कि काँग्रेस पार्टी जमींदारी प्रथा को खत्म करना चाहती थी और मुस्लिम लीग जमींदारी प्रथा का समर्थन करती हुई प्रतीत होती थी। तीसरे, मुस्लिम लीग मुसलमानों की एकमात्र प्रवक्ता होने पर बल दे रही थी।

प्रश्न 10.
पाकिस्तान का नाम का प्रस्ताव सर्वप्रथम किसने रखा था? लिखिए।
उत्तर:
पाकिस्तान अथवा पाकस्तान (पंजाब, अफगानिस्तान, कश्मीर, सिन्ध और बिलोचिस्तान) नाम सबसे पहले कैम्ब्रिज में पढ़ने वाले पंजाबी मुसलमान छात्र चौधरी रहमत अली ने 1933 और 1935 में लिखित अपने दो पचों में गढ़ा। रहमत अली इस नई इकाई के लिए अलग राष्ट्रीय हैसियत चाहता था। 1930 के दशक में किसी ने भी उसकी बात को गंभीरता से नहीं लिया। यहाँ तक कि मुस्लिम लीग तथा अन्य मुस्लिम नेताओं ने भी उसके इस विचार को खारिज कर दिया था।

प्रश्न 11.
पाकिस्तान का प्रस्ताव कब रखा गया? क्या सभी मुस्लिम नेता पाकिस्तान निर्माण या विभाजन के पक्ष में थे?
उत्तर:
23 मार्च 1940 को मुस्लिम लीग ने उपमहाद्वीप के मुस्लिम बहुल इलाकों के लिए कुछ स्वायत्तता की माँग का प्रस्ताव पेश किया। इस अस्पष्ट से प्रस्ताव में कहीं भी विभाजन या पाकिस्तान का जिक्र नहीं था बल्कि इस प्रस्ताव को लिखने वाले पंजाब के प्रधानमंत्री सिकन्दर हयात खान ने 1 मार्च, 1941 को पंजाब असेम्बली में अपने भाषण में कहा था कि “वह ऐसे पाकिस्तान की अवधारणा का विरोध करते हैं जिसमें यहाँ मुस्लिम राज और बाकी जंगह हिन्दू राज होगा।”

प्रश्न 12.
क्या ‘सारे जहाँ से अच्छा हिन्दोस्तां हमारा’ गीत के लेखक मो. इकबाल अलग पाकिस्तान निर्माण के पक्ष में थे? स्पष्ट कीजिए।
उत्तर:
1930 में मुस्लिम लीग के अध्यक्षीय भाषण में मो. इकबाल ने ‘उत्तर-पश्चिमी भारतीय मुस्लिम राज्य’ की आवश्यकता पर जोर दिया था। अपने भाषण में इकबाल एक नए देश के उदय पर नहीं बल्कि पश्चिमोत्तर भारत में मुस्लिम बहुल इलाकों को एकीकृत शिथिल भारतीय संघ के भीतर एक स्वायत्त इकाई की स्थापना पर जोर दे रहे थे। इससे स्पष्ट होता है कि मो. इकबाल विभाजन के पक्ष में नहीं थे।

प्रश्न 13.
क्या मुस्लिम नेताओं और स्वयं जिन्ना ने पाकिस्तान की माँग को गंभीरता से उठाया था? यदि नहीं तो क्यों?
उत्तर:
प्रारम्भ में मुस्लिम नेताओं तथा स्वयं मोहम्मद अली जिन्ना ने पाकिस्तान की माँग को गंभीरता से नहीं उठाया था क्योंकि जिन्ना इस माँग को एक सौदेबाजी के पैंतरे के रूप में प्रयोग कर रहे थे। उनका उद्देश्य ब्रिटिश सरकार द्वारा काँग्रेस को मिलने वाली रिवायतों पर रोक लगाने तथा मुसलमानों के लिए और रियायतें हासिल करना था। द्वितीय विश्वयुद्ध के कारण अंग्रेजों को स्वतन्त्रता के बारे में औपचारिक वार्ताएं कुछ समय के लिए टालनी पड़ीं।

JAC Class 12 History Important Questions Chapter 14 विभाजन को समझना : राजनीति, स्मृति, अनुभव

प्रश्न 14.
1945 में दोबारा शुरू हुई वार्ताएँ क्यों टूट गई?
उत्तर:
(1) अँग्रेज इस बात पर सहमत हुए कि एक केन्द्रीय कार्यकारिणी सभा बनाई जाएगी, जिसके सभी सदस्य भारतीय होंगे सिवाय वायसराव और सशस्त्र सेनाओं के सेनापति के।
(2) लेकिन यह वार्ता टूट गई। जिना इस बात पर अड़े हुए थे कि कार्यकारिणी सभा के मुस्लिम सदस्यों का चुनाव करने का अधिकार मुस्लिम लीग के अलावा और किसी को नहीं है। उनका कहना था कि अगर मुस्लिम सदस्य किसी फैसले का विरोध करते हैं तो उसे कम से कम दो-तिहाई सदस्यों की सहमति से ही पारित किया जाना चाहिए।

प्रश्न 15.
उर्दू कवि मोहम्मद इकबाल का उत्तरी- पश्चिमी भारतीय मुस्लिम राज्य से क्या आशय था?
उत्तर:
1930 ई. में मुस्लिम लीग के अधिवेशन में अध्यक्षीय भाषण देते हुए उर्दू कवि मोहम्मद इकबाल ने उत्तरी- पश्चिमी भारतीय मुस्लिम राज्य की आवश्यकता पर जोर दिया था परन्तु इस भाषण में इकबाल एक नए देश के उदय पर नहीं बल्कि पश्चिमोत्तर भारत में मुस्लिम बहुल इलाकों को भारतीय संघ के भीतर एक स्वायत्त इकाई की स्थापना पर जोर दे रहे थे।

प्रश्न 16.
विभाजन के लिए भारत को कितने भागों में वर्गीकृत किया गया था ? नाम लिखिए।
उत्तर:
विभाजन के लिए भारत को तीन भागों में वर्गीकृत किया गया, जो निम्नलिखित हैं – समूह ‘क’-हिन्दू बहुल प्रान्त समूह ‘ख’- पश्चिमोत्तर मुस्लिम बहुल प्रान्त समूह ‘ग’ असम सहित पूर्वोत्तर के मुस्लिम प्रान्त।

प्रश्न 17.
यूनियनिस्ट पार्टी क्या थी? इसके प्रमुख नेता का नाम बताइए कैबिनेट मिशन योजना से अपना समर्थन वापस लेने के पश्चात् मुस्लिम लीग ने क्या कार्यवाही की?
उत्तर:
यूनियनिस्ट पार्टी पंजाब में हिन्दू, मुस्लिम एवं सिख भूस्वामियों के हितों का प्रतिनिधित्व करने वाली राजनीतिक पार्टी थी। यह पार्टी 1923 से 1947 के मध्य बहुत अधिक शक्तिशाली थी। इस पार्टी के प्रमुख नेता, सिकन्दर हयात खान थे जो पंजाब के प्रधानमन्त्री भी रहे थे। कैबिनेट मिशन योजना से अपना समर्थन वापस लेने के पश्चात् मुस्लिम लीग ने पाकिस्तान की अपनी माँग को वास्तविकता प्रदान करने के लिए 16 अगस्त, 1946 को प्रत्यक्ष कार्यवाही दिवस मनाने का फैसला किया।

प्रश्न 18.
उर्दू के प्रतिभाशाली कहानीकार सआदत हसन मंटो ने अपने लेखन के बारे में क्या कहा?
उत्तर:
सआदत हसन मंटो ने लिखा है, “लम्बे अर्से तक मैं देश के बँटवारे से अपनी उथल-पुथल के नतीजों को स्वीकार करने से इनकार करता रहा महसूस तो मैं अब भी यही करता पर मुझे लगता है कि आखिरकार मैंने अपने आप पर तरस खाए या हताश हुए बगैर उस खौफनाक सच्चाई को मंजूर कर लिया है। इस प्रक्रिया में मैंने इंसान के बनाए हुए लहू के इस समंदर से अनोखी आय (चमक) वाले मोतियों को निकालने की कोशिश की।”

प्रश्न 19.
विभाजन के बारे में बहुत सी कहानी, कविता और फिल्में लिखी व बनाई गई हैं, उनमें से कुछ का संक्षेप में वर्णन कीजिये।
उत्तर:
विभाजन से सम्बन्धित यहाँ पर हम विभिन्न भाषाओं के लेखकों तथा उनकी रचनाओं के नाम दे रहे हैं।
उर्दू-सआदत हसन मंटो, राजेन्दर सिंह बेदी, इंतेजार हुसैन, फैज अहमद फैज दास।
हिन्दी-भीष्म साहनी (तमस), कमलेश्वर, राही मासूम रजा (नीम का पेड़)।
पंजाबी-संत सिंह सेखो, अमृता प्रीतम
बंगला-नरेन्द्रनाथ मित्रा, सैयद वली उल्ला, दिनेश

प्रश्न 20.
काँग्रेस ने भारत विभाजन को किस उद्देश्य से स्वीकार किया?
अथवा
क्या भारत का विभाजन अपरिहार्य था ? स्पष्ट करें।
उत्तर:
(1) कैबिनेट मिशन की असफलता के बाद विभाजन कमोबेश अपरिहार्य हो गया था। महात्मा गाँधी और खान अब्दुल गफ्फार खान को छोड़कर शेष सभी कांग्रेसी नेता विभाजन को अब अवश्यंभावी परिणाम मान चुके थे।
(2) लोग द्वारा प्रत्यक्ष कार्यवाही का फैसला लेने से कलकत्ता में दंगे भड़क गये थे जिन्होंने देश की शान्ति भंग कर दी थी। शीघ्र शान्ति की स्थापना हेतु कांग्रेस नेताओं को बँटवारे के लिए अपनी सहमति देनी पड़ी।

JAC Class 12 History Important Questions Chapter 14 विभाजन को समझना : राजनीति, स्मृति, अनुभव

प्रश्न 21.
1946 के प्रान्तीय चुनावों में काँग्रेस और मुस्लिम लीग की स्थिति में क्या अन्तर आया?
उत्तर:
(1) 1946 के प्रान्तीय चुनावों में सामान्य सीटों पर तो काँग्रेस को एकतरफा सफलता मिली। 91.3 प्रतिशत गैर मुस्लिम वोट काँग्रेस के खाते में गये।
(2) मुसलमानों के लिए आरक्षित सीटों पर मुस्लिम लीग को भी ऐसी ही बेजोड़ सफलता मिनी मध्य प्रान्त में उसने सभी 30 आरक्षित सीटें जीतीं और मुस्लिम वोटों में से 86.6 प्रतिशत उसके उम्मीदवारों को मिले। सभी प्रान्तों की कुल 509 आरक्षित सीटों में से 442 सीटें मुस्लिम लीग के पास गई।

प्रश्न 22.
कैबिनेट मिशन के प्रस्ताव को कांग्रेस और मुस्लिम लीग ने मानने से क्यों इन्कार कर दिया था?
उत्तर:
(1) कांग्रेस चाहती थी कि प्रान्तों को अपनी इच्छा का समूह चुनने का अधिकार मिलना चाहिए। कांग्रेस इस बात से भी असन्तुष्ट थी कि प्रारम्भ में प्रान्तों की समूहबद्धता अनिवार्य होगी परन्तु संविधान बन जाने के बाद उनके पास समूहों से निकलने का अधिकार होगा। (2) मुस्लिम लीग की माँग थी कि प्रान्तों की समूहबद्धता अनिवार्य हो जिसमें समूह ‘ख’ तथा ‘ग’ के पास भविष्य में संघ से अलग होने का अधिकार होना चाहिए।

प्रश्न 23.
” अंग्रेजों द्वारा 1909 ई. में मुसलमानों के लिए बनाए गए पृथक् चुनाव क्षेत्रों का साम्प्रदायिक राजनीति की प्रकृति पर गहरा प्रभाव पड़ा।” इस कथन की व्याख्या कीजिए।
उत्तर:
कुछ विद्वानों का तर्क है कि अंग्रेजों द्वारा 1909 ई. में मुसलमानों के लिए बनाए गए पृथक् चुनाव क्षेत्रों (जिनका 1919 में विस्तार किया गया) का साम्प्रदायिक राजनीति की प्रकृति पर गहरा प्रभाव पड़ा पृथक् चुनाव क्षेत्रों की व्यवस्था से मुसलमान विशेष चुनाव क्षेत्रों में अपने प्रतिनिधि चुन सकते थे। इस व्यवस्था में राजनेताओं को लालच रहता था कि वह सामुदायिक नारों का प्रयोग करें एवं अपने धार्मिक समुदाय के व्यक्तियों को अनुचित लाभ पहुँचाएँ। इसी प्रकार से उभरती हुई आधुनिक राजनीतिक व्यवस्था में धार्मिक अस्मिताओं का सक्रिय प्रयोग होने लगा। अब सामुदायिक अस्मिताओं का सम्बन्ध केवल विश्वास एवं आस्था के अन्तर से नहीं था बल्कि अब धार्मिक अस्मिताएँ समुदायों के मध्य बढ़ रहे विरोधों से जुड़ गई। यद्यपि भारतीय राजनीति पर पृथक् चुनाव क्षेत्रों का बहुत प्रभाव पड़ा।

प्रश्न 24.
क्या कांग्रेस ने कैबिनेट मिशन के प्रस्तावों को स्वीकार किया? संक्षेप में बताइए ।
उत्तर:
प्रारम्भ में कांग्रेस ने कैबिनेट मिशन के प्रस्तावों को स्वीकार कर लिया लेकिन यह समझौता अधिक दिनों तक नहीं चल पाया, कांग्रेस चाहती थी कि प्रान्तों को अपनी इच्छा का समूह चुनने का अधिकार मिलना चाहिए। कांग्रेस कैबिनेट मिशन के इस स्पष्टीकरण से भी सन्तुष्ट नहीं थी कि प्रारम्भ में यह समूहबद्धता अनिवार्य होगी लेकिन एक बार संविधान बन जाने के उपरान्त उनके पास समूहों से, निकलने का अधिकार प्राप्त होगा और परिवर्तित परिस्थितियों में नए चुनाव कराए जाएँगे। अन्ततः कांग्रेस ने कैबिनेट मिशन के प्रस्तावों को अस्वीकार कर लिया।

प्रश्न 25.
मार्च, 1947 से लगभग साल भर तक देश में रक्तपात चलता रहा।” इसका क्या कारण था?
उत्तर:
लगभग साल भर तक देश में रक्तपात चलते रहने का प्रमुख कारण यह था कि शासन की संस्थाएँ बिखर चुकी थीं। शासन-तन्त्र पूरी तरह नष्ट हो चुका था। अंग्रेज अधिकारी निर्णय लेना नहीं चाहते थे और हस्तक्षेप करने में संकोच कर रहे थे किसी को भी ज्ञात नहीं था कि सत्ता किसके हाथ में है और पीड़ित लोग कहाँ शिकायत करें। भारतीय दलों के अधिकांश नेता स्वतन्त्रता के बारे में जारी वार्ताओं में व्यस्त थे। अंग्रेज भारत छोड़ने की तैयारी में लगे थे।

प्रश्न 26.
1947 के विभाजन में क्षेत्रीय विविधताओं का उल्लेख कीजिए।
उत्तर:

  • विभाजन का सबसे खूनी और विनाशकारी रूप पंजाब में देखा गया
  • उत्तर प्रदेश, बिहार, मध्य प्रदेश और हैदराबाद (आन्ध्र प्रदेश) के बहुत सारे परिवार पचास के दशक तथा साठ के दशक के प्रारम्भिक वर्षों में भी पाकिस्तान जाकर बसते रहे।
  • बंगाल में लोगों का पलायन अधिक लम्बे समय तक चलता रहा। लोग अन्तर्राष्ट्रीय सीमा के आर-पार जाते रहे।
  • पंजाब और बंगाल में स्त्रियों और लड़कियों पर अत्याचार किए गए।

निबन्धात्मक प्रश्न

प्रश्न 1.
आपके अनुसार क्या भारत विभाजन आवश्यक था? विस्तार से उल्लेख कीजिए।
अथवा
भारत विभाजन की माँग के प्रति काँग्रेस और गाँधीजी के रवैये की विवेचना कीजिए। आखिरकार विभाजन की माँग क्यों स्वीकार कर ली गई?
उत्तर:
भारत विभाजन की माँग के प्रति काँग्रेस और गाँधीजी का रवैया – कैबिनेट मिशन योजना तक तो काँग्रेस और गाँधीजी दोनों का दृष्टिकोण भारत विभाजन के प्रति नकारात्मक था। वे किसी भी कीमत पर भारत को विभाजन नहीं चाहते थे।

विभाजन की माँग को स्वीकार करने के कारण –
काँग्रेस द्वारा भारत विभाजन की माँग को निम्न कारणों से स्वीकार कर लिया गया –

(1) ब्रिटिश सरकार की ‘फूट डालो और राज करो’ नीति-1909 में ब्रिटिश सरकार ने सांप्रदायिक चुनाव पद्धति लागू कर भारत विभाजन के बीज बोए और फिर वह निरन्तर जिन्ना व मुस्लिम लीग को प्रश्रय देते रहे और जिन्ना की हठ को स्वीकार करते हुए, माउंटबेटन योजना में विभाजन की घोषणा कर दी। इस प्रकार विभाजन की स्थिति अचानक स्वतंत्रता व सत्ता हस्तांतरण के साथ आयी। उस समय उसे स्वीकार करने के अलावा कोई अन्य विकल्प काँग्रेस के पास नहीं रह गया था।

(2) साम्प्रदायिक तनाव-920 और 1930 के दशकों में कई घटनाओं के कारण साम्प्रदायिक तनावों में वृद्धि हुई। मुसलमान मस्जिद के सामने संगीत, गोरक्षा आन्दोलन, आर्य समाज द्वारा संचालित शुद्धि आन्दोलन आदि से नाराज थे। दूसरी ओर हिन्दू तबलीग (प्रचार) और तंजीम (संगठन) के विस्तार से नाराज थे। इससे साम्प्रदायिक तनाव को प्रोत्साहन मिला।.

JAC Class 12 History Important Questions Chapter 14 विभाजन को समझना : राजनीति, स्मृति, अनुभव

(3) पाकिस्तान की माँग-23 मार्च, 1940 को मुस्लिम लीग ने लाहौर अधिवेशन में मुस्लिम बहुल क्षेत्रों के लिए स्वायत्ता की माँग का प्रस्ताव प्रस्तुत किया।

(4) अंतरिम सरकार की विफलता-अंतरिम सरकार में सम्मिलित मुस्लिम लीग के प्रतिनिधियों ने कदम-कदम पर रुकावटें पैदा कर उसे विफल कर दिया।

(5) 1946 के चुनावों में लीग को आरक्षित सीटों पर मिली अच्छी सफलता-1946 में दुबारा हुए प्रांतीय चुनावों में कुल 509 आरक्षित सीटों में से 442 मुस्लिम लीग के पास गई थीं। अब वह मुस्लिम मतदाताओं के बीच सबसे प्रभुत्वशाली पार्टी के रूप में उभरी थी।

(6) ब्रिटिश सरकार की धमकी-20 फरवरी, 1947 को ब्रिटिश प्रधानमंत्री लॉर्ड एटली ने घोषणा की कि जून, 1948 तक अंग्रेज भारत छोड़ देंगे।

(7) साम्प्रदायिक दंगे-मुस्लिम लीग ने पाकिस्तान की अपनी माँग को मनवाने के लिए 16 अगस्त, 1946 को ‘प्रत्यक्ष कार्यवाही दिवस’ मनाने की घोषणा कर दी। उस दिन कलकत्ता में भीषण दंगा भड़क उठा जिसमें हजारों लोग मारे गए।

प्रश्न 2.
दिल्ली अब बच जायेगी।” 1947 के साम्प्रदायिक दंगों के संदर्भ में गाँधीजी के लिए कहे गए उक्त कथन की सत्यता सिद्ध कीजिये।
अथवा
सांप्रदायिक सौहार्द बनाने में गाँधीजी के योगदान का वर्णन कीजिए।
अथवा
स्वतंत्रता प्राप्त होने के साथ भड़के सांप्रदायिक दंगों में गाँधीजी ने कैसे अकेली फौज की तरह कार्य किया? लिखिए।
उत्तर:
स्वतंत्रता प्राप्ति के साथ-साथ सांप्रदायिक दंगे भड़क उठे हिन्दू और मुसलमान दोनों आपसी भाई चारे को भूल गए। इस सारी उथल-पुथल में सांप्रदायिक सद्भाव बहाल करने के लिए एक आदमी की बहादुराना कोशिशें आखिरकार रंग लाने लगीं।
(1) अहिंसा का सहारा-77 साल के बुजुर्ग गांधीजी ने अपने जीवनपर्यन्त सिद्धान्त को एक बार फिर आजमाया और अपना सब कुछ दाँव पर लगा दिया।

(2) गाँधीजी की पदयात्रा- गाँधीजी पूर्वी बंगाल के नोआखली (वर्तमान बांग्लादेश) से बिहार तक के गाँवों में उसके बाद कलकत्ता व दिल्ली के दंगों में झुलसी झोंपड़- पट्टियों की यात्रा पर निकल पड़े।

(3) गांधीजी पूर्वी बंगाल में-अक्टूबर, 1946 में पूर्वी बंगाल के मुसलमान हिन्दुओं को अपना निशाना बना रहे थे। गाँधी वहाँ पैदल गाँव-गाँव घूमे और स्थानीय मुसलमानों को समझाया कि ये हिन्दुओं को न मारें तथा उनकी रक्षा करें।

(4) गाँधीजी दिल्ली में दिल्ली में गाँधीजी ने दोनों समुदायों को भरोसा दिलाया तथा पारस्परिक विश्वास और भरोसा कायम रखने की सलाह दी।

(5) गाँधीजी शीशगंज गुरुद्वारे में 28 नवम्बर, 1947 को गुरुनानक जयंती के अवसर पर गुरुद्वारा शीशगंज में सिखों की एक सभा को संबोधित करने गये तो उन्होंने देखा कि दिल्ली का दिल कहलाने वाले चाँदनी चौक में एक भी मुसलमान सड़क पर नहीं था।
गाँधीजी अनशन पर मुसलमानों को शहर से बाहर खदेड़ने की सोच से तंग आकर उन्होंने अनशन शुरू किया। इस अनशन में उनके साथ पाकिस्तान से आए शरणार्थी हिन्दू व सिख भी बैठते थे।

मौलाना आजाद ने लिखा है कि “इस अनशन का असर ‘आसमानी बिजली’ की तरह हुआ।” लोगों को मुसलमानों के सफाए की बात में निरर्थकता दिखाई देने लगी। मगर हिंसा का यह नंगा नाच आखिरकार गाँधीजी के बलिदान के साथ ही खत्म हुआ। इस प्रकार हम देखते हैं कि जो कार्य एक फौज नहीं कर सकती थी उसे बुजुर्ग गाँधीजी ने अपने अकेले दम पर करके दिखाया। इसलिए उन्हें एक अकेली फौज कहकर सम्मानित किया गया।

प्रश्न 3.
शोधकर्ता के सामने लाहौर विश्वविद्यालय में जो घटनाएँ घटित हुई उनका वर्णन करते हुए बताइए कि इन घटनाओं से क्या निष्कर्ष निकला?
उत्तर:
शोधकर्ता पंजाब विश्वविद्यालय लाहौर में विभाजन के समय हुए दंगों के विषय पर शोध करने गया था। वह भारतीय नागरिक था लेकिन वह स्वयं को दक्षिण एशियाई नागरिक मानता था। उसके मन में हिन्दुस्तान या पाकिस्तान का कोई भेदभाव नहीं था। शोध करते समय उसके सामने तीन घटनाएँ घटित हुईं जिनसे अलग-अलग निष्कर्ष निकलते हैं। प्रथम घटना” मैं तो सिर्फ अपने अब्बा पर चढ़े हुए कर्ज को चुका रहा हूँ।”

शोधकर्ता विश्वविद्यालय के इतिहास विभाग के पुस्तकालय में जाया करता था तो वहाँ अब्दुल लतीफ नामक धर्मनिष्ठ अधेड़ आयु का व्यक्ति उसकी बहुत मदद करता था। जब शोधकर्ता ने उससे मदद देने के आरे में जानकारी चाही तो उसका जवाब सुनकर शोधकर्ता अवाक् रह गया। अब्दुल लतीफ जानते थे कि शोधकर्ता भारतीय है जहाँ बँटवारे में उसका पूरा खानदान खत्म हो गया था सिवाय उसके पिता के उसके पिता की जान एक बुजुर्ग हिन्दू महिला ने बचाई थी इसलिए वह अपने को ऋणी मानते हुए शोधकर्ता की मदद कर रहा था।

इस घटना से पता चलता है कि अब्दुल लतीफ एक दयालु और एहसानमंद व्यक्ति था जिसके मन में भारत के प्रति नफरत नहीं थी। यह घटना बताती है कि अब्दुल लतीफ मजहब के बजाय इंसानी रिश्ते को महत्व देने वाला व्यक्ति था। वह मजहबी दंगों को सिर्फ एक पागलपन मानता था। दूसरी घटना “बरसों हो गए, मैं किसी पंजाबी मुसलमान से नहीं मिला।”

शोधकर्ता के सामने दूसरी घटना लाहौर के एक यूथ हॉस्टल के मैनेजर के साथ घटी जिसने भारतीय होने के कारण शोधकर्ता को हॉस्टल में स्थान देने से मना कर दिया लेकिन शोधकर्ता को चाय पिलाई और अपने साथ दिल्ली में घटी घटना सुनाई। जब वह एक सरदार पहाड़गंज का पता पूछता है तो सरदार उसे रुकने को कहता है।

JAC Class 12 History Important Questions Chapter 14 विभाजन को समझना : राजनीति, स्मृति, अनुभव

मैनेजर उसकी आवाज से डर गया कि अब यह सरदार मुझे खत्म कर देगा क्योंकि उसने अपना नाम इकबाल अहमद तथा निवासी लाहौर बताया था लेकिन उसका डर सही नहीं था। सरदार ने आते ही उसे अपनी बाँहों में कसकर भींच लिया और वह भीगी आँखों से बोला “बरसों हो गए, मैं किसी पंजाबी मुसलमान से नहीं मिला। मैं मिलने को तरस रहा था पर यहाँ पंजाबी बोलने वाले मुसलमान मिलते ही नहीं।” इस घटना से पता चलता है कि विभाजन के बाद भी हिन्दू और मुसलमान एक-दूसरे से मिलने के लिए आतुर रहते थे।

तीसरी घटना-“ना, नहीं तुम कभी हमारे नहीं हो सकते।” शोधकर्ता को लाहौर में एक व्यक्ति मिला जो धोखे से उसे पाकिस्तानी समझ कर उसे वहीं रहने की जिद करने लगा लेकन शोधकर्ता ने बताया कि वह पाकिस्तानी नहीं है, हिन्दुस्तानी है यह सुनते ही उसके मुँह से न चाहते हुए ये शब्द निकले, “ना, नहीं तुम कभी हमारे नहीं हो सकते। तुम्हारे लोगों (भारतीयों) ने हमारे पूरे गाँव को 1947 में साफ कर दिया था। हम कट्टर दुश्मन हैं और हमेशा रहेंगे।” यह घटना बताती है कि तीसरा व्यक्ति मजहब को महत्त्व दे रहा था, उसके अन्दर भारत के प्रति अपार घृणा थी। उसमें इंसानियत का जज्बा (भावना नहीं था।

प्रश्न 4.
“देश के विभाजन के दौरान जहाँ दोनों तरफ मारकाट मची थी वहीं पर कुछ लोग पीड़ितों की मदद करके मानवता और सद्भावना की मिसाल कायम कर रहे थे।” अपनी पाठ्यपुस्तक में से पढ़ी हुई किसी ऐसी घटना का वर्णन कीजिए।
अथवा
डॉ. खुशदेव सिंह मानवता व सद्भावना की जिन्दा मिसाल थे। इस कथन की व्याख्या कीजिए।
उत्तर:
देश में बँटवारे के समय दंगों के दौरान जहाँ लोग निर्दोषों का खून बहा रहे थे वहीं दूसरी तरफ डॉ. खुशदेव सिंह जैसे लोग बिना जाति और मजहब का विचार किए पीड़ितों की सेवा में जी-जान से जुटे थे पीड़ितों को राहत पहुँचाना ही उनका मजहब था और ईमान था इतिहासकारों ने अगणित कहानियाँ उजागर की हैं कि किस तरह बहुत सारे लोग बँटवारे के समय एक-दूसरे की मदद कर रहे थे। ये आपसी हमदर्दी और साझेदारी नए मौकों के खुलने और सदमों पर विजय की कहानियाँ हैं। डॉ. खुशदेव सिंह की कहानी भी इसी प्रकार की कहानी है –

डॉ.खुशदेव सिंह – डॉ. खुशदेव सिंह हमारे सामने एक बेहतरीन मिसाल हैं। डॉ. खुशदेव सिंह एक सिख डॉक्टर थे जो तपेदिक (टी.बी. रोग के विशेषज्ञ थे। वे उस समय धर्मपुर में तैनात थे जो आजकल हिमाचल प्रदेश में है। दिन- रात लगकर डॉ. सिंह ने असंख्य प्रवासी मुसलमानों, सिखों और हिन्दुओं को बिना किसी भेदभाव के एक कोमल स्पर्श, भोजन, आश्रय और सुरक्षा प्रदान की।

धर्मपुर के लोगों में उनके इंसानी जज्बे और सहृदयता के प्रति गहरी आस्था और विश्वास पैदा हो गया था। उन पर लोगों को वैसा ही भरोसा था जैसा दिल्ली और कई जगह के मुसलमानों को गाँधीजी पर था। एक मुस्लिम मुहम्मद उमर ने अपनी चिट्ठी में डॉ. खुशदेव सिंह को लिखा था, “पूरी विनम्रता से मैं यह कहना चाहता हूँ कि मुझे आपके अलावा किसी की शरण में सुरक्षा दिखाई नहीं देती। इसलिए मेहरबानी करके आप मुझे अपने अस्पताल में एक सीट दे दीजिए।”

डॉ. खुशदेव सिंह के संस्मरण – डॉ. खुशदेव सिंह द्वारा किए गए अथक प्रयासों के बारे में उनके संस्मरणों लव इज स्ट्रांगर दैन हेट ए रिमेम्बेरेन्स ऑफ 1947 ( मुहब्बत नफरत से ज्यादा ताकतवर होती है-1947 की यादें) से पता चलता है। यहाँ डॉक्टर साहब ने अपने कामों को बयान करते हुए लिखा है कि यह ” एक इंसान होने के नाते बिरादर इंसानों के प्रति अपनी जिम्मेदारी का निर्वाह करते हुए मेरी एक छोटी सी कोशिश थी।”

उन्होंने 1949 में कराची की दो संक्षिप्त यात्राओं का गर्व से जिक्र किया है। उनके पुराने दोस्तों और धर्मपुर में उनसे मदद लेने वालों को कराची हवाई अड्डे पर उनके साथ कुछ यादगार घंटे बिताने का मौका मिला। पहले से उन्हें जानने वाले 6 पुलिस कांस्टेबल उन्हें लेकर हवाई जहाज तक गए और जहाज पर चढ़ते हुए उन्हें सलामी दी। “मैंने हाथ जोड़कर उनका अभिवादन किया। मेरी आँखों में आँसू छलक आए थे।”

प्रश्न 5.
भारत विभाजन को गृहयुद्ध या महाध्वंस क्यों कहा गया है? यह महाध्वंस नात्सी महाध्वंस से किस रूप में भिन्न है?
उत्तर:
भारत विभाजन – गृहयुद्ध तथा महाध्वंस के रूप में –
भारत विभाजन के दौरान जो चौतरफा हिंसा हुई, उसमें कई लाख लोग मारे गये न जाने कितनी औरतों का बलात्कार और अपहरण हुआ। मोटे रूप से इसमें मरने वालों की संख्या दो लाख से पाँच लाख तक रही तथा लगभग डेढ़ करोड़ लोगों को एक सरहद से दूसरी सरहद में जाना पड़ा इसे एक सामान्य विभाजन, एक व्यवस्थित संवैधानिक फैसला तथा आपसी रजामंदी के आधार पर इलाके और सम्पत्तियों का सामान्य बँटवारा भर नहीं कहा जा सकता।

JAC Class 12 History Important Questions Chapter 14 विभाजन को समझना : राजनीति, स्मृति, अनुभव

गृहयुद्ध – कुछ इतिहासकारों ने इसे 16 माह का गृहयुद्ध कहा है। उनका तर्क रहा है कि पाले के दोनों तरफ पूरी की पूरी जनसंख्या का दुश्मनों की तरह सफाया कर देने के लिए सुनियोजित कोशिशें की जा रही थीं और इसके लिए संगठित गिरोह कमर कसे खड़े थे। जिन्दा बच जाने वाले लोग इस विभाजन के दौर को इन शब्दों में व्यक्त करते हैं- माशल-ला (मार्शल लॉ)’. ‘मारामारी’, ‘रौला’ या ‘हुल्लड़’।

महाध्वंस (होलोकॉस्ट) – विभाजन के दौरान हुई हत्याओं, बलात्कार, आगजनी और लूटपाट को देखते हुए समकालीन प्रेक्षकों और विद्वानों ने इसके लिए ‘मराध्वंस’ शब्द का उल्लेख करते हुए इस सामूहिक जनसंहार की भयानकता को रेखांकित किया है। एक दृष्टि से देखें तो भारत विभाजन की इस भीषणता को ‘महाध्वंस’ शब्द से ही समझा जा सकता है क्योंकि यह हादसा इतना जघन्य था कि ‘विभाजन’, ‘बंटवारे’ जैसे शब्दों से उसके सारे पहलू सामने नहीं आते। इससे यह भी समझने में मदद मिलती है कि यूरोपीय महाध्वंस (नात्सी जर्मन महाध्वंस) की तरह हमारे समकालीन सरोकारों में भी विभाजन का इतना ज्यादा जिक्र क्यों आता है?

JAC Class 12 History Important Questions Chapter 14 विभाजन को समझना : राजनीति, स्मृति, अनुभव

यूरोपीय महाध्वंस और भारत-विभाजन के महाध्वंस में अन्तर यूरोपीय महाध्वंस और भारत विभाजन के महाध्वंस में एक गुणात्मक अन्तर सरकारी भूमिका को लेकर था। 1947-49 में विभाजन के दौरान भारतीय उपमहाद्वीप में जनसफाए की कोई सरकारी मुहिम नहीं चली थी जबकि नारसी जर्मनी के महाध्वंस में सरकार मुख्य भूमिका निभा रही थी। वहाँ लोगों को मारने के लिए नियन्त्रण और संगठन की तमाम आधुनिक तकनीकों का इस्तेमाल किया गया था, लेकिन भारत विभाजन के समय जो नस्ली सफाया हुआ वह सरकारी निकायों की नहीं बल्कि धार्मिक समुदायों के स्वयंभू प्रतिनिधियों की कारगुजारी थी।

प्रश्न 6.
सांप्रदायिकता से क्या अभिप्राय है? क्या भारत-पाक बँटवारा प्रत्यक्ष रूप से सांप्रदायिक तनावों का ही परिणाम है?
उत्तर:
सांप्रदायिकता से आशय सांप्रदायिकता उस राजनीति को कहा जाता हैं जो धार्मिक समुदायों के बीच विरोध और झगड़े पैदा करती है। यथा –
(1) ऐसी राजनीति धार्मिक पहचान को बुनियादी और अटल मानती है सांप्रदायिक राजनीतिज्ञ शर्मिक पहचान को मजबूत बनाना चाहते हैं। वे इसे लोगों की एक स्वाभाविक अस्मिता मानकर पेश करते हैं, मानो लोग ऐसी पहचान लेकर पैदा हुए हों, मानो ये अस्मिताएँ इतिहास और समय के दौर से गुजरते हुए बदलती नहीं हैं।

(2) सांप्रदायिकता किसी भी समुदाय में एकता पैदा करने के लिए आंतरिक अन्तरों को दबाती है उस समुदाय की एकता पर जोर देती है और उस समुदाय को किसी न किसी अन्य समुदाय के विरुद्ध लड़ने के लिए प्रेरित करती है।

(3) सांप्रदायिकता किसी चिह्नित ‘गैर’ के विरुद्ध घृणा की राजनीति को पोषित करती है। उदाहरण के लिए मुस्लिम सांप्रदायिकता हिन्दुओं को ‘गैर’ बताकर उनका विरोध करती है और ऐसे ही हिन्दू सांप्रदायिकता मुसलमानों को गैर बताकर उनके विरुद्ध डटी रहती है। इस पारस्परिक घृणा से हिंसा की राजनीति को बढ़ावा मिलता है।

उपर्युक्त विवेचन से स्पष्ट होता है कि सांप्रदायिकता धार्मिक अस्मिता का विशेष तरह से राजनीतिकरण है जो धार्मिक समुदायों में परस्पर घृणा फैलाकर झगड़े पैदा करवाने की कोशिश करता है तथा हिंसा की राजनीति को बढ़ावा देता है किसी भी बहुधार्मिक देश में ‘धार्मिक राष्ट्रवाद’ शब्दों का अर्थ भी सांप्रदायिकता के करीब-करीब हो सकता है। ऐसे देश में यदि कोई व्यक्ति किसी धार्मिक समुदाय को राष्ट्र मानता है, तो वह विरोध और झगड़ों के बीज बो रहा है।

प्रश्न 7.
“भारत का बँटवारा एक साम्प्रदायिक राजनीति का आखिरी बिन्दु था।” इस कथन की समालोचना कीजिए।
उत्तर:
भारत का बँटवारा एक साम्प्रदायिक राजनीति का आखिरी बिन्दु कुछ विद्वानों की मान्यता है कि भारत का बँटवारा एक ऐसी साम्प्रदायिक राजनीति का आखिरी बिन्दु था, जो बीसवीं शताब्दी के प्रारम्भिक दशकों में शुरू हुई। इसकी पुष्टि अग्रलिखित तथ्यों से होती है –
(1) मुसलमानों के लिए बनाए गए पृथक् निर्वाचन क्षेत्र अंग्रेजों द्वारा 1909 में मुसलमानों के लिए बनाए गए पृथक् चुनाव क्षेत्रों का सांप्रदायिक राजनीति की प्रकृति पर गहरा प्रभाव पड़ा पृथक् चुनाव क्षेत्रों की वजह से मुसलमान विशेष चुनाव क्षेत्रों में अपने प्रतिनिधि चुन सकते थे। इस व्यवस्था में राजनीतिज्ञों ने सामुदायिक नारों का इस्तेमाल किया तथा अपने धार्मिक समुदाय के व्यक्तियों को नाजायज तरीके से लाभ पहुँचाने की कोशिश की। इससे धार्मिक अस्मिताओं का क्रियाशील प्रयोग होने लगा। इस प्रकार सांप्रदायिक चुनावी राजनीति ने इन धार्मिक अस्मिताओं को अधिक गहरा तथा पक्का किया और अब धार्मिक अस्मिताएँ समुदायों के बीच हो रहे विरोधों से जुड़ गई।

(2) अंग्रेजों की फूट डालो और शासन करो नीति-ब्रिटिश साम्राज्य को स्थायी रूप से बनाये रखने के लिए ब्रिटिश सरकार ने ‘फूट डालो और शासन करो’ की नीति अपनाई ।

(3) 1920-30 के दशक में बढ़ते सांप्रदायिक तनाव-कुछ इतिहासकारों का मत है कि 1920-30 के दशकों में कई घटनाओं की वजह से हिन्दू-मुसलमानों में तनाव उभरे। जैसे- मुसलमानों को ‘मस्जिद के सामने संगीत’, ‘गो-रक्षा आन्दोलन’ और आर्य समाज की ‘शुद्धि’ की कोशिशें जैसे मुद्दों पर गुस्सा आया तो हिन्दू 1923 के बाद के तबलीग (प्रचार) और तंजीम (संगठन) के विस्तार से उत्तेजित हुए।

जैसे-जैसे मध्यवर्गीय प्रचारक और सांप्रदायिक कार्यकर्ता अपने-अपने समुदायों में लोगों को दूसरे समुदाय के खिलाफ एकजुट करते हुए ज्यादा एकजुटता बनाने लगे, वैसे-वैसे देश के विभिन्न भागों में दंगे फैलते गए, समुदायों के बीच भेदभाव गहरे होते गए और हिंसात्मक गतिविधियों में वृद्धि होने लगी।

(4) साम्प्रदायिक दंगे-मुस्लिम लीग ने पाकिस्तान की अपनी माँग को मनवाने के लिए 16 अगस्त, 1946 को ‘प्रत्यक्ष कार्यवाही दिवस’ मनाने की घोषणा कर दी। उस दिन कलकत्ता में भीषण दंगा भड़क उठा जिसमें हजारों लोग मारे गए। इससे भी साम्प्रदायिकता को बढ़ावा मिला।

JAC Class 12 History Important Questions Chapter 14 विभाजन को समझना : राजनीति, स्मृति, अनुभव

भारत-पाक बँटवारा प्रत्यक्ष रूप से सांप्रदायिक तनावों का परिणाम नहीं से हुआ इस सबके बावजूद यह कहना गलत है कि बँटवारा केवल सीधे-सीधे बढ़ते हुए सांप्रदायिक तनावों की वजह ‘क्योंकि सांप्रदायिक कलह तो 1947 से पहले भी होती थी, पर इसके कारण लाखों लोगों के घर नहीं उजड़े। अतः पहले की साम्प्रदायिक राजनीति और विभाजन में गुणात्मक अन्तर है। बँटवारे के पीछे ब्रिटिश शासन के आखिरी दशक की घटनाएँ उत्तरदायी रही हैं।

प्रश्न 8.
भारत के विभाजन में ब्रिटिश शासन के अन्तिम दशक के कौन-कौन से कारकों को प्रमुख उत्तरदायी माना जाता है?
उत्तर:
भारत के विभाजन के उत्तरदायी कारक- ब्रिटिश शासन के अन्तिम दशक में निम्नलिखित घटनाओं व कारकों को भारत के विभाजन के लिए उत्तरदायी माना जाता है –
(1) 1937 के प्रांतीय चुनाव और कॉंग्रेस मंत्रालय- प्रांतीय संसदों के गठन के लिए 1937 में पहली बार चुनाव कराए गए। इन चुनावों में कांग्रेस के परिणाम अच्छे रहे। उसने 11 में से 7 प्रांतों में अपनी सरकारें बनाई। मुसलमानों के लिए आरक्षित चुनाव क्षेत्रों में कांग्रेस और लीग दोनों का प्रदर्शन अच्छा नहीं रहा। इन चुनावों के बाद मंत्रिमण्डल के निर्माण में काँग्रेस ने जिस तरह मुस्लिम लीग की उपेक्षा की, उसने पाकिस्तान के निर्माण की नींव रख दी।

(2) काँग्रेस के मुस्लिम जनसम्पर्क कार्यक्रम की असफलता-1937 के चुनावों के बाद काँग्रेस को अपने ‘मुस्लिम जनसम्पर्क’ कार्यक्रम में कोई खास सफलता नहीं मिल पायी थी।

(3) मुस्लिम लीग का पाकिस्तान प्रस्ताव, 1940-पाकिस्तान की स्थापना की माँग धीरे-धीरे ठोस रूप ले रही थी। 23 मार्च, 1940 को मुस्लिम लीग ने उपमहाद्वीप के मुस्लिम बहुल इलाकों के लिए कुछ स्वायत्तता की माँग का प्रस्ताव पेश किया। यह भारत के विभाजन या पृथक् पाकिस्तान राष्ट्र की माँग नहीं थी, बल्कि पश्चिमोत्तर भारत में मुस्लिम बहुल इलाकों को एकीकृत किन्तु शिथिल भारत- संघ के भीतर एक स्वायत्त इकाई की स्थापना की माँग थी।

(4) 1942 का भारत छोड़ो आन्दोलन और भारत-की स्वतंत्रता के लिए वार्तायें आरम्भ 1942 के शुरू हुए विशाल भारत छोड़ो आन्दोलन के परिणामस्वरूप अंग्रेजों को भारत की स्वतंत्रता के बारे में भारतीयों से बातें करने के लिए झुकना पड़ा और उसके अफसरों को संभावित सत्ता हस्तान्तरण के बारे में भारतीय पक्षों के साथ बातचीत करने के लिए तैयार होना पड़ा। सत्ता के हस्तान्तरण का पेचीदा प्रश्न ही विभाजन का प्रमुख कारक
बना।

(5) जिन्ना की हठधर्मिता-1945 में वार्ताओं के दौरान अंग्रेज इस बात पर सहमत हुए कि एक केन्द्रीय कार्यकारिणी सभा बनायी जायेगी जिसके सभी सदस्य भारतीय होंगे सिवाय वायसराय और सशस्त्र सेनाओं के सेनापति के उनकी राय में यह पूर्ण स्वतंत्रता की ओर शुरुआती कदम था लेकिन सत्ता हस्तान्तरण के बारे में जिना की इस हठधर्मिता के कारण वार्ता टूट गई क्योंकि वे इस बात पर अड़े हुए थे कि कार्यकारिणी सभा के मुस्लिम सदस्यों का चुनाव करने का अधिकार मुस्लिम लीग के अलावा और किसी को नहीं है।

(6) कैबिनेट मिशन की असफलता मार्च, 1946 में ब्रिटिश मंत्रिमंडल ने लीग की माँग का अध्ययन करने एवं स्वतंत्र भारत के लिए एक उचित राजनीतिक रूपरेखा सुझाने के लिए कैबिनेट मिशन दिल्ली भेजा, जिसने भारत का दौरा कर एक ढीले-ढाले त्रिस्तरीय महासंघ का सुझाव दिया। काँग्रेस और मुस्लिम लीग दोनों दलों ने इसे अस्वीकृत कर दिया।

(7) काँग्रेस द्वारा विभाजन को स्वीकार करना-यह एक बहुत महत्वपूर्ण पड़ाव था क्योंकि इसके बाद विभाजन कमोबेश अपरिहार्य हो गया था। काँग्रेस के ज्यादातर नेता इसे शासद मगर अवश्यंभावी परिणाम मान चुके थे। गाँधीजी और खान अब्दुल गफ्फार खान ही अब केवल विभाजन के विरोधी रह गये थे। मार्च, 1947 में काँग्रेस हाईकमान ने पंजाब को मुस्लिम बहुल हिन्दू- सिख बहुल दो हिस्सों में बाँटने के प्रस्ताव की मंजूरी दे दी।

प्रश्न 9.
1937 में प्रान्तीय चुनाव व कांग्रेस की भूमिका पर एक लेख लिखिए।
उत्तर:
1937 में प्रान्तीय चुनाव व कांग्रेस की भूमिका प्रान्तीय संसदों के गठन के लिए 1937 में पहली बार चुनाव कराये गए। इन चुनावों में मताधिकार केवल 10 से 12 प्रतिशत लोगों के पास था। इन चुनावों में कांग्रेस की स्थिति अच्छी रही। उसने 11 प्रान्तों में से 5 प्रान्तों में पूर्ण बहुमत प्राप्त किया और 7 प्रान्तों में अपनी सरकार बनाई। मुसलमानों के लिए आरक्षित चुनाव क्षेत्रों में कांग्रेस का प्रदर्शन अच्छा नहीं रहा परन्तु मुस्लिम लीग को भी इन क्षेत्रों में बहुत अच्छी सफलता नहीं मिली। उसे इस चुनाव में सम्पूर्ण मुस्लिम वोट का केवल 44 प्रतिशत हिस्सा ही मिला। उत्तर पश्चिमी सीमा प्रान्त में उसे एक सीट भी नहीं मिली। पंजाब की 84 आरक्षित सीटों में उसे केवल 2 प्राप्त हुई और सिन्ध में से 13 प्राप्त हुई।

संयुक्त प्रान्त में मुस्लिम लीग द्वारा कांग्रेस के साथ मिलकर सरकार बनाने का प्रयास संयुक्त प्रान्त में मुस्लिम लीग कांग्रेस के साथ मिलकर सरकार बनाना चाहती थी। परन्तु यहाँ कांग्रेस का सम्पूर्ण बहुमत था इसलिए उसने मुस्लिम लीग की इस माँग को अस्वीकार कर दिया। मुस्लिम लीग की यह मान्यता थी कि मुस्लिम हितों का प्रतिनिधित्व एक मुस्लिम दल ही कर सकता है और कांग्रेस एक हिन्दू दल है। मुहम्मद अली जिन्ना इस जिद्द पर अड़े हुए थे कि मुस्लिम लीग को मुसलमानों का एकमात्र प्रवक्ता माना जाए। परन्तु जिन्ना की इस बात से बहुत कम लोग सहमत थे।

JAC Class 12 History Important Questions Chapter 14 विभाजन को समझना : राजनीति, स्मृति, अनुभव

कांग्रेस मंत्रालयों द्वारा इस खाई को गहरा करना – कांग्रेस मंत्रालयों ने भी इस खाई को और गहरा कर दिया। संयुक्त प्रान्त में पार्टी ने गठबन्धन सरकार बनाने के सम्बन्ध में मुस्लिम लीग के प्रस्ताव को अस्वीकृत कर दिया था क्योंकि मुस्लिम लीग जमींदारी प्रथा का समर्थन कर रही थी जबकि कांग्रेस जमींदारी प्रथा को समाप्त करना चाहती थी।

कांग्रेस को मुस्लिम जनसम्पर्क कार्यक्रम में सफलता न मिलना – कांग्रेस को अपने मुस्लिम जनसम्पर्क कार्यक्रम में भी सफलता नहीं मिली। इस प्रकार कांग्रेस के धर्मनिरपेक्ष और एडियल बयानों से रूढ़िवादी मुसलमान और मुसलमान भू-स्वामी तो चिन्ता में ही पड़ गए, कांग्रेस मुसलमानों को अपनी ओर आकर्षित करने में भी सफल नहीं हो पाई। मौलाना आजाद ने 1937 में यह प्रश्न किया था कि कांग्रेस के सदस्यों को मुस्लिम लीग में शामिल होने की छूट तो नहीं है परन्तु उन्हें हिन्दू महासभा में शामिल होने से नहीं रोका जाता है। उनका कहना था कि कम से कम मध्य प्रान्त (वर्तमान मध्य प्रदेश) में यही स्थिति थी।

JAC Class 12 History Important Questions Chapter 13 महात्मा गांधी और राष्ट्रीय आंदोलन : सविनय अवज्ञा और उससे आगे

Jharkhand Board JAC Class 12 History Important Questions Chapter 13 महात्मा गांधी और राष्ट्रीय आंदोलन : सविनय अवज्ञा और उससे आगे Important Questions and Answers.

JAC Board Class 12 History Important Questions Chapter 12 महात्मा गांधी और राष्ट्रीय आंदोलन : सविनय अवज्ञा और उससे आगे

बहुविकल्पीय प्रश्न (Multiple Choice Questions)

1. गाँधीजी दक्षिणी अफ्रीका से भारत वापस आए
(क) जनवरी, 1915 में
(ख) मार्च, 1915 में
(ग) दिसम्बर, 1915 में
(घ) सितम्बर, 1915 में
उत्तर:
(क) जनवरी, 1915 में

2. गाँधीजी ने अपना पहला सत्याग्रह किया था –
(क) दक्षिणी अमेरिका में
(ख) दक्षिणी अफ्रीका में
(ग) दक्षिणी आस्ट्रेलिया में
(घ) दक्षिणी भारत में
उत्तर:
(ख) दक्षिणी अफ्रीका में

3. गाँधीजी के राजनीतिक गुरु थे –
(क) बाल गंगाधर तिलक
(ख) महादेव गोविन्द रानाडे
(ग) गोपालकृष्ण गोखले
(घ) वल्लभ भाई पटेल
उत्तर:
(ग) गोपालकृष्ण गोखले

JAC Class 12 History Important Questions Chapter 13 महात्मा गांधी और राष्ट्रीय आंदोलन : सविनय अवज्ञा और उससे आगे

4. गाँधीजी ने फसल चौपट होने पर लगान माफ करने की माँग कहाँ की ?
(क) सूरत में
(ख) चंपारन में
(ग) बिहार में
(घ) खेड़ा में
उत्तर:
(घ) खेड़ा में

5. जलियाँवाला बाग काण्ड हुआ था –
(क) लाहौर में।
(ख) अमृतसर में
(ग) कराची में
(घ) कलकत्ता में
उत्तर:
(ख) अमृतसर में

6. खिलाफत आन्दोलन चलाने वाले नेता थे –
(क) मोहम्मद अली व शौकत अली
(ख) जिला- जवाहरलाल
(ग) गाँधीजी और सरदार पटेल
(घ) इनमें से कोई नहीं
उत्तर:
(क) मोहम्मद अली व शौकत अली

7. काला विधेयक किसे कहा जाता है?
(क) इलबर्ट
(ख) रॉलेट एक्ट
(ग) शिक्षा बिल
(घ) इनमें से कोई नहीं
उत्तर:
(ख) रॉलेट एक्ट

JAC Class 12 History Important Questions Chapter 13 महात्मा गांधी और राष्ट्रीय आंदोलन : सविनय अवज्ञा और उससे आगे

8. सरकारी आँकड़ों के अनुसार 1921 में कुल हड़तालें हुई –
(क) 496
(ख) 396
(ग) 284
(घ) 398
उत्तर:
(ख) 396

9. गाँधीजी ने नमक यात्रा (दांडी मार्च) की थी –
(क) मार्च, 1930 में
(ख) जून, 1931 में
(ग) मार्च, 1931 में
(घ) अप्रैल, 1930 में
उत्तर:
(क) मार्च, 1930 में

10. अमृतसर में जलियाँवाला बाग काण्ड हुआ था-
(क) 13 अप्रैल, 1919 में
(ख) 14 फरवरी, 1919 में
(ग) 17 अप्रैल, 1919 में
(घ) 25 दिसम्बर, 1919 में
उत्तर:
(क) 13 अप्रैल, 1919 में

11. भारतीय राष्ट्र का पिता माना गया है-
(क) महात्मा गाँधी को
(ख) पं. जवाहरलाल नेहरू कॉ
(ग) सुभाष चन्द्र बोस को
(घ) सरदार वल्लभ भाई पटेल को
उत्तर:
(क) महात्मा गाँधी को

12. निम्न में से किस राष्ट्रवादी नेता ने सम्पूर्ण देशभर में रॉलट एक्ट के खिलाफ एक अभियान चलाया था?
(क) पं. जवाहरलाल नेहरू
(ख) डॉ. राजेन्द्र प्रसाद
(ग) महात्मा गाँधी
(घ) सरदार पटेल
उत्तर:
(ग) महात्मा गाँधी

13. चरखे के साथ भारतीय राष्ट्रवाद की सर्वाधिक स्थायी पहचान बन गए।
(क) डॉ. राजेन्द्र प्रसाद
(ख) पं. जवाहरलाल नेहरू
(ग) महात्मा गाँधी
(घ) शहिद अमीन
उत्तर:
(ग) महात्मा गाँधी

14. निम्न में से किस वर्ष अंग्रेज सदस्यों वाला साइमन कमीशन भारत आया था –
(क) सन् 1928
(ग) सन् 1936
(ख) सन् 1930
(घ) सन् 1942
उत्तर:
(क) सन् 1928

JAC Class 12 History Important Questions Chapter 13 महात्मा गांधी और राष्ट्रीय आंदोलन : सविनय अवज्ञा और उससे आगे

15. कांग्रेस ने पूर्ण स्वराज्य की घोषणा अपने किस अधिवेशन में की थी?
(क) सूरत अधिवेशन
(ख) लाहौर अधिवेशन
(ग) बम्बई अधिवेशन
(घ) नागपुर अधिवेशन
उत्तर:
(क) सूरत अधिवेशन

16. गोलमेज सम्मेलन का आयोजन हुआ –
(क) लखनऊ में
(ग) लन्दन में
(ख) कोलम्बो में
(घ) जयपुर
उत्तर:
(ग) लन्दन में

17. निम्न में से किस वर्ष नया गवर्नमेंट ऑफ इण्डिया एक्ट पारित हुआ?
(क) सन् 1935
(ग) सन् 1940
(ख) सन् 1936
(घ) सन् 1956
उत्तर:
(क) सन् 1935

18. निम्न में से किस गोलमेज सम्मेलन में महात्मा गाँधी ने निम्न जातियों के लिए पृथक निर्वाचिका की माँग का विरोध किया –
(क) प्रथम
(ख) द्वितीय
(ग) तृतीय
(घ) चतुर्थ
उत्तर:
(ख) द्वितीय

19. “मुझे सम्राट का सर्वोच्च मन्त्री इसलिए नहीं नियुक्त किया गया है कि मैं ब्रिटिश साम्राज्य के टुकड़े-टुकड़े कर दूँ।” यह कथन किस ब्रितानी प्रधानमन्त्री का था?
(क) विंस्टन चर्चिल
(ख) मारग्रेट पैचर
(ग) जेम्स मैकडोनाल्ड
(घ) लार्ड वेवेल
उत्तर:
(क) विंस्टन चर्चिल

20. ‘अंग्रेजों भारत छोड़ो आन्दोलन कब प्रारम्भ हुआ?
(क) अप्रैल, 1941 में
(ख) अगस्त, 1942 में
(ग) जनवरी, 1943 में
(घ) अक्टूबर, 1946 में
उत्तर:
(ख) अगस्त, 1942 में

21. निम्न में से किस राजनेता द्वारा ‘ए बंच ऑफ लेटर्स’ का संकलन किया गया-
(क) जवाहरलाल नेहरू द्वारा
(ख) महात्मा गाँधी द्वारा
(ग) डॉ. राजेन्द्र प्रसाद द्वारा
(घ) डॉ. राधाकृष्णन द्वारा
उत्तर:
(क) जवाहरलाल नेहरू द्वारा

JAC Class 12 History Important Questions Chapter 13 महात्मा गांधी और राष्ट्रीय आंदोलन : सविनय अवज्ञा और उससे आगे

रिक्त स्थानों की पूर्ति कीजिए –

1. मोहनदास करमचंद गाँधी दो दशक रहने के बाद …………… में अपनी गृहभूमि भारत वापस आ गए।
2. गाँधीजी ने सन् ……………. में भारत छोड़ा था।
3. ……………. भी गाँधीजी की तरह गुजराती मूल के लंदन में प्रशिक्षित वकील थे।
4. गांधीजी ने …………….. एक्ट के खिलाफ अभियान चलाया।
5. ……………. आन्दोलन मुहम्मद अली और शौकत अली के नेतृत्व में भारतीय मुसलमानों का एक आन्दोलन था।
6. सरकारी आँकड़ों के मुताबिक 1921 में …………… हुई जिनमें ‘लाख श्रमिक शामिल थे।
7. …………… के विद्रोह के बाद पहली बार ……………… आन्दोलन के परिणामस्वरूप अंग्रेजी राज की नींव हिल गई।
8. फरवरी 1922 में किसानों के एक समूह ने संयुक्त प्रान्त के ……………. पुरवा में एक पुलिस स्टेशन पर आक्रमण कर उसमें आग लगा दी।
9. गाँधीजी को मार्च, 1922 में ……………. के आरोप में गिरफ्तार कर लिया गया
10. 1929 में दिसम्बर के अन्त में कांग्रेस ने अपना वार्षिक अधिवेशन …………… शहर में किया।
उत्तर:
1 जनवरी, 1915
2. 1893
3 मोहम्मद अली जिन्ना
4. गॅलेट
5. खिलाफत
6. 396
6 7.1857, असहयोग
8. चौरी-चौरा
9 राजद्रोह
10 लाहौर।

अतिलघूत्तरात्मक प्रश्न

प्रश्न 1.
कौनसा कानून ‘काला कानून’ कहलाता था ?
उत्तर:
रॉलेट एक्ट।

प्रश्न 2.
गांधीजी के लिए ‘महात्मा’ शब्द का प्रयोग किसने किया?
उत्तर:
गुरु रवीन्द्रनाथ टैगोर ने।

प्रश्न 3.
खिलाफत का उद्देश्य क्या था?
उत्तर:
खिलाफत का उद्देश्य था-खलीफा पद की पुनर्स्थापना करना।

प्रश्न 4.
चौरी-चौरा कांड कहाँ हुआ था?
उत्तर:
उत्तरप्रदेश के गोरखपुर जिले के चौरी-चौरा नामक जगह पर।

प्रश्न 5.
दांडी मार्च कब और कितने सदस्यों के साथ गांधीजी ने शुरू किया?
उत्तर:
12 मार्च, 1930 को 78 सदस्यों के साथ।

प्रश्न 6.
तृतीय गोलमेज सम्मेलन के विचार-विमर्श की परिणति किस कानून के रूप में सामने आयी ?
उत्तर:
1935 का भारत सरकार कानून।

JAC Class 12 History Important Questions Chapter 13 महात्मा गांधी और राष्ट्रीय आंदोलन : सविनय अवज्ञा और उससे आगे

प्रश्न 7.
महात्मा गाँधी के तीन सत्याग्रह आन्दोलनों का उल्लेख कीजिए जो असहयोग आन्दोलन से पहले प्रारम्भ किये गये थे।
उत्तर:

  • चंपारन सत्याग्रह
  • अहमदाबाद सत्याग्रह
  • खेड़ा सत्याग्रह।

प्रश्न 8.
हिन्दू-मुस्लिम एकता के लिए गाँधीजी द्वारा किस आन्दोलन का समर्थन किया गया?
उत्तर:
खिलाफत आन्दोलन का।

प्रश्न 9.
गाँधीजी द्वारा कौन से गोलमेज सम्मेलन में भाग लिया गया?
उत्तर:
द्वितीय गोलमेज सम्मेलन में।

प्रश्न 10.
पूर्ण स्वराज्य का प्रस्ताव काँग्रेस के किस अधिवेशन में जारी किया गया?
उत्तर:
1929 के लाहौर अधिवेशन में।

प्रश्न 11.
1905-07 के स्वदेशी आन्दोलन ने कौन से तीन उग्रवादी काँग्रेसी नेताओं को जन्म दिया?
उत्तर:

  • बाल गंगाधर तिलक
  • विपिनचन्द्र पाल और
  • लाला लाजपत राय।

प्रश्न 12.
1915 से पहले के काँग्रेस के किन्हीं दो प्रमुख उदारवादी नेताओं के नाम लिखिये।
उत्तर:
(1) गोपालकृष्ण गोखले
(2) सुरेन्द्रनाथ बनर्जी।

प्रश्न 13.
भारत में गाँधीजी की पहली महत्त्वपूर्ण सार्वजनिक उपस्थिति कब हुई ?
उत्तर:
फरवरी, 1916 में बनारस हिन्दू विश्वविद्यालय के उद्घाटन समारोह में।

प्रश्न 14.
असहयोग आन्दोलन का क्या प्रभाव हुआ?
उत्तर:
असहयोग आन्दोलन के बाद भारतीय राष्ट्रवाद जन-आन्दोलन में बदल गया।

प्रश्न 15.
16 अगस्त, 1946 को जिला ने पाकिस्तान की स्थापना की माँग के समर्थन में कौनसा दिवस मनाने का आह्वान किया?
उत्तर:
प्रत्यक्ष कार्यवाही दिवस।

JAC Class 12 History Important Questions Chapter 13 महात्मा गांधी और राष्ट्रीय आंदोलन : सविनय अवज्ञा और उससे आगे

प्रश्न 16.
भारतीय राष्ट्रीय आन्दोलन के अन्तर्गत गाँधीजी द्वारा संचालित आन्दोलनों के नाम लिखिए।
उत्तर:

  • असहयोग आन्दोलन
  • सविनय अवज्ञा आन्दोलन
  • भारत छोड़ो आन्दोलन।

प्रश्न 17.
गाँधीजी ने असहयोग आन्दोलन कब स्थगित किया?
उत्तर:
12 फरवरी, 1922 को

प्रश्न 18.
गांधीजी ने असहयोग आन्दोलन क्यों स्थगित कर दिया?
उत्तर:
चौरी-चौरा की हिंसात्मक घटना के कारण।

प्रश्न 19.
किस उद्योगपति ने राष्ट्रीय आन्दोलन का खुला समर्थन किया?
उत्तर:
जी. डी. बिड़ला ने।

प्रश्न 20.
गांधीजी किन नामों से पुकारे जाते थे?
उत्तर:
‘गाँधी बाबा’, ‘गाँधी महाराज’, ‘महात्मा’ के नामों से।

प्रश्न 21.
कांग्रेस ने अपना वार्षिक अधिवेशन लाहौर में कब किया?
उत्तर:
1929 में दिसम्बर के अन्त में।

प्रश्न 22.
दिसम्बर, 1929 में कांग्रेस का लाहौर अधिवेशन किसकी अध्यक्षता में आयोजित किया गया?
उत्तर:
पं. जवाहरलाल नेहरू की अध्यक्षता में।

प्रश्न 23.
सम्पूर्ण देश में ‘स्वतन्त्रता दिवस’ कब मनाया गया?
उत्तर:
26 जनवरी, 19301

JAC Class 12 History Important Questions Chapter 13 महात्मा गांधी और राष्ट्रीय आंदोलन : सविनय अवज्ञा और उससे आगे

प्रश्न 24.
गाँधीजी ने दाण्डी मार्च कब शुरू किया और किस स्थान से किया ?
उत्तर:
(1) 12 मार्च, 1930 को
(2) साबरमती आश्रम से।

प्रश्न 25.
गांधीजी ने नमक सत्याग्रह कब शुरू किया ?
उत्तर:
6 अप्रैल, 1930 को दाण्डी यात्रा की समाप्ति पर नमक बनाकर।

प्रश्न 26.
नमक- कर कानून को तोड़ने की घोषणा गाँधीजी ने कब की थी और कहाँ की थी?
उत्तर:
(1) 5 अप्रैल, 1930 को
(2) झण्डी में

प्रश्न 27.
नमक सत्याग्रह में किस महिला ने बढ़- चढ़कर हिस्सा लिया था?
उत्तर:
कमलादेवी चट्टोपाध्याय ने।

JAC Class 12 History Important Questions Chapter 13 महात्मा गांधी और राष्ट्रीय आंदोलन : सविनय अवज्ञा और उससे आगे

प्रश्न 28.
किस महिला ने गाँधीजी को सलाह दी थी कि वह अपने आन्दोलन को पुरुषों तक ही सीमित न रखें?
उत्तर:
कमलादेवी चट्टोपाध्याय ने।

प्रश्न 29.
पहला गोलमेज सम्मेलन कब आयोजित किया गया और कहाँ किया गया?
उत्तर:
(1) नवम्बर 1930 में।
(2) लन्दन में।

प्रश्न 30.
गाँधी-इरविन समझौता कब हुआ ?
उत्तर:
5 मार्च, 1931 को

प्रश्न 31.
दूसरा गोलमेज सम्मेलन कब और कहाँ आयोजित किया गया?
उत्तर:
(1) दिसम्बर, 1931 में
(2) लन्दन में।

प्रश्न 32.
गाँधीजी किस गोलमेज सम्मेलन में शामिल हुए?
उत्तर:
दूसरे गोलमेज सम्मेलन में।

प्रश्न 33.
गवर्नमेंट ऑफ इण्डिया एक्ट कब पारित हुआ?
उत्तर:
1935 में

प्रश्न 34.
1937 के आम चुनाव में कितने प्रान्तों में से कांग्रेस के प्रधानमंत्री सत्ता में आए ?
उत्तर:
11 प्रान्तों में से 8 प्रान्तों के प्रधानमंत्री।

प्रश्न 35.
कांग्रेसी मन्त्रिमण्डलों ने कब त्याग-पत्र दे दिया ?
उत्तर:
अक्टूबर, 1939 में।

प्रश्न 36.
दलितों को पृथक् निर्वाचिका का अधिकार दिए जाने की माँग किस दलित नेता ने की थी?
उत्तर:
डॉ. बी. आर. अम्बेडकर।

JAC Class 12 History Important Questions Chapter 13 महात्मा गांधी और राष्ट्रीय आंदोलन : सविनय अवज्ञा और उससे आगे

प्रश्न 37.
मुस्लिम लीग ने पाकिस्तान के निर्माण की माँग कब की थी?
उत्तर:
मार्च, 1940 में

प्रश्न 38.
“मैं सम्राट का सर्वोच्च मन्त्री इसलिए नहीं नियुक्त किया गया हूँ कि मैं ब्रिटिश साम्राज्य के टुकड़े- टुकड़े कर दूँ।” यह किसका कथन था ?
उत्तर:
ब्रिटिश प्रधानमंत्री विंस्टन चर्चिल का।

प्रश्न 39.
सर स्टेफर्ड क्रिप्स भारत कब आए?
उत्तर:
मार्च, 1942 में।

प्रश्न 40.
ब्रिटिश सरकार ने स्टेफर्ड क्रिप्स को भारत क्यों भेजा था ?
उत्तर:
राजनीतिक गतिरोध को दूर करने हेतु।

प्रश्न 41.
भारत छोड़ो आन्दोलन’ कब व किसने आरम्भ किया?
उत्तर:
(1) 8 अगस्त, 1942 को
(2) गाँधीजी ने।

प्रश्न 42.
कांग्रेस के किस नेता ने भारत छोड़ो आन्दोलन में भूमिगत प्रतिरोध गतिविधियों में सक्रिय भाग लिया ?
उत्तर:
जय प्रकाश नारायण ने।

प्रश्न 43.
ऐसे दो स्थानों के नाम लिखिए जहाँ आन्दोलनकारियों ने अपनी स्वतन्त्र सरकारें स्थापित कर ली थीं।
उत्तर:
(1) सतास
(2) मेदिनीपुर।

प्रश्न 44.
कैबिनेट मिशन भारत कब आया ?
उत्तर:
23 मार्च, 1946

प्रश्न 45.
पाकिस्तान के निर्माण के लिए मुस्लिम लीग ने ‘प्रत्यक्ष कार्यवाही दिवस’ का कब आह्वान किया?
उत्तर:
16 अगस्त, 19461

प्रश्न 46.
दिल्ली में संविधान सभा के अध्यक्ष ने गाँधीजी को किसकी उपाधि प्रदान की थी?
उत्तर:
‘राष्ट्रपिता’ की।

JAC Class 12 History Important Questions Chapter 13 महात्मा गांधी और राष्ट्रीय आंदोलन : सविनय अवज्ञा और उससे आगे

प्रश्न 47.
गाँधीजी के जीवनी लेखक कौन थे?
उत्तर:
जी. डी. तेन्दुलकर।

प्रश्न 48.
खेड़ा में गाँधीजी ने किसानों के लिए क्या किया?
उत्तर:
गांधीजी ने खेड़ा में फसल चौपट होने पर राज्य सरकार से किसानों का लगान माफ करने की माँग की।

प्रश्न 49.
दाण्डी मार्च पर टिप्पणी लिखिए।
उत्तर:
गांधीजी ने 12 मार्च, 1930 को नमक कानून हेतु साबरमती आश्रम से दाण्डी की यात्रा 24 दिनों में पूरी की।

प्रश्न 50.
“महात्मा गाँधी जन नेता थे।” इस कथन की विवेचना कीजिये।
उत्तर:
गाँधीजी के नेतृत्व में राष्ट्रीय आन्दोलन में हजारों किसानों, मजदूरों, कारीगरों, बुद्धिजीवियों ने भाग लिया। वे आम लोगों की तरह रहते थे।

प्रश्न 51.
रॉलेट एक्ट क्या था?
उत्तर:
इसके अनुसार किसी को भी बिना कारण बताए जेल में अनिश्चित काल के लिए बन्द किया जा सकता था।

प्रश्न 52.
दक्षिणी अफ्रीका से लौटने पर किसने महात्मा गाँधी को क्या सलाह दी थी?
उत्तर:
गोपाल कृष्ण गोखले ने गाँधीजी को एक वर्ष तक ब्रिटिश भारत की यात्रा करने की सलाह दी थी।

प्रश्न 53.
द्वितीय गोलमेज सम्मेलन सफल क्यों नहीं हुआ?
उत्तर:
भारत की स्वतन्त्रता की माँग को स्वीकार न करने तथा साम्प्रदायिकता की समस्या का समाधान न होने के कारण।

प्रश्न 54.
दक्षिण अफ्रीका ने ही गाँधीजी को ‘महात्मा’ बनाया। यह कथन किस इतिहासकार का है?
उत्तर:
चन्द्रन देवनेसन ने।

प्रश्न 55.
खिलाफत आन्दोलन क्या था?
उत्तर:
खिलाफत आन्दोलन तुर्की के खलीफा की पुनर्स्थापना के लिए भारतीय मुसलमानों का एक आन्दोलन था।

प्रश्न 56.
असहयोग आन्दोलन के दो कारण बताइये।
उत्तर:
(1) रॉलेट एक्ट’ के कारण भारतीयों में असन्तोष था
(2) जलियाँवाला बाग हत्याकाण्ड के कारण भारतीयों में आक्रोश व्याप्त था।

प्रश्न 57.
चौरीचौरा की घटना क्या थी?
उत्तर:
फरवरी, 1922 में सत्याग्रहियों द्वारा चौरी- चौरा (गोरखपुर) में पुलिस थाने में आग लगा दी जिससे 22 पुलिसकर्मी (एक थानेदार तथा इक्कीस सिपाही) जलकर मर गये।

JAC Class 12 History Important Questions Chapter 13 महात्मा गांधी और राष्ट्रीय आंदोलन : सविनय अवज्ञा और उससे आगे

नाम
प्रश्न 58.
गाँधीजी के चार प्रमुख सहयोगियों के लिखिए जिन्होंने भारतीय राष्ट्रीय आन्दोलन में सक्रिय भाग लिया।
उत्तर:

  • सरदार वल्लभ भाई पटेल
  • सुभाष चन्द्र बोस
  • जवाहर लाल नेहरू
  • अबुल कलाम आजाद।

प्रश्न 59.
असहयोग आन्दोलन के दो प्रभाव बताइये।
उत्तर:
(1) राष्ट्रीय आन्दोलन का क्षेत्र व्यापक हो गया।
(2) राष्ट्रवाद का सन्देश भारत के सुदूर भागों तक फैल गया।

प्रश्न 60.
गांधीजी के दो सामाजिक सुधारों का उल्लेख कीजिये
उत्तर:
(1) छुआछूत के उन्मूलन पर बल देना।
(2) बाल विवाह की कुप्रथा को समाप्त करने पर बल देना ।

प्रश्न 61.
गांधीजी ने चरखा चलाने पर क्यों बल दिया? कोई दो तर्क दीजिये।
अथवा
चरखा राष्ट्रवाद का प्रतीक क्यों चुना गया ?
उत्तर:
(1) चरखा गरीबों को पूरक आय दे सकता था
(2) यह लोगों को स्वावलम्बी बना सकता था।

प्रश्न 62.
1929 में दिसम्बर के अना में आयोजित कांग्रेस का लाहौर अधिवेशन किन दो दृष्टियों से महत्त्वपूर्ण था?
उत्तर:
(1) जवाहरलाल नेहरू का अध्यक्ष के रूप मैं चुनाव
(2) ‘पूर्ण स्वराज’ की उद्घोषणा।

प्रश्न 63.
‘स्वतन्त्रता दिवस’ मनाए जाने के तुरन्त बाद महात्मा गाँधी ने क्या घोषणा की थी?
उत्तर:
गाँधीजी ने नमक कानून तोड़ने के लिए एक यात्रा का नेतृत्व करने की घोषणा की।

प्रश्न 64.
गाँधीजी ने नमक कानून को किसकी संज्ञा दी थी?
उत्तर:
ब्रिटिश भारत के सर्वाधिक घृणित कानूनों में से एक कानून की।

JAC Class 12 History Important Questions Chapter 13 महात्मा गांधी और राष्ट्रीय आंदोलन : सविनय अवज्ञा और उससे आगे

प्रश्न 65.
गाँधीजी ने नमक-कानून के विरुद्ध आन्दोलन करने का क्यों निश्चय किया?
उत्तर:
(1) घरेलू प्रयोग के लिए भी नमक बनाना निषिद्ध था।
(2) लोगों को ऊंचे दाम पर नमक खरीदने के लिए बाध्य किया गया।

प्रश्न 66.
गाँधीजी के अनुसार नमक विरोध का प्रतीक क्यों था?.
अथवा
गाँधीजी ने नमक सत्याग्रह क्यों शुरू किया?
उत्तर:
(1) नमक एकाधिकार लोगों को ग्राम उद्योग से वंचित करता था
(2) भूखे लोगों से हजार प्रतिशत से अधिक की वसूली करना।

प्रश्न 67.
नमक सत्याग्रह के दो कार्यक्रमों का उल्लेख कीजिये।
उत्तर:
(1) वकीलों द्वारा ब्रिटिश अदालतों का बहिष्कार करना
(2) विद्यार्थियों द्वारा सरकारी शिक्षा संस्थानों का बहिष्कार करना।

प्रश्न 68.
नमक सत्याग्रह के दौरान गाँधीजी ने क्या आह्वान किया था?
उत्तर:
(1) स्थानीय अधिकारी सरकारी नौकरियाँ छोड़कर स्वतन्त्रता संघर्ष में शामिल हों ।
(2) ऊंची जाति के लोग दलितों की सेवा करें।

प्रश्न 69.
अमेरिकी समाचार पत्रिका ‘टाइम’ ने क्या कहकर गाँधीजी का मजाक उड़ाया था?
उत्तर:
पत्रिका ने गाँधीजी के ‘तकुए जैसे शरीर तथा ‘मकड़ी जैसे पेडू’ का मजाक उड़ाया था।

प्रश्न 70.
नमक सत्याग्रह की दो विशेषताएँ बताइये।
उत्तर:
(1) यह पहला राष्ट्रीय आन्दोलन था जिसमें महिलाओं ने भी बड़ी संख्या में भाग लिया।
(2) यह आन्दोलन पूर्ण अहिंसात्मक था।

प्रश्न 71.
जालियाँवाला बाग कहाँ स्थित है?
उत्तर:
अमृतसर में

प्रश्न 72.
खिलाफत आन्दोलन का उद्देश्य क्या था?
उत्तर:
खलीफा पद की पुनर्स्थापना करना।

प्रश्न 73.
चौरी-चौरा काण्ड कर्ब व कहाँ हुआ था?
उत्तर:
5 फरवरी, 1922 में गोरखपुर।

JAC Class 12 History Important Questions Chapter 13 महात्मा गांधी और राष्ट्रीय आंदोलन : सविनय अवज्ञा और उससे आगे

प्रश्न 74.
साइमन कमीशन भारत क्यों आया?
उत्तर:
भारतीय उपनिवेश को स्थितियों की जाँच- पड़ताल करने के लिए।

प्रश्न 75.
बरदौली में किसान सत्याग्रह किस वर्ष हुआ?
उत्तर:
सन् 1928 में।

प्रश्न 76.
26 जनवरी 1930 को स्वतन्त्रता दिवस मनाए जाने के पश्चात् गाँधीजी ने क्या घोषणा की?
उत्तर:
गाँधीजी ने यह घोषणा की कि वे ब्रिटिश भारत के सर्वाधिक घृणित कानून को तोड़ने के लिए एक यात्रा का नेतृत्व करेंगे।

प्रश्न 77.
गाँधीजी ने अपनी नमक यात्रा की पूर्व सूचना किस अंग्रेज वायसराय को दी थी?
उत्तर:
लॉर्ड इर्बिन को।

प्रश्न 78.
अखिल बंगाल सविनय अवज्ञा परिषद् का गठन किसने किया?
उत्तर:
जे. एम. सेन गुप्ता ने

प्रश्न 79.
गाँधी-इरविन समझौते की दो शर्तों का उल्लेख कीजिए।
उत्तर:
(1) गाँधीजी सविनय अवज्ञा आन्दोलन को वापस ले लेंगे।
(2) ब्रिटिश सत्याग्रहियों को मुक्त कर देगी।
सरकार बन्दी बनाए गए

प्रश्न 80.
गाँधीजी को लन्दन से खाली हाथ क्यों लौटना पड़ा?
उत्तर:
ब्रिटिश सरकार ने भारत को स्वतन्त्रता प्रदान करने का आश्वासन नहीं दिया तथा वह साम्प्रदायिकता की समस्या हल नहीं कर सकी।

प्रश्न 81.
गाँधीजी ने निम्न जातियों के लिए पृथक् निर्वाचिका की मांग का विरोध क्यों किया ?
उत्तर:
पृथक् निर्वाचिका की माँग से दलितों का समाज की मुख्य धारा में एकीकरण नहीं हो पायेगा।

प्रश्न 82.
कांग्रेसी मन्त्रिमण्डलों ने क्यों त्याग-पत्र दे दिया?
उत्तर:
ब्रिटिश सरकार ने द्वितीय युद्ध के बाद भारत को स्वतन्त्रता देने की कांग्रेस की माँग को अस्वीकार कर दिया था।

प्रश्न 83.
दूसरे गोलमेज सम्मेलन में गाँधीजी को किन तीन दलों ने चुनौती दी थी?
उत्तर:
(1) मुस्लिम लीग
(2) राजे-रजवाड़े
(3) डॉ. बी. आर. अम्बेडकर।

JAC Class 12 History Important Questions Chapter 13 महात्मा गांधी और राष्ट्रीय आंदोलन : सविनय अवज्ञा और उससे आगे

प्रश्न 84.
कांग्रेस ने मुस्लिम लीग के किस सिद्धान्त को कभी स्वीकार नहीं किया?
उत्तर:
दो राष्ट्र सिद्धान्त’।

प्रश्न 85.
किस व्यक्ति ने गाँधीजी की हत्या की भी और कब?
उत्तर:
30 जनवरी 1948 को नाथूराम गोडसे ने।

प्रश्न 86.
अमेरिका की टाइम पत्रिका’ ने गांधीजी के बलिदान की तुलना किससे की थी?
उत्तर:
अब्राहम लिंकन के बलिदान से।

प्रश्न 87.
किस समाचार-पत्र में गाँधीजी उन पत्रों को प्रकाशित करते थे, जो उन्हें लोगों से मिलते थे?
उत्तर:
‘हरिजन’ में।

प्रश्न 88.
पं. जवाहर लाल नेहरू ने राष्ट्रीय आन्दोलन के दौरान उन्हें लिखे गए पत्रों का एक संकलन तैयार किया था। उसे उन्होंने किस नाम से प्रकाशित किया?
उत्तर:
‘ए बंच ऑफ ओल्ड लेटर्स’ (पुराने पत्रों का पुलिन्दा) ।

लघुत्तरात्मक प्रश्न

प्रश्न 1.
गाँधीजी के दक्षिण अफ्रीका में उनके द्वारा किए गए कार्यों को संक्षेप में लिखिए।
उत्तर:
गाँधीजी ने दक्षिण अफ्रीका की सरकार के रंग भेदभाव के विरोध में सत्याग्रह का सहारा लिया। उन्होंने वहाँ विभिन्न धर्मों के बीच सौहार्द बढ़ाने का प्रयास किया। गाँधीजी ने उच्च जातीय भारतीयों से दलितों एवं महिलाओं के प्रति भेदभाव का व्यवहार न करने के लिए चेतावनी दी। वास्तव में दक्षिण अफ्रीका ही उनके सत्याग्रह की प्रथम पाठशाला बना तथा उसने ही उन्हें ‘महात्मा’ बना दिया।

प्रश्न 2.
गाँधीजी के रचनात्मक कार्यों पर एक टिप्पणी लिखिये।
उत्तर:
गांधीजी ने बुनियादी शिक्षा, ग्राम उद्योग संघ, तालीमी संघ और गौ रक्षा संप स्थापित किये। उन्होंने समाज में फैली शोषण व्यवस्था को समाप्त करने पर बल दिया। उन्होंने कुटीर उद्योगों के प्रोत्साहन के लिए कार्य किया। चरखा और खादी उनके आर्थिक तंत्र के मुख्य आधार थे। उन्होंने दलितोद्धार, शराबबन्दी, नारी सशक्तिकरण तथा हिन्दू-मुस्लिम एकता को प्रोत्साहन दिया।

प्रश्न 3.
“दक्षिण अफ्रीका ने ही गाँधीजी को महात्मा बनाया।” यह कथन किसका है? इसके पक्ष में तर्क दीजिए।
उत्तर:
इतिहासकार चंदन देवनेसन ने कहा था कि गाँधीजी ने दक्षिण अफ्रीका में ही पहली बार अहिंसात्मक विरोध के अपने विशेष तरीकों की प्रयोग किया जिसे सत्याग्रह का नाम दिया गया। यहीं पर उन्होंने विभिन्न धर्मों के मध्य सद्भावना बढ़ाने का प्रयास किया तथा उच्च जातीय भारतीयों को दलितों एवं महिलाओं के प्रति भेदभाव के व्यवहार के लिए चेतावनी दी।

प्रश्न 4.
गाँधीजी ने खिलाफत आन्दोलन को असहयोग आन्दोलन का अंग क्यों बनाया?
उत्तर:
गांधीजी ने असहयोग आन्दोलन को विस्तार एवं मजबूती देने के लिए खिलाफत आन्दोलन को इसका अंग बनाया। उन्हें यह विश्वास था कि असहयोग को खिलाफत के साथ मिलाने से भारत के दो प्रमुख धार्मिक समुदाय हिन्दू और मुसलमान आपस में मिलकर औपनिवेशिक शासन का अन्त कर देंगे।

JAC Class 12 History Important Questions Chapter 13 महात्मा गांधी और राष्ट्रीय आंदोलन : सविनय अवज्ञा और उससे आगे

प्रश्न 5.
महात्मा गाँधी के अमरीकी जीवनी लेखक लुई फिशर ने गाँधीजी के बारे में क्या लिखा है?
उत्तर:
लुई फिशर ने लिखा कि “असहयोग भारत और गाँधीजी के जीवन के एक युग का ही नाम हो गया। असहयोग शान्ति की दृष्टि से नकारात्मक किन्तु प्रभाव की दृष्टि से सकारात्मक था। इसके लिए प्रतिवाद, परित्याग एवं स्व- अनुशासन आवश्यक थे। यह स्वशासन के लिए एक प्रशिक्षण था।”

प्रश्न 6.
दलितों के लिए पृथक् निर्वाचन क्षेत्र का विरोध गाँधीजी द्वारा क्यों किया गया था? स्पष्ट कीजिए।
उत्तर:
गोलमेज सम्मेलन के दौरान गांधीजी ने दमित वर्गों के लिए पृथक निर्वाचन क्षेत्र के प्रस्ताव का विरोध करते हुए कहा था, “अस्पृश्यों के लिए पृथक् निर्वाचिका का प्रावधान करने से उनकी दासता स्थायी रूप ले लेगी। क्या आप चाहते हैं कि ‘अस्पृश्य’ हमेशा ‘अस्पृश्य’ ही बने रहें? पृथक् निर्वाचिका से उनके प्रति कलंक का यह भाव अधिक मजबूत हो जायेगा। जरूरत इस बात की है कि अस्पृश्यतां का विनाश किया जाए।

प्रश्न 7.
1939 में कॉंग्रेस मंत्रिमण्डल ने सरकार से इस्तीफा क्यों दिया?
उत्तर:
1937 में सीमित मताधिकार के आधार पर हुए चुनावों में काँग्रेस की 11 में से 8 प्रांतों में सरकारें बनीं। सितम्बर, 1939 में दूसरा विश्व युद्ध शुरू हो गया। महात्मा गाँधी और जवाहरलाल नेहरू ने कहा कि अगर अंग्रेज युद्ध की समाप्ति पर स्वतंत्रता देने को राजी हों तो काँग्रेस उनके युद्ध प्रयासों में सहायता दे सकती है सरकार ने कांग्रेस का प्रस्ताव खारिज कर दिया। इसके विरोध में काँग्रेस ममण्डलों ने अक्टूबर, 1939 में इस्तीफा दे दिया।

प्रश्न 8.
प्रत्यक्ष कार्यवाही दिवस’ का आह्वान किसने किया था और इसका क्या परिणाम रहा?
उत्तर:
कैबिनेट मिशन की असफलता के बाद जिना ने पाकिस्तान की स्थापना के लिए लीग की माँग के समर्थन में एक ‘प्रत्यक्ष कार्यवाही दिवस’ का आह्वान किया। इसके लिए 16 अगस्त, 1946 का दिन तय किया गया। उसी दिन कलकत्ता में खूनी संघर्ष शुरू हो गया। यह हिंसा कलकत्ता से शुरू होकर ग्रामीण बंगाल, बिहार, संयुक्त प्रांत तथा पंजाब तक फैल गई। कुछ स्थानों पर हिन्दुओं ने मुसलमानों को तथा मुसलमानों ने हिन्दुओं को अपना निशाना बनाया।

प्रश्न 9.
“महात्मा गाँधी भारतीय राष्ट्र के पिता थे।” कैसे?
उत्तर:
गाँधीजी भारतीय स्वतन्त्रता संघर्ष में भाग लेने वाले सभी नेताओं में सर्वाधिक प्रभावशाली और सम्मानित थे। उन्होंने किसानों, मजदूरों, कारीगरों, व्यापारियों, बुद्धिजीवियों, हिन्दुओं, मुसलमानों, सभी भारतीयों को संगठित किया और उनमें राष्ट्रीयता की भावना का प्रसार किया। उन्होंने 1920 से 1947 तक असहयोग आन्दोलन, सविनय अवज्ञा आन्दोलन एवं भारत छोड़ो आन्दोलन का नेतृत्व किया और सम्पूर्ण भारत में राष्ट्रवाद का और स्वाधीनता प्राप्त करने का जोश भर दिया।

प्रश्न 10.
गाँधी इर्विन समझौता कब हुआ? इसकी शर्तें बताइए। रैडिकल राष्ट्रवादियों ने गाँधी इर्विन समझौते की आलोचना क्यों की?
उत्तर:
गाँधी इर्विन समझौता 5 मार्च 1931 को हुआ था। इस समझौते में निम्न बातों पर सहमति बनी –

  1. सविनय अवज्ञा आन्दोलन को वापस लेना
  2. समस्त राजनैतिक कैदियों की रिहाई
  3. तटीय क्षेत्रों में नमक उत्पादन की अनुमति देना।

रैडिकल राष्ट्रवादियों ने गांधी इर्विन समझौते की आलोचना की। क्योंकि गाँधीजी अंग्रेजी वायसराय से भारतीयों के लिए राजनीतिक स्वतन्त्रता का आश्वासन प्राप्त नहीं कर पाये थे। गाँधीजी को इस सम्भावित लक्ष्य की प्राप्ति के लिए केवल वार्ताओं का आश्वासन मिला था।

प्रश्न 11.
क्रिप्स मिशन भारत कब आया ? क्रिप्स वार्ता क्यों टूट गयी?
उत्तर:
क्रिप्स मिशन मार्च, 1942 में भारत आया। सर स्टेफर्ड क्रिप्स के साथ वार्ता में कांग्रेस ने इस बात पर बल दिया कि यदि धुरी शक्तियों के विरुद्ध ब्रिटेन कांग्रेस का समर्थन चाहता है तो उसे व्यवसाय की कार्यकारी परिषद में किसी भारतीय को रक्षा सदस्य नियुक्त करना होगा। ब्रिटिश सरकार द्वारा असहमति देने पर यह वार्ता टूट गयी।

प्रश्न 12.
गाँधीजी के प्रारम्भिक जीवन तथा दक्षिण अफ्रीका में उनके कार्यकलापों पर संक्षिप्त टिप्पणी लिखिए।
उत्तर:
सन् 1915 से पूर्व लगभग 22 वर्षों तक मोहनदास करमचंद गाँधी (महात्मा गाँधी) विदेशों में रहे। इन वर्षों का अधिकांश हिस्सा उन्होंने दक्षिण अफ्रीका में व्यतीत किया। गाँधीजी एक वकील के रूप में दक्षिण अफ्रीका गए थे और बाद में वे इस क्षेत्र के भारतीय समुदायों के नेता बन गए। गाँधीजी ने दक्षिण अफ्रीका में प्रथम बार वहाँ की सरकार की रंग-भेद एवं जातीय भेद के विरुद्ध सत्याग्रह के रूप में अपना अहिंसात्मक तरीके से विरोध किया तथा विभिन्न धर्मों के मध्य सौहार्द बढ़ाने का प्रयास किया।

JAC Class 12 History Important Questions Chapter 13 महात्मा गांधी और राष्ट्रीय आंदोलन : सविनय अवज्ञा और उससे आगे

प्रश्न 13.
रॉलेट एक्ट सत्याग्रह से ही गाँधीजी एक सच्चे राष्ट्रीय नेता बन गए।” व्याख्या कीजिये।
उत्तर:
गांधीजी ने ‘रॉलेट एक्ट’ के विरुद्ध सम्पूर्ण देश में आन्दोलन चलाया। इसकी सफलता से उत्साहित होकर गाँधीजी ने ब्रिटिश शासन के विरुद्ध ‘असहयोग आन्दोलन’ की माँग कर दी। जो लोग भारतीय उपनिवेशवाद को समाप्त करना चाहते थे, उनसे आग्रह किया गया कि वे स्कूलों, कालेजों तथा न्यायालयों का बहिष्कार करें तथा कर न चुकाएँ। गाँधीजी ने कहा कि असहयोग आन्दोलन के द्वारा भारत एक वर्ष के भीतर स्वराज प्राप्त कर लेगा।

प्रश्न 14.
खिलाफत आन्दोलन’ से आप क्या समझते हैं ?
उत्तर:
खिलाफत आन्दोलन (1919-1920) मुहम्मद अली और शौकत अली के नेतृत्व में संचालित भारतीय मुसलमानों का एक आन्दोलन था। इस आन्दोलन की निम्नलिखित मांगें थीं—पूर्व में आटोमन साम्राज्य के सभी इस्लामी पवित्र स्थानों पर तुर्की सुल्तान अथवा खलीफा का नियन्त्रण बना रहे जजीरात-उल-अरब इस्लामी सम्प्रभुता के अधीन रहे तथा खलीफा के पास काफी क्षेत्र हों। गाँधीजी ने खिलाफत आन्दोलन का समर्थन किया।

प्रश्न 15.
26 जनवरी, 1930 को स्वतन्त्रता दिवस को किस रूप में मनाए जाने की गाँधीजी ने अपील की?
उत्तर:
गाँधीजी ने सुझाव दिया कि 26 जनवरी को सभी गाँवों और शहरों में स्वतन्त्रता दिवस के रूप में मनाया जाए, संगोष्ठियां आयोजित की जाएं तथा राष्ट्रीय ध्वज को फहराए जाने से समारोहों की शुरुआत की जाए। दिन में सूत कातने, दलितों की सेवा करने, हिन्दुओं व मुसलमानों के पुनर्मिलन आदि के कार्यक्रम आयोजित किये जाएं। इस दिन लोग यह प्रतिज्ञा लेंगे कि भारतीय लोगों को भी स्वतन्त्रता प्राप्त करने का अधिकार है।

प्रश्न 16.
गाँधीजी ने नमक सत्याग्रह क्यों शुरू किया?
उत्तर:

  1. प्रत्येक भारतीय के घर में नमक का प्रयोग होता था, परन्तु उन्हें घरेलू प्रयोग के लिए नमक बनाने का अधिकार नहीं था
  2. उन्हें दुकानों से ऊँचे दाम पर नमक खरीदने के लिए बाध्य किया जाता था।
  3. नमक के उत्पादन तथा बिक्री पर सरकार का एकाधिकार था, जो बहुत अलोकप्रिय था।
  4. यह भारतीयों को बहुमूल्य सुलभ ग्राम उद्योग से वंचित करता था।
  5. यह राष्ट्रीय सम्पदा के लिए विनाशकारी था।

प्रश्न 17.
लन्दन में आयोजित द्वितीय गोलमेज सम्मेलन में गाँधीजी के दावे को किन तीन पार्टियों से चुनौती सहन करनी पड़ी?
उत्तर:
(1) मुस्लिम लीग का कहना था कि वह मुस्लिम अल्पसंख्यकों के हित में काम करती है। कांग्रेस मुस्लिम अल्पसंख्यकों के हित में काम नहीं करती है।
(2) राजे-रजवाड़ों का दावा था कि कांग्रेस का उनके नियन्त्रण वाले भू-भाग पर कोई अधिकार नहीं है।
(3) डॉ. भीमराव अम्बेडकर का कहना था कि गांधीजी और कांग्रेस पार्टी दलितों का प्रतिनिधित्व नहीं करते।

JAC Class 12 History Important Questions Chapter 13 महात्मा गांधी और राष्ट्रीय आंदोलन : सविनय अवज्ञा और उससे आगे

प्रश्न 18.
स्टेफर्ड क्रिप्स मिशन क्यों असफल हो गया?
उत्तर:
मार्च, 1942 में ब्रिटिश सरकार ने स्टेफर्ड क्रिप्स को वार्ता हेतु भारत भेजा। कांग्रेस ने इस बात पर बल दिया कि यदि धुरी शक्तियों से भारत की रक्षा के लिए ब्रिटिश सरकार कांग्रेस का समर्थन चाहती है, तो वायसराय को सबसे पहले अपनी कार्यकारी परिषद में किसी भारतीय को एक रक्षा-सदस्य के रूप में नियुक्त करना चाहिए। इसी बात पर वार्ता टूट गई और स्टेफर्ड क्रिप्स खाली हाथ स्वदेश लौट गए।

प्रश्न 19.
नमक सत्याग्रह का महत्त्व प्रतिपादित कीजिये।
उत्तर:

  1. इस घटना ने गाँधीजी को संसार भर में प्रसिद्ध कर दिया।
  2. इस सत्याग्रह में भारतीय महिलाओं ने भारी संख्या में हिस्सा लिया। स्वियों ने शराब की दुकानों तथा विदेशी वस्त्रों की दुकानों पर धरना दिया और अपने आप को गिरफ्तारी के लिए पेश किया।
  3. इस सत्याग्रह से अंग्रेजों को पता चल गया कि अब उनका राज बहुत दिन नहीं टिक सकेगा और उन्हें भारतीयों को भी सत्ता में हिस्सा देना पड़ेगा।

प्रश्न 20.
महात्मा गाँधी ने असहयोग आन्दोलन वापस क्यों लिया?
उत्तर:
5 फरवरी, 1922 को उत्तरप्रदेश के गोरखपुर जिले में स्थित चौरी-चौरा नामक गाँव में पुलिस ने कांग्रेस के सत्याग्रहियों पर गोलियाँ चलाई, तो भीड़ क्रुद्ध हो उठी और उसने एक थाने में आग लगा दी। इसके फलस्वरूप एक थानेदार तथा 21 सिपाहियों की मृत्यु हो गई। चौरी- चौरा की इस हिंसात्मक घटना से गाँधीजी को प्रबल आघात पहुँचा और उन्होंने असहयोग आन्दोलन को स्थगित कर दिया।

प्रश्न 21.
दाण्डी यात्रा की प्रमुख घटनाओं की न्व्याख्या कीजिये। भारतीय राष्ट्रीय आन्दोलन के इतिहास में इसका क्या महत्त्व है?
उत्तर:
12 मार्च, 1930 को गाँधीजी ने अपने 78 आश्रमवासियों को लेकर साबरमती आश्रम से दाण्डी नामक स्थान की ओर प्रस्थान किया। उन्होंने अपनी यात्रा पैदल चल कर 24 दिन में तय की। 6 अप्रैल, 1930 को वहाँ उन्होंने नमक बनाकर कानून का उल्लंघन किया। इस प्रकार सम्पूर्ण भारत में नमक सत्याग्रह शुरू हो गया। इस आन्दोलन ने राष्ट्रीय आन्दोलन को व्यापक बनाया, स्वियों में जागृति पैदा की इस आन्दोलन से अंग्रेजों को पता चल गया कि अब उनका राज बहुत दिनों तक नहीं टिक सकेगा।

प्रश्न 22.
जलियाँवाला बाग हत्याकाण्ड पर टिप्पणी लिखिए।
उत्तर:
रॉलेट एक्ट तथा अपने लोकप्रिय नेताओं की गिरफ्तारी के विरुद्ध 13 अप्रैल, 1919 को अमृतसर के जलियाँवाला बाग में लोगों ने एक सार्वजनिक सभा आयोजित की जनरल डायर ने शान्तिप्रिय तथा निहत्थे लोगों पर गोलियाँ चलाने का आदेश दिया। इस बर्बरतापूर्ण कार्यवाही में 400 लोग मारे गए तथा सैकड़ों लोग घायल हो गए। इससे भारतीय जनता में तीव्र आक्रोश उत्पन्न हुआ और सम्पूर्ण देश में इस हत्याकाण्ड की कटु आलोचना की गई।

प्रश्न 23.
“चम्पारन, अहमदाबाद एवं खेड़ा में की गई पहल ने गाँधीजी को एक राष्ट्रवादी नेता के रूप में उभारा।” स्पष्ट कीजिए।
उत्तर:
चम्पारन, अहमदाबाद एवं खेड़ा में की गई पहल ने गाँधीजी को एक राष्ट्रवादी नेता के रूप में उभारा। गाँधीजी में गरीबों के प्रति गहरी सहानुभूति थी। वर्ष 1917 का अधिकांश समय महात्मा गाँधी को बिहार के चम्पारन जिले में किसानों के लिए काश्तकारी की सुरक्षा साथ-साथ अपनी पसन्द की फसल उपजाने की स्वतन्त्रता दिलाने में बीता। गाँधीजी ने भारत में सत्याग्रह का पहला प्रयोग 1917 ई. में चम्पारन में ही किया था।

JAC Class 12 History Important Questions Chapter 13 महात्मा गांधी और राष्ट्रीय आंदोलन : सविनय अवज्ञा और उससे आगे

वर्ष 1918 ई. में गाँधीजी गुजरात के अपने गृह राज्य में दो अभियानों में व्यस्त रहे। सर्वप्रथम उन्होंने अहमदाबाद के एक श्रम विवाद में हस्तक्षेप करके कपड़ा मिलों में कार्य करने वाले श्रमिकों के लिए काम करने की बेहतर स्थितियों की माँग की। इसके पश्चात् उन्होंने खेड़ा में फसल चौपट होने पर राज्य में किसानों का लगान माफ करने की माँग की। इस प्रकार कहा जा सकता है कि इन आन्दोलनों ने गाँधीजी को एक राष्ट्रवादी नेता के रूप में उभारा।

प्रश्न 24.
असहयोग आन्दोलन से भारतीयों ने क्या उम्मीदें लगा रखी थीं? स्पष्ट कीजिए।
उत्तर:
1920 ई. के कलकत्ता अधिवेशन में महात्मा गाँधी के प्रस्ताव ‘असहयोग आन्दोलन’ से भारतवासियों को अत्यधिक आशाएँ थीं। इसे हम निम्नलिखित बिन्दुओं के माध्यम से समझ सकते हैं –

  1. विदेशी वस्तुओं के बहिष्कार से देशी वस्तुओं को बढ़ावा मिलेगा।
  2. सरकारी उत्सवों का बहिष्कार कर देशी उत्सवों को प्रोत्साहन तथा पुनः प्रतिष्ठा प्राप्त होगी।
  3. साम्प्रदायिक रूप से हिन्दू तथा मुस्लिमों में एकता स्थापित होगी।
  4. राष्ट्र को एकता के सूत्र में बाँधने तथा राष्ट्रवाद को बढ़ाने में सहायता प्राप्त होगी।
  5. इस आन्दोलन से विभिन्न भारतीय नेताओं को एक मंच अवश्य प्राप्त होगा।

प्रश्न 25.
खिलाफत आन्दोलन की प्रमुख मांगों का संक्षिप्त वर्णन कीजिए।
उत्तर:
खिलाफत आन्दोलन (1919-20 ) मुहम्मद अली जिन्ना एवं शौकत अली के नेतृत्व में भारतीय मुसलमानों का एक आन्दोलन था। इस आन्दोलन की प्रमुख माँगें निम्नलिखित थीं –

  • पहले के ऑटोमन साम्राज्य के समस्त इस्लामी पवित्र स्थानों पर तुर्की के सुल्तान अथवा खलीफा का नियन्त्रण बना रहे।
  • जजीरात-उल-अरब ( अरब, सीरिया, इराक, फिलिस्तीन ) इस्लामी सम्प्रभुता के अधीन रहें।
  • खलीफा के पास इतने क्षेत्र हों कि वह इस्लामी विश्वास को सुरक्षित रखने योग्य बना सके।

प्रश्न 26.
मार्च 1922 में महात्मा गाँधी को राजद्रोह के आरोप में गिरफ्तारी के पश्चात् सजा सुनाते समय जस्टिस एन. ब्रूमफील्ड ने क्या टिप्पणी की?
उत्तर:
मार्च 1922 ई. में महात्मा गाँधीजी को राजद्रोह के आरोप में गिरफ्तार कर लिया गया। उन पर जाँच की कार्यवाही करने वाली समिति की अध्यक्षता करने वाले जज जस्टिस सी. एन. ब्रूमफील्ड ने उन्हें सजा सुनाते समय महत्त्वपूर्ण भाषण दिया। जज ने अपनी टिप्पणी में लिखा ” इस बात को नकारना असम्भव होगा कि मैंने आज तक जिनकी जांच की है या करूंगा आप उनसे अलग श्रेणी के हैं इस तथ्य को नकारना असम्भव होगा कि आपके लाखों देशवासियों की दृष्टि में आप एक महान् देश-भक्त व नेता हैं।

यहाँ तक कि राजनीति में जो लोग आपसे अलग विचार रखते हैं वे भी आपको उच्च आदर्शों और पवित्र जीवन वाले व्यक्ति के रूप में देखते हैं।” चूँकि गाँधीजी ने कानून की अवहेलना की थी अतः उस न्यायपीठ के लिए गाँधीजी को 6 वर्ष की सजा सुनाया जाना आवश्यक था। लेकिन जज ब्रूमफील्ड ने कहा कि “यदि भारत में घट रही घटनाओं की वजह से सरकार के लिए सजा के इन वर्षों कराना सम्भव हुआ तो इससे नहीं होगा।” में कमी और आपको मुक्त मुझसे ज्यादा कोई प्रसन्न

प्रश्न 27.
कांग्रेस के लाहौर अधिवेशन पर टिप्पणी लिखिए।
उत्तर:
दिसम्बर, 1929 में पं. जवाहरलाल नेहरू की अध्यक्षता में लाहौर में कांग्रेस का अधिवेशन शुरू हुआ। इसके अनुसार, 26 जनवरी, 1930 को देश के विभिन्न स्थानों पर राष्ट्रीय ध्वज फहरा कर और देशभक्ति के गीत गाकर ‘स्वतन्त्रता दिवस’ मनाया गया। यह अधिवेशन दो दृष्टियों से महत्त्वपूर्ण था –
(1) पं. जवाहरलाल नेहरू का अध्यक्ष के रूप में चुनाव, जो बुवा पीढ़ी को नेतृत्व की छड़ी सौंपने का प्रतीक था।
(2) इसमें पूर्ण स्वराज की घोषणा की गई।

प्रश्न 28.
दाण्डी यात्रा के समय गाँधीजी ने वसना गाँव में ऊँची जाति वालों को संबोधित करते हुए क्या कहा था?
उत्तर:
गांधीजी ने उच्च जाति के लोगों से कहा, “यदि आप स्वराज के हक में आवाज उठाते हैं तो आपको दलितों की सेवा करनी पड़ेगी। सिर्फ नमक कर या अन्य करों की समाप्ति से ही स्वराज नहीं मिल जायेगा। स्वराज के लिए आपको अपनी उन गलतियों के लिए प्रायश्चित करना पड़ेगा जो आपने दलितों के साथ की हैं। स्वराज के लिए हिन्दू, मुसलमान, पारसी और सिक्ख सबको एकजुट होना पड़ेगा। ये स्वराज प्राप्त करने की सीढ़ियाँ हैं।”

JAC Class 12 History Important Questions Chapter 13 महात्मा गांधी और राष्ट्रीय आंदोलन : सविनय अवज्ञा और उससे आगे

प्रश्न 29.
गाँधीजी और नेहरूजी के आग्रह पर काँग्रेस ने अल्पसंख्यकों के अधिकारों पर कौनसा प्रस्ताव पारित किया था?
उत्तर:
काँग्रेस ने ‘दो राष्ट्र सिद्धान्त’ को कभी स्वीकार नहीं किया था। जब उसे अपनी इच्छा के विरुद्ध विभाजन पर मंजूरी देनी पड़ी तो भी उसका दृढ़ विश्वास था कि “भारत बहुत सारे धर्मों और नस्लों का देश है और उसे ऐसे ही बनाए रखना चाहिए।” पाकिस्तान में हालात जो भी रहें, भारत “एक लोकतांत्रिक धर्मनिरपेक्ष राष्ट्र होगा जहाँ सभी नागरिकों को पूर्ण अधिकार प्राप्त होंगे तथा धर्म के आधार पर भेदभाव के बिना सभी को राज्य के द्वारा संरक्षण का अधिकार होगा।”

प्रश्न 30.
भारत विभाजन के समय हुए दंगों में गाँधीजी ने लोगों से क्या अपील की ?
उत्तर:
गांधीजी के जीवनी लेखक डी. जी. तेंदुलकर ने लिखा है कि सितम्बर और अक्टूबर के दौरान गाँधीजी “पीड़ितों को सांत्वना देते हुए अस्पतालों और शरणार्थी शिविरों में चक्कर लगा रहे थे।” उन्होंने “सिवानों, हेन्दुओं और मुसलमानों से अपील की कि वे अतीत को भुलाकर अपनी पीड़ा पर ध्यान देने के बजाय एक दूसरे के प्रति भाईचारे का हाथ बढ़ाने तथा शान्ति से रहने का संकल्प लें।”

प्रश्न 31.
गाँधीजी का भारत में राष्ट्रवाद के आधार को और अधिक व्यापक बनाने में किस प्रकार सफल रहे?
उत्तर:
महात्मा गाँधी का जनता से अनुरोध निस्सन्देह कपट से मुक्त था। भारतीय राजनीतिक के सन्दर्भ में तो बिना किसी संकोच के यह कहा जा सकता है कि वह अपने प्रयत्नों से राष्ट्रवाद के आधार को और अधिक व्यापक बनाने में सफल रहे। निम्न बिन्दुओं से यह तथ्य स्पष्ट है –

  • गाँधीजी के नेतृत्व में भारत के विभिन्न भागों में कांग्रेस की नयी शाखाएँ खोली गयीं।
  • रजवाड़ों में राष्ट्रवादी सिद्धान्त को बढ़ावा देने के लिए प्रजामण्डलों की स्थापना की गई। हम
  • गाँधीजी ने राष्ट्रवादी सन्देश का प्रसारे अंग्रेजी भाषा में करने की बजाय मातृभाषा में करने को प्रोत्साहन दिया।

प्रश्न 32.
1943 में हुए सतारा आन्दोलन की महानता और विशेषता का संक्षेप में वर्णन कीजिए।
उत्तर:
1943 में महाराष्ट्र में सतारा जिले के कुछ नेताओं ने सेवा दलों और तूफान दलों (ग्रामीण इकाई) के साथ मिलकर एक प्रति (समानान्तर ) सरकार की स्थापना कर ली थी। उन्होंने सतारा में जन अदालतों का आयोजन किया और सम्पूर्ण महाराष्ट्र में रचनात्मक कार्य किए। कुनबी किसानों के दबदबे और दलितों के सहयोग से चलने वाली सतारा की प्रति सरकार, ब्रिटिश सरकार द्वारा किए जा रहे दमन के बावजूद 1946 के चुनाव तक चलती रही।

प्रश्न 33.
आप कैसे कह सकते हैं कि गाँधीजी सर्वसाधारण के पक्षधर एवं हिमायती थे? 1916 से 1918 के मध्य की घटनाओं से इस कथन की पुष्टि कीजिये।
उत्तर:
(1) फरवरी, 1916 में बनारस हिन्दू विश्वविद्यालय के उद्घाटन के समय गाँधीजी ने बोलते समय मजदूरों और गरीबों की उपेक्षा किये जाने की आलोचना की।
(2) गाँधीजी ने कहा कि हमारे लिए स्वशासन या स्वराज का तब तक कोई अर्थ नहीं है जब तक हम किसानों से उनके श्रम का लगभग सम्पूर्ण लाभ स्वयं या अन्य लोगों को ले लेने की अनुमति देते रहेंगे। दिसम्बर, 1916 में उन्होंने चंपारन में तथा 1918 में अहमदाबाद और खेड़ा में सत्याग्रह किये।

प्रश्न 34.
1919 में पास किए गए रौलेट एक्ट के प्रति भारतीय जनमानस की प्रतिक्रिया का वर्णन कीजिए।
उत्तर:
1919 में रॉलेट एक्ट पास किया गया जिसके अनुसार –
(1) अंग्रेज बिना किसी कारण के भारतीयों को गिरफ्तार कर सकते थे तथा बिना मुकदमा चलाए उन्हें जेल में रख सकते थे।
(2) पंजाब जाते समय गाँधीजी को गिरफ्तार कर लिया गया तथा स्थानीय नेता भी गिरफ्तार कर लिए गए।
(3) 13 अप्रैल, 1919 को अमृतसर के जलियाँवाला बाग में जनरल डायर ने निहत्थी व निर्दोष जनता पर गोलियां चलवाई। इस भीषण नरसंहार में 400 लोग मारे गए।

प्रश्न 35.
गाँधीजी की दाण्डी यात्रा के बारे में विभिन्न स्रोतों द्वारा किन-किन बातों का पता लगा? लिखिए।
उत्तर:
(1) 12 मार्च, 1930 को गांधीजी ने साबरमती आश्रम से दाण्डी के लिए कूच किया।
(2) पुलिस रिपोर्ट के अनुसार जगह-जगह गाँधीजी ने भाषण दिए जिसमें दलितों को उनका हक देने, सभी धर्मावलम्बियों को एकजुट होने का आह्वान किया।
(3) अमेरिकी पत्रिका टाइम ने पहले गांधीजी के कमजोर शरीर का मजाक उड़ाया। परन्तु बाद में टाइम ने लिखा कि यात्रा को भारी जनसमर्थन मिल रहा है।

JAC Class 12 History Important Questions Chapter 13 महात्मा गांधी और राष्ट्रीय आंदोलन : सविनय अवज्ञा और उससे आगे

प्रश्न 36.
दलितों के उत्थान में डॉ. बी. आर. अम्बेडकर के प्रमुख योगदानों को स्पष्ट कीजिए।
उत्तर:
डॉ. अम्बेडकर दलितों के मसीहा थे। उन्होंने अपना तन-मन-धन दलितों के उत्थान में लगा दिया। उन्होंने प्रथम गोलमेज कान्फ्रेन्स में भाग लिया और उनकी दयनीय दशा पर प्रकाश डाला। उन्होंने 1931 में द्वितीय गोलमेज सम्मेलन में सवर्ण हिन्दुओं द्वारा दलितों के शोषण किए जाने की निन्दा की और उनके लिए पृथक् निर्वाचिका की माँग की। उन्होंने दलितों के लिए स्कूल खुलवाये।

प्रश्न 37.
“भारत छोड़ो आन्दोलन सही मायने में जन-आन्दोलन था?” समालोचना कीजिये।
उत्तर:
अगस्त, 1942 में गाँधीजी ने अपना तीसरा बड़ा आन्दोलन ‘अंग्रेजो भारत छोड़ो’ प्रारम्भ किया। यह आन्दोलन सही मायने में एक जन आन्दोलन था जिसमें लाखों आम हिन्दुस्तानी शामिल थे। इस आन्दोलन ने युवा वर्ग को बहुत बड़ी संख्या में अपनी ओर आकर्षित किया। उन्होंने अपने कॉलेज छोड़कर जेल का रास्ता अपनाया। इस आन्दोलन के दौरान सतारा में स्वतंत्र’ सरकार भी बनी, जो 1946 तक चलती रही। वस्तुतः 1942 का आन्दोलन वास्तव में जन-आन्दोलन था।

प्रश्न 38.
जस्टिस सी. एन. बूमफील्ड ने गाँधीजी को सजा सुनाते हुए क्या कहा?
उत्तर:
जस्टिस सी. अपनी टिप्पणी में लिखा, “इस तथ्य को नकारना असम्भव होगा कि आपके लाखों देशवासियों की दृष्टि में आप एक महान देशभक्त और नेता हैं। यहाँ तक कि राजनीति में जो लोग आपसे भिन्न विचार रखते हैं वे भी आपको उच्च आदर्शों और पवित्र जीवन वाले व्यक्ति के रूप में देखते हैं। चूँकि गाँधीजी ने कानून की अवहेलना की थी, अतः उस न्यायपीठ के लिए गाँधीजी को 6 वर्षों की जेल की सजा सुनाया जाना जरूरी था।”

प्रश्न 39.
“गाँधीजी भारतीय राष्ट्रवाद को सम्पूर्ण भारतीय लोगों का और अधिक अच्छे ढंग से प्रतिनिधित्व करने में सक्षम बनाना चाहते थे।” स्पष्ट कीजिए।
उत्तर:
गाँधीजी ने महसूस किया कि भारतीय राष्ट्रवाद वकीलों, डॉक्टरों और जमींदारों जैसे विशिष्ट वर्गों द्वारा निर्मित था। फरवरी, 1916 में गाँधीजी ने बनारस हिन्दू ‘विश्वविद्यालय में भाषण देते हुए कहा था कि “हमारी मुक्ति केवल किसानों के माध्यम से ही हो सकती है, न तो वकील, न डॉक्टर, न ही जमींदार इसे सुरक्षित रख सकते हैं।” अतः गाँधीजी लाखों किसानों और मजदूरों को भारतीय राष्ट्रवाद का अभिन्न अंग बनाना चाहते थे।

प्रश्न 40.
गाँधीजी के बारे में कौनसी चमत्कारिक शक्तियों की अफवाहें फैली हुई थीं?
उत्तर:
(1) गाँधीजी के बारे में यह अफवाह भी फैली हुई थी कि उन्हें राजा द्वारा किसानों के दुःखों एवं कष्टों के निवारण के लिए भेजा गया था तथा उनके पास सभी स्थानीय अधिकारियों के निर्देशों को अस्वीकृत करने की शक्ति थी
(2) गांधीजी की शक्ति ब्रिटिश सम्राट से उत्कृष्ट है और उनके आगमन से ब्रिटिश शासक जिलों से भाग जायेंगे। (3) गाँधीजी की आलोचना करने वाले गाँवों के लोगों के घर गिर गए या उनकी फसलें नष्ट हो गई।

प्रश्न 41.
गाँधीजी सामान्य जन से घनिष्ठ रूप से जुड़े हुए थे।” व्याख्या कीजिए।
उत्तर:
गांधीजी आम लोगों की तरह रहते थे, उनकी ही तरह के वस्त्र पहनते थे तथा उनकी भाषा में बोलते थे। गाँधीजी लोगों के बीच एक साधारण धोती में जाते थे। वे किसानों, मजदूरों, कारीगरों, गरीबों, दलितों से गहरी सहानुभूति रखते थे। वे प्रतिदिन कुछ समय के लिए चरखा चलाते थे। वे मानसिक एवं शारीरिक परिश्रम में कोई भेद नहीं मानते थे।

निबन्धात्मक प्रश्न

प्रश्न 1.
निम्नलिखित पर संक्षिप्त टिप्पणियाँ लिखिए –
(1) रॉलेट एक्ट
(ii) खिलाफत आन्दोलन।
उत्तर:
(i) रॉलेट एक्ट यद्यपि प्रथम विश्व- बुद्ध (1914-18) के दौरान भारतवासियों ने अंग्रेजों की तन-मन-धन से सहायता की थी, परन्तु विश्वयुद्ध की समाप्ति के पश्चात् ब्रिटिश सरकार ने भारतीय राष्ट्रीय आन्दोलन को कुचलने के लिए कठोर कानून बनाये। 1919 में ब्रिटिश सरकार ने रॉलेट एक्ट पास किया जिसके अनुसार किसी भी भारतीय को बिना किसी जाँच के कारावास में बन्द किया जा सकता था। रॉलेट एक्ट के पारित किये जाने से गाँधीजी को प्रबल आघात पहुँचा। उन्होंने इस काले कानून के विरुद्ध एक देशव्यापी अभियान चलाया। गाँधीजी की अपील पर अनेक नगरों में हड़ताल की गई।

JAC Class 12 History Important Questions Chapter 13 महात्मा गांधी और राष्ट्रीय आंदोलन : सविनय अवज्ञा और उससे आगे

दिल्ली में लोगों ने ब्रिटिश सरकार के विरुद्ध एक जुलूस निकाला जिस पर पुलिस ने गोलियाँ चलाई जिससे अनेक लोग मारे गए। गाँधीजी को पलवल (हरियाणा) नामक रेलवे स्टेशन पर गिरफ्तार कर लिया गया। इससे भारतवासियों में तीव्र आक्रोश उत्पन्न हुआ। पंजाब के लोगों ने रॉलेट एक्ट का घोर विरोध किया।

ब्रिटिश सरकार ने अमृतसर के स्थानीय नेताओं को गिरफ्तार कर लिया। इसके विरोध में 13 अप्रैल, 1919 को अमृतसर के जलियाँवाला बाग में लोगों ने एक विशाल सभा आयोजित की। एक अंग्रेज ब्रिगेडियर डाबर ने प्रदर्शनकारियों पर गोलियाँ चलवार्थी, जिससे 400 से अधिक लोग मारे गए और हजारों घायल हो गए, इस पर गाँधीजी ने ब्रिटिश सरकार के विरुद्ध असहयोग आन्दोलन चलाने का निर्णय किया।

(ii) खिलाफत आन्दोलन खिलाफत आन्दोलन (1919-20 ) मुहम्मद अली एवं शौकत अली के नेतृत्व में भारतीय मुसलमानों का एक आन्दोलन था।

इस आन्दोलन की प्रमुख माँगें निम्नलिखित –

  • पहले के आटोमन साम्राज्य के सभी इस्लामी पवित्र स्थानों पर तुर्की सुल्तान अथवा खलीफा का नियन्त्रण बना रहे।
  • जंजीरात-उल-अरब इस्लामी सम्प्रभुता के अधीन रहे।
  • खलीफा के पास इतने क्षेत्र हों कि वह इस्लामी विश्वास को सुरक्षित रखने योग्य बन सके।

गाँधीजी ने असहयोग आन्दोलन को सफल बनाने के लिए खिलाफत आन्दोलन को इसका अंग बनाया। उन्होंने हिन्दुओं और मुसलमानों में एकता उत्पन्न करने के लिए खिलाफत आन्दोलन का समर्थन किया।

प्रश्न 2.
तीनों गोलमेज सम्मेलनों के विषय में विस्तारपूर्वक लिखिए।
उत्तर:
तीनों गोलमेज सम्मेलनों को निम्नलिखित शीर्षकों के माध्यम से समझ सकते हैं –
(1) प्रथम गोलमेज सम्मेलन भारतीय संविधान पर विचार करने के लिये लन्दन में 12 नवम्बर, 1930 को प्रथम गोलमेज सम्मेलन का आयोजन किया गया। इस सम्मेलन में कुल 57 भारतीयों ने भागीदारी की। यह वह समय था जब भारत के प्रायः सभी प्रमुख नेता सविनय अवज्ञा आन्दोलन के कारण जेल में बन्द थे। इस सम्मेलन में प्रायः कुछ साम्प्रदायिक समस्याओं पर ही चर्चा हुई। इस सम्मेलन में दुर्भाग्य से कोई निर्णय नहीं लिया जा सका।

(2) द्वितीय गोलमेज सम्मेलन – प्रथम गोलमेज सम्मेलन की असफलता के उपरान्त द्वितीय गोलमेज सम्मेलन की सफलता के लिये अंग्रेजों ने अत्यधिक प्रयास किये, इसके लिये उन्होंने गाँधीजी को रिहा कर दिया। यहाँ गाँधी- इर्विन में समझौता हुआ था। गाँधीजी ने द्वितीय गोलमेज सम्मेलन में भाग लेना स्वीकार किया। 7 सितम्बर, 1931 को लन्दन में द्वितीय गोलमेज सम्मेलन का आयोजन किया गया।

इसमें गाँधीजी ने कांग्रेस के प्रतिनिधि के रूप में भाग लिया। गाँधीजी का कहना था कि उनकी पार्टी भारत का प्रतिनिधित्व करती है, परन्तु उनके इस दावे को तीन पार्टियों ने चुनौती दी थी। इस सम्मेलन में भाग ले रहे मुस्लिम लीग के जिन्ना का कहना था कि उनकी पार्टी मुस्लिम अल्पसंख्यकों के हित में काम करती है। अनेक रियासतों के प्रतिनिधि भी इस सम्मेलन में सम्मिलित हुए। उनका दावा था कि कांग्रेस का उनके नियन्त्रण वाले भू-भाग पर कोई प्रभुत्व नहीं है। तीसरी चुनौती डॉ. भीमराव रामजी अम्बेडकर की ओर से थी। उनका कहना था कि गांधीजी व कांग्रेस पार्टी निचली जातियों का प्रतिनिधित्व नहीं करती है।

(3) तृतीय गोलमेज सम्मेलन-जिन दिनों भारत में महात्मा गाँधी ने सशक्तता के साथ सविनय अवज्ञा आन्दोलन चला रखा था उसी समय लन्दन में तृतीय गोलमेज सम्मेलन का आयोजन हो रहा है। इंग्लैण्ड की एक मुख्य लेबर पार्टी ने इसमें भाग नहीं लिया। भारत में कांग्रेस पार्टी ने भी इस सम्मेलन का पूर्ण रूप से बहिष्कार किया था। वे भारतीय प्रतिनिधि जो सिर्फ अंग्रेजों की हाँ में हाँ मिलाते थे, ने इस सम्मेलन में भाग लिया। इस सम्मेलन में लिये गये निर्णयों को श्वेत-पत्र के रूप में प्रकाशित किया गया। इस श्वेत-पत्र के आधार पर 1935 का भारत सरकार अधिनियम पारित किया गया। कांग्रेस के अनेक नेताओं को गिरफ्तार कर जेल में डाल दिया गया।

प्रश्न 3.
असहयोग आन्दोलन एक तरह का प्रतिरोध कैसे था? उल्लेख कीजिए।
अथवा
असहयोग आन्दोलन के कारणों का मूल्यांकन कीजिए।
अथवा
महात्मा गाँधी द्वारा चलाए गए असहयोग आन्दोलन के कार्यक्रमों एवं प्रगति पर एक निबन्ध लिखिए।
अथवा
महात्मा गाँधी द्वारा संचालित असहयोग आन्दोलन कब आरम्भ हुआ? इसके उद्देश्य तथा कार्यक्रम क्या थे? यह आन्दोलन क्यों समाप्त हुआ?
अथवा
असहयोग आन्दोलन पर एक लेख लिखिए। उत्तर- असहयोग आन्दोलन के कारण 1920 में गांधीजी द्वारा संचालित असहयोग आन्दोलन के निम्नलिखित कारण थे –
(1) रॉलेट एक्ट 1919 में ब्रिटिश सरकार ने रॉलेट एक्ट पारित किया जिसके अनुसार किसी भी भारतीय को बिना मुकदमा चलाए जेल में बन्द किया जा सकता था। गाँधीजी ने देशभर में ‘रॉलेट एक्ट’ के विरुद्ध एक अभियान चलाया। पंजाब जाते समय गाँधीजी को गिरफ्तार कर लिया गया।

JAC Class 12 History Important Questions Chapter 13 महात्मा गांधी और राष्ट्रीय आंदोलन : सविनय अवज्ञा और उससे आगे

(2) जलियाँवाला बाग हत्याकाण्ड-13 अप्रैल, 1919 को अमृतसर के जलियाँवाला बाग में एक सार्वजनिक सभा आयोजित की गई। जनरल डायर ने निहत्थे लोगों पर गोलियाँ चलाना शुरू कर दिया। इस बर्बरतापूर्ण कार्यवाही में 400 लोग मारे गए तथा सैकड़ों लोग घायल हो गए।

(3) खिलाफत आन्दोलन-गाँधीजी ने हिन्दुओं और मुसलमानों में एकता उत्पन्न करने के लिए खिलाफत आन्दोलन का समर्थन किया। असहयोग आन्दोलन के कार्यक्रम असहयोग आन्दोलन के कार्यक्रमों के अन्तर्गत निम्नलिखित बातें सम्मिलित थीं –

  • सरकारी स्कूलों तथा कॉलेजों का बहिष्कार करना
  • सरकारी उपाधियों तथा अवैतनिक पदों का बहिष्कार करना
  • सरकारी न्यायालयों का बहिष्कार करना
  • विदेशी वस्तुओं का बहिष्कार करना तथा स्वदेशी वस्तुओं का प्रयोग करना।

आन्दोलन की प्रगति ( असहयोग आन्दोलन प्रतिरोध के रूप में ) – कलकत्ता अधिवेशन में असहयोग आन्दोलन के प्रस्ताव को बहुमत से स्वीकार कर लिया गया। हजारों विद्यार्थियों ने सरकारी स्कूलों तथा कॉलेजों का बहिष्कार किया तथा वकीलों ने अदालत जाने मना कर दिया। अनेक नगरों में हड़तालें हुई। 1921 में 396 हड़तालें हुई जिनमें 6 लाख श्रमिक शामिल थे। उत्तरी आंध्र के पहाड़ी लोगों ने वन्य कानूनों का उल्लंघन किया। अवध के किसानों ने कर नहीं चुकाया। स्वदेशी का प्रचार हुआ और विदेशी वस्त्रों का बहिष्कार किया गया। सरकार ने अनेक कांग्रेसी नेताओं को जेलों में बन्द कर दिया।

चौरी-चौरा काण्ड-5 फरवरी, 1922 को उत्तरप्रदेश के गोरखपुर जिले में स्थित चौरी-चौरा नामक गाँव में पुलिस ने कांग्रेस के सत्याग्रहियों पर गोलियाँ चलाई तो भीड़ क्रुद्ध हो उठी और उसने एक थाने में आग लगा दी। इसके फलस्वरूप एक थानेदार तथा 21 सिपाहियों की मृत्यु हो गई। चौरी-चौरा काण्ड से गाँधीजी को प्रबल आघात पहुँचा और उन्होंने असहयोग आन्दोलन को स्थगित कर दिया। 10 मार्च, 1922 को सरकार ने गांधीजी को गिरफ्तार कर लिया और न्यायाधीश ग्रूमफील्ड ने उन्हें 6 वर्ष के कारावास की सजा दी।

असहयोग आन्दोलन का महत्त्व एवं प्रभाव –
(1) सकारात्मक आन्दोलन लुई फिशर के अनुसार असहयोग भारत और गाँधीजी के जीवन के एक युग का ही नाम हो गया। असहयोग शान्ति की दृष्टि से नकारात्मक किन्तु प्रभाव की दृष्टि से बहुत सकारात्मक था। इसके लिए प्रतिवाद, परित्याग तथा स्व-अनुशासन आवश्यक थे यह स्वशासन के लिए एक प्रशिक्षण था।

(2) अंग्रेजी शासन की नींव हिलना-1857 के विद्रोह के बाद पहली बार असहयोग आन्दोलन के परिणामस्वरूप अंग्रेजी शासन की नींव हिल गई।

(3) जन-आन्दोलन-असहयोग आन्दोलन ने राष्ट्रीय आन्दोलन को व्यापक एवं जनप्रिय बना दिया।

(4) राष्ट्रीयता का प्रसार असहयोग आन्दोलन ने देशवासियों में राष्ट्रीयता का प्रसार किया 1922 तक गाँधीजी ने भारतीय राष्ट्रवाद को एकदम परिवर्तित कर दिया।

प्रश्न 4.
ब्रिटिश सरकार ने गोलमेज सम्मेलनों का आयोजन क्यों किया? काँग्रेस का इन सम्मेलनों के प्रति क्या रुख रहा? इनका क्या परिणाम निकला?
अथवा
गोलमेज सम्मेलन क्यों आयोजित किये गए? इनके कार्यों की विवेचना कीजिये।
उत्तर:
पृष्ठभूमि भारत में लागू किए जाने वाले संवैधानिक सुधारों के बारे में चर्चा करने के लिए ब्रिटिश सरकार ने तीन बार गोलमेज सम्मेलनों का आयोजन लंदन में किया। लेकिन इन सम्मेलनों का कोई भी सार्थक परिणाम नहीं निकला।

(1) प्रथम गोलमेज सम्मेलन – भारतीय राजनीतिक गतिरोध को दूर करने के उद्देश्य से 1930 में प्रथम गोलमेज सम्मेलन का लंदन में आयोजन किया गया। इसमें ब्रिटिश सरकार के समर्थक मुस्लिम लीग तथा हिन्दू महासभा के प्रतिनिधि भी शामिल हुए लेकिन कॉंग्रेस का कोई भी प्रतिनिधि इसमें शामिल नहीं हुआ क्योंकि उस समय सविनय अवज्ञा आन्दोलन चल रहा था। आधारभूत मुद्दों पर किसी सहमति के बिना 1931 में यह सम्मेलन स्थगित कर दिया गया। कॉंग्रेस की गैरहाजिरी में यह सम्मेलन अपने उद्देश्य में सफल नहीं हो पाया।

(2) द्वितीय गोलमेज सम्मेलन – 1931 के आखिर में दूसरा गोलमेज सम्मेलन लन्दन में आयोजित किया गया। इसमें गाँधीजी ने कांग्रेस के प्रतिनिधि के रूप में भाग लिया। इस सम्मेलन में गांधीजी ने मुस्लिम लीग की पृथक् निर्वाचिका की माँग का विरोध करते हुए पूर्ण स्वराज्य की माँग की। गाँधीजी का कहना था कि उनका दल सम्पूर्ण भारत का प्रतिनिधित्व करता है।

JAC Class 12 History Important Questions Chapter 13 महात्मा गांधी और राष्ट्रीय आंदोलन : सविनय अवज्ञा और उससे आगे

परन्तु मुस्लिम लीग, राजे- रजवाड़ों तथा डॉ. अम्बेडकर ने गाँधीजी के दावे को चुनौती दी। मुस्लिम लीग के अनुसार कांग्रेस मुसलमानों का, राजे- रजवाड़ों के अनुसार कांग्रेस देशी रियासतों का तथा डॉ. अम्बेडकर के अनुसार कांग्रेस दलितों का प्रतिनिधित्व नहीं करती। इस प्रकार दूसरा सम्मेलन भी किसी परिणाम पर नहीं पहुँचा और गाँधीजी को खाली हाथ लौटना पड़ा।

(3) तृतीय गोलमेज सम्मेलन राजनीतिक स्थिति की समीक्षा के लिए तथा संवैधानिक सुधार लागू करने के लिए ब्रिटिश सरकार ने 1932 में तीसरा गोलमेज सम्मेलन आयोजित किया। काँग्रेस ने इसमें भाग नहीं लिया। इसमें इंग्लैण्ड के एक मुख्य राजनीतिक दल लेबर पार्टी ने भी भाग नहीं लिया। इसमें कुछ ऐसे भारतीय प्रतिनिधि सम्मिलित हुए जो अंग्रेजों की हाँ में हाँ मिलाते थे। अन्त में तीनों गोलमेज सम्मेलनों में हुई चर्चाओं के आधार पर ब्रिटिश सरकार ने 1933 में एक श्वेत पत्र जारी किया, जिसके आधार पर 1935 का भारत सरकार का अधिनियम पारित किया गया।

प्रश्न 5.
“भारत छोड़ो आन्दोलन ब्रिटिश शासन के खिलाफ गाँधीजी का तीसरा बड़ा आन्दोलन था।” व्याख्या कीजिए।
उत्तर:
क्रिप्स मिशन की विफलता के पश्चात् महात्मा गाँधी ने ब्रिटिश शासन के खिलाफ अपना तीसरा बड़ा आन्दोलन छेड़ने का फैसला किया। अगस्त, 1942 ई. में शुरू किए गए इस आन्दोलन को ‘अंग्रेज भारत छोड़ो’ के नाम से जाना गया।

भारत छोड़ो आन्दोलन प्रारम्भ करने के कारण –
(i) अंग्रेजों की साम्राज्यवादी नीति- सितम्बर, 1939 में द्वितीय विश्व युद्ध प्रारम्भ हो गया। महात्मा गाँधी व जवाहरलाल नेहरू दोनों ही हिटलर व नात्सियों के आलोचक थे। तदनुरूप उन्होंने फैसला किया कि यदि अंग्रेज युद्ध समाप्त होने के पश्चात् भारत को स्वतन्त्रता देने पर सहमत हों तो कांग्रेस उनके बुद्ध प्रयासों में सहायता दे सकती है। ब्रिटिश सरकार ने कांग्रेस के इस प्रस्ताव को खारिज कर दिया। इस घटनाक्रम ने अंग्रेजी साम्राज्यवादी नीति के विरुद्ध आन्दोलन प्रारम्भ करने हेतु प्रोत्साहित किया।

(i) क्रिप्स मिशन की असफलता द्वितीय विश्व युद्ध में कांग्रेस व गाँधीजी का समर्थन प्राप्त करने के लिए तत्कालीन ब्रिटिश प्रधानमन्त्री विंस्टन चर्चिल ने अपने एक | मन्त्री सर स्टेफर्ड क्रिप्स को भारत भेजा। क्रिप्स के साथ वार्ता में कांग्रेस ने इस बात पर जोर दिया कि यदि धुरी शक्तियों से भारत की रक्षा के लिए ब्रिटिश शासन कांग्रेस का समर्थन चाहता है तो वायसराय को सबसे पहले अपनी कार्यकारी परिषद् में किसी भारतीय को एक रक्षा सदस्य के रूप में नियुक्त करना चाहिए। इसी बात पर वार्ता टूट गयी। क्रिप्स मिशन की विफलता के पश्चात् गाँधीजी ने अंग्रेजों भारत छोड़ो आन्दोलन प्रारम्भ करने का फैसला किया।

भारत छोड़ो आन्दोलन का प्रारम्भ-9 अगस्त, 1942 ई. को गाँधीजी के नेतृत्व में भारत छोड़ो आन्दोलन प्रारम्भ हो गया। अंग्रेजों ने इस आन्दोलन को दबाने के लिए बड़ी कठोरता से काम लिया। कांग्रेस को अवैध घोषित कर दिया गया तथा सभाओं, जुलूसों व समाचार-पत्रों पर कठोर प्रतिबन्ध लगा दिए गए। इसके बावजूद देशभर के युवा कार्यकर्ता हड़तालों एवं तोड़फोड़ की कार्यवाहियों के माध्यम से आन्दोलन चलाते रहे। कांग्रेस में जयप्रकाश नारायण जैसे समाजवादी सदस्य भूमिगत होकर अपनी गतिविधियों को चलाते रहे।
आन्दोलन का अन्त-अंग्रेजों ने भारत छोड़ो आन्दोलन के प्रति कठोर रवैया अपनाया फिर भी इस विद्रोह का दमन करने में साल भर से अधिक समय लग गया।

आन्दोलन का महत्त्व भारत छोड़ो आन्दोलन में लाखों की संख्या में आम भारतीयों ने भाग लिया तथा हड़तालों एवं तोड़-फोड़ के माध्यम से आन्दोलन को आगे बढ़ाते रहे। इस आन्दोलन के कारण भारत की ब्रिटिश औपनिवेशिक सरकार यह बात अच्छी तरह जान गई कि जनता में कितना व्यापक असन्तोष है। सरकार समझ गई कि अब वह भारत में ज्यादा दिनों तक शासन नहीं कर पायेगी।

प्रश्न 6.
भारत को स्वतंत्र कराने में गाँधीजी के योगदान का वर्णन कीजिए।’
अथवा
भारतीय स्वतंत्रता संग्राम में महात्मा गाँधी के योगदान का वर्णन कीजिये।
उत्तर:
भारत को स्वतंत्र कराने में गाँधीजी का योगदान भारत को आजादी दिलाने में गाँधीजी की भूमिका मुख्य थी। उन्होंने सत्याग्रह और शान्तिपूर्ण अहिंसा का सहारा लेकर ताकतवर ब्रिटिश साम्राज्य को झुकने पर मजबूर कर दिया। भारत को स्वतंत्र कराने में गांधीजी के योगदान को निम्न बिन्दुओं के अन्तर्गत स्पष्ट किया गया है –
(1) भारतीय राष्ट्र के पिता-राष्ट्रवाद के इतिहास में प्रायः एक अकेले व्यक्ति को राष्ट्र निर्माण के साथ जोड़कर देखा जाता है। महात्मा गाँधी को भारतीय राष्ट्र का ‘पिता’ माना गया है क्योंकि गाँधीजी स्वतंत्रता संघर्ष में भाग लेने वाले सभी नेताओं में सर्वाधिक प्रभावी और सम्मानित थे।

(2) भारतीय राष्ट्रीय आन्दोलनों का कुशलतापूर्वक संचालन –

  • असहयोग आन्दोलन – 1920 में गांधीजी ने असहयोग आन्दोलन चलाया जिसमें लाखों लोगों ने हिस्सा लिया। उनके कहने पर भारतीयों ने चाहे वे क्लर्क थे, वकील थे या कारीगर थे, सबने अपना काम करना बन्द कर दिया। विद्यार्थियों ने विद्यालय जाना छोड़ दिया। सभी आजादी की लड़ाई में कूद पड़े।
  • सविनय अवज्ञा आन्दोलन – 1930 में गाँधीजी ने सविनय अवज्ञा आन्दोलन प्रारम्भ कर दिया। उन्होंने नमक कानून तोड़ा जिसके लिए उन्हें जेल जाना पड़ा।
  • भारत छोड़ो आन्दोलन अगस्त – 1942 में गाँधीजी ने ‘अंग्रेजो भारत छोड़ो आन्दोलन शुरू किया। गाँधीजी ने इसके लिए ‘करो या मरो’ का नारा बुलन्द किया था।

(3) स्वदेशी आन्दोलन – गाँधीजी ने भारतीय राष्ट्रीय आन्दोलनों के अन्तर्गत स्वदेशी आन्दोलन का समर्थन किया।

(4) भारतीय राष्ट्रीय आन्दोलन को विशिष्टवर्गीय आन्दोलन से जनांदोलन बनाया-भारतीय राष्ट्रीय आन्दोलन में गाँधीजी का सबसे बड़ा योगदान राष्ट्रीय आन्दोलन को जन-आन्दोलन में परिणत करने का था। गाँधीजी ने अपनी पहली महत्त्वपूर्ण सार्वजनिक सभा, जो फरवरी, 1916 में बनारस हिन्दू विश्वविद्यालय के उद्घाटन समारोह में हुई, में अपने भाषण में मजदूर गरीबों की ओर ध्यान न देने के कारण भारतीय विशिष्ट वर्ग को आड़े हाथों लिया। इसके बाद उन्होंने अपनी वेशभूषा को गरीब भारतीयों की वेशभूषा के अनुरूप ढाला ताकि गरीब जनता उसे अपने जैसा समझते हुए राष्ट्रीय आन्दोलन से जुड़े इसके अतिरिक्त उन्होंने विभिन्न धर्मों के बीच सौहार्द बढ़ाने का प्रयास किया।

(5) समाज सुधारक गाँधीजी महान समाज सुधारक भी थे। उनका विश्वास था कि स्वतंत्रता के योग्य बनने के लिए भारतीयों को बाल विवाह और छुआछूत जैसी सामाजिक बुराइयों से मुक्त होना पड़ेगा। एक मत के भारतीयों को दूसरे मत के भारतीयों के लिए सच्चा संयम लाना होगा और इस प्रकार उन्होंने हिन्दू-मुसलमानों के बीच सौहार्द पर बल दिया। उन्होंने विदेशी वस्त्रों के बहिष्कार और स्वदेशी के अपनाने तथा खादी पहनने पर बल दिया।

JAC Class 12 History Important Questions Chapter 13 महात्मा गांधी और राष्ट्रीय आंदोलन : सविनय अवज्ञा और उससे आगे

(6) हिन्दू-मुस्लिम एकता के प्रबल समर्थक गाँधीजी हिन्दू-मुस्लिम एकता के प्रबल समर्थक थे। उन्होंने साम्प्रदायिक दंगों का घोर विरोध किया और देश में शान्ति बनाए रखने की अपील की। उन्होंने हिन्दू-मुस्लिम एकता बनाये रखने के लिए अपने प्राणों तक का बलिदान कर दिया।

प्रश्न 7.
महात्मा गाँधी केवल राजनीतिक नेता ही नहीं थे, वे एक समाज सुधारक तथा आर्थिक सुधारक भी थे। स्पष्ट कीजिये।
उत्तर:
एक राजनीतिज्ञ के रूप में गांधीजी का महत्त्व तब पता चलता है जब उन्होंने अहिंसक आन्दोलन चलाकर एक साम्राज्यवादी ताकत को हिलाकर रख दिया और देश को आजादी दिलाकर ही दम लिया लेकिन वे एक समाज सुधारक और आर्थिक सुधारक भी थे। यथा –
(1) गाँधीजी एक समाज सुधारक के रूप में- प्रथमतः, गाँधीजी ने हिन्दू-मुस्लिम एकता स्थापित की तथा उन्हें अंग्रेजों के विरुद्ध लड़ने के लिए एकजुट किया। इसका उदाहरण खिलाफत आन्दोलन के साथ असहयोग आन्दोलन को जोड़ना था। दूसरे, समाज सुधारक के रूप में उन्होंने जातीय प्रथा का। विरोध किया। उन्होंने यथासम्भव दलितों का उद्धार किया तथा पहली बार अछूतों को हरिजन नाम से संबोधित किया। उनके उद्धार के लिए हरिजन नामक पत्र निकाला। ये छुआछूत के विरुद्ध लड़े तथा सभी वर्गों और धर्मों से सौहार्दपूर्ण सम्बन्ध बनाने पर बल दिया। तीसरे, उन्होंने बाल विवाह, छुआछूत का विरोध किया तथा भारतीयों को मत-मतांतरों के बीच सच्चा संगम लाने पर बल दिया।

(2) आर्थिक सुधारक के रूप में एक आर्थिक सुधारक के रूप में उन्होंने निम्न प्रमुख कार्य किए
(i) आर्थिक स्थिति के सुधार के लिए उन्होंने चरखा चलाने तथा खादी पहनने पर बल दिया जिससे देशी कपड़ा उद्योग को बढ़ावा मिला। गाँवों के विकास और कुटीर उद्योगों को बढ़ावा देने पर जोर दिया, जिन्हें छोटी पूँजी से परिवार के सदस्य घर से चलाकर अतिरिक्त आय प्राप्त कर सकते थे। उन्होंने मशीनीकरण की नीति का विरोध किया। वे इस बात के विरुद्ध थे कि धन का केन्द्रीकरण केवल कुछ ही लोगों के हाथों में हो जाए।

(ii) उन्होंने अहमदाबाद के मिल मजदूरों को अपना वेतन बढ़वाने के लिए हड़ताल करने को कहा तथा भारत के सभी वर्गों में आर्थिक समानता लाने पर जोर दिया।

(iii) गाँधीजी किसानों के हित के लिए लड़े तथा उनकी रक्षा के लिए स्वयं आगे आए। चंपारन सत्याग्रह, खेड़ा सत्याग्रह इसके उदाहरण हैं। गाँधीजी के उपर्युक्त कार्यों के आधार पर हम सकते हैं कि गाँधीजी केवल एक राजनीतिक नेता ही नहीं थे अपितु वे समाज सुधारक तथा आर्थिक सुधारक के रूप में भी हमारे सामने आए वे आजादी दिलाने के साथ-साथ देश की आर्थिक व सामाजिक स्थिति में भी सुधार लाए।

प्रश्न 8.
“1919 तक महात्मा गाँधी ऐसे राष्ट्रवादी के रूप में उभर चुके थे, जिनमें गरीबों के प्रति गहरी सहानुभूति थी।” उदाहरण सहित कथन को सिद्ध कीजिए।
अथवा
भारत में गाँधीजी द्वारा किए गए शुरुआती सत्याग्रहों का वर्णन कीजिए। ये सत्याग्रह गाँधीजी के राजनीतिक जीवन में कहाँ तक सहायक हुए ?
अथवा
चम्पारन सत्याग्रह का संक्षिप्त विवरण दीजिये।
उत्तर:
1915 में दक्षिणी अफ्रीका से भारत लौटने के बाद गाँधीजी ने प्रारम्भ में कई सत्याग्रह किए जिनका विवरण इस प्रकार है –
(1) चम्पारन सत्याग्रह – गाँधीजी ने अपना प्रथम सत्याग्रह 1917 में बिहार के चंपारन नामक स्थान पर किया। वहाँ पर जो किसान नील की खेती करते थे उन पर यूरोपीय निलहे बहुत अत्याचार करते थे। उन किसानों ने गाँधीजी को अपनी समस्या बताई। इस पर गाँधीजी चंपारन पहुँचे। अन्त में सरकार ने किसानों की शिकायतों को दूर करने हेतु कदम उठाये।

(2) खेड़ा सत्याग्रह – 1918 में गुजरात के खेड़ा जिले में फसल खराब होने से किसानों की हालत खराब हो गई। किसानों ने लगान देने से मना कर दिया। गाँधीजी ने उनकी बात का समर्थन किया। यहाँ भी सरकार को झुकना पड़ा और यह निर्णय लिया गया कि जो किसान लगान देने में सक्षम हैं उन्हें ही जमा कराने का आदेश दें। अतः कुछ समय बाद यह आन्दोलन खत्म हो गया।

(3) अहमदाबाद के मिल मजदूरों का संघर्ष – 1918 में अहमदाबाद के कपड़ा मिल मजदूरों ने अपने वेतन को | बढ़वाने के लिए मिल मालिकों से कहा लेकिन मिल- मालिकों ने मना कर दिया। इस पर मजदूरों ने हड़ताल कर दी। गाँधीजी ने मिल मजदूरों की माँग को जायज बताया और अनशन शुरू कर दिया। अन्त में मिल मालिकों ने उनकी बात मान ली। गाँधीजी ने अपना अनशन समाप्त कर दिया तथा हड़ताल भी खत्म हो गई।

JAC Class 12 History Important Questions Chapter 13 महात्मा गांधी और राष्ट्रीय आंदोलन : सविनय अवज्ञा और उससे आगे

(4) सर्वसाधारण के प्रति सहानुभूति-गाँधीजी की आम भारतीयों के प्रति गहरी सहानुभूति थी वे किसानों के शोषण से दुःखी थे। फरवरी, 1916 में उन्होंने बनारस हिन्दू विश्वविद्यालय में भाषण देते हुए कहा था कि “हमारी मुक्ति केवल किसानों के माध्यम से ही हो सकती है, न तो वकील, न डॉक्टर, न ही जमींदार इसे सुरक्षित रख सकते हैं।” गाँधीजी का कहना था कि हमारे लिए स्वशासन या स्वराज का तब तक कोई अर्थ नहीं है, जब तक हम किसानों से उनके श्रम का लगभग सम्पूर्ण लाभ स्वयं या अन्य लोगों को ले लेने की अनुमति देते रहेंगे।

(5) रोलेट एक्ट सत्याग्रह 1919 में गाँधीजी ने ‘रोलेट एक्ट’ के विरुद्ध सम्पूर्ण देश में सत्याग्रह चलाया। दिल्ली और अनेक शहरों में लोगों ने सरकार के विरुद्ध प्रदर्शन किया और विशाल जुलूस निकाले पंजाब जाते समय गाँधीजी को गिरफ्तार कर लिया गया। इसके अतिरिक्त हजारों कार्यकर्ताओं को जेलों में बंद कर दिया गया। उपर्युक्त आन्दोलनों का कुशल नेतृत्व करने के कारण गाँधीजी भारत के राजनीतिक आकाश पर छा गये और एक जननेता के रूप में प्रतिष्ठित हो गए। 1919 तक गाँधीजी ऐसे राष्ट्रवादी के रूप में उभर चुके थे, जिनमें गरीबों के प्रति गहरी सहानुभूति थी।

प्रश्न 9.
1930 में गाँधीजी द्वारा संचालित सविनय अवज्ञा आन्दोलन का वर्णन कीजिये। यह आन्दोलन कहाँ तक सफल हुआ?

अथवा
सविनय अवज्ञा आन्दोलन का वर्णन कीजिये। इसका हमारे स्वतन्त्रता संग्राम पर क्या प्रभाव पड़ा?
उत्तर:
सविनय अवज्ञा आन्दोलन गाँधीजी के नेतृत्व में संचालित सविनय अवज्ञा आन्दोलन के निम्नलिखित कारण थे –
1. साइमन कमीशन – 3 फरवरी, 1928 को साइमन कमीशन जब बम्बई पहुँचा तो उसका प्रबल विरोध हुआ क्योंकि इसमें कोई भी भारतीय सदस्य नहीं था। सरकार की दमनकारी नीति के कारण जनता में तीव्र आक्रोश व्याप्त था।

2. पूर्ण स्वराज्य की माँग-दिसम्बर, 1929 में पं. जवाहरलाल नेहरू की अध्यक्षता में लाहौर में कॉंग्रेस का अधिवेशन शुरू हुआ। 31 दिसम्बर, 1929 को अधिवेशन में पूर्ण स्वराज्य का प्रस्ताव पास किया गया। सविनय अवज्ञा आन्दोलन (दांडी यात्रा) का प्रारम्भ स्वतंत्रता दिवस मनाए जाने के तुरन्त बाद गाँधीजी ने घोषणा की कि वे ब्रिटिश भारत के सर्वाधिक घृणित ‘नमक- कानून’ को तोड़ने के लिए एक यात्रा का नेतृत्व करेंगे। नमक पर राज्य का एकाधिपत्य बहुत अलोकप्रिय था। लोगों को दुकानों से ऊँचे दाम पर नमक खरीदने के लिए बाध्य किया जाता था। यह राष्ट्रीय सम्पदा के लिए विनाशकारी था। नमक कर लोगों को बहुमूल्य सुलभ ग्राम उद्योग से वंचित करता था।

12 मार्च, 1930 को गाँधीजी ने अपने 78 आश्रमवासियों को साथ लेकर साबरमती आश्रम से दांडी (डाण्डी) नामक स्थान की ओर प्रस्थान किया। उन्होंने लगभग 200 मील की यात्रा पैदल चलकर 24 दिन में तब की 5 अप्रैल, 1930 को वे दाण्डी पहुँचे और 6 अप्रैल को वहाँ उन्होंने मुट्ठीभर नमक बनाकर कानून का उल्लंघन किया और सविनय अवज्ञा आन्दोलन शुरू किया।

सविनय अवज्ञा आन्दोलन की प्रगति –

  1. देश के विशाल भाग में किसानों ने दमनकारी औपनिवेशिक वन कानूनों का उल्लंघन किया।
  2. कुछ कस्बों में फैक्ट्री कामगार हड़ताल पर चले गये।
  3. वकीलों ने ब्रिटिश अदालतों का बहिष्कार किया।
  4. विद्यार्थियों ने सरकारी शिक्षा संस्थाओं में पढ़ने से इनकार कर दिया।
  5. विदेशी वस्तुओं का बहिष्कार करना, विदेशी वस्त्रों की होली जलाना तथा स्वदेशी का प्रयोग करना, सविनय अवज्ञा आन्दोलन का एक अन्य कार्यक्रम था।

(1) अंग्रेजों की साम्राज्यवादी नीति- सितम्बर, 1939 में द्वितीय विश्व युद्ध शुरू हो गया। कांग्रेस ने अंग्रेजों को युद्ध में समर्थन देने के लिए दो प्रमुख माँगें प्रस्तुत की –

  •  युद्ध की समाप्ति के बाद भारत को स्वतंत्रता प्रदान की जाए।
  • युद्धकाल में केन्द्र में भारतीयों की राष्ट्रीय सरकार का गठन किया जाए। ब्रिटिश सरकार ने इन मांगों को ठुकरा दिया। अन्ततः 1939 में 8 प्रांतों में कॉंग्रेसी मंत्रिमण्डलों ने त्याग पत्र दे दिया और उन्होंने अंग्रेजों के विरुद्ध आन्दोलन प्रारम्भ करने का निश्चय कर लिया।

(2) क्रिप्स मिशन की विफलता – द्वितीय विश्व युद्ध में काँग्रेस का सहयोग प्राप्त करने की दृष्टि से चर्चिल ने अपने एक मंत्री सर स्टेफर्ड क्रिप्स को भारत भेजा। क्रिप्स के साथ वार्ता में कांग्रेस ने इस बात पर जोर दिया कि अगर धुरी शक्तियों से भारत की रक्षा के लिए ब्रिटिश शासन काँग्रेस का समर्थन चाहता है तो वायसराय को सबसे पहले अपनी कार्यकारी परिषद में किसी भारतीय को एक रक्षा सदस्य के रूप में नियुक्त करना चाहिए। इसी बात पर वार्ता टूट गई। क्रिप्स मिशन की विफलता के बाद गाँधीजी ने ‘अंग्रेजो भारत छोड़ो’ आन्दोलन शुरू करने का फैसला लिया। भारत छोड़ो आन्दोलन का प्रारम्भ- 8 अगस्त, 1942 को मुम्बई में काँग्रेस के विशेष अधिवेशन में गाँधीजी का ‘भारत छोड़ो’ प्रस्ताव पास कर दिया गया।

JAC Class 12 History Important Questions Chapter 13 महात्मा गांधी और राष्ट्रीय आंदोलन : सविनय अवज्ञा और उससे आगे

आन्दोलन की प्रगति – सरकार ने भारत छोड़ो आन्दोलन की घोषणा के बाद 9 अगस्त, 1942 को ही गांधीजी, नेहरूजी आदि अनेक प्रमुख नेताओं को गिरफ्तार कर लिया। इसके बावजूद सम्पूर्ण भारत में हड़तालें और सरकार विरोधी प्रदर्शन हुए। काँग्रेस में जयप्रकाश नारायण जैसे समाजवादी सदस्य भूमिगत प्रतिरोध गतिविधियों में सबसे ज्यादा सक्रिय थे।
पश्चिम में सतारा और पूर्व में मेदिनीपुर जैसे कई जिलों में ‘स्वतंत्र’ सरकार (प्रति सरकार) की स्थापना कर दी गई थी।

आन्दोलन का अन्त- अंग्रेजों ने आन्दोलन के प्रति सख्त रवैया अपनाया, फिर भी इस विद्रोह को दबाने में सरकार को सालभर से अधिक समय लगा। भारत छोड़ो आन्दोलन का महत्त्व और परिणाम इस आन्दोलन के निम्न प्रमुख परिणाम निकले –

(1) भारत में राजनीतिक जागृति- भारत छोड़ो आन्दोलन सही मायने में एक जन-आन्दोलन था जिसमें लाखों आम हिन्दुस्तानी शामिल थे। इसके फलस्वरूप भारत में राजनीतिक जागृति में वृद्धि हुई। अब ब्रिटिश सरकार को पता चल गया कि अब वह भारत में अधिक दिनों तक शासन नहीं कर पाएगी।

(2) राष्ट्रीय आन्दोलन में युवकों का प्रवेश – इस आन्दोलन ने युवाओं को बड़ी संख्या में अपनी ओर आकर्षित किया। उन्होंने अपने कॉलेज छोड़कर जेल का रास्ता अपनाया।

(3) मुस्लिम लीग ने पंजाब व सिन्ध में अपनी पहचान बनाई – इस आन्दोलन के परिणामस्वरूप मुस्लिम लीग को पंजाब तथा सिन्ध में अपना प्रभाव बढ़ाने का अवसर मिला।

JAC Class 12 Political Science Important Questions Chapter 6 लोकतांत्रिक व्यवस्था का संकट

Jharkhand Board JAC Class 12 Political Science Important Questions Chapter 6 लोकतांत्रिक व्यवस्था का संकट Important Questions and Answers.

JAC Board Class 12 Political Science Important Questions Chapter 6 लोकतांत्रिक व्यवस्था का संकट

बहुचयनात्मक प्रश्न

1. जनता पार्टी के शासनकाल में भारत के प्रधानमंत्री कौन थे?
(क) चौ. देवीलाल
(ख) चौ. चरण सिंह
(ग) मोरारजी देसाई
(घ) ए.बी. वाजपेयी
उत्तर:
(ग) मोरारजी देसाई

2. श्रीमती इंदिरा गांधी ने भारत में आपातकाल की घोषणा कब की थी?
(क) 18 जून, 1975
(ख) 25 जून, 1975
(ग) 5 जुलाई, 1975
(घ) 10 जून, 1975
उत्तर:
(ख) 25 जून, 1975

3. भारत में प्रतिबद्ध नौकरशाही तथा प्रतिबद्ध न्यायपालिका की घोषणा को किसने जन्म दिया?
(क) इंदिरा गाँधी
(ख) लालबहादुर शास्त्री
(ग) मोरारजी देसाई
(घ) जवाहरलाल नेहरू
उत्तर:
(क) इंदिरा गाँधी

4. समग्र क्रान्ति के प्रतिपादक कौन थे?
(क) जयप्रकाश नारायण
(ख) मोरारजी देसाई
(ग) महात्मा गाँधी
(घ) गोपाल कृष्ण गोखले
उत्तर:
(क) जयप्रकाश नारायण

JAC Class 12 Political Science Important Questions Chapter 6 लोकतांत्रिक व्यवस्था का संकट

5. निम्न में से नक्सलवादी आन्दोलन से किसका सम्बन्ध है?
(क) सुरेश कलमाड़ी
(ख) चारु मजूमदार
(ग) ममता बैनर्जी
(घ) जयललिता
उत्तर:
(ख) चारु मजूमदार

6. आपातकाल के समय भारत के राष्ट्रपति कौन थे?
(क) ज्ञानी जेलसिंह
(ख) फखरुद्दीन अली अहमद
(ग) आर. वैंकटरमन
(घ) डॉ. राजेन्द्र प्रसाद
उत्तर:
(ख) फखरुद्दीन अली अहमद

7. शाह आयोग की स्थापना कब की गई ?
(क) 1977
(ख) 1978
(ग) 1985
(घ) 1980
उत्तर:
(क) 1977

8. 1971 के चुनाव में कांग्रेस ने कौनसा नारा दिया?
(क) जय जवान जय किसान
(ख) अच्छे दिन
(ग) गरीबी हटाओ
(घ) कट्टर सोच नहीं युवा जोश
उत्तर:
(ग) गरीबी हटाओ

9. आपातकाल का प्रावधान संविधान के किस अनुच्छेद में किया गया है?
(क) अनुच्छेद 350
(ख) अनुच्छेद 42
(ग) अनुच्छेद 351
(घ) अनुच्छेद 352
उत्तर:
(ग) गरीबी हटाओ

10. संविधान के अनुच्छेद 352 के अनुसार आपातकाल की घोषणा कौन कर सकता है?
(क) राष्ट्रपति
(ख) प्रधानमंत्री
(ग) लोकसभा अध्यक्ष
(घ) राज्यसभा अध्यक्ष
उत्तर:
(क) राष्ट्रपति

रिक्त स्थानों की पूर्ति कीजिए:

1. सामाजिक और सांप्रदायिक गड़बड़ी की आशंका के मद्देनजर सरकार ने आपातकाल के दौरान …………………और ……….. पर प्रतिबंध लगा दिया।
उत्तर:
राष्ट्रीय स्वयंसेवक संघ, जमात-ए-इस्लामी

2. …………………और ………………जैसे अखबारों ने प्रेस पर लगी सेंसरशिप का विरोध किया।
उत्तर:
इंडियन एक्सप्रेस, स्टेट्समैन

3. संविधान के 42वें अनुच्छेद में संशोधन करते हुए देश की विधायिका का कार्यकाल 6 से ………………..साल कर दिया गया।
उत्तर:
6

4. वर्तमान स्थिति में अंदरूनी आपातकाल सिर्फ ………………… की स्थिति में लगाया जा सकता है।
उत्तर:
सशस्त्र विद्रोह

5. काँग्रेस पार्टी में टूट के पश्चात् मोरारजी देसाई ……………………. पार्टी में शामिल हुए।
उत्तर:
काँग्रेस (ओ)

6. चौधरी चरण सिंह ने ……………………..पार्टी की स्थापना की।
उत्तर:
लोकदल

अतिलघूत्तरात्मक प्रश्न

प्रश्न 1.
लोकसभा का पाँचवाँ आम चुनाव किस वर्ष में हुआ था ?
उत्तर:
1971 में।
प्रश्न 2. भारत में आन्तरिक आपातकाल के समय प्रधानमंत्री कौन था ?
उत्तर:
श्रीमती इंदिरा गाँधी।

प्रश्न 3.
बिहार आन्दोलन के प्रमुख नेता कौन थे?
उत्तर:
जयप्रकाश नारायण।

JAC Class 12 Political Science Important Questions Chapter 6 लोकतांत्रिक व्यवस्था का संकट

प्रश्न 4.
1975 में आपातकाल संविधान के किस अनुच्छेद के अन्तर्गत लगाया गया?
अथवा
25 जून, 1975 को आपातकाल की घोषणा संविधान के किस अनुच्छेद के तहत की गई?
उत्तर:
अनुच्छेद 352 के अन्तर्गत।

प्रश्न 5.
आपातकाल लागू करने का तात्कालिक कारण क्या था?
उत्तर:
आपातकाल लागू करने का तात्कालिक कारण था। श्रीमती इन्दिरा गाँधी के निर्वाचन को इलाहाबाद हाईकोर्ट द्वारा अवैध घोषित करना तथा विपक्षी दलों द्वारा उनके इस्तीफे की माँग करना।

प्रश्न 6.
लोकतंत्र की बहाली का प्रतीक कौंन बना?
उत्तर:
जयप्रकाश नारायण।

प्रश्न 7.
प्रतिबद्ध न्यायपालिका से क्या अभिप्राय है?
उत्तर:
प्रतिबद्ध न्यायपालिका से अभिप्राय है कि न्यायपालिका शासक दल और उसकी नीतियों के प्रति निष्ठावान रहे।

प्रश्न 8.
शाह आयोग का गठन किसलिए किया गया?
उत्तर:
आपातकाल की जाँच हेतु।

प्रश्न 9.
भारत में आपातकाल की जाँच के लिए किस आयोग का गठन किया गया?
उत्तर:
शाह आयोग का।

प्रश्न 10.
1977 के चुनावों में कौनसी पार्टी की करारी हार हुई?
उत्तर:
कांग्रेस पार्टी की।

JAC Class 12 Political Science Important Questions Chapter 6 लोकतांत्रिक व्यवस्था का संकट

प्रश्न 11.
देश के प्रथम गैर-कांग्रेसी प्रधानमंत्री कौन थे?
उत्तर:
मोरारजी देसाई।

प्रश्न 12.
1975 में किस नेता ने सेना, पुलिस और सरकारी कर्मचारियों को आह्वान किया कि वे सरकार के अनैतिक व अवैधानिक आदेशों का पालन न करें।
उत्तर:
जयप्रकाश नारायण ने।

प्रश्न 13.
किस वर्ष नागरिक स्वतन्त्रता एवं लोकतान्त्रिक अधिकारों के लिए संघ का नाम बदल कर नागरिक स्वतन्त्रताओं के लिए लोगों का संघ रख दिया गया?
उत्त
सन् 1980 में।

प्रश्न 14.
1973 में किस न्यायाधीश को तीन वरिष्ठ न्यायाधीशों की अनदेखी करके भारत का मुख्य न्यायाधीश नियुक्त किया गया?
उत्तर:
न्यायाधीश ए. एन. रे।

प्रश्न 15.
केशवानंद भारती मुकदमे का निर्णय कब हुआ?
उत्तर:
सन् 1973 में।

प्रश्न 16.
किस मुकदमे में संविधान के मूलभूत ढाँचे की धारणा का जन्म हुआ?
उत्तर:
केशवानंद भारती मुकदमा।

JAC Class 12 Political Science Important Questions Chapter 6 लोकतांत्रिक व्यवस्था का संकट

प्रश्न 17.
जनता पार्टी की स्थापना कब हुई?
उत्तर:
सन् 1977।

प्रश्न 18.
1977 का चुनाव विपक्ष ने किस नारे से लड़ा?
उत्तर:
लोकतन्त्र बचाओ नारे से।

प्रश्न 19.
आपातकाल की अवधि कब तक रही?
उत्तर;
1975 से 1977 तक।

प्रश्न 20.
1975 में भारत में आपातकाल की घोषणा किसने की थी?
उत्तर:
1975 में भारत में आपातकाल की घोषणा श्रीमती इन्दिरा गाँधी की सिफारिश पर तत्कालीन राष्ट्रपति फखरुद्दीन अली अहमद ने की थी।

प्रश्न 21.
1974 की रेल हड़ताल के प्रमुख नेता कौन थे?
उत्तर
जार्ज फर्नांडिस ।

प्रश्न 22.
1970 के दशक के किन दो वर्षों में कीमतों में अधिक वृद्धि हुई?
उत्तर:
1973 और 1974 के दो वर्षों में।

JAC Class 12 Political Science Important Questions Chapter 6 लोकतांत्रिक व्यवस्था का संकट

प्रश्न 23.
1974 के बिहार आन्दोलन का प्रमुख नारा क्या था?
उत्तर:
” सम्पूर्ण क्रान्ति अब नारा है— भावी इतिहास हमारा है। ”

प्रश्न 24.
जनता पार्टी के पतन का कोई एक कारण बताएँ।
उत्तर:
जनता पार्टी की आन्तरिक गुटबाजी|

प्रश्न 25.
1977 के चुनावों में जनता पार्टी को कुल कितनी सीटें प्राप्त हुईं?
उत्तर:
कुल 295 सीटें प्राप्त हुईं?

प्रश्न 26.
1977 के चुनावों में कांग्रेस पार्टी को कितनी सीटें प्राप्त हुईं?
उत्तर:
कुल 154 सीटें मिलीं।

JAC Class 12 Political Science Important Questions Chapter 6 लोकतांत्रिक व्यवस्था का संकट

प्रश्न 27.
प्रेस सेंसरशिप से आप क्या समझते हैं?
अथवा
प्रेस सेंसरशिप क्या है?
उत्तर:
प्रेस सेंसरशिप के अन्तर्गत अखबारों को कोई भी खबर छापने से पहले उसकी अनुमति सरकार से लेना अनिवार्य है।

प्रश्न 28.
अनुच्छेद 352 क्या है?
उत्तर:
भारतीय संविधान के अनुच्छेद 352 के अन्तर्गत देश में आपातकाल की घोषणा की जा सकती है। 1962, 1971 एवं 1975 में की गई आपात की घोषणा अनुच्छेद 352 के अन्तर्गत ही की गई थी।

प्रश्न 29.
शाह आयोग की स्थापना का प्रमुख उद्देश्य क्या था?
उत्तर:
शाह आयोग की स्थापना आपातकाल के दौरान की गई कार्यवाही तथा सत्ता के दुरुपयोग, अतिचार और कदाचार के विविध पहलुओं की जाँच करने के लिए की गई।

प्रश्न 30.
1977 में स्वतन्त्रता से सम्बन्धित किस संगठन का निर्माण हुआ?
उत्तर:
1977 में स्वतन्त्रता से सम्बन्धित नागरिक स्वतन्त्रता एवं लोकतान्त्रिक अधिकारों के लिए लोगों के संघ का निर्माण हुआ।

JAC Class 12 Political Science Important Questions Chapter 6 लोकतांत्रिक व्यवस्था का संकट

प्रश्न 31.
किस भारतीय नेता ने वचनबद्ध नौकरशाही एवं वचनबद्ध न्यायपालिका की धारणा को जन्म दिया?
उत्तर:
श्रीमती इंदिरा गाँधी ने वचनबद्ध नौकरशाही एवं वचनबद्ध न्यायपालिका की धारणा का प्रतिपादन किया।

प्रश्न 32.
1980 के मध्यावधि चुनाव क्यों करवाने पड़े?
उत्तर:
जनता पार्टी की सरकार की अक्षमता एवं अस्थिरता के कारण 1980 के मध्यावधि चुनाव करवाने पड़े।

प्रश्न 33.
शाह आयोग के अनुसार निवारक नजरबंदी के कानून के तहत कितने लोगों को गिरफ्तार किया गया?
उत्तर:
शाह आयोग के अनुसार एक लाख ग्यारह हजार लोगों को गिरफ्तार किया गया।

प्रश्न 34.
जनता पार्टी के किन्हीं चार प्रमुख नेताओं के नाम लिखिए।
उत्तर:
जनता पार्टी के चार प्रमुख नेता थे। मोरारजी देसाई, चरणसिंह, जयप्रकाश नारायण, सिकन्दर बख्त।

प्रश्न 35.
समग्र क्रान्ति से क्या अभिप्राय है? इसके प्रतिपादक कौन थे?
उत्तर:
समग्र क्रान्ति का अर्थ है चारों तरफ परिवर्तन। इसके प्रतिपादक जयप्रकाश नारायण थे।

JAC Class 12 Political Science Important Questions Chapter 6 लोकतांत्रिक व्यवस्था का संकट

प्रश्न 36.
जयप्रकाश नारायण की समग्र क्रान्ति के चार पहलू कौन-कौनसे हैं?
उत्तर:
जयप्रकाश नारायण की समग्र क्रान्ति के चार पहलू हैं। संघर्ष, निर्माण, प्रचार, संगठन।

प्रश्न 37.
निवारक नजरबंदी से आपका क्या अभिप्राय है?
उत्तर:
निवारक नजरबंदी अधिनियम के अंतर्गत सरकार उन लोगों को हिरासत में ले सकती है जिन पर भविष्य में अपराध करने की आशंका होती है।

प्रश्न 38.
आपातकाल के विरोध का प्रतीक कौन बन गया था?
उत्तर:
जयप्रकाश नारायण।

प्रश्न 39.
रेल हड़ताल कब हुई थी?
उत्तर:
1974

प्रश्न 40.
किस हिंदी लेखक ने आपातकाल के विरोध में पद्मश्री की पदवी लौटा दी?
उत्तर:
फणीश्वरनाथ ‘रेणु’।

प्रश्न 41.
1975 के आपातकाल की वजह क्या बताई गई?
उत्तर:
अंदरूनी गड़बड़ी।

प्रश्न 42.
1977 के चुनाव में इंदिरा गांधी और संजय गाँधी ने चुनाव किस क्षेत्र से लड़े?
उत्तर:
1977 के चुनाव में इंदिरा रायबरेली से तथा संजय गाँधी अमेठी से चुनाव लड़े।

JAC Class 12 Political Science Important Questions Chapter 6 लोकतांत्रिक व्यवस्था का संकट

प्रश्न 43.
1971 के चुनाव में काँग्रेस पार्टी को कितनी सीटें मिलीं?
उत्तर:
353

प्रश्न 44.
1974 के बिहार आंदोलन का नारा क्या था?
उत्तर:
सम्पूर्ण क्रांति अब नारा है भावी इतिहास हमारा है।

प्रश्न 45.
1974 के मार्च महीने में बिहार के छात्रों द्वारा आंदोलन क्यों छेड़ा गया ?
उत्तर:
1974 के मार्च महीने में बढ़ती हुई कीमतों, खाद्यान्न के अभाव, बेरोजगारी और भ्रष्टाचार के खिलाफ बिहार के छात्रों ने आंदोलन छेड़ा।

लघूत्तरात्मक प्रश्न

प्रश्न 1.
1977 के चुनावों में कौन-कौनसी पार्टियाँ विजयी रहीं?
उत्तर:
1977 के लोकसभा चुनावों में जनता पार्टी विजयी रही। इस पार्टी की सरकार में मोरारजी देसाई भारत के प्रथम गैर-कांग्रेसी सरकार के प्रधानमंत्री बने। जनता पार्टी और उसके साथी दलों को लोकसभा की कुल 542 सीटों में से 330 सीटें मिलीं। खुद जनता पार्टी अकेले 295 सीटों पर विजयी रही और उसे स्पष्ट बहुमत मिला।

प्रश्न 2.
आपातकाल के कोई दो कारण बताइये।
उत्तर:
आपातकाल के कारण है।

  1. आंतरिक गड़बड़ी के आधार पर 1975 में आपातकाल घोषित किया गया।
  2. श्रीमती गाँधी के चुनावों को इलाहाबाद उच्च न्यायालय द्वारा अवैध घोषित करना तथा विपक्षी दलों द्वारा श्रीमती गाँधी से इस्तीफे की माँग करना।

प्रश्न 3.
जयप्रकाश नारायण की समग्र क्रान्ति पर संक्षिप्त नोट लिखिए।
उत्तर:
जयप्रकाश नारायण के अनुसार समग्र क्रान्ति का अर्थ है। चारों तरफ परिवर्तन। जयप्रकाश नारायण ने समग्र क्रान्ति के चार पहलू बताये निर्माण, प्रचार संगठन और संघर्ष वर्तमान स्थिति के सम्बन्ध में उनका मत था कि हमें निर्माण कार्य पर ध्यान देना होगा। युवा वर्ग दहेज, जाति-भेद, अस्पृश्यता, सम्प्रदायवाद आदि के विरुद्ध एकजुट होकर कार्य करें।

JAC Class 12 Political Science Important Questions Chapter 6 लोकतांत्रिक व्यवस्था का संकट

प्रश्न 4.
1974 की रेल हड़ताल पर संक्षिप्त टिप्पणी लिखिए।
उत्तर:
1974 में रेल की सबसे बड़ी और राष्ट्रव्यापी हड़ताल हुई। रेलवे कर्मचारियों के संघर्ष से संबंधित राष्ट्रीय समन्वय समिति ने जॉर्ज फर्नान्डिस के नेतृत्व में रेलवे कर्मचारियों की एक राष्ट्रव्यापी हड़ताल का आह्वान किया। बोनस और सेवा से जुड़ी शर्तों के संबंध में अपनी माँगों को लेकर सरकार पर दबाव बनाने के लिए हड़ताल का यह आह्वान किया गया था। सरकार इन माँगों के खिलाफ थी। ऐसे में भारत के इस सबसे बड़े सार्वजनिक उद्यम के कर्मचारी 1974 के मई महीने में हड़ताल पर चले गए।

प्रश्न 5.
वचनबद्ध न्यायपालिका से क्या अभिप्राय है?
उत्तर:
वचनबद्ध न्यायपालिका: ऐसी न्यायपालिका जो एक दल विशेष या सरकार विशेष के प्रति वफादार हो तथा उसके निर्देशों एवं आदेशों के अनुसार ही चले, उसे वचनबद्ध न्यायपालिका कहा जाता है।

प्रश्न 6.
भारतीय संविधान में न्यायपालिका की स्वतन्त्रता हेतु क्या-क्या प्रावधान किये गये हैं?
उत्तर:
न्यायपालिका की स्वतन्त्रता हेतु संवैधानिक उपबन्ध निम्न हैं।

  1. न्यायाधीशों की नियुक्ति राष्ट्रपति द्वारा की जाती है।
  2. न्यायाधीशों की योग्यता का संविधान में वर्णन किया गया है।
  3. न्यायाधीश एक निश्चित आयु पर सेवानिवृत्त होते हैं।
  4. न्यायाधीशों को केवल महाभियोग द्वारा ही पद से हटाया जा सकता है।

प्रश्न 7.
बिहार आन्दोलन क्या था?
उत्तर:
बिहार आन्दोलन:
बिहार आन्दोलन सन् 1974 में जयप्रकाश नारायण के नेतृत्व में चलाया गया। जयप्रकाश नारायण ने इसे पूर्ण या व्यापक क्रान्ति भी कहा है। जयप्रकाश ने 1975 में बिहार के लोगों को सम्बोधित करते हुए कहा था कि बिहार आन्दोलन का उद्देश्य समाज एवं व्यक्ति के सभी पक्षों में एक क्रान्तिकारी परिवर्तन लाना

प्रश्न 8.
गुजरात आन्दोलन 1974 को स्पष्ट कीजिए।
उत्तर:
1974 के जनवरी माह में गुजरात के छात्रों ने खाद्यान्न, खाद्य तेल तथा अन्य आवश्यक वस्तुओं की बढ़ती हुई कीमत तथा उच्च पदों पर जारी भ्रष्टाचार के खिलाफ आन्दोलन छेड़ दिया। इस आन्दोलन में बड़ी राजनीतिक पार्टियाँ भी शरीक हो गईं। फलतः गुजरात में राष्ट्रपति शासन लगा दिया गया।

JAC Class 12 Political Science Important Questions Chapter 6 लोकतांत्रिक व्यवस्था का संकट

प्रश्न 9.
1977 के चुनावों में जनता पार्टी की जीत के कोई दो कारण लिखिए।
अथवा
1977 के चुनावों में कांग्रेस पार्टी की पराजय के किन्हीं दो कारणों का उल्लेख कीजिए।
उत्तर:
1977 में जनता पार्टी की जीत व कांग्रेस की हार के कारण निम्न हैं।

  1. इन्दिरा गाँधी की घटती लोकप्रियता: श्रीमती गाँधी द्वारा की गई आपातकाल की घोषणा से उनकी लोकप्रियता घट गई थी। इससे कांग्रेस की हार हुई
  2. जयप्रकाश नारायण का व्यक्तित्व: जयप्रकाश नारायण इस दौर के सबसे करिश्माई व्यक्तित्व थे। उन्हें अपार जन समर्थन प्राप्त था। जनता पार्टी को जिताने में उनका महत्त्वपूर्ण योगदान रहा।

प्रश्न 10.
संक्षेप में बताइए कि 1975-76 के 18 माह के आपातकाल के दौरान क्या-क्या हुआ था?
उत्तर:
इंदिरा सरकार ने गरीबों के हित के लिए बीस सूत्री कार्यक्रम की घोषणा की और उसे तेजी से लागू करने का प्रयास किया। इस कार्यक्रम में भूमि सुधार, भू-पुनर्वितरण, खेतीहर मजदूरों के पारिश्रमिक पर पुनर्विचार, प्रबंधन में कामगारों की भागीदारी, बंधुआ मजदूरी की समाप्ति इत्यादि मामले शामिल थे। देश के बड़े नेताओं को गिरफ्तार किया गया। प्रेस पर कई तरह की पाबंदी लगाई। प्रेस सेंसरशिप का बड़ा ढाँचा तैयार किया । झुग्गी-झोंपड़ियों को हटा दिया गया। इस काल के दौरान पुलिस की यातनाएँ और पुलिस हिरासत में यातनाएँ दी गईं। अनिवार्य रूप से नसबंदी के कार्यक्रम चलाए गए।

प्रश्न 11.
नागरिक स्वतन्त्रता के संगठनों की किन्हीं दो समस्याओं का वर्णन कीजिए।
उत्तर:

  1. प्रायः नागरिक स्वतन्त्रता के संगठनों को राष्ट्रविरोधी कहकर इनकी आलोचना की जाती है।
  2. कुछ तथाकथित ऐसे समाज सुधार भी इन संगठनों से जुड़ गए हैं, जिनके लिए यह केवल एक व्यवसाय है।

प्रश्न 12.
चारू मजूमदार कौन थे?
उत्तर:
चारू मजूमदार:
चारु मजूमदार एक क्रान्तिकारी समाजवादी नेता थे। उनका जन्म 1918 में हुआ। उन्होंने सी. पी. आई. पार्टी छोड़कर कम्युनिस्ट पार्टी ऑफ इण्डिया (मार्क्सवादी-लेनिनवादी) की स्थापना की। यह पार्टी नक्सलवादी आन्दोलन की प्रेरणास्रोत बनी। वे क्रान्तिकारी हिंसा के समर्थक थे।

JAC Class 12 Political Science Important Questions Chapter 6 लोकतांत्रिक व्यवस्था का संकट

प्रश्न 13.
लोकनायक जयप्रकाश नारायण का संक्षिप्त परिचय दीजिए।
उत्तर:
लोकनायक जयप्रकाश नारायण का जन्म 1902 में हुआ। उन्होंने कांग्रेस सोशलिस्ट पार्टी की स्थापना की । उन्होंने भारत छोड़ो आन्दोलन में हिस्सा लिया । स्वतन्त्रता के बाद उन्होंने नेहरू मन्त्रिमण्डल में शामिल होने से मना किया। वे बिहार आन्दोलन के प्रमुख नेता थे। 1979 में उनकी मृत्यु हो गई।

प्रश्न 14.
जगजीवन राम पर संक्षिप्त टिप्पणी लिखिए।
उत्तर:
जगजीवन राम: जगजीवन राम भारत के महान् स्वतन्त्रता सेनानी और बिहार राज्य के उच्चकोटि के कांग्रेसी नेता थे। इनका जन्म 1908 में हुआ। ये स्वतन्त्र भारत के पहले केंद्रीय मंत्रिमण्डल में श्रम मंत्री बने। 1952 से 1977 तक उन्होंने अनेक मंत्रालयों की जिम्मेदारी निभाई। वे देश की संविधान सभा के सदस्य थे। वे 1952 से लेकर मृत्युपर्यन्त तक सांसद रहे। 1977 से 1979 तक देश के उपप्रधानमंत्री पद पर रहे। उन्होंने अपना सम्पूर्ण जीवन दलितों की सेवा में बिताया और उनकी सेवा के लिए हमेशा तैयार रहते थे। 1986 में उनका निधन हो गया।

प्रश्न 15.
चौधरी चरणसिंह के जीवन पर संक्षिप्त नोट लिखिए।
उत्तर:
चौधरी चरणसिंह: चौधरी चरणसिंह का जन्म 1902 में हुआ। वे महान् स्वतन्त्रता सेनानी और प्रारम्भ में उत्तर प्रदेश की राजनीति में सक्रिय रहे। वे ग्रामीण एवं कृषि विकास की नीति और कार्यक्रमों के कट्टर समर्थक थे। 1967 में कांग्रेस पार्टी को छोड़कर उन्होंने भारतीय क्रान्ति दल का गठन किया। वे दो बार उत्तर प्रदेश के मुख्यमंत्री बने वे जयप्रकाश के क्रांति आन्दोलन से जुड़े और 1977 में जनता पार्टी के संस्थापकों में से एक थे। 1977 से 1979 तक वे भारत के उपप्रधानमंत्री और गृह मंत्री रहे। उन्होंने लोक दल की स्थापना की। वे कुछ महीनों के लिए जुलाई, 1979 से जनवरी, 1980 के बीच भारत के प्रधानमंत्री रहे। चौधरी चरणसिंह का निधन 1987 में हुआ।

प्रश्न 16.
मोरारजी देसाई का संक्षिप्त परिचय दीजिए।
उत्तर:
मोरारजी देसाई: मोरारजी देसाई एक महान् स्वतन्त्रता सेनानी और गांधीवादी नेता थे। इनका जन्म 1896 में हुआ। वे बम्बई (वर्तमान में मुम्बई ) राज्य के मुख्यमंत्री रहे। उन्होंने 1966 में कांग्रेस संसदीय पार्टी का नेतृत्व सम्भाला। वे 1967-1969 तक देश के उपप्रधानमंत्री रहे । वे कांग्रेस पार्टी में सिंडिकेट के एक प्रमुख सदस्य थे। बाद में जनता पार्टी के द्वारा प्रधानमंत्री निर्वाचित हुए और वे देश में पहले गैर-कांग्रेसी प्रधानमंत्री बने। उन्होंने 1977 से 1979 के मध्य लगभग 18 महीने इस पद पर कार्य किया। 1995 में उनका देहान्त हो गया।

प्रश्न 17.
प्रतिबद्ध नौकरशाही पर संक्षिप्त टिप्पणी लिखिए।
उत्तर:
प्रतिबद्ध नौकरशाही: प्रतिबद्ध नौकरशाही का अर्थ है कि नौकरशाही किसी विशिष्ट राजनीतिक दल के सिद्धान्तों एवं नीतियों से बंधी हुई रहती है और उस दल के निर्देशन में ही कार्य करती है। प्रतिबद्ध नौकरशाही निष्पक्ष एवं स्वतन्त्र होकर कार्य नहीं करती। इसका कार्य किसी दल विशेष की योजनाओं को बिना कोई प्रश्न उठाए आँखें मूंद कर लागू करना होता है। लोकतान्त्रिक देशों में नौकरशाही प्रतिबद्ध नहीं होती। परन्तु साम्यवादी देशों में जैसे कि चीन में वचनबद्ध नौकरशाही पायी जाती है। भारत में प्रतिबद्ध नौकरशाही से आशय किसी दल के सिद्धान्तों के प्रति वचनबद्ध न होकर संविधान के प्रति वचनबद्धता ह ।

प्रश्न 18.
भारत में वचनबद्ध न्यायपालिका की धारणा का उदय कैसे हुआ?
उत्तर:
भारत में वचनबद्ध न्यायपालिका का उदय: केशवानन्द भारती मुकदमे की सुनवाई सर्वोच्च न्यायालय की एक 13 सदस्यीय संविधान पीठ ने की। 13 में से 9 न्यायाधीशों ने यह निर्णय दिया कि संसद मौलिक अधिकारों सहित संविधान में संशोधन कर सकती है, परन्तु संविधान के मूलभूत ढाँचे में परिवर्तन नहीं कर सकती। इस निर्णय से सरकार एवं न्यायपालिका में मतभेद बढ़ गए, क्योंकि 1973 में सरकार का नेतृत्व श्रीमती इंदिरा गाँधी कर रही थीं। अतः यह विवाद श्रीमती गांधी एवं न्यायालय के बीच हुआ जिसमें जीत न्यायालय की हुई क्योंकि न्यायालय ने संसद की संविधान में संशोधन करने की शक्ति को सीमित कर दिया। इसी कारण श्रीमती गाँधी ने वचनबद्ध न्यायपालिका की धारणा को आगे बढ़ाया। 1975 में आपातकाल के समय वचनबद्ध न्यायपालिका का सिद्धान्त कार्यपालिका का सिद्धान्त बन गया।

JAC Class 12 Political Science Important Questions Chapter 6 लोकतांत्रिक व्यवस्था का संकट

प्रश्न 19.
भारत में वचनबद्ध (प्रतिबद्ध ) न्यायपालिका के लिए सरकार द्वारा प्रयोग किये गये किन्हीं तीन उपायों का वर्णन करें।
अथवा
भारत में वचनबद्ध न्यायपालिका की धारणा को उदाहरण सहित समझाइये
उत्तर:
भारत में वचनबद्ध न्यायपालिका के उदाहरण- भारत में वचनबद्ध न्यायपालिका के प्रमुख उदाहरण निम्नलिखित हैं।

  1. न्यायाधीशों की नियुक्ति में वरिष्ठता की अनदेखी: श्रीमती गाँधी ने वचनबद्ध न्यायपालिका के लिए न्यायाधीशों की नियुक्ति में तीन वरिष्ठ न्यायाधीशों की वरिष्ठता की अनदेखी करके श्री ए. एन. रे को सर्वोच्च न्यायालय का मुख्य न्यायाधीश नियुक्त किया।
  2. न्यायाधीशों का स्थानान्तरण: श्रीमती गाँधी ने वचनबद्ध न्यायपालिका के लिए न्यायाधीशों के स्थानान्तरण का सहारा लिया। उन्होंने 1981 में मद्रास उच्च न्यायालय के न्यायाधीश इस्माइल को केरल उच्च न्यायालय का मुख्य न्यायाधीश बनाकर भेजा।
  3. अन्य पदों पर नियुक्तियाँ: सरकार ने सेवानिवृत्त न्यायाधीशों में से उन्हें राज्यपाल, राजदूत, मन्त्री या किसी आयोग का अध्यक्ष नियुक्त किया, जो सरकार के प्रति वफादार थ।

प्रश्न 20.
उन कारकों का उल्लेख कीजिए जिनके कारण पिछड़े राज्यों में नक्सली आन्दोलन हुआ।
उत्तर:

  1. बंधुआ मजदूरी।
  2. जमींदारों द्वारा शोषण।
  3. बाहरी लोगों द्वारा संसाधनों का शोषण।

JAC Class 12 Political Science Important Questions Chapter 6 लोकतांत्रिक व्यवस्था का संकट

प्रश्न 21.
भारत के एक जिम्मेदार नागरिक के रूप में आप आपातकाल की आलोचना किन आधारों पर करते हैं?
अथवा
भारतीय लोकतंत्र पर आपातकाल के कोई चार दुष्प्रभाव बताइये।
उत्तर:
भारतीय लोकतंत्र पर आपातकाल के दुष्प्रभाव: भारतीय लोकतंत्र पर आपातकाल के निम्नलिखित दुष्प्रभाव पड़े; जिनके कारण हम आपातकाल की आलोचना करते हैं।

  1. लोकतांत्रिक कार्यप्रणाली का ठप होना- आपातकाल में लोगों को सार्वजनिक तौर पर सरकार के विरोध करने की लोकतांत्रिक कार्यप्रणाली को ठप कर दिया गया।
  2. निवारक नजरबंदी कानून का दुरुपयोग – आपातकाल में निवारक नजरबंदी कानून का दुरुपयोग करते हुए लगभग 1 लाख 11 हजार लोगों को गिरफ्तार किया गया। इन्हें अपनी गिरफ्तारी की चुनौती का अधिकार भी नहीं दिया गया।
  3. प्रेस पर नियंत्रण: आपातकाल के दौरान सरकार ने प्रेस की आजादी पर रोक लगा दी।
  4. संविधान का 42वाँ संशोधन: आपातकाल के दौरान ही संविधान का 42वाँ संशोधन पारित हुआ । इसके जरिये संविधान के अनेक हिस्सों में बदलाव किये गये।

प्रश्न 22.
आपातकाल के सबकों का उल्लेख कीजिए।
उत्तर:
आपातकाल के सबक: आपातकाल से भारतीय राजनीतिक व्यवस्था को निम्न सबक मिले-

  1. विरोधी दलों और मतदाताओं ने जितनी अपनी राजनीतिक जागृति दिखाई, इससे साबित हो गया कि बड़े से बड़ा तानाशाह नेता भी भारत से लोकतन्त्र विदा नहीं कर सकता।
  2. आपातकाल के बाद संविधान में अच्छे सुधार किए गए। अब आपातकाल सशक्त स्थिति में लगाया जा सकता था । ऐसा तभी हो सकता था जब मन्त्रिमण्डल लिखित रूप से राष्ट्रपति को ऐसा परामर्श दे।
  3. आपातकाल में भी न्यायालयों में व्यक्ति के नागरिक अधिकारों की रक्षा करने की भूमिका सक्रिय रहेगी और नागरिक अधिकारों की रक्षा तत्परता से होने लगी।

प्रश्न 23.
जनता पार्टी की सरकार पर एक संक्षिप्त टिप्पणी लिखिए।
उत्तर:
जनता पार्टी की सरकार- जनता पार्टी की सरकार के सम्बन्ध में निम्नलिखित विचार प्रकट किये जा सकते हैं।

  1. 1977 के चुनावों के बाद बनी जनता पार्टी की सरकार में कोई तालमेल नहीं था। पहले प्रधानमंत्री के पद को लेकर मोरारजी देसाई, चरणसिंह और जगजीवन राम में खींचतान हुई। अंतत: मोरारजी देसाई प्रधानमंत्री बने।
  2. जनता पार्टी अनेक राजनैतिक दलों का संगठन था। इन दलों में निरन्तर खींचा-तानी बनी रही और यह दल कुछ ही महीनों तक अपनी एकता बनाए रख सका।
  3. इस पार्टी की सरकार के पास एक निश्चित दिशा या दीर्घकालीन कल्याणकारी बहुजनप्रिय कार्यक्रम या सर्वमान्य नेतृत्व या न्यूनतम साध्य कार्यक्रम नहीं था।
  4. 18 मास के पश्चात् मोरारजी देसाई को त्यागपत्र देना पड़ा। 1980 के नए चुनावों में जनता पार्टी की हार हुई और कांग्रेस पुनः सत्ता में आयी।

JAC Class 12 Political Science Important Questions Chapter 6 लोकतांत्रिक व्यवस्था का संकट

प्रश्न 24.
विरोधी दलों के विरोध तथा कांग्रेस की टूट ने आपातकाल की पृष्ठभूमि कैसे तैयार की?
उत्तर:
आपातकाल की पृष्ठभूमि के कारण:

  1. 1967 के चुनावों के बाद कुछ प्रान्तों में विरोधी दलों या संयुक्त विरोधी दलों की सरकार बनी। वे केन्द्र में सत्ता में आना चाहते थे।
  2. कांग्रेस के विपक्ष में जो दल थे उन्हें लग रहा था कि सरकारी प्राधिकार को निजी प्राधिकार मान कर इस्तेमाल किया जा रहा है और राजनीति हद से ज्यादा व्यक्तिगत होती जा रही है।
  3. कांग्रेस टूट से इंदिरा गाँधी और उनके विरोधियों के बीच मतभेद गहरे हो गये थे।
  4. इस अवधि में न्यायपालिका और सरकार के आपसी रिश्तों में भी तनाव आए। सर्वोच्च न्यायालय ने सरकार की कई पहलकदमियों को संविधान के विरुद्ध माना सरकार ने न्यायपालिका को प्रगति विरोधी बताया तथा 1975 में इलाहाबाद उच्च न्यायालय ने श्रीमती इन्दिरा गाँधी के चुनाव को अवैध करार दिया था।
  5. जयप्रकाश नारायण समग्र क्रान्ति की बात कर रहे थे। ऐसी सभी घटनाओं ने आपातकाल के लिए पृष्ठभूमि तैयार की।

प्रश्न 25.
“ 1967 के बाद देश की कार्यपालिका तथा न्यायपालिका के सम्बन्धों में आए तनावों ने आपातकाल की पृष्ठभूमि तैयार की थी।” इस कथन को संक्षेप में स्पष्ट कीजिए।
उत्तर;
1967 के बाद भारत में श्रीमती इंदिरा गाँधी एक कद्दावर नेता के रूप में उभरीं और 1973-74 तक उनकी लोकप्रियता अपने चरम पर थी लेकिन इस दौर में दलगत प्रतिस्पर्धा कहीं ज्यादा तीखी और ध्रुवीकृत हो चली थी। इस अवधि में न्यायपालिका और सरकार के सम्बन्धों में तनाव आए सर्वोच्च न्यायालय ने सरकार की कई पहलकदमियों को संविधान के विरुद्ध माना कांग्रेस पार्टी का मानना था कि अदालत का यह रवैया लोकतन्त्र के सिद्धान्तों और संसद की सर्वोच्चता के विरुद्ध है। कांग्रेस ने यह आरोप भी लगाया कि अदालत एक यथास्थितिवादी संस्था है और यह संस्था गरीबों को लाभ पहुँचाने वाले कल्याण कार्यक्रमों को लागू करने की राह में रोड़ा अटका रही है।

प्रश्न 26.
आपातकाल के संवैधानिक एवं उत्तर संवैधानिक पक्षों का वर्णन कीजिए।
उत्तर:
आपातकाल के संवैधानिक एवं उत्तर: संवैधानिक पक्ष- आपातकाल के समय कुछ संवैधानिक एवं उत्तर संवैधानिक पक्ष भी सामने आए। श्रीमती गांधी ने संविधान में 39वाँ संवैधानिक संशोधन किया। इस संशोधन द्वारा राष्ट्रपति, उपराष्ट्रपति, प्रधानमंत्री एवं स्पीकर के चुनाव से सम्बन्धित मुकदमों की सुनवाई की सर्वोच्च न्यायालय की शक्ति समाप्त कर दी गई।

इस संशोधन को पास करने का मुख्य उद्देश्य श्रीमती गाँधी को इलाहाबाद उच्च न्यायालय द्वारा दिये गए निर्णय से राहत दिलाना था।  विरोधी पक्ष ने 39 वें संशोधन को संविधान के मूल ढांचे के विरुद्ध बताया परन्तु उच्च न्यायालय की पीठ के पाँच में से चार न्यायाधीशों ने 39वें संशोधन को वैध ठहराया तथा इस संशोधन के आधार पर श्रीमती गाँधी के निर्वाचन को पूर्ण रूप से वैध ठहराया।

प्रश्न 27.
बिहार आन्दोलन की घटनाओं की संक्षिप्त व्याख्या कीजिए।
उत्तर:
बिहार आन्दोलन: बिहार आन्दोलन प्रशासन में भ्रष्टाचारी एवं अयोग्य कर्मचारियों के विरुद्ध लोकनायक जयप्रकाश नारायण द्वारा चलाया गया आन्दोलन था। 1974 में इस आन्दोलन का श्रीगणेश हुआ । इस आन्दोलन का मुख्य उद्देश्य समाज एवं व्यक्ति के सभी पक्षों में एक क्रान्तिकारी परिवर्तन लाना है। इस उद्देश्य की प्राप्ति के लिए इस आन्दोलन को एक लम्बे समय तक चलाए जाने पर बल दिया गया था।

बिहार आन्दोलन में अनुसूचित जातियों एवं अनुसूचित जनजातियों की सामाजिक एवं आर्थिक समस्याओं को भी हल करने का प्रयास किया गया। जयप्रकाश नारायण ने बिहार आन्दोलन के चार पक्षों का वर्णन किया  प्रथम संघर्ष, द्वितीय निर्माण, तृतीय प्रचार तथा चतुर्थ संगठन । जयप्रकाश नारायण ने तत्कालीन परिस्थितियों में निर्माण कार्य पर अधिक जोर दिया।

JAC Class 12 Political Science Important Questions Chapter 6 लोकतांत्रिक व्यवस्था का संकट

प्रश्न 28.
आपातकाल के संदर्भ में विवादों को स्पष्ट कीजिये।
उत्तर:
आपातकाल के संदर्भ में विवाद: आपातकाल के संदर्भ में सरकार तथा विपक्षी दलों के बीच विवाद निम्नलिखित थे।
(अ) आपातकाल लगाना उचित था।: आपातकाल के औचित्य के सम्बन्ध में सरकार के तर्क ये थे:

  1. भारत में लोकतंत्र है और विपक्षी दलों को निर्वाचित शासक दल अपनी नीतियों के अनुसार शासन चलाने दें। देश में लगातार गैर-संसदीय राजनीति से अस्थिरता पैदा होती है।
  2. षडयंत्रकारी ताकतें सरकार के प्रगतिशील कार्यक्रमों में अडंगे लगा रही थीं।
  3. ये ताकतें श्रीमती गाँधी को गैर-संवैधानिक साधनों के बूते सत्ता से बेदखल करना चाहती थी।

(ब) आपातकाल लगाना अनुचित था आपातकाल लगाना अनुचित था। इसके सम्बन्ध में विपक्षी दलों के तर्क येथे

  1. भारत में स्वतंत्रता के आंदोलन से लेकर लगातार भारत में जन आंदोलन का एक सिलसिला रहा है तथा लोकतंत्र में लोगों को सार्वजनिक तौर पर सरकार के विरोध का अधिकार होना चाहिए। इनके कारण आपातकाल लगाना अनुचित था। किया।
  2. जन आंदोलनों से खतरा देश की एकता और अखंडता को नहीं, बल्कि शासक दल और प्रधानमंत्री को था।
  3. श्रीमती इन्दिरा गाँधी ने निजी ताकतों को बचाने के लिए संवैधानिक आपातकालीन प्रावधानों का दुरुपयोग अतः आपातकाल लागू करना अनुचित था।

प्रश्न 29.
नक्सली आंदोलन के दो परिणामों का उल्लेख कीजिए।
उत्तर:
नक्सली आंध्रप्रदेश, पश्चिमी बंगाल, बिहार और आसपास के क्षेत्रों में मार्क्सवादी और लेनिनवादी कृषि कार्यकर्ता थे जिन्होंने आर्थिक अन्याय और असमानता के खिलाफ बड़े पैमाने पर आंदोलन किए और किसानों को भूमि का पुनर्वितरण करने की माँग की।

प्रश्न 30.
बिहार में 1974 के छात्र आंदोलन के कारकों को स्पष्ट कीजिए। जयप्रकाश नारायण ने इस आंदोलन के लिए क्या शर्तें रखीं?
उत्तर:
बिहार आंदोलन के निम्न कारण थे।
बढ़ती हुई कीमत। सर्वोच्चता के विरुद्ध है। कांग्रेस ने यह आरोप भी लगाया कि अदालत एक यथास्थितिवादी संस्था है और यह संस्था गरीबों को लाभ पहुँचाने वाले कल्याण कार्यक्रमों को लागू करने की राह में रोड़ा अटका रही है।

प्रश्न 26.
आपातकाल के संवैधानिक एवं उत्तर संवैधानिक पक्षों का वर्णन कीजिए।
उत्तर:
आपातकाल के संवैधानिक एवं उत्तर: संवैधानिक पक्ष- आपातकाल के समय कुछ संवैधानिक एवं उत्तर संवैधानिक पक्ष भी सामने आए। श्रीमती गांधी ने संविधान में 39वाँ संवैधानिक संशोधन किया। इस संशोधन द्वारा राष्ट्रपति, उपराष्ट्रपति, प्रधानमंत्री एवं स्पीकर के चुनाव से सम्बन्धित मुकदमों की सुनवाई की सर्वोच्च न्यायालय की शक्ति समाप्त कर दी गई। इस संशोधन को पास करने का मुख्य उद्देश्य श्रीमती गाँधी को इलाहाबाद उच्च न्यायालय द्वारा दिये गए निर्णय से राहत दिलाना था। विरोधी पक्ष ने 39 वें संशोधन को संविधान के मूल ढांचे के विरुद्ध बताया परन्तु उच्च न्यायालय की पीठ के पाँच में से चार न्यायाधीशों ने 39वें संशोधन को वैध ठहराया तथा इस संशोधन के आधार पर श्रीमती गाँधी के निर्वाचन को पूर्ण रूप से वैध ठहराया।

JAC Class 12 Political Science Important Questions Chapter 6 लोकतांत्रिक व्यवस्था का संकट

प्रश्न 27.
बिहार आन्दोलन की घटनाओं की संक्षिप्त व्याख्या कीजिए।
उत्तर:
बिहार आन्दोलन: बिहार आन्दोलन प्रशासन में भ्रष्टाचारी एवं अयोग्य कर्मचारियों के विरुद्ध लोकनायक जयप्रकाश नारायण द्वारा चलाया गया आन्दोलन था। 1974 में इस आन्दोलन का श्रीगणेश हुआ। इस आन्दोलन का मुख्य उद्देश्य समाज एवं व्यक्ति के सभी पक्षों में एक क्रान्तिकारी परिवर्तन लाना है। इस उद्देश्य की प्राप्ति के लिए इस आन्दोलन को एक लम्बे समय तक चलाए जाने पर बल दिया गया था।

बिहार आन्दोलन में अनुसूचित जातियों एवं अनुसूचित जनजातियों की सामाजिक एवं आर्थिक समस्याओं को भी हल करने का प्रयास किया गया। जयप्रकाश नारायण ने बिहार आन्दोलन के चार पक्षों का वर्णन किया प्रथम संघर्ष, द्वितीय निर्माण, तृतीय प्रचार तथा चतुर्थ संगठन। जयप्रकाश नारायण ने तत्कालीन परिस्थितियों में निर्माण कार्य पर अधिक जोर दिया।

प्रश्न 28.
आपातकाल के संदर्भ में विवादों को स्पष्ट कीजिये।
उत्तर:
आपातकाल के संदर्भ में विवाद: आपातकाल के संदर्भ में सरकार तथा विपक्षी दलों के बीच विवाद निम्नलिखित थे।
(अ) आपातकाल लगाना उचित था आपातकाल के औचित्य के सम्बन्ध में सरकार के तर्क ये थे।

  1. भारत में लोकतंत्र है और विपक्षी दलों को निर्वाचित शासक दल अपनी नीतियों के अनुसार शासन चलाने दें। देश में लगातार गैर-संसदीय राजनीति से अस्थिरता पैदा होती है।
  2. षडयंत्रकारी ताकतें सरकार के प्रगतिशील कार्यक्रमों में अडंगे लगा रही थीं।
  3. ये ताकतें श्रीमती गाँधी को गैर-संवैधानिक साधनों के बूते सत्ता से बेदखल करना चाहती थी।

(ब) आपातकाल लगाना अनुचित था। आपातकाल लगाना अनुचित था। इसके सम्बन्ध में विपक्षी दलों के तर्क येथे

  1. भारत में स्वतंत्रता के आंदोलन से लेकर लगातार भारत में जन आंदोलन का एक सिलसिला रहा है। तथा लोकतंत्र में लोगों को सार्वजनिक तौर पर सरकार के विरोध का अधिकार होना चाहिए। इनके कारण आपातकाल लगाना अनुचित था।
  2. जन आंदोलनों से खतरा देश की एकता और अखंडता को नहीं, बल्कि शासक दल और प्रधानमंत्री को था।
  3. श्रीमती इन्दिरा गाँधी ने निजी ताकतों को बचाने के लिए संवैधानिक आपातकालीन प्रावधानों का दुरुपयोग अतः आपातकाल लागू करना अनुचित था।

प्रश्न 29.
नक्सली आंदोलन के दो परिणामों का उल्लेख कीजिए।
उत्तर:
नक्सली आंध्रप्रदेश, पश्चिमी बंगाल, बिहार और आसपास के क्षेत्रों में मार्क्सवादी और लेनिनवादी कृषि कार्यकर्ता थे जिन्होंने आर्थिक अन्याय और असमानता के खिलाफ बड़े पैमाने पर आंदोलन किए और किसानों को भूमि का पुनर्वितरण करने की माँग की।

प्रश्न 30.
बिहार में 1974 के छात्र आंदोलन के कारकों को स्पष्ट कीजिए। जयप्रकाश नारायण ने इस आंदोलन के लिए क्या शर्तें रखीं?
उत्तर:
बिहार आंदोलन के निम्न कारण थे।

  1. बढ़ती हुई कीमत।
  2. खाद्यान्न के अभाव।
  3. बेरोजगारी और भ्रष्टाचार में वृद्धि।

छात्रों ने अपने आंदोलन की अगुवाई के लिए जयप्रकाश नारायण को बुलावा भेजा। जेपी ने छात्रों का निमंत्रण इस शर्त पर स्वीकार किया कि आंदोलन अहिंसक रहेगा और अपने को सिर्फ बिहार तक सीमित नहीं रखेगा।

JAC Class 12 Political Science Important Questions Chapter 6 लोकतांत्रिक व्यवस्था का संकट

प्रश्न 31.
गुजरात के छात्र आंदोलन के क्या परिणाम हुए?
उत्तर:
गुजरात के छात्र आंदोलन के राष्ट्रीय स्तर की राजनीति पर दूरगामी प्रभाव हुए। इस आंदोलन में बड़ी राजनीतिक पार्टियाँ भी शरीक हो गईं। इस प्रकार आंदोलन ने विकराल रूप धारण कर लिया। ऐसे में गुजरात में राष्ट्रपति शासन लगा दिया गया। राज्य में दुबारा विधानसभा के चुनाव की माँग उठने लगी।

प्रश्न 32.
जयप्रकाश नारायण द्वारा बिहार के छात्रों के आंदोलन में साथ देने का क्या परिणाम हुआ?
उत्तर:
जयप्रकाश नारायण द्वारा बिहार आंदोलन के साथ जुड़ते ही इस आंदोलन ने राजनीतिक चरित्र ग्रहण किया और राष्ट्रव्यापी अपील आई। जीवन के हर क्षेत्र के लोग आंदोलन से जुड़ गए। बिहार की कांग्रेस सरकार को बर्खास्त करने की माँग की। बिहार की सरकार के खिलाफ लगातार घेराव, बंद और हड़ताल का एक सिलसिला चल पड़ा। इस आंदोलन का प्रभाव राष्ट्रीय राजनीति पर भी पड़ा।

प्रश्न 33.
आपातकाल की घोषणा के साथ राजनीति में क्या बदलाव आता है?
उत्तर:
आपातकाल की घोषणा के साथ ही शक्तियों के बँटवारे का संघीय ढाँचा व्यावहारिक तौर पर निष्प्रभावी हो जाता है और सारी शक्तियाँ केन्द्र सरकार के हाथ में चली आती हैं। दूसरे, सरकार चाहे तो ऐसी स्थिति में किसी एक अथवा सभी मौलिक अधिकारों पर रोक लगा सकती है अथवा उनमें कटौती कर सकती है।

प्रश्न 34.
आपातकाल के संदर्भ में दो विवादों का वर्णन कीजिए।
उत्तर:

  1. आपातकाल भारतीय राजनीति का सर्वाधिक विवादास्पद प्रकरण है। इसके दो विवाद निम्न हैं।
  2. खाद्यान्न के अभाव।
  3. बेरोजगारी और भ्रष्टाचार में वृद्धि।

छात्रों ने अपने आंदोलन की अगुवाई के लिए जयप्रकाश नारायण को बुलावा भेजा। जेपी ने छात्रों का निमंत्रण इस शर्त पर स्वीकार किया कि आंदोलन अहिंसक रहेगा और अपने को सिर्फ बिहार तक सीमित नहीं रखेगा।

JAC Class 12 Political Science Important Questions Chapter 6 लोकतांत्रिक व्यवस्था का संकट

प्रश्न 31.
गुजरात के छात्र आंदोलन के क्या परिणाम हुए?
उत्तर:
गुजरात के छात्र आंदोलन के राष्ट्रीय स्तर की राजनीति पर दूरगामी प्रभाव हुए। इस आंदोलन में बड़ी राजनीतिक पार्टियाँ भी शरीक हो गईं। इस प्रकार आंदोलन ने विकराल रूप धारण कर लिया। ऐसे में गुजरात में राष्ट्रपति शासन लगा दिया गया। राज्य में दुबारा विधानसभा के चुनाव की माँग उठने लगी।

प्रश्न 32.
जयप्रकाश नारायण द्वारा बिहार के छात्रों के आंदोलन में साथ देने का क्या परिणाम हुआ?
उत्तर:
जयप्रकाश नारायण द्वारा बिहार आंदोलन के साथ जुड़ते ही इस आंदोलन ने राजनीतिक चरित्र ग्रहण किया और राष्ट्रव्यापी अपील आई। जीवन के हर क्षेत्र के लोग आंदोलन से जुड़ गए। बिहार की कांग्रेस सरकार को बर्खास्त करने की माँग की। बिहार की सरकार के खिलाफ लगातार घेराव, बंद और हड़ताल का एक सिलसिला चल पड़ा। इस आंदोलन का प्रभाव राष्ट्रीय राजनीति पर भी पड़ा।

प्रश्न 33.
आपातकाल की घोषणा के साथ राजनीति में क्या बदलाव आता है?
उत्तर:
आपातकाल की घोषणा के साथ ही शक्तियों के बँटवारे का संघीय ढाँचा व्यावहारिक तौर पर निष्प्रभावी हो जाता है और सारी शक्तियाँ केन्द्र सरकार के हाथ में चली आती हैं। दूसरे, सरकार चाहे तो ऐसी स्थिति में किसी एक अथवा सभी मौलिक अधिकारों पर रोक लगा सकती है अथवा उनमें कटौती कर सकती है।

प्रश्न 34.
आपातकाल के संदर्भ में दो विवादों का वर्णन कीजिए।
उत्तर:
आपातकाल भारतीय राजनीति का सर्वाधिक विवादास्पद प्रकरण है। इसके दो विवाद निम्न हैं।

  1. आपातकाल की घोषणा की जरूरत को लेकर विभिन्न दृष्टिकोणों का होना।
  2. सरकार ने संविधान प्रदत्त अधिकारों का इस्तेमाल करके व्यावहारिक तौर पर लोकतांत्रिक कामकाज को ठप्प कर दिया था।

निबन्धात्मक प्रश्न

प्रश्न 1.
1975 में आंतरिक आपातकाल क्यों घोषित किया गया? इस दौरान कौनसे परिणाम महसूस किये गये? स्पष्ट कीजिए।
अथवा
भारतीय सरकार के अनुसार राष्ट्रीय आपातकाल की घोषणा के क्या कारण थे? आपातकाल की घोषणा से पूर्व क्या घटनाएँ घटित हुई थीं? कीजिए।
अथवा
1975 में श्रीमती इंदिरा गांधी द्वारा लागू किये गये आपातकाल की घोषणा के प्रमुख कारणों का वर्णन
उत्तर:

  • आपातकाल के कारण 1975 में आन्तरिक राष्ट्रीय आपातकाल के प्रमुख ‘कारण निम्नलिखित हैं।
    1. 1971 के युद्ध में अत्यधिक व्यय: 1971 के भारत-पाक युद्ध तथा बांग्लादेशी शरणार्थियों पर हुए व्यय का भारतीय अर्थव्यवस्था पर प्रतिकूल असर पड़ा। इससे लोगों में असन्तोष फैल गया।
    2. कृषिगत तथा औद्योगिक उत्पादन में कमी: 1972-1973 में भारत में फसल भी अच्छी नहीं हुई तथा औद्योगिक उत्पादन में भी निरन्तर कमी आ रही थी। इससे कृषक तथा औद्योगिक कर्मचारियों में असन्तोष बढ़ रहा था।
    3. रेलवे की हड़ताल: 1975 में की गई आपातकालीन घोषणा का एक कारण रेलवे कर्मचारियों द्वारा की गई हड़ताल भी थी जिससे यातायात व्यवस्था बिल्कुल खराब हो गई।
    4. बिहार आंदोलन: जयप्रकाश नारायण के नेतृत्व में किया जा रहा बिहार आन्दोलन भी 1975 में आपातकाल की घोषणा का एक प्रमुख कारण था।
    5. श्रीमती गाँधी के चुनाव को अवैध घोषित करना: 1975 के आपातकाल का एक अन्य महत्त्वपूर्ण कारण इलाहाबाद उच्च न्यायालय द्वारा उनके निर्वाचन को अवैध घोषित करना था। इसके बाद विपक्षी दल श्रीमती गाँधी के त्यागपत्र की माँग करने लगा।
  • आपातकाल के दौरान महंसूस किये गए परिणाम:
    1. आपातकाल की घोषणा की जरूरत को लेकर विभिन्न दृष्टिकोणों का होना।
    2. सरकार ने संविधान प्रदत्त अधिकारों का इस्तेमाल करके व्यावहारिक तौर पर लोकतांत्रिक कामकाज को ठप्प कर दिया था।
  • आपातकाल के दौरान महसूस किये गए परिणाम:दिया
    1. आपातकाल में लोगों को सार्वजनिक तौर पर सरकार का विरोध करने की लोकतांत्रिक प्रणाली को ठप्प कर
    2. इस काल में निवारक नजरबंदी कानून का दुरुपयोग किया गया।
    3. इस दौरान सरकार ने प्रेस की आजादी पर रोक लगा दी।
    4. इस दौरान 42वें संविधान संशोधन के द्वारा संविधान के अनेक हिस्सों में बदलाव किये गये।

प्रश्न 2.
1977 में कांग्रेस पार्टी की पराजय के प्रमुख कारणों का विवेचन कीजिए।
उत्तर;
1977 में कांग्रेस की पराजय के कारण 1977 के चुनावों में कांग्रेस की पराजय के पीछे निम्नलिखित कारण जिम्मेदार रहे।

  1. आपातकाल की घोषणा: श्रीमती गाँधी द्वारा लागू किये गये आपातकाल के विरुद्ध सभी गैर-कांग्रेसी राजनीतिक दल एकजुट हो गये।
  2. आपातकाल के दौरान अत्याचार: श्रीमती गाँधी ने आपातकाल के दौरान मीसा कानून तथा अनिवार्य नसबंदी के द्वारा लोगों पर अनेक अत्याचार किये। इससे लोगों में कांग्रेस पार्टी के विरुद्ध असन्तोष पैदा हुआ।
  3. कीमतों में अत्यधिक वृद्धि: श्रीमती गाँधी की सरकार सभी प्रकार के उपाय करके भी कीमतों की वृद्धि को नहीं रोक पा रही थी तथा 1971 के चुनावों में उनके द्वारा दिया गया गरीबी हटाओ का नारा भी दम तोड़ता नजर आ रहा था।
  4. प्रेस पर प्रतिबन्ध: आपातकाल के समय श्रीमती गाँधी ने प्रेस पर प्रतिबन्ध लगा दिया। इस तरह के प्रतिबन्ध से भी लोगों में असन्तोष था।
  5. कर्मचारियों की दयनीय स्थिति: तत्कालीन समय में वस्तुओं की कीमतों में अत्यधिक वृद्धि से सरकारी कर्मचारियों की दशा खराब होने लगी थी। वे सरकार से नाराज हो गए थे।
  6. जगजीवन राम का त्यागपत्र: आपातकाल के समय जगजीवन राम जैसे श्रीमती गाँधी के वफादार नेता भी उनके साथ नहीं रहे तथा कांग्रेस एवं सरकार से त्यागपत्र दे दिया।

प्रश्न 3.
1977 के लोकसभा चुनावों के किन्हीं तीन महत्त्वपूर्ण परिणामों एवं निष्कर्षों पर प्रकाश डालिए।
उत्तर:
1977 के लोकसभा चुनावों के निष्कर्ष एवं परिणाम: 1977 के चुनावों के कुछ निष्कर्ष इस प्रकार हैं।

  1. कांग्रेस पार्टी के एकाधिकार की समाप्ति: 1977 के चुनावों का सबसे महत्त्वपूर्ण निष्कर्ष यह रहा पिछले तीन दशकों से चली आ रही कांग्रेसी सत्ता का एकाधिकार समाप्त हो गया।
  2. केन्द्र में प्रथम गैर-कांग्रेस सरकार का निर्माण: 1977 के चुनावों के पश्चात् केन्द्र में पहली बार गैर- कांग्रेस सरकार का निर्माण हुआ। विपक्षी दलों ने मिलकर जनता पार्टी के झण्डे के नीचे गठबन्धन सरकार का निर्माण किया। जनता पार्टी को मार्च, 1977 के लोकसभा के चुनाव में भारी सफलता प्राप्त हुई। 542 सीटों में से जनता पार्टी गठबन्धन को 330 सीटें मिलीं। इस प्रकार जनता पार्टी ने कांग्रेस को करारी शिकस्त दी, अतः जनता पार्टी की सरकार बनी।
  3. पिछड़े वर्गों की भलाई का मुद्दा प्रभावी हो गया: 1977 के चुनावों के बाद अप्रत्यक्ष रूप से पिछड़े वर्गों की भलाई का मुद्दा भारतीय राजनीति पर हावी होना शुरू हुआ क्योंकि 1977 के चुनाव परिणामों पर पिछड़ी जातियों के मतदान का असर पड़ा था।

JAC Class 12 Political Science Important Questions Chapter 6 लोकतांत्रिक व्यवस्था का संकट

प्रश्न 4.
नागरिक स्वतन्त्रताओं के संगठनों के उदय का वर्णन कीजिए।
उत्तर:
नागरिक स्वतन्त्रताओं के संगठनों का उदय: आपातकाल से पूर्व तथा इसके पश्चात् नागरिक स्वतन्त्रता संगठनों के उदय एवं प्रसार में कुछ प्रगति हुई, जिसका वर्णन इस प्रकार है।
1. नव निर्माण समिति का गठन:
जयप्रकाश नारायण ने बिहार आंदोलन के समय दलविहीन लोकतन्त्र तथा सम्पूर्ण क्रान्ति का नारा दिया। इस उद्देश्य की पूर्ति के लिए उन्होंने विनोबा भावे के भूदान आन्दोलन को भी इसमें शामिल किया। जयप्रकाश नारायण ने इसके लिए गुजरात एवं बिहार के युवा छात्रों को शामिल किया तथा एक नवनिर्माण समिति का गठन किया। इस समिति का उद्देश्य सार्वजनिक जीवन से भ्रष्टाचार को समाप्त करना था।

2. नक्सलवादी संगठनों का उदय:
नक्सलवादी पद्धति बंगाल, केरल एवं आन्ध्रप्रदेश में सक्रिय थी। नक्सलवादी वामदलों से अलग हुआ गुट था, क्योंकि इनका वामदलों से मोह भंग हो गया था। इन्होंने लोगों की स्वतन्त्रता एवं माँगों पूरा करने के लिए हिंसा का सहारा लिया, परन्तु सरकारों ने इस प्रकार के आन्दोलन को समाप्त करने के लिए बल प्रयोग का सहारा लिया।

3. नागरिक स्वतन्त्रता एवं लोकतान्त्रिक अधिकारों के लिए लोगों का संघ: नागरिक स्वतन्त्रता एवं लोकतान्त्रिक अधिकारों के लिए लोगों के संघ का उदय अक्टूबर, 1976 में हुआ। संगठन ने न केवल आपातकाल में बल्कि सामान्य परिस्थितियों में भी लोगों को अपने अधिकारों के प्रति सतर्क रहने के लिए कहा। 1980 में नागरिक स्वतन्त्रता एवं लोकतान्त्रिक अधिकारों के लिए लोगों के संघ का नाम बदलकर ‘नागरिक स्वतन्त्रताओं के लिए लोगों का संघ’ रख दिया गया तथा इस संगठन का प्रचार-प्रसार पूरे देश में किया गया। यह संगठन नागरिक अधिकारों की रक्षा के लिए जनमत तैयार करवाता था।

प्रश्न 5.
वचनबद्ध न्यायपालिका से आप क्या समझते हैं? श्रीमती इन्दिरा गाँधी की सरकार ने इसके लिए क्या प्रयत्न किये?
उत्तर:
वचनबद्ध न्यायपालिका: वचनबद्ध न्यायपालिका से तात्पर्य न्यायपालिका का सरकार के प्रति प्रतिबद्ध होना या सरकार की नीतियों का आंख मूंद कर पालन करने से है। वचनबद्ध न्यायपालिका के लिए सरकार द्वारा प्रयोग किए गए उपाय थे तत्कालीन प्रधानमन्त्री श्रीमती गांधी की सरकार ने न्यायपालिका की वचनबद्धता के लिए अग्रलिखित उपाय किये

  1. न्यायाधीशों की नियुक्ति में वरिष्ठता के सिद्धान्त की अनदेखी: श्रीमती गाँधी ने वचनबद्ध न्यायपालिका के लिए न्यायाधीशों की नियुक्ति में वरिष्ठता की अनदेखी की तथा उन न्यायाधीशों को पदोन्नत किया, जो सरकार के प्रति वफादार थे।
  2. न्यायाधीशों का स्थानान्तरण: श्रीमती गाँधी ने वचनबद्ध न्यायपालिका के लिए न्यायाधीशों के स्थानान्तरण का सहारा भी लिया।
  3. न्यायपालिका की आलोचना: न्यायाधीशों द्वारा लिए जाने वाले निर्णयों की प्रायः अधिकारियों द्वारा आलोचना की जाती थी, जबकि ऐसा किया जाना संविधान के विरुद्ध था।
  4. अस्थायी न्यायाधीशों की नियुक्ति: सरकार अस्थायी तौर पर न्यायाधीशों की नियुक्ति करके न्यायाधीशों की कार्यप्रणाली एवं व्यवहार का अध्ययन करती थी, कि वह सरकार के पक्ष में कार्य कर रहा है, या विपक्ष में।
  5. अन्य पदों पर नियुक्तियाँ: सरकार ने सेवानिवृत्त न्यायाधीशों में से उन्हें राज्यपाल, राजदूत या किसी आयोग का अध्यक्ष नियुक्त किया, जो सरकार के प्रति वफादार थे अथवा सरकार की नीतियों के अनुसार चलते थे।
  6. कार्यवाहक मुख्य न्यायाधीश का प्रावधान: कार्यवाहक मुख्य न्यायाधीश के संवैधानिक प्रावधानों को भी वचनबद्ध न्यायपालिका के लिए प्रयोग किया गया।

प्रश्न 6.
नक्सलवादी आन्दोलन क्या है? भारतीय राजनीति में इसकी भूमिका का मूल्यांकन कीजिए।
अथवा
नक्सलवादी आन्दोलन पर एक निबन्ध लिखिए।
उत्तर:
1. नक्सलवादी आन्दोलन की पृष्ठभूमि:
पश्चिम बंगाल के पर्वतीय जिले दार्जिलिंग के नक्सलबाड़ी पुलिस थाने के इलाके में 1967 में एक किसान विद्रोह उठ खड़ा हुआ। इस विद्रोह की अगुवाई मार्क्सवादी कम्युनिस्ट पार्टी के स्थानीय नेताओं ने की। नक्सलबाड़ी पुलिस थाने से शुरू होने वाला यह आंदोलन भारत के कई राज्यों में फैल गया। इस आंदोलन को नक्सलवादी आंदोलन के रूप में जाना जाता है।

2. सी.पी.आई. से अलग होना और गुरिल्ला युद्ध प्रणाली को अपनाना:
1969 में नक्सलवादी सी. पी. आई. (एम.) से अलग हो गए और उन्होंने सी.पी.आई. (मार्क्सवादी-लेनिनवादी) नाम से एक नई पार्टी ‘चारु मजूमदार’ के नेतृत्व में बनायी। इस पार्टी की दलील थी कि भारत में लोकतन्त्र एक छलावा है। इस पार्टी ने क्रान्ति करने के लिए गोरिल्ला युद्ध की रणनीति अपनायी।

3. नक्सलवादियों के कार्यक्रम:
नक्सलवादी आंदोलन ने धनी भूस्वामियों से जमीन बलपूर्वक छीन कर गरीब और भूमिहीन लोगों को दी। इस आंदोलन के समर्थक अपने राजनीतिक लक्ष्यों को प्राप्त करने के लिए हिंसक साधनों के इस्तेमाल के पक्ष में दलील देते थे।

4. नक्सलवाद का प्रसार एवं प्रभाव:
1970 के दशक में भारत में 9 राज्यों के लगभग 75 जिले नक्सलवाद की हिंसा से प्रभावित हुए। इनमें अधिकांश बहुत पिछड़े इलाके हैं और यहाँ आदिवासियों की संख्या बहुत अधिक है। नक्सलवादी हिंसा व नक्सल विरोधी कार्यवाहियों में अब तक हजारों लोग जान गँवा चुके हैं।

JAC Class 12 Political Science Important Questions Chapter 6 लोकतांत्रिक व्यवस्था का संकट

प्रश्न 7.
निम्न बिन्दुओं का वर्णन आपातकाल 1975 की पृष्ठभूमि के संदर्भ में कीजिए
(अ) गुजरात व बिहार आंदोलन
(ब) सरकार व न्यायपालिका में संघर्ष।
उत्तर:
( अ ) गुजरात आंदोलन:
गुजरांत में नव निर्माण आंदोलन के प्रभावस्वरूप गुजरात के मुख्यमंत्री को त्यागपत्र देना पड़ा। आपातकाल की घोषणा का यह भी एक कारण बना। बिहार आंदोलन बिहार आंदोलन प्रशासन में भ्रष्टाचारी व अयोग्य कर्मचारियों के विरुद्ध लोकनायक जयप्रकाश नारायण द्वारा चलाया गया आंदोलन था। बिहार आंदोलन में अनुसूचित जातियों एवं अनुसूचित जनजातियों की सामाजिक एवं आर्थिक समस्याओं को भी हल करने का प्रयास किया गया। यह बिहार आंदोलन भी 1975 में आपातकाल की घोषणा का एक प्रमुख कारण बना।

(ब) सरकार व न्यापालिका में संघर्ष:
1973-74 की अवधि में न्यायपालिका और सरकार के सम्बन्धों में भी तनाव आए। सर्वोच्च न्यायालय ने सरकार की नई पहलकदमियों को संविधान के विरुद्ध माना। सरकार का मानना था कि अदालत का यह रवैया लोकतंत्र के सिद्धान्तों और संसद की सर्वोच्चता के विरुद्ध है। यह सरकार के गरीबों को लाभ पहुँचाने वाले कल्याण कार्यक्रमों को लागू करने की राह में रोड़ा अटका रही है। इसके साथ ही इलाहाबाद उच्च न्यायालय ने श्रीमती गाँधी के निर्वाचन को अवैध घोषित कर दिया तथा श्रीमती गाँधी को छ: वर्ष तक संसद की सदस्यता न ग्रहण करने की सजा दी गई। सरकार और न्यायपालिका के इस संघर्ष ने आपातकाल की घोषणा की पूर्ण पृष्ठभूमि तैयार कर दी।

प्रश्न 8.
1975 के आपातकालीन घोषणा और क्रियान्वयन से देश के राजनैतिक माहौल, प्रेस, नागरिक अधिकारों, न्यायपालिका के निर्णयों और संविधान पर क्या प्रभाव पड़े? विस्तार में लिखिए ।
उत्तर:
इंदिरा सरकार के आपातकाल की घोषणा पर विरोध- आंदोलनों में कमी आई क्योंकि आपातकाल के दौरान अनेक विपक्षी नेताओं को जेल में डाल दिया गया। राजनीतिक माहौल तनावग्रस्त हो गया था। आपातकालीन प्रावधानों का हवाला देते हुए सरकार ने प्रेस की आजादी पर रोक लगा दी। इसे प्रेस सेंसरशिप के नाम से जाना गया। सामाजिक और सांप्रदायिक गड़बड़ी की आशंका के मद्देनजर सरकार ने राष्ट्रीय स्वयंसेवक संघ और जमात-ए- इस्लामी जैसे संगठनों पर रोक लगा दी। आपातकालीन प्रावधानों के अंतर्गत नागरिकों के मौलिक अधिकार निष्प्रभावी हो गए।

उनके पास यह अधिकार भी नहीं रहा कि मौलिक अधिकारों की बहाली के लिए अदालत का दरवाजा खटखटाएँ। निवारक नजरबंदी के अंतर्गत लोगों को गिरफ्तार इसलिए नहीं किया जाता था कि उन्होंने कोई अपराध किया है बल्कि इसके विपरीत, इस प्रावधान के अंतर्गत लोगों को इस आशंका से गिरफ्तार किया जाता था कि भविष्य में वे कोई अपराध कर सकते हैं। इस अधिनियम के अंतर्गत सरकार ने बड़े पैमाने पर गिरफ्तारियाँ कीं। गिरफ्तार लोग अपनी गिरफ्तारी को चुनौती नहीं दे पाते थे। गिरफ्तार लोगों अथवा उनके पक्ष से किन्हीं और ने उच्च न्यायालय और सर्वोच्च न्यायालय में कई मामले दायर किए, किन्तु सरकार का कहना था कि गिरफ्तार लोगों को कोई कारण बताना कतई जरूरी नहीं है।

अनेक उच्च न्यायालयों ने फैसला दिया कि आपातकाल की घोषणा के बावजूद अदालत किसी व्यक्ति द्वारा दायर की गई ऐसी बंदी प्रत्यक्षीकरण याचिका को विचार के लिए स्वीकार कर सकती है। परंतु 1976 के अप्रैल माह में सर्वोच्च न्यायालय ने उच्च न्यायालय के फैसले को उलट दिया और सरकार की दलील को सही ठहराया। इसका सामान्य शब्दों में यही अर्थ था कि आपातकाल के दौरान सरकार, नागरिक से जीवन और आजादी का अधिकार वापस ले सकती है। आपातकाल के समय जो नेता गिरफ्तारी से बच गए थे वे भूमिगत हो गए और सरकार के खिलाफ मुहिम चलाई।

पद्मभूषण से सम्मानित कन्नड़ लेखक शिवम कारंत और पद्मश्री से सम्मानित हिन्दी लेखक फणीश्वर नाथ ‘रेणु’ ने अपनी-अपनी पदवी वापस कर दी। संसद ने संविधान के सामने कई नई चुनौतियाँ खड़ी कीं। इंदिरा गाँधी के मामले में इलाहाबाद उच्च न्यायालय के फैसले की पृष्ठभूमि में संशोधन हुआ। इस संशोधन के अनुसार प्रधानमंत्री, राष्ट्रपति और उपराष्ट्रपति पद के निर्वाचन को अदालत में चुनौती नहीं दी जा सकती। आपातकाल के दौरान ही संविधान में 42वाँ संशोधन किया गया।

JAC Class 12 Political Science Important Questions Chapter 6 लोकतांत्रिक व्यवस्था का संकट

प्रश्न 9.
बिहार आंदोलन से जुड़े विभिन्न पहलुओं पर विचार करते हुए संक्षिप्त सार लिखिए।
उत्तर:
बिहार के आंदोलन के समय वहाँ पर काँग्रेस की सरकार थी। यहाँ आंदोलन छात्र आंदोलन के रूप में शुरू हुआ। यह आंदोलन न केवल बिहार तक बल्कि अनेक वर्षों तक राष्ट्रीय राजनीति पर दूरगामी प्रभाव डालने वाला साबित हुआ। बिहार आंदोलन के कारण 1974 के मार्च माह में बढ़ती हुई कीमतों, खाद्यान्न के अभाव, बेरोजगारी और भ्रष्टाचार के खिलाफ बिहार के छात्रों ने आंदोलन छेड़ दिया। आंदोलन के क्रम में उन्होंने जयप्रकाश नारायण को बुलावा भेजा। जेपी तब सक्रिय राजनीति छोड़ चुके थे और सामाजिक कार्यों में लगे थे। छात्रों ने अपने आंदोलन की अगुवाई के लिए जयप्रकाश नारायण को बुलावा भेजा।

जेपी ने छात्रों का निमंत्रण इस शर्त पर स्वीकार किया कि आंदोलन अहिंसक रहेगा और अपने को सिर्फ बिहार तक सीमित नहीं रखेगा। इस प्रकार छात्र आंदोलन ने एक राजनीतिक चरित्र ग्रहण किया और उसके भीतर राष्ट्रव्यापी अपील आई जीवन के हर क्षेत्र के लोग अब आंदोलन से आ जुड़े। आंदोलन की प्रगति और सार जयप्रकाश नारायण ने बिहार की कांग्रेस को बर्खास्त करने की माँग की। उन्होंने सामाजिक, आर्थिक और राजनीतिक दायरे में ‘सम्पूर्ण क्रांति’ का आह्वान किया ताकि ‘सच्चे लोकतंत्र’ की स्थापना की जा सके। बिहार की सरकार के खिलाफ लगातार घेराव, बंद और हड़ताल का एक सिलसिला चल पड़ा। बहराल, सरकार ने इस्तीफा देने से इनकार कर दिया। 1974 के बिहार आंदोलन का एक प्रसिद्ध नारा था, “संपूर्ण क्रांति अब नारा है- भावी इतिहास हमारा है।”

प्रभाव-आंदोलन का प्रभाव राष्ट्रीय राजनीति पर पड़ना शुरू हुआ। जयप्रकाश नारायण चाहते थे यह आंदोलन देश के दूसरे हिस्से में भी फैले। जेपी के नेतृत्व में चल रहे आंदोलन के साथ ही साथ रेलवे के कर्मचारियों ने भी एक राष्ट्रवादी हड़ताल का आह्वान किया। इससे देश के रोजमर्रा के कामकाज के ठप्प हो जाने का खतरा पैदा हो गया। 1975 में जेपी ने जनता के ‘संसद मार्च’ का नेतृत्व किया। देश की राजधानी में अब तक इतनी बड़ी रैली नहीं हुई थी। जेपी को भारतीय जनसंघ, काँगेस (ओ), भारतीय लोकदल, सोशलिस्ट पार्टी जैसे गैर- काँग्रेसी का समर्थन मिला। इन दलों ने जेपी को इंदिरा के विकल्प के रूप में पेश किया।

टिप्पणी-जेपी के विचारों और उनके द्वारा अपनायी नई जन-प्रतिरोध की रणनीति की आलोचनाएँ भी मुखर हुईं गुजरात और बिहार, दोनों ही राज्यों के आंदोलन को कांग्रेस विरोधी आंदोलन माना गया। कहा गया कि ये आंदोलन राज्य सरकार के खिलाफ नहीं बल्कि इंदिरा गाँधी के नेतृत्व के खिलाफ चलाए गए हैं। इंदिरा गाधी का मानना था कि ये आंदोलन उनके प्रति व्यक्तिगत विरोध से प्रेरित हैं।

प्रश्न 10.
1975 के आपातकाल से सीखे गए तीन सबकों का विश्लेषण कीजिए।
उत्तर:

  1. 1975 की आपातकाल से एकबारगी भारतीय लोकतंत्र की ताकत और कमजोरियाँ उजागर हो गईं। हालाँकि बहुत से पर्यवेक्षक मानते हैं कि आपातकाल के दौरान भारत लोकतांत्रिक नहीं रह गया था। लेकिन यह भी ध्यान देने की बात है कि थोड़े ही दिनों के अंदर कामकाज फिर से लोकतांत्रिक ढर्रे पर लौट आया। इस तरह आपातकाल का एक सबक तो यही है कि भारत से लोकतंत्र को विदा कर पाना बहुत कठिन हैं।
  2. आपातकाल से संविधान में वर्णित आपातकाल के प्रावधानों के कुछ अर्थगत उलझाव भी प्रकट हुए, जिन्हें बाद में सुधार लिया गया। अब ‘अंदरूनी’ आपातकाल सिर्फ ‘सशस्त्र विद्रोह’ की स्थिति में लगाया जा सकता है। इसके लिए यह भी जरूरी है कि आपातकाल की धारणा की सलाह मंत्रिमंडल राष्ट्रपति को लिखित में दे।
  3. आपातकाल से हर कोई नागरिक अधिकारों के प्रति ज्यादा सचेत हुआ। आपातकाल की सम्पत्ति के बाद अदालतों ने व्यक्ति के नागरिक अधिकारों की रक्षा में सक्रिय भूमिका निभाई।  यपालिका आपातकाल के वक्त नागरिक अधिकारों की रक्षा में तत्पर हो गई। आपातकाल के बाद नागरिक अधिकारों के कई संगठन वजूद में आए।

JAC Class 12 Political Science Important Questions Chapter 5 कांग्रेस प्रणाली : चुनौतियाँ और पुनर्स्थापना

Jharkhand Board JAC Class 12 Political Science Important Questions Chapter 5 कांग्रेस प्रणाली : चुनौतियाँ और पुनर्स्थापना Important Questions and Answers.

JAC Board Class 12 Political Science Important Questions Chapter 5 कांग्रेस प्रणाली : चुनौतियाँ और पुनर्स्थापना

बहुचयनात्मक प्रश्न

1. कांग्रेस पार्टी का प्रभुत्व केन्द्र में कब तक रहा?
(क) 1947 से 1990 तक
(ग) 1947 से 1977 तक
(ख) 1947 से 1960 तक
(घ) 1947 से 1980 तक।
उत्तर:
(ग) 1947 से 1977 तक

2. किस वर्ष इन्दिरा गाँधी ने सोवियत संघ के साथ बीस वर्ष के लिए शान्ति, मित्रता तथा सहयोग की सन्धि पर हस्ताक्षर किए थे?
(कं) जून, 1972
(ख) अक्टूबर, 1977
(ग) अगस्त, 1947
(घ) अंगस्त, 1971
उत्तर:
(घ) अंगस्त, 1971

3. गरीबी हटाओ का नारा किसने दिया?
(क) सुभाषचन्द्र बोस ने
(ख) लाल बहादुर शास्त्री ने
(ग) जवाहरलाल नेहरू ने
(घ) इन्दिरा गाँधी ने।
उत्तर:
(घ) इन्दिरा गाँधी ने।

4. भारत ने प्रथम परमाणु परीक्षण कब किया?
(क) 1974
(ख) 1975
(ग) 1976
(घ) 1977
उत्तर:
(क) 1974

5. ‘जय जवान जय किसान’ का नारा किसने दिया-
(क) लाल बहादुर शास्त्री ने
(ख) इंदिरा गाँधी ने
(ग) जवाहरलाल नेहरू ने
(घ) मोरारजी देसाई ने।
उत्तर:
(क) लाल बहादुर शास्त्री ने

6. बांग्लादेश का निर्माण हुआ
(क) सन् 1966 में
(ख) सन् 1970 में
(ग) सन् 1971 में
(घ) इन्दिरा गाँधी ने।
उत्तर:
(ग) सन् 1971 में

1. निम्नलिखित का मिलान कीजिए

1. जवाहरलाल नेहरू के उपरान्त (क) 1966
2. ताशकन्द समझौता (ख) लालबहादुर शास्त्री देश के दूसरे प्रधानमन्त्री बने।
3. मोरारजी का सम्बन्ध (ग) सिंडीकेट से
4. बांग्लादेश का निर्माण (घ) 1971
5. वी.वी. गिरि (ङ) आन्ध्र प्रदेश के मजदूर नेता
6. कर्पूरी ठाकुर (च) बिहार के नेता

उत्तर:

1. जवाहरलाल नेहरू के उपरान्त (ख) लालबहादुर शास्त्री देश के दूसरे प्रधानमन्त्री बने।
2. ताशकन्द समझौता (क) 1966
3. मोरारजी का सम्बन्ध (ग) सिंडीकेट से
4. बांग्लादेश का निर्माण (घ) 1971
5. वी.वी. गिरि (ङ) बिहार के नेता
6. कर्पूरी ठाकुर (च) आन्ध्र प्रदेश के मजदूर नेता


रिक्त स्थानों की पूर्ति कीजिए 

1. ………………….. में देश के पहले प्रधानमंत्री जवाहरलाल नेहरू की मृत्यु हो गई।
उत्तर:
1964

2. 1960 के दशक को ………………… की संज्ञा दी गई।
उत्तर:
खतरनाक दशक

3. लाल बहादुर शास्त्री ……………. से ………………… तक भारत के प्रधानमंत्री रहे।
उत्तर:
1964, 1966

4. 10 जनवरी ……………. को ……………….. में शास्त्रीजी का निधन हो गया।
उत्तर:
1966, ताशकंद

5. सी. नटराजन अन्नादुरई ने 1949 में ……………….. का बतौर राजनीतिक पार्टी गठन किया।
उत्तर:
द्रविड़ मुन्नेत्र कषगम

6. पंजाब में बनी संयुक्त विधायक दल की सरकार को ……………….. की सरकार कहा गया।
उत्तर:
पॉपुलर यूनाइटेड फ्रंट

अतिलघूत्तरात्मक प्रश्न

प्रश्न 1.
1967 में कांग्रेस के प्रतिष्ठित नेताओं के समूह को किस नाम से जाना जाता था?
उत्तर:
सिंडिकेट।

प्रश्न 2.
‘जय जवान जय किसान’ का नारा किसने दिया?
उत्तर:
लाल बहादुर शास्त्री ने।

प्रश्न 3.
1971 के चुनावों में कांग्रेसी नेताओं को कुल कितनी सीटें प्राप्त हुईं?
उत्तर:
352 सीटें प्राप्त हुईं।

JAC Class 12 Political Science Important Questions Chapter 5 कांग्रेस प्रणाली : चुनौतियाँ और पुनर्स्थापना

प्रश्न 4.
विरोधी दलों को लगा कि इन्दिरा गाँधी की अनुभवहीनता और कांग्रेस की आन्तरिक गुटबन्दी से उन्हें कांग्रेस को सत्ता से हटाने का एक अवसर मिला। राम मनोहर लोहिया ने इस रणनीति को क्या नाम दिया?
उत्तर:
गैर-कांग्रेसवाद।

प्रश्न 5.
चन्द्रशेखर, चरणजीत यादव, मोहन धारिया तथा कृष्णकान्त जैसे नेता किस गुट में शामिल थे?
उत्तर:
युवा तुर्क गुट में

प्रश्न 6.
कामराज, एस.के. पाटिल तथा निजलिंगप्पा जैसे नेता किस समूह के सदस्य माने जाते थे?
उत्तर:
सिंडिकेट

प्रश्न 7.
प्रारम्भ में कांग्रेस का चुनाव चिह्न क्या था तथा वर्तमान में इस दल का चिह्न क्या है?
उत्तर:
प्रारम्भ में कांग्रेस का चुनाव चिह्न बैलों की जोड़ी था और वर्तमान में हाथ का निशान है।

प्रश्न 8.
कांग्रेस का विभाजन कब हुआ? अथवा कांग्रेस में पहली फूट कब पड़ी?
उत्तर:
1969 में।

प्रश्न 9.
चौथे आम चुनावों के पश्चात् केन्द्र में किस पार्टी की सरकार बनी?
उत्तर:
चौथे आम चुनाव के पश्चात् केन्द्र में कांग्रेस पार्टी की सरकार बनी।

प्रश्न 10.
लाल बहादुर शास्त्री का उत्तराधिकारी कौन बना?
उत्तर:
श्रीमती इन्दिरा गाँधी।

प्रश्न 11.
1969 में राष्ट्रपति के निर्वाचन के लिए इन्दिरा गाँधी ने किस उम्मीदवार का साथ दिया था?
उत्तर:
श्रीमती गांधी ने निर्दलीय उम्मीदवार वी.वी. गिरि का साथ दिया।

प्रश्न 12.
दस सूत्री कार्यक्रम कब लागू किया गया?
उत्तर:
दस सूत्री कार्यक्रम 1967 में लागू किया गया।

प्रश्न 13.
पाँचवीं लोकसभा के चुनाव कब हुए?
उत्तर:
पाँचवीं लोकसभा के चुनाव 1971 में हुए।

JAC Class 12 Political Science Important Questions Chapter 5 कांग्रेस प्रणाली : चुनौतियाँ और पुनर्स्थापना

प्रश्न 14.
1971 में कांग्रेस (आर) का नेतृत्व किस नेता ने किया?
उत्तर:
1971 में कांग्रेस (आर) का नेतृत्व श्रीमती इंदिरा गांधी ने किया।

प्रश्न 15.
कांग्रेस ऑर्गनाइजेशन से आपका क्या तात्पर्य है?
उत्तर:
1969 के नवंबर तक सिंडिकेट की अगुवाई वाले कांग्रेसी खेमे को कांग्रेस ऑर्गनाइजेशन कहा जाता था।

प्रश्न 16.
पुरानी कांग्रेस से क्या अभिप्राय है?
उत्तर:
मोरारजी देसाई के समर्थक कांग्रेस के समूह को पुरानी कांग्रेस की संज्ञा दी गई।

प्रश्न 17.
गैर-कांग्रेसवाद से क्या अभिप्राय है?
उत्तर:
यह वह राजनीतिक विचारधारा है जो मुख्यतः कांग्रेस विरोधी और उसे सत्ता से अलग करने के लिए समाजवादी नेता राम मनोहर लोहिया द्वारा प्रस्तुत की गई।

प्रश्न 18.
कांग्रेस सिंडिकेट से क्या अभिप्राय है?
उत्तर:
कांग्रेस पार्टी के नेता जिनमें कामराज, एस. के. पाटिल, निजलिंगप्पा तथा मोरारजी देसाई (इनमें कुछ को हम इंदिरा विरोधी भी कह सकते हैं) आदि के समूह को कांग्रेस सिंडिकेट के नाम से जाना जाता है।

प्रश्न 19.
प्रिवी पर्स से क्या अभिप्राय है?
उत्तर:
भूतपूर्व देशी राजाओं को दिए जाने वाले विशेष भत्ते आदि को प्रिवी पर्स के नाम से जाना जाता है।

प्रश्न 20.
लोकसभा का पाँचवाँ आम चुनाव कब हुआ था?
उत्तर:
1971 में। इंदिरा गाँधी के नेतृत्व वाले गुट को कांग्रेस का रिक्विजिस्ट खेमा कहा जाता है।

प्रश्न 21.
ग्रैंड अलायंस से क्या अभिप्राय है?
उत्तर:
कांग्रेस (ओ) और इंदिरा विरोधी विपक्षी पार्टियों और गैर-साम्यवादी दलों को ग्रैंड अलायंस की संज्ञा दी जाती है।

प्रश्न 22.
कामराज योजना क्या थी?
उत्तर:
कामराज योजना के अनुसार 1963 में सभी वरिष्ठ कांग्रेसी नेताओं ने पार्टी के पदों से त्यागपत्र दे दिया ताकि उनकी जगह युवा कार्यकर्ताओं को दी जा सके।

प्रश्न 23.
आया राम-गया राम राजनीति से आप क्या समझते हैं?
उत्तर:
आया राम-गया राम से अभिप्राय नेताओं द्वारा अपने व्यक्तिगत स्वार्थ के लिए निरन्तर दल-बदल करना है।

प्रश्न 24.
10 सूत्री कार्यक्रम कब और क्यों लागू किया गया?
उत्तर:
10 सूत्री कार्यक्रम श्रीमती इंदिरा गांधी द्वारा 1967 में कांग्रेस की प्रतिष्ठा को पुनर्स्थापित करने के लिए लागू किया गया।

JAC Class 12 Political Science Important Questions Chapter 5 कांग्रेस प्रणाली : चुनौतियाँ और पुनर्स्थापना

प्रश्न 25.
राजनीतिक भूकम्प से क्या तात्पर्य है?
उत्तर:
1967 के चुनावों में कांग्रेस को केन्द्र एवं राज्य स्तर पर हुई गहरी हानि को चुनाव विश्लेषकों ने कांग्रेस के लिए इसे राजनीतिक भूकम्प कहा।

प्रश्न 26.
किंगमेकर किसे कहते हैं?
उत्तर:
किसी नेता को प्रधानमंत्री या मुख्यमंत्री बनाने में महत्त्वपूर्ण भूमिका निभाने वाले नेताओं को किंगमेकर कहा है।

प्रश्न 27.
1971 के चुनावों में कांग्रेस पार्टी की जीत का क्या कारण था?
उत्तर:
1971 के चुनावों में कांग्रेस पार्टी की जीत का मुख्य कारण श्रीमती इंदिरा गांधी का चमत्कारिक नेतृत्व कांग्रेस की समाजवादी नीतियाँ तथा गरीबी हटाओ का नारा था।

प्रश्न 28.
कांग्रेस में युवा तुर्क के बारे में आप क्या जानते हैं?
उत्तर:
युवा तुर्क कांग्रेस में युवाओं से सम्बन्धित था। इसमें चन्द्रशेखर, चरणजीत यादव, मोहन धारिया, कृष्णकान्त एवं आर. के. सिन्हा जैसे युवा कांग्रेसी शामिल थे।

प्रश्न 29.
किन्हीं चार राज्यों के नाम लिखिए जहाँ 1967 के चुनाव के बाद कांग्रेसी सरकारें बनीं।
उत्तर:
महाराष्ट्र, आन्ध्रप्रदेश, मध्यप्रदेश, गुजरात।

प्रश्न 30.
किन्हीं चार राज्यों के नाम लिखिए जहाँ 1967 के चुनावों के बाद गैर-कांग्रेसी सरकारें बनीं।
उत्तर:
पंजाब, राजस्थान, उत्तरप्रदेश, पश्चिम बंगाल

प्रश्न 31.
राममनोहर लोहिया कौन थे?
उत्तर:
राममनोहर लोहिया समाजवादी नेता एवं विचारक थे। 1963 से 1967 तक लोकसभा के सदस्य रहे। वे गैर-कांग्रेसवाद के रणनीतिकार थे। उन्होंने नेहरू की नीतियों का विरोध किया।

प्रश्न 32.
ताशकन्द समझौते पर हस्ताक्षर करने वाले दो महान् राष्ट्राध्यक्षों के नाम लिखिए।
उत्तर:

  1. भारत के पूर्व प्रधानमंत्री लालबहादुर शास्त्री
  2. पाकिस्तान के पूर्व राष्ट्रपति मोहम्मद अयूब खान।

प्रश्न 33.
प्रिवी पर्स की समाप्ति कब हुई?
उत्तर:
1971 में कांग्रेस को मिली जीत के बाद इंदिरा गाँधी ने संविधान में संशोधन करवाकर प्रिवी पर्स की समाप्ति करवा दी।

प्रश्न 34.
इंदिरा गाँधी की हत्या कब की गई?
उत्तर:
इदिरा गाँधी की हत्या 31 अक्टूबर, 1984 के दिन हुई थी।

JAC Class 12 Political Science Important Questions Chapter 5 कांग्रेस प्रणाली : चुनौतियाँ और पुनर्स्थापना

प्रश्न 35.
मद्रास प्रांत को अब किस नाम से जाना जाता है ?
उत्तर:
तमिलनाडु।

लघूत्तरात्मक प्रश्न

प्रश्न 1.
पण्डित जवाहरलाल नेहरू का राजनीतिक उत्तराधिकारी कौन था ?
उत्तर:
जवाहरलाल की मृत्यु के बाद लालबहादुर शास्त्री को नेहरू का उत्तराधिकारी बनाया गया। शास्त्री लगभग 18 महीने प्रधानमंत्री पद पर रहे। उनके शासन काल में 1965 का पाकिस्तान युद्ध हुआ। भारत को शानदार सफलता प्राप्त हुई। शास्त्रीजी ने ‘जय जवान जय किसान’ का नारा दिया। इस प्रकार शास्त्री एक प्रधान समझौताकर्ता, मध्यस्थ तथा समन्वयकार थे।

प्रश्न 2.
लालबहादुर शास्त्री की मृत्यु के बाद भारत का प्रधानमंत्री कौन बना?
उत्तर:
लालबहादुर शास्त्री की मृत्यु के बाद श्रीमती इंदिरा गांधी प्रधानमंत्री बनीं। प्रधानमंत्री बनने के बाद श्रीमती कृषि क्षेत्र को बढ़ावा दिया, गरीबी को हटाने के लिए कार्यक्रम घोषित किया, देश की सेनाओं का आधुनिकीकरण किया तथा 1974 में पोकरण में ऐतिहासिक परमाणु विस्फोट किया। 1971 में भारत ने पाकिस्तान को युद्ध में हराया, जिसके कारण बांग्लादेश नाम का एक नया देश अस्तित्व में आया।

प्रश्न 3.
1966 में कांग्रेस दल के वरिष्ठ नेताओं ने प्रधानमंत्री के पद के लिए श्रीमती गाँधी का साथ क्यों दिया?
उत्तर:
कांग्रेस के वरिष्ठ नेताओं ने संभवतः यह सोचकर 1966 में श्रीमती गाँधी का साथ प्रधानमंत्री पद के लिए दिया कि प्रशासनिक और राजनैतिक मामलों में खास अनुभव नहीं होने के कारण समर्थन व दिशा निर्देशन के लिए वे उन पर निर्भर रहेंगी।

JAC Class 12 Political Science Important Questions Chapter 5 कांग्रेस प्रणाली : चुनौतियाँ और पुनर्स्थापना

प्रश्न 4.
1967 के चौथे आम चुनाव में कांग्रेस दल की मुख्य चुनौतियाँ बताइये।
उत्तर:
1967 के चौथे आम चुनाव में कांग्रेस दल की मुख्य चुनौतियाँ इस प्रकार रहीं।

  1. राजनीतिक प्रतिस्पर्द्धा अब गहन हो गई थी और ऐसे में कांग्रेस को अपना प्रभुत्व बरकरार रखने में मुश्किलें आ रही थीं।
  2. विपक्ष अब पहले की अपेक्षा कम विभाजित तथा कहीं ज्यादा ताकतवर था। गैर-कांग्रेसवाद के नारे के साथ वह काफी एकजुट हो गया था।
  3. कांग्रेस को अन्दरूनी चुनौतियों का सामना भी करना पड़ रहा था क्योंकि यह पार्टी के अन्दर विभिन्नता को थाम कर नहीं चल पा रही थी।

प्रश्न 5.
भारत में सम्पन्न चौथे आम चुनाव परिणामों की संक्षेप में व्याख्या कीजिए।
अथवा
भारत में 1967 के चुनाव परिणामों को राजनीतिक भूचाल क्यों कहा गया?
उत्तर:
चौथे आम चुनाव के परिणाम इस प्रकार रहे।

  1. इन चुनावों में भारतीय मतदाताओं ने कांग्रेस को वैसा समर्थन नहीं दिया, जो पहले तीन आम चुनावों में दिया था।
  2. लोकसभा की कुल 520 सीटों में से कांग्रेस को केवल 283 सीटें ही मिल पाईं तथा मत प्रतिशत में भी भारी गिरावट आई।
  3. इन्दिरा गाँधी के मंत्रिमंडल के आधे मंत्री चुनाव हार गये थे।
  4. इसके साथ-साथ कांग्रेस को 8 राज्य विधानसभाओं में भी हार का सामना करना पड़ा। इसलिए अनेक राजनीतिक पर्यवेक्षकों ने इन अप्रत्याशित चुनाव परिणामों को राजनीतिक भूचाल की संज्ञा दी।

प्रश्न 6.
1969 में राष्ट्रपति के निर्वाचन में श्रीमती इंदिरा गांधी ने किस उम्मीदवार का साथ दिया?
उत्तर:
1969 के राष्ट्रपति के चुनाव में इंदिरा गांधी ने निर्दलीय उम्मीदवार वी. वी. गिरि का साथ दिया और कांग्रेस के अधिकृत उम्मीदवार नीलम संजीव रेड्डी चुनाव हार गए। यह चुनाव श्रीमती गाँधी का कांग्रेस में वर्चस्व स्थापित करने में मील का पत्थर सिद्ध हुआ।

प्रश्न 7.
कांग्रेस की फूट (विभाजन) के किन्हीं दो कारणों का उल्लेख कीजिए।
उत्तर:

  1. 1967 के चुनावों में मिली हार के बाद कुछ कांग्रेसी दक्षिणपंथी दलों के साथ जबकि कुछ कांग्रेसी वामपंथी दलों के साथ समझौते का समर्थन कर रहे थे, जिससे कांग्रेस में मतभेद बढ़ा
  2. कांग्रेस पार्टी के प्रमुख सदस्यों में 1967 में होने वाले राष्ट्रपति के चुनावों को लेकर मतभेद था जो अन्ततः कांग्रेस के विभाजन का तात्कालिक कारण बना।

प्रश्न 8.
कांग्रेस की फूट के पश्चात् कौनसे दो राजनीतिक दल उभरे?
उत्तर:
1969 में कांग्रेस विभाजन के पश्चात् जो दो राजनीतिक दल सामने आए उनके नाम कांग्रेस (आर) (Congress Requisitioned) तथा कांग्रेस (ओ) (Congress Organisation) थे। कांग्रेस (आर) का नेतृत्व श्रीमती गाँधी कर रही थीं, वहीं कांग्रेस (ओ) का नेतृत्व सिंडिकेट कर रहा था जिसके अध्यक्ष निजलिंगप्पा थे। गई?

प्रश्न 9.
आया राम गया राम’ किस वर्ष से संबंधित घटना है तथा यह टिप्पणी किस व्यक्ति के सम्बन्ध में की
उत्तर:
‘आया राम गया राम’ सन् 1967 के वर्ष से संबंधित घटना है। ‘आया राम गया राम’ नामक टिप्पणी कांग्रेस के हरियाणा राज्य के एक विधायक गया लाल के सम्बन्ध में की गई थी जिसने एक पखवाड़े के अन्दर तीन दफा अपनी पार्टी बदली।

प्रश्न 10.
1960 के दशक में कांग्रेस प्रणाली को पहली बार चुनौती क्यों मिली?
उत्तर:
कांग्रेस प्रणाली को 1960 के दशक में पहली बार चुनौती मिली क्योंकि राजनीतिक प्रतिस्पर्धा गहन हो चली थी और ऐसे में कांग्रेस को अपना प्रभुत्व बरकरार रखने में मुश्किलें आ रही थीं। विपक्ष अब पहले की अपेक्षा कम विभाजित – और ज्यादा ताकतवर था । कांग्रेस को अंदरूनी चुनौतियाँ भी झेलनी पड़ीं, क्योंकि अब यह पार्टी अपने अंदर की विभिन्नता को एक साथ थामकर नहीं चल पा रही थी।

प्रश्न 11.
1969-1971 के दौरान इन्दिरा गाँधी की सरकार द्वारा सामना किये जाने वाली किन्हीं दो समस्याओं का उल्लेख कीजिये।
उत्तर:
श्रीमती इन्दिरा गाँधी ने सन् 1969-71 के दौरान निम्नलिखित दो चुनौतियों का सामना किया।

  1. उन्हें सिंडिकेट (मोरारजी देसाई के प्रभुत्व वाले कांग्रेस का दक्षिणपंथी गुट) के प्रभाव से मुक्त होकर अपना निजी मुकाम बनाने की चुनौती थी।
  2. कांग्रेस ने 1967 के चुनावों में जो जमीन खोई थी उसे वापस हासिल करना था।

JAC Class 12 Political Science Important Questions Chapter 5 कांग्रेस प्रणाली : चुनौतियाँ और पुनर्स्थापना

प्रश्न 12.
पांचवीं लोकसभा के चुनावों पर संक्षिप्त नोट लिखिए।
उत्तर:
पांचवीं लोकसभा के चुनाव 1971 में सम्पन्न हुए और चुनावों में श्रीमती गाँधी की कांग्रेस (आर) पार्टी ने शानदार सफलता प्राप्त की। लोकसभा की 518 सीटों में से कांग्रेस ने अकेले ही 352 सीटें जीतीं। इन चुनावों में कांग्रेस ने अपनी विरोधी पार्टियों को बुरी तरह से हराया।

प्रश्न 13.
किसने पाँचवीं लोकसभा निर्वाचन के समय ‘गरीबी हटाओ’ का नारा बुलन्द किया?
उत्तर:
1971 के पांचवीं लोकसभा के चुनावों में श्रीमती इंदिरा गांधी ने गरीबी हटाओ का नारा बुलन्द किया। कांग्रेस पार्टी ने इस चुनाव में भारी सफलता हासिल की तथा 352 स्थानों पर विजय प्राप्त की। कांग्रेस (ओ) ने केवल 10 प्रतिशत मत तथा केवल 16 स्थान प्राप्त किए।

प्रश्न 14.
पांचवीं लोकसभा के चुनावों में कांग्रेस की जीत के दो कारण बताएँ।
उत्तर:

  1. 1971 में पांचवीं लोकसभा के चुनावों में कांग्रेस की जीत का सबसे बड़ा कारण श्रीमती गाँधी का चमत्कारिक नेतृत्व था।
  2. 1971 में श्रीमती गांधी की जीत का सबसे महत्त्वपूर्ण कारण इनके द्वारा की गई समाजवादी नीति सम्बन्धी पहल तथा ‘गरीबी हटाओ’ का नारा था।

प्रश्न 15.
सिंडिकेट पर दक्षिणपंथियों से गुप्त समझौते करने के आरोप क्यों लगे?
उत्तर:
कांग्रेस ने जब राष्ट्रपति पद के लिए नीलम संजीव रेड्डी को आधिकारिक उम्मीदवार घोषित किया, तो कार्यवाहक राष्ट्रपति वी. वी. गिरि ने राष्ट्रपति का चुनाव लड़ने की घोषणा कर दी तथा श्रीमती इन्दिरा गांधी ने अंतरात्मा के आधार पर. मत देने की बात कहकर वी.वी. गिरि का समर्थन कर दिया, तो निजलिंगप्पा ने जनसंघ तथा स्वतन्त्र पार्टी

प्रश्न 7.
कांग्रेस की फूट ( विभाजन) के किन्हीं दो कारणों का उल्लेख कीजिए।
उत्तर:

  1. 1967 के चुनावों में मिली हार के बाद कुछ कांग्रेसी दक्षिणपंथी दलों के साथ जबकि कुछ कांग्रेसी वामपंथी दलों के साथ समझौते का समर्थन कर रहे थे, जिससे कांग्रेस में मतभेद बढ़ा।
  2. कांग्रेस पार्टी के प्रमुख सदस्यों में 1967 में होने वाले राष्ट्रपति के चुनावों को लेकर मतभेद था जो अन्ततः कांग्रेस के विभाजन का तात्कालिक कारण बना।

प्रश्न 8.
कांग्रेस की फूट के पश्चात् कौनसे दो राजनीतिक दल उभरे?
उत्तर:
969 में कांग्रेस विभाजन के पश्चात् जो दो राजनीतिक दल सामने आए उनके नाम कांग्रेस (आर) (Congress Requisitioned) तथा कांग्रेस (ओ) (Congress Organisation) थे। कांग्रेस (आर) का नेतृत्व श्रीमती गाँधी कर रही थीं, वहीं कांग्रेस (ओ) का नेतृत्व सिंडिकेट कर रहा था जिसके अध्यक्ष निजलिंगप्पा थे गई?

प्रश्न 9.
आया राम गया राम’ किस वर्ष से संबंधित घटना है तथा यह टिप्पणी किस व्यक्ति के सम्बन्ध में की गई?
उत्तर:
‘आया राम गया राम’ सन् 1967 के वर्ष से संबंधित घटना है। ‘ आया राम गया राम’ नामक टिप्पणी कांग्रेस के हरियाणा राज्य के एक विधायक गया लाल के सम्बन्ध में की गई थी जिसने एक पखवाड़े के अन्दर तीन दफा अपनी पार्टी बदली।

प्रश्न 10.
1960 के दशक में कांग्रेस प्रणाली को पहली बार चुनौती क्यों मिली?
उत्तर:
कांग्रेस प्रणाली को 1960 के दशक में पहली बार चुनौती मिली क्योंकि राजनीतिक प्रतिस्पर्धा गहन हो चली थी और ऐसे में कांग्रेस को अपना प्रभुत्व बरकरार रखने में मुश्किलें आ रही थीं। विपक्ष अब पहले की अपेक्षा कम विभाजित और ज्यादा ताकतवर था। कांग्रेस को अंदरूनी चुनौतियाँ भी झेलनी पड़ीं, क्योंकि अब यह पार्टी अपने अंदर की विभिन्नता को एक साथ थामकर नहीं चल पा रही थी।

JAC Class 12 Political Science Important Questions Chapter 5 कांग्रेस प्रणाली : चुनौतियाँ और पुनर्स्थापना

प्रश्न 11.
1969-1971 के दौरान इन्दिरा गाँधी की सरकार द्वारा सामना किये जाने वाली किन्हीं दो समस्याओं का उल्लेख कीजिये।
उत्तर:
श्रीमती इन्दिरा गाँधी ने सन् 1969-71 के दौरान निम्नलिखित दो चुनौतियों का सामना।

  1. उन्हें सिंडिकेट (मोरारजी देसाई के प्रभुत्व वाले कांग्रेस का दक्षिणपंथी गुट) के प्रभाव से मुक्त होकर अपना निजी मुकाम बनाने की चुनौती थी।
  2. कांग्रेस ने 1967 के चुनावों में जो जमीन खोई थी उसे वापस हासिल करना था।

प्रश्न 12.
पांचवीं लोकसभा के चुनावों पर संक्षिप्त नोट लिखिए।
उत्तर:
पांचवीं लोकसभा के चुनाव 1971 में सम्पन्न हुए और चुनावों में श्रीमती गाँधी की कांग्रेस (आर) पार्टी ने शानदार सफलता प्राप्त की। लोकसभा की 518 सीटों में से कांग्रेस ने अकेले ही 352 सीटें जीतीं। इन चुनावों में कांग्रेस ने अपनी विरोधी पार्टियों को बुरी तरह से हराया।

प्रश्न 13.
किसने पाँचवीं लोकसभा निर्वाचन के समय ‘गरीबी हटाओ’ का नारा बुलन्द किया?
उत्तर:
1971 के पांचवीं लोकसभा के चुनावों में श्रीमती इंदिरा गांधी ने गरीबी हटाओ का नारा बुलन्द किया। कांग्रेस पार्टी ने इस चुनाव में भारी सफलता हासिल की तथा 352 स्थानों पर विजय प्राप्त की। कांग्रेस (ओ) ने केवल 10 प्रतिशत मत तथा केवल 16 स्थान प्राप्त किए।

प्रश्न 14.
पांचवीं लोकसभा के चुनावों में कांग्रेस की जीत के दो कारण बताएँ।
उत्तर:

  1. 1971 में पांचवीं लोकसभा के चुनावों में कांग्रेस की जीत का सबसे बड़ा कारण श्रीमती गाँधी का चमत्कारिक नेतृत्व था।
  2. 1971 में श्रीमती गांधी की जीत का सबसे महत्त्वपूर्ण कारण इनके द्वारा की गई समाजवादी नीति सम्बन्धी पहल तथा ‘गरीबी हटाओ’ का नारा था।

प्रश्न 15.
सिंडिकेट पर दक्षिणपंथियों से गुप्त समझौते करने के आरोप क्यों लगे?
उत्तर:
कांग्रेस ने जब राष्ट्रपति पद के लिए नीलम संजीव रेड्डी को आधिकारिक उम्मीदवार घोषित किया, तो कार्यवाहक राष्ट्रपति वी. वी. गिरि ने राष्ट्रपति का चुनाव लड़ने की घोषणा कर दी तथा श्रीमती इन्दिरा गांधी ने अंतरात्मा के आधार पर मत देने की बात कहकर वी.वी. गिरि का समर्थन कर दिया, तो निजलिंगप्पा ने जनसंघ तथा स्वतन्त्र पार्टी जैसे दक्षिणपंथी पार्टियों से अनुरोध किया कि वे अपना मत नीलम संजीव रेड्डी के पक्ष में डालें । इस पर जगजीवन राम तथा फखरुद्दीन अहमद ने सिंडीकेट पर दक्षिणपंथियों के साथ गुप्त समझौता करने का आरोप लगाया।

JAC Class 12 Political Science Important Questions Chapter 5 कांग्रेस प्रणाली : चुनौतियाँ और पुनर्स्थापना

प्रश्न 16.
लालबहादुर शास्त्री के जीवन पर संक्षिप्त नोट लिखिए।
उत्तर:
श्री लालबहादुर शास्त्री ( 1904-1966 ):
श्री लालबहादुर शास्त्री का जन्म 1904 में हुआ। 1930 से स्वतन्त्रता आंदोलन में भागीदारी की और उत्तरप्रदेश मंत्रिमण्डल में मंत्री रहे। उन्होंने कांग्रेस पार्टी के महासचिव का पदभार सँभाला। वे 1951 – 56 तक केन्द्रीय मंत्रिमंडल में मंत्री पद पर रहे। इसी दौरान रेल दुर्घटना की नैतिक जिम्मेदारी लेते हुए उन्होंने रेल मंत्री के पद से इस्तीफा दे दिया था। 1957-64 के बीच वह मंत्री पद पर रहे। जून, 1964 से अक्टूबर, 1966 तक वह भारत के प्रधानमंत्री पद पर रहे तथा

1965 के भारत-पाक युद्ध में उन्होंने ‘जय जवान जय किसान’ का नारा दिया तथा जीत हासिल की। उन्होंने देश में हरित क्रांति को आगे बढ़ाने के लिए प्रयोग किया तथा भारत को खाद्यान्न के क्षेत्र में आत्मनिर्भर देश बनाया। लेकिन शास्त्रीजी का देहान्त 10 अक्टूबर, 1966 को ताशकंद के उजबेग शहर में हो गया। वे प्रधानमंत्री पद पर अधिक समय तक नहीं रहे परंतु वे एक युद्धवीर साबित हुए।

प्रश्न 17.
श्रीमती इंदिरा गांधी के जीवन का संक्षिप्त परिचय देते हुए लाल बहादुर शास्त्री के उत्तराधिकारी के रूप में श्रीमती गाँधी पर संक्षिप्त नोट लिखिए।
उत्तर:
श्रीमती इन्दिरा गाँधी का संक्षिप्त परिचय – इंदिरा प्रियदर्शनी 1917 में जवाहर लाल नेहरू के परिवार में उत्पन्न हुईं। वह शास्त्रीजी के देहान्त के पश्चात् पहली महिला प्रधानमंत्री बनीं। 1966 से 1977 तक और फिर 1980 से 1984 तक भारत की प्रधानमंत्री रहीं। युवा कांग्रेस कार्यकर्ता के रूप में स्वतंत्रता आंदोलन में श्रीमती गाँधी की भागीदारी रहीं। 1958 में वह कांग्रेस के अध्यक्ष पद पर आसीन हुईं तथा 1964 से 1966 तक शास्त्री मंत्रिमण्डल में केन्द्रीय मन्त्री पद पर रहीं।

1967, 1971 और 1980 के आम चुनावों में कांग्रेस पार्टी ने श्रीमती गाँधी के नेतृत्व में सफलता प्राप्त की। उन्होंने ‘गरीबी हटाओ’ का लुभावना नारा दिया, 1971 में युद्ध में विजय का श्रेय और प्रिवी पर्स की समाप्ति, बैंकों के राष्ट्रीयकरण, आण्विक परीक्षण तथा पर्यावरण संरक्षण के कदम उठाए। 31 अक्टूबर, 1984 के दिन इनकी हत्या कर दी गई।

प्रश्न 18.
दल-बदल पर संक्षिप्त टिप्पणी लिखिए।
अथवा
आया राम-गया राम से क्या अभिप्राय है?
उत्तर:
दल-बदल – भारत की राजनीति में ध्यानाकर्षण करने वाली कुरीति ( बुराई ) दल-बदल रही है। 1967 के आम चुनाव की एक खास बात दल-बदल की रही। इस विशेषता के कारण कई राज्यों में सरकारों के बनने और बिगड़ने की महत्त्वपूर्ण घटनाएँ दृष्टिगोचर हुईं। जब कोई जनप्रतिनिधि किसी विशेष राजनैतिक पार्टी का चुनाव चिह्न लेकर चुनाव लड़े और वह चुनाव जीत जाए और अपने स्वार्थ के लिए या किसी अन्य कारण से मूल दल छोड़कर किसी अन्य दल में शामिल हो जाए, इसे दल-बदल कहते हैं। 1967 के सामान्य चुनावों के बाद कांग्रेस को छोड़ने वाले विधायकों ने तीन राज्यों हरियाणा, मध्यप्रदेश और उत्तर प्रदेश में गैर-कांग्रेसी सरकारों को गठित करने में अहम भूमिका निभाई। इस दल-बदल के कारण ही देश में एक मुहावरा या लोकोक्ति – ‘ आया राम-गया राम’ बहुत प्रसिद्ध हो गया।

प्रश्न 19.
के. कामराज कौन थे? उनके जीवन के बारे में संक्षेप में परिचय दीजिए।
उत्तर:
के. कामराज़ का जन्म 1903 में हुआ था। वे देश के महान स्वतन्त्रता एक प्रमुख नेता के रूप में अत्यधिक ख्याति प्राप्त की। सेनानी थे। उन्होंने कांग्रेस के उन्हें मद्रास (तमिलनाडु) के मुख्यमंत्री के पद पर रहने का सौभाग्य मिला। मद्रास प्रान्त में शिक्षा का प्रसार और स्कूली बच्चों को दोपहर का भोजन देने की योजना लागू करने के लिए उन्हें अत्यधिक ख्याति प्राप्त हुई। उन्होंने 1963 में कामराज योजना नाम से मशहूर एक प्रस्ताव रखा जिसके अंतर्गत उन्होंने सभी वरिष्ठ कांग्रेसी नेताओं को त्यागपत्र दे देने का सुझाव दिया, ताकि जो अपेक्षाकृत कांग्रेस पार्टी के युवा कार्यकर्ता हैं वे पार्टी की कमान संभाल सकें। वह कांग्रेस पार्टी के अध्यक्ष भी रहे। 1975 में उनका देहान्त हो गया।

प्रश्न 20.
1967 के गैर-कांग्रेसवाद एवं चुनावी बदलाव का वर्णन करें।
अथवा
1967 में हुए चौथे आम चुनाव पर संक्षिप्त टिप्पणी लिखें।
उत्तर:
1967 के चौथे आम चुनावों में भारतीय मतदाताओं ने कांग्रेस को वैसा समर्थन नहीं दिया जो पहले तीन आम चुनावों में दिया था। केन्द्र में जहाँ कांग्रेस मुश्किल से बहुमत प्राप्त कर पाई वहीं 8 राज्य विधान सभाओं (बिहार, केरल, मद्रास, उड़ीसा, पंजाब, राजस्थान, उत्तरप्रदेश तथा पश्चिमी बंगाल) में हार का सामना करना पड़ा। लोकसभा की कुल 520 सीटों में से कांग्रेस को केवल 283 सीटें ही मिल पाईं।

अतः जिस कांग्रेस पार्टी ने पहले तीन आम चुनावों में जिन विरोधी दलों को बुरी तरह से हराया वे दल चौथे लोकसभा चुनाव में बहुत अधिक सीटों पर चुनाव जीत गए। इसी तरह राज्यों में भी कांग्रेस की स्थिति 1967 के आम चुनावों में ठीक नहीं थी। 1967 के चुनाव बहुत बड़े उलट-फेर वाले रहे। पहली बार भारत में बड़े पैमाने पर गैर-कांग्रेसवाद की लहर चली तथा राज्यों में कांग्रेस का एकाधिकार समाप्त हो गया।

JAC Class 12 Political Science Important Questions Chapter 5 कांग्रेस प्रणाली : चुनौतियाँ और पुनर्स्थापना

प्रश्न 21.
इंदिरा गांधी बनाम सिंडिकेट पर टिप्पणी लिखिए। इंदिरा गांधी बनाम सिंडिकेट
उत्तर:
सन् 1967 के आम चुनावों में कांग्रेस को मिली हार के बाद कुछ कांग्रेसी दक्षिणपंथी दलों के साथ जबकि कुछ कांग्रेसी वामपंथी दलों के साथ समझौते का समर्थन कर रहे थे। दक्षिणपंथी दलों के साथ समर्थन का पक्ष लेने वाले नेताओं में रामराज, एस. के. पाटिल, निजलिंगप्पा तथा मोरारजी देसाई प्रमुख थे। इनके समूह को सिण्डीकेट के नाम से जाना जाता है। वामपंथी दलों के साथ समझौते के समर्थक युवा तुर्क कांग्रेसी तथा प्रधानमंत्री इन्दिरा गाँधी थीं।

इन दोनों के बीच मतभेद 1967 में राष्ट्रपति के पद के प्रत्याशी को लेकर मतभेद हुए जो पार्टी के विभाजन का कारण बने, 1969 में श्रीमती गाँधी द्वारा मोरारजी देसाई से वित्त विभाग वापस लेने के कारण श्रीमती इन्दिरा गाँधी और सिंडिकेट के बीच मतभेद अत्यन्त बढ़ गये। फलतः 1969 में कांग्रेस का विभाजन हो गया।

प्रश्न 22.
1969 में कांग्रेस पार्टी में विभाजन के क्या कारण थे?
अथवा
1969 में कांग्रेस पार्टी के विभाजन के प्रमुख कारणों का संक्षेप में वर्णन कीजिए।
उत्तर:
कांग्रेस पार्टी के विभाजन के कारण: 1969 में कांग्रेस पार्टी में विभाजन या फूट के प्रमुख कारण निम्नलिखित हैं।

  1. दक्षिणपंथी एवं वामपंथी विषय पर कलह: कांग्रेस के सदस्यों में इस बात पर मतभेद मुखर हो गया था कि वे वामपंथी दलों के साथ मिलकर लड़े या दक्षिणपंथी दलों के साथ।
  2. राष्ट्रपति पद के उम्मीदवार के विषय में मतभेद: कांग्रेस में 1967 में होने वाले राष्ट्रपति पद के प्रत्याशी को लेकर मतभेद था जो पार्टी के विभाजन का कारण बना।
  3. युवा तुर्क और सिण्डीकेट के बीच कलह: युवा तुर्क और सिण्डीकेट के मध्य मतभेद था। जहाँ युवा तुर्क बैंकों के राष्ट्रीयकरण एवं राजाओं के प्रिवी पर्स को समाप्त करने के पक्ष में था वहीं सिण्डीकेट इसका विरोध कर रहा था।
  4. मोरारजी देसाई से वित्त विभाग वापस लेना: 1969 में श्रीमती गाँधी द्वारा मोरारजी देसाई से वित्त विभाग वापस लेने के कारण भी मतभेद बढ़ा।

प्रश्न 23.
1971 के लोकसभा चुनाव में कांग्रेस की पुनर्स्थापना के कारणों को स्पष्ट कीजिए।
अथवा
1971 के चुनाव के बाद किन कारणों से कांग्रेस की पुनर्स्थापना हुई?
अथवा
पाँचवीं लोकसभा (1971) में कांग्रेस पार्टी की जीत के मूल कारणों का वर्णन कीजिए।
उत्तर:
पांचवीं लोकसभा के चुनावों में कांग्रेस की विजय के कारण: पांचवीं लोकसभा में कांग्रेस की विजय के प्रमुख कारण निम्नलिखित हैं।

  1. श्रीमती गांधी का चमत्कारिक नेतृत्व:1971 के चुनावों में कांग्रेस के पीछे श्रीमती गांधी के चमत्कारिक नेतृत्व का हाथ रहा।
  2. समाजवादी नीतियाँ: 1971 के चुनावों में श्रीमती गांधी ने समाजवादी नीतियाँ अपनायीं। उन्होंने प्रत्येक चुनाव रैली में समाजवाद के विषय में बढ़-चढ़ कर बातें कीं।
  3. गरीबी हटाओ का नारा: इस नारे के बल पर श्रीमती गांधी ने अधिकांश गरीबों के वोट बटोरे।
  4. कांग्रेस दल पर श्रीमती गांधी की पकड़: 1969 में कांग्रेस पार्टी के विभाजन के पश्चात् श्रीमती गाँधी पूरी तरह पार्टी पर छा गईं। कोई भी नेता उनकी आज्ञा का उल्लंघन नहीं कर सकता था।

JAC Class 12 Political Science Important Questions Chapter 5 कांग्रेस प्रणाली : चुनौतियाँ और पुनर्स्थापना

प्रश्न 24.
1971 के चुनाव के परिणामस्वरूप बदली हुई कांग्रेस व्यवस्था की प्रकृति कैसी थी?
उत्तर:
1971 के चुनाव के बाद कांग्रेस व्यवस्था की प्रकृति: 1971 के चुनाव के बाद बदली हुई कांग्रेस व्यवस्था की प्रकृति निम्नलिखित थी।

  1. 1971 के चुनाव के पश्चात् श्रीमती इन्दिरा गाँधी ने कांग्रेस को अपने ऊपर निर्भर बना दिया। अब यहाँ उनके आदेश सर्वोपरि बनने प्रारंभ हुए। सिंडीकेट जैसे अनौपचारिक प्रभावशाली नेताओं का समूह राजनीतिक मंच से गायब हो गया।
  2. 1971 के पश्चात् कांग्रेस का संगठन भिन्न-भिन्न विचारधाराओं वाले समूहों के समावेशी किस्म का नहीं रहा। अब यह अनन्य एकाधिकारिता वाला बन गया था।
  3. इन्दिरा गाँधी अब धनी उद्योगपतियों, सौदागरों तथा राजनीतिज्ञों के समूह अथवा सिंडीकेट के हाथों अब कठपुतली नहीं रहीं।

प्रश्न 25.
गरीबी हटाओ की राजनीति से आप क्या समझते हैं?
अथवा
गरीबी हटाओ की राजनीति पर संक्षिप्त टिप्पणी लिखिए।
उत्तर:
गरीबी हटाओ की राजनीति:
1971 के चुनावों में श्रीमती इंदिरा गाँधी के गरीबी हटाओ के नारे को मतदाताओं ने अधिक पसन्द करते हुए श्रीमती गांधी को भारी विजय दिलाई। गरीबी हटाओ कार्यक्रम के अन्तर्गत 1970- 1971 से नीतियाँ एवं कार्यक्रम बनाये जाने लगे तथा इस कार्यक्रम पर अधिक से अधिक धन खर्च किया जाने लगा। परन्तु 1971 के भारत-पाकिस्तान युद्ध एवं विश्व स्तर पर पैदा हुए तेल संकट के कारण श्रीमती गाँधी ने गरीबी हटाओ कार्यक्रम की राशि में कटौती करना शुरू कर दिया जिससे यह नारा कमजोर पड़ने लगा और केवल पांच साल के अन्दर श्रीमती गाँधी की गरीबी हटाओ की राजनीति असफल हो गई । परिणामतः 1977 के चुनावों में श्रीमती गांधी को पराजय का सामना करना पड़ा तथा जनता पार्टी को सत्ता प्राप्त हुई।

प्रश्न 26.
प्रारंभ में कांग्रेस का चुनाव चिह्न क्या था? कितने वर्ष बाद कांग्रेस फूट का शिकार बनी? अब इस दल का चुनाव चिह्न क्या है?
उत्तर:
प्रारंभ में कांग्रेस का चुनाव चिह्न दो बैलों की जोड़ी था। आजादी के 22 वर्ष गुजरते गुजरते कांग्रेस में व्यापक फूट हो गई थी। अब इस दल का चुनाव चिह्न हाथ का पंजा है।

प्रश्न 27.
राममनोहर लोहिया का संक्षिप्त परिचय दीजिए।
उत्तर:
राममनोहर लोहिया: राममनोहर लोहिया का जन्म 1910 में हुआ। वे समाजवादी नेता एवं विचारक थे तथा स्वतन्त्रता सेनानी एवं सोशलिस्ट पार्टी के सदस्य थे। वे मूल पार्टी में विभाजन के बाद सोशलिस्ट पार्टी एवं बाद में संयुक्त सोशलिस्ट पार्टी के नेता बने। 1963 से 1967 तक लोकसभा सांसद रहे। वे ‘मैन काइंड’ एवं ‘जन’ के सम्पादक बने। गैर-यूरोपीय समाजवादी सिद्धान्त के विकास में उनका मौलिक योगदान रहा।

वे भारतीय राजनीति के क्षितिज पर गैर- कांग्रेसवादी विचारधारा के संयोजनकर्ता और रणनीतिकार के रूप में उभरे। उन्होंने जीवन भर दलित और पिछड़े वर्गों को आरक्षण दिए जाने की वकालत की। उन्होंने प्रारम्भ में नेहरू के खिलाफ मोर्चा खोला। भाषा की दृष्टि से वे अंग्रेजी के घोर विरोधी थे। उनका देहान्त 1967 में हुआ।

प्रश्न 28.
प्रिवी पर्स से आप क्या समझते हैं? इस व्यवस्था की समाप्ति के विभिन्न प्रयासों पर प्रकाश डालिए।
अथवा
प्रिवी पर्स का क्या अर्थ है? 1970 में इंदिरा गाँधी इसे क्यों समाप्त कर देना चाहती थीं? प्रिवी पर्स की समाप्ति पर संक्षिप्त टिप्पणी लिखिए।
उत्तर:
प्रिवी पर्स से आशय भारत में स्वतन्त्रता प्राप्ति के समय देशी रियासतों को भारतीय संघ में सम्मिलित किया गया तथा इनके तत्कालीन शासकों को जीवनयापन हेतु विशेष धनराशि एवं भत्ते दिये जाने की व्यवस्था की गई। इसे प्रिवी पर्स के नाम से जाना जाता है। प्रिवीपर्स की समाप्ति श्रीमती इंदिरा गांधी की सरकार ने प्रिवी पर्स को समाप्त करने के लिए 1970 में संविधान में संशोधन का प्रयास किया, लेकिन राज्य सभा में यह मंजूरी नहीं पा सका। इसके बाद सरकार ने एक अध्यादेश जारी किया, लेकिन इसे सर्वोच्च न्यायालय ने निरस्त कर दिया। श्रीमती गांधी ने इसे 1971 के चुनावों में एक बड़ा मुद्दा बनाया और 1971 में मिली भारी जीत के बाद संविधान में संशोधन किया गया। इस प्रकार प्रिवी पर्स की समाप्ति की राह में मौजूदा कानूनी अड़चनें समाप्त हुईं।

प्रश्न 29.
1971 के चुनावों में इंदिरा गाँधी की नाटकीय जीत के लिए कौन-कौनसे प्रमुख कारक जिम्मेदार थे?
उत्तर:
हालाँकि 1971 के चुनावी मुकाबले में काँग्रेस की स्थिति अत्यंत ही कमजोर थी, इसके बावजूद नयी काँग्रेस के साथ एक ऐसी बात थी, जिसका उसके बड़े विपक्षियों के पास अभाव था। नयी काँग्रेस के पास एक मुद्दा था; एक एजेंडा और कार्यक्रम था। विपक्ष के ‘इंदिरा हटाओ’ के विपरीत इंदिरा गाँधी ने लोगों के सामने ‘गरीबी हटाओ’ नामक सकारात्मक कार्यक्रम रखा। यह इंदिरा गाँधी की नाटकीय जीत के लिए एक प्रमुख कारक साबित हुआ।

प्रश्न 30.
लाल बहादुर शास्त्री की मृत्यु के बाद वरिष्ठ कांग्रेस नेता ने इंदिरा गाँधी को प्रधानमंत्री के रूप में समर्थन क्यों दिया?
उत्तर:
लाल बहादुर शास्त्री की मृत्यु के बाद वरिष्ठ कांग्रेस नेता ने इंदिरा गाँधी को प्रधानमंत्री के रूप में समर्थन दिया क्योंकि।

  1. इंदिरा गाँधी जवाहरलाल नेहरू की बेटी थीं और वह पूर्व में काँग्रेस की अध्यक्ष रह चुकी थीं तथा शास्त्री के मंत्रिमंडल में मंत्री भी रह चुकी थीं।
  2. वरिष्ठ नेताओं का मानना था कि इंदिरा गाँधी की प्रशासनिक और राजनीतिक अनुभवहीनता उन्हें समर्थन और मार्गदर्शन के लिए उन पर निर्भर होने पर मजबूर करेंगी और श्रीमती गाँधी उनकी सलाहों पर अमल करेंगी।

प्रश्न 31.
1967 में इंदिरा गाँधी सरकार ने भारतीय रुपये का अवमूल्यन क्यों किया?
उत्तर:
इंदिरा गाँधी सरकार ने 1967 के आर्थिक संकट की जाँच के लिए भारतीय रुपये का अवमूल्यन किया। परिणामतः 1 अमरीकी डॉलर की कीमत 5 रुपये थी जो बढ़कर 7 रुपये हो गई।

  1. आर्थिक स्थिति की विकटता के कारण कीमतों में तेजी से वृद्धि हुई।
  2. लोग आवश्यक वस्तुओं की कीमतों में वृद्धि, खाद्यान्न की कमी, बढ़ती हुई बेरोजगारी और देश की दयनीय आर्थिक स्थिति को लेकर विरोध पर उतर आए।
  3. साम्यवादी और समाजवादी पार्टी ने व्यापक समानता के लिए संघर्ष छेड़ दिया।

प्रश्न 32.
काँग्रेस को दूसरी बार राजनीतिक उत्तराधिकार की चुनौती का सामना कैसे करना पड़ा?
उत्तर:
काँग्रेस को दूसरी बार राजनीतिक उत्तराधिकार की चुनौती का सामना लाल बहादुर शास्त्री की मृत्यु के उपरांत करना पड़ा।

  1. यह चुनौती मोरारजी देसाई और इंदिरा गाँधी के बीच एक गुप्तदान के माध्यम से हल करने के लिए एक गहन प्रतियोगिता के साथ शुरू हुई।
  2. इस चुनाव में इंदिरा गाँधी ने मोरारजी देसाई को हरा दिया था। उन्हें कांग्रेस पार्टी के दो-तिहाई से अधिक सांसदों ने अपना मत दिया था।
  3. नेतृत्व के लिए प्रतिस्पर्धा के बावजूद पार्टी में सत्ता का हस्तांतरण बड़े शांतिपूर्ण ढंग से सम्पन्न हो गया।

प्रश्न 33.
गठबंधन के नए युग में संयुक्त विधायक दल की स्थिति क्या थी?
उत्तर:
1967 के चुनावों से गठबंधन की परिघटना सामने आयी। क्योंकि किसी पार्टी को बहुमत नहीं मिला था, इसलिए अनेक गैर-कांग्रेसी पार्टियों ने एकजुट होकर संयुक्त विधायक दल बनाया और गैर-कांग्रेसी सरकार को समर्थन दिया। इसी कारण इन सरकारों को संयुक्त विधायक दल की सरकार कहा गया-।

  1. बिहार में बनी संयुक्त विधायक दल की सरकार में वामपंथी: सीपीआई और दक्षिणपंथी जनसंघ शामिल था।
  2. पंजाब में बनी संयुक्त विधायक दल की सरकार को ‘पॉपुलर यूनाइटेड फ्रंट’ की सरकार कहा गया। इसमें परस्पर प्रतिस्पर्धी अकाली दल – संत ग्रुप और मास्टर ग्रुप शामिल थे।

JAC Class 12 Political Science Important Questions Chapter 5 कांग्रेस प्रणाली : चुनौतियाँ और पुनर्स्थापना

प्रश्न 34.
1970 के दशक की शुरुआत में इंदिरा गाँधी की लोकप्रियता बढ़ाने वाले तीन कारकों का विश्लेषण कीजिए।
अथवा
इंदिरा गाँधी सरकार ने अपनी छवि को समाजवादी बनाने के लिए क्या-क्या कदम उठाए?
उत्तर:

  1. 1970 के दशक में इंदिरा गाँधी ने भूमि सुधार के मौजूदा कानूनों के क्रियान्वय के लिए जबरदस्त अभियान चलाए। उन्होंने भू- परिसीमन के कुछ और कानून भी बनवाए।
  2. दूसरे राजनीतिक दलों पर अपनी निर्भरता समाप्त करने, संसद में अपनी पार्टी की स्थिति मजबूत करने और अपने कार्यक्रमों के पक्ष में जनादेश हासिल करने के लिए 1970 के दिसंबर में लोकसभा भंग करने की सिफारिश की।
  3. भारत-पाक युद्ध में बाँग्लादेश को एक स्वतंत्र राष्ट्र के रूप में स्थापित करने के संकट ने भी इंदिरा गाँधी की लोकप्रियता को बढ़ावा दिया।

प्रश्न 35.
1967 के चुनाव को भारत के राजनीतिक और चुनावी इतिहास में एक ऐतिहासिक वर्ष क्यों माना गया? टिप्पणी लिखिए।
उत्तर:
1967 के चुनाव को भारत के राजनीतिक और चुनावी इतिहास में ऐतिहासिक वर्ष माना गया क्योंकि।

  1. यह चुनाव नेहरू के बिना पहली बार आयोजित किया गया था । इस चुनाव का फैसला काँग्रेस के पक्ष में नहीं था और इसके नतीजों ने राष्ट्रीय और राज्य दोनों स्तरों पर काँग्रेस को झटका दिया।
  2. इंदिरा गाँधी मंत्रिमंडल के आधे मंत्री चुनाव हार गए थे। इसमें तमिलनाडु से कामराज, महाराष्ट्र से एस. के. पाटिल, पश्चिम बंगाल से अतुल्य घोष और बिहार से के. बी. सहाय इत्यादि शामिल हैं।
  3. चुनावी इतिहास में यह पहली घटना थी जब किसी गैर- काँग्रेसी दल को किसी राज्य में पूर्ण बहुमत मिला।

निबन्धात्मक प्रश्न

प्रश्न 1.
पण्डित जवाहर लाल नेहरू के बाद राजनीतिक उत्तराधिकार पर एक निबन्ध लिखिए।
उत्तर:
सन् 1964 में नेहरू की मृत्यु के बाद यह आशंका व्यक्त की जा रही थी कि भारत में कांग्रेस पार्टी में उत्तराधिकार के लिए संघर्ष छिड़ जायेगा और कांग्रेस पार्टी बिखर जायेगी। लेकिन यह सब गलत साबित हुआ और पं. नेहरू की मृत्यु के बाद उनके राजनैतिक उत्तराधिकारी के रूप में पहले लालबहादुर शास्त्री तथा बाद में श्रीमती इन्दिरा गाँधी ने सफलतापूर्वक कार्य किया।
1. श्री लालबहादुर शास्त्री:
लालबहादुर शास्त्री ने 6 जून, 1964 को भारत के प्रधानमंत्री पद की शपथ ली। जब शास्त्रीजी ने नेहरू की मृत्यु के बाद कार्यभार सँभाला उस समय देश अनेक संकटों का सामना कर रहा था। देश में अनाज की कमी थी। देश का मनोबल नीचा था, सीमान्त क्षेत्रों में तनाव विद्यमान था आदि। शास्त्रीजी ने इन चुनौतियों का दृढ़तापूर्वक सामना किया तथा कृषि पर बल देते हुए ‘अधिक अन्न उगाओ के साथ ही ‘जय जवान जय किसान’ का मशहूर नारा भी दिया।

1965 में पाकिस्तान के आक्रमण का शास्त्रीजी की सूझ-बूझ एवं कुशल नेतृत्व से युद्ध में विजय प्राप्त की । सोवियत संघ के प्रयासों से 1966 में भारत-पाकिस्तान के बीच ताशकंद में समझौता हुआ और ताशकन्द में ही जनवरी, 1966 में शास्त्रीजी की संदिग्ध परिस्थितियों में मृत्यु हो गई।

2. श्रीमती इंदिरा गांधी: शास्त्रीजी की मृत्यु के पश्चात् श्रीमती इन्दिरा गाँधी को प्रधानमंत्री बनाया गया। अपने प्रधानमंत्री काल में श्रीमती गांधी ने ऐसे कई कार्य किए जिससे देश प्रगति कर सके। उन्होंने कृषि कार्यों को बढ़ावा दिया। गरीबी को हटाने के लिए कार्यक्रम घोषित किया तथा देश की सेनाओं का आधुनिकीकरण किया। 1974 में पोकरण में ऐतिहासिक परमाणु परीक्षण किया। 1971 में हुए भारत-पाक युद्ध में भारत की निर्णायक विजय हुई तथा बांग्लादेश का निर्माण हुआ। इस युद्ध में विजय के बाद विश्व में भारत की प्रतिष्ठा बढ़ी।

प्रश्न 2.
1971 में पांचवीं लोकसभा चुनावों में कांग्रेस की विजय पर एक संक्षिप्त नोट लिखिए।
अथवा
पांचवीं लोकसभा में कांग्रेस पार्टी की जीत के प्रमुख कारणों का विवेचन कीजिए।
उत्तर:
1971 का लोकसभा चुनाव इंदिरा गांधी के लिए मील का पत्थर साबित हुआ। श्रीमती गाँधी पार्टी अनुभवी नेताओं को छोड़कर, निर्वाचकों के साथ सीधा संपर्क स्थापित करने के लिए निकल पड़ीं और चुनावों में अप्रत्याशित सफलता प्राप्त की। श्रीमती गाँधी की पार्टी को लोकसभा की 518 सीटों में से 352 सीटें प्राप्त हुईं। कांग्रेस की इस अप्रत्याशित सफलता के पीछे अनेक कारण जिम्मेदार रहे, जो इस प्रकार हैं।

  1. श्रीमती गांधी का प्रभावशाली नेतृत्व: 1971 के लोकसभा चुनावों में कांग्रेस पार्टी की सफलता का मूल कारण श्रीमती गाँधी का चमत्कारिक एवं प्रभावशाली नेतृत्व रहा।
  2. पार्टी पर श्रीमती गांधी की पकड़ व नियन्त्रण: इस चुनाव में श्रीमती गांधी की अपने दल पर पूरी पकड़ एवं प्रभाव था।
  3. गरीबी हटाओ का नारा: 1971 में कांग्रेस पार्टी की सफलता का सबसे महत्त्वपूर्ण कारण उनके द्वारा दिया गया गरीबी हटाओ का नारा था । इस नारे के बल पर श्रीमती गांधी ने अधिकांश गरीबों के वोट अपनी झोली में डाले।
  4. समाजवादी नीतियों एवं कार्यक्रमों का निर्धारण: 1971 के चुनावों में कांग्रेस की अप्रत्याशित जीत का एक और कारण बैंकों का राष्ट्रीयकरण, राजाओं के प्रीविपर्स बन्द करना तथा श्रीमती गाँधी की अन्य समाजवादी नीतियों को भी माना जाता है। इस प्रकार श्रीमती गांधी का चमत्कारिक नेतृत्व गरीबी हटाओ का नारा, बैंकों का राष्ट्रीयकरण व समाजवादी नीतियों का अपनाना ऐसे मुद्दे थे जिन्होंने 1971 के चुनावों में कांग्रेस पार्टी को भारी सफलता दिलाई।

प्रश्न 3.
1970 के दशक में इंदिरा गाँधी की सरकार की लोकप्रियता बढ़ाने वाले कारकों का विस्तारपूर्वक विश्लेषण कीजिए
उत्तर:
1. लोकसभा का पांचवा आमचुनाव 1971 में हुआ था। चुनावी मुकाबला काँग्रेस (आर) के विपरीत जान पड़ रहा था। हर किसी को विश्वास था कि काँग्रेस पार्टी की असली सांगठनिक ताकत काँग्रेस (ओ) के नियंत्रण में है। इसके अतिरिक्त, सभी बड़ी गैर – साम्यवादी और गैर-कांग्रेसी विपक्षी पार्टियों ने एक चुनावी गठबंधन बना लिया था। से ‘ग्रैंड अलायंस’ कहा गया। एसएसपी, पीएसपी, भारतीय जनसंघ, स्वतंत्र पार्टी और भारतीय क्रांतिदल, चुनाव में एक छतरी के नीचे आ गए। शासक दल ने भारतीय कम्युनिस्ट पार्टी के साथ गठजोड़ किया। इसके बावजूद नयी काँग्रेस के साथ एक ऐसी बात थी, जिसका उसके बड़े विपक्षियों के पास अभाव था।

2. नयी कॉंग्रेस के पास एक मुद्दा था; एक एजेंडा और कार्यक्रम था। ‘ग्रैंड अलायंस’ के पास कोई सुसंगत राजनीतिक कार्यक्रम नहीं था। विपक्षी गठबंधन के ‘इंदिरा हटाओ’ के विपरीत इंदिरा गाँधी के ‘गरीबी हटाओ’ नामक सकारात्मक कार्यक्रम रखा।

3. इंदिरा गाँधी ने सार्वजनिक क्षेत्र की संवृद्धि, ग्रामीण भू-स्वामित्व और शहरी संपदा के परिसीमन, आय और अवसरों की असमानता की समाप्ति तथा ‘प्रिवी पर्स’ की समाप्ति पर अपने चुनाव अभियान में जोर दिया। ‘गरीबी हटाओ’ के नारे से इंदिरा गाँधी ने वंचित तबकों खासकर भूमिहीन किसान, दलित और आदिवासी, अल्पसंख्यक, महिला और बेरोजगार नौजवानों के बीच अपने समर्थन का आधार तैयार करने की कोशिश की।

‘गरीबी हटाओ’ का नारा और इससे जुड़ा हुआ कार्यक्रम इंदिरा गाँधी की राजनीतिक रणनीति थी। परिणामस्वरूप इंदिरा गाँधी की काँग्रेस (आर) ने 352 सीटें और 44 प्रतिशत वोट हासिल किये। इस जीत के साथ इंदिरा गाँधी की अगुवाई वाली काँग्रेस ने अपने दावे को साबित कर दिया और भारतीय राजनीति में फिर अपना प्रभुत्व स्थापित किया।

JAC Class 12 Political Science Important Questions Chapter 5 कांग्रेस प्रणाली : चुनौतियाँ और पुनर्स्थापना

प्रश्न 4.
काँग्रेस ‘सिंडिकेट’ पर विस्तार में लिखिए।
उत्तर:
कांग्रेसी नेताओं के एक समूह को अनौपचारिक तौर पर ‘सिंडिकेट’ के नाम से इंगित किया जाता था। इस समूह के नेताओं का पार्टी के संगठन पर नियंत्रण था। ‘सिंडिकेट’ के अगुवा मद्रास प्रांत के भूतपूर्व मुख्यमंत्री और फिर काँग्रेस पार्टी के अध्यक्ष रह चुके के. कामराज थे। इसमें प्रांतों के ताकतवर नेता जैसे बंबई सिटी के एस. के. पाटिल, मैसूर के एस. निजलिंगप्पा, आंध्र प्रदेश के एन. संजीव रेड्डी और पश्चिम बंगाल के अतुल्य घोष शामिल थे। लालबहादुर शास्त्री और उसके बाद इंदिरा गाँधी दोनों ही सिंडिकेट की सहायता से प्रधानमंत्री के पद पर आरूढ़ हुए थे।

इंदिरा गाँधी के पहले मंत्रिपरिषद् में इस समूह की निर्णायक भूमिका रही। इसने तब नीतियों के निर्माण और क्रियान्वयन में अहम भूमिका निभायी थी। कांग्रेस के विभाजित होने के बाद सिंडिकेट के नेताओं और उनके प्रति निष्ठावान काँग्रेसी काँग्रेस (ओ) में ही रहे । चूँकि इंदिरा गाँधी की काँग्रेस (आर) ही लोकप्रियता की कसौटी पर सफल रहीं, इसलिए भारतीय राजनीति के ये बड़े और ताकतवर नेता 1971 के बाद प्रभावहीन हो गए।

प्रश्न 5.
1969 के दौरान काँग्रेस में विभाजन के लिए जिम्मेदार कारकों का विस्तार में उल्लेख कीजिए।
उत्तर:
काँग्रेस में औपचारिक विभाजन 1969 में राष्ट्रपति चुनावों के दौरान उम्मीदवार के नामांकन के मुद्दे पर हुआ।

  • राष्ट्रपति जाकिर हुसैन की मृत्यु के कारण राष्ट्रपति का पद रिक्त हो गया और इंदिरा गाँधी की असहमति के बावजूद सिंडिकेट ने तत्कालीन लोकसभा अध्यक्ष एन. संजीव रेड्डी को काँग्रेस पार्टी की तरफ से राष्ट्रपति पद के उम्मीदवार के रूप में खड़ा किया।
  • इस स्थिति का प्रतिकार करने के लिए इंदिरा गाँधी ने तत्कालीन उपराष्ट्रपति वी.वी. गिरि को बढ़ावा दिया कि वे एक स्वतंत्र उम्मीदवार के रूप में राष्ट्रपति पद के लिए नामांकन भरें।
  • चुनाव के दौरान तत्कालीन काँग्रेस अध्यक्ष एस. निजलिंगप्पा ने ‘ह्विप’ जारी किया कि काँग्रेसी सांसद और विधायक संजीव रेड्डी को वोट डालें।
  • दूसरी तरफ इंदिरा गाँधी ने छुपे तौर पर वी.वी. गिरि को समर्थन करते हुए विधायकों और सांसदों को अंतरात्मा की आवाज पर तथा अपनी मनमर्जी से वोट डालने का आह्वान किया।
  • आखिरकार राष्ट्रपति पद के चुनाव में वी.वी. गिरि विजयी हुए।
  • कांग्रेस पार्टी के आधिकारिक उम्मीदवार की हार से पार्टी का टूटना तय हो गया; और इस प्रकार काँग्रेस पार्टी का विभाजन दो गुटों में हो गया।
    1. सिंडिकेट की अगुवाई वाले काँग्रेसी खेमा को काँग्रेस (ओ) के नाम से तथा
    2. इंदिरा गाँधी की अगुवाई वाले काँग्रेसी खेमे को काँग्रेस (आर) के नाम से जाना गया।
  • इन दोनों दलों को क्रमशः ‘पुरानी काँग्रेस’ और ‘नयी काँग्रेस’ भी कहा जाता था।

प्रश्न 6.
1967 के चौथे आम चुनाव से पहले गंभीर आर्थिक संकट की जाँच करें। चुनावी फैसले का भी आकलन करें।
उत्तर:
इंदिरा गाँधी सरकार ने 1967 के आर्थिक संकट की जाँच के लिए भारतीय रुपये का अवमूल्यन किया फलस्वरूप पहले के वक्त में 1 अमरीकी डॉलर की कीमत 5 रुपये थी जो बढ़कर 7 रुपये हो गई।

  • आर्थिक संकट के कारण कीमतों में तेजी से इजाफा हुआ।
  • आवश्यक वस्तुओं की कीमतों में वृद्धि, बेरोजगारी आदि को लेकर जनता विरोध करने लगी।
  • साम्यवादी और समाजवादी पार्टी ने व्यापक समानता के लिए संघर्ष छेड़ दिया। चौथा आम चुनाव पहली बार नेहरू की गैर मौजूदगी में हुआ था।
    1. चुनाव के परिणामों से काँग्रेस को राष्ट्रीय और प्रांतीय स्तर पर धक्का लगा।
    2. इंदिरा गाँधी के मंत्रिमंडल के आधे मंत्री चुनाव हार गए थे। तमिलनाडु से कामराज, महाराष्ट्र से एस. पाटिल, पश्चिम बंगाल से अतुल्य घोष इत्यादि दिग्गजों को मुँह की खानी पड़ी थी।
    3. काँग्रेस को सात राज्यों में बहुमत नहीं मिला और दो अन्य राज्यों में दलबदल के कारण यह पार्टी सरकार नहीं बना पायी।
    4. चुनावी इतिहास में यह पहली घटना थी जब किसी गैर – काँग्रेसी दल को किसी राज्य में पूर्ण बहुमत मिला।

JAC Class 12 Political Science Important Questions Chapter 4 भारत के विदेश संबंध

Jharkhand Board JAC Class 12 Political Science Important Questions Chapter 4 भारत के विदेश संबंध Important Questions and Answers.

JAC Board Class 12 Political Science Important Questions Chapter 4 भारत के विदेश संबंध

बहुचयनात्मक प्रश्न

1. भारतीय विदेश नीति के जनक हैं।
(क) पंडित जवाहरलाल नेहरू
(ग) वल्लभ भाई पटेल
(ख) डॉ. राजेन्द्र प्रसाद
(घ) मौलाना अबुल कलाम आजाद।
उत्तर:
(क) पंडित जवाहरलाल नेहरू

2. भारतीय विदेश नीति किन कारकों से प्रभावित है।
(क) सांस्कृतिक कारक
(ग) अन्तर्राष्ट्रीय कारक
(ख) घरेलू कारक
(घ) घरेलू तथा अन्तर्राष्ट्रीय कारक।
उत्तर:
(घ) घरेलू तथा अन्तर्राष्ट्रीय कारक।

3. बाण्डुंग सम्मेलन सम्पन्न हुआ।
(क) 1954 में
(ख) 1955 में
(ग) 1956 में
(घ) 1957 में।
उत्तर:
(ख) 1955 में

4. भारतीय विदेश नीति की प्रमुख विशेषता है।
(क) पंचशील
(ख) सैनिक गुट
(ग) गुटबन्दी
(घ) उदासीनता।
उत्तर:
(क) पंचशील

JAC Class 12 Political Science Important Questions Chapter 4 भारत के विदेश संबंध

5. गुटनिरपेक्ष आन्दोलन के प्रणेता हैं।
(क) पं. नेहरू
(ख) नासिर
(ग) मार्शल टीटो
(घ) उपर्युक्त सभी।
उत्तर:
(घ) उपर्युक्त सभी।

6. पंचशील के सिद्धान्त किसके द्वारा घोषित किये गये थे?
(क) नेहरू
(ग) राजीव गाँधी
(ख) लाल बहादुर शास्त्री
(घ) अटल बिहारी वाजपेयी।
उत्तर:
(क) नेहरू

7. पहला परमाणु परीक्षण भारत में कब किया गया था?
(क) 1971
(ख) 1974
(ग) 1980
(घ) 1985।
उत्तर:
(ख) 1974

8. निम्न का सही मिलान कीजिए।

(क) घाना (i) जवाहरलाल नेहरू
(ख) मिस्र (ii) एन कुमा
(ग) भारत (iii) नासिर
(घ) इंडोनेशिया (iv) सुकर्णो
(ङ) यूगोस्लाविया (v) टीटो

उत्तर:

(क) घाना (ii) एन कुमा
(ख) मिस्र (iii) नासिर
(ग) भारत (i) जवाहरलाल नेहरू
(घ) इंडोनेशिया (iv) सुकर्णो
(ङ) यूगोस्लाविया (v) टीटो

9. भारतीय संविधान के किस अनुच्छेद में अंतर्राष्ट्रीय शांति और सुरक्षा को बढ़ावा देने के लिए नीति-निर्देशक सिद्धांत का उल्लेख किया गया हैं।
(क) अनुच्छेद 47
(ख) अनुच्छेद 50
(ग) अनुच्छेद 51
(घ) अनुच्छेद 49
उत्तर:
(ग) अनुच्छेद 51

JAC Class 12 Political Science Important Questions Chapter 4 भारत के विदेश संबंध

10. दूसरे विश्वयुद्ध के दौरान आई. एन. ए. का गठन किसने किया था?
(क) जवाहरलाल नेहरू
(ग) सरदार पटेल
(ख) सुभाषचंद्र बोस
(घ) भगत सिंह
उत्तर:
(ख) सुभाषचंद्र बोस

रिक्त स्थानों की पूर्ति कीजिए:

1. पंडित नेहरू के अनुसार हर देश की आजादी बुनियादी तौर पर ………………………. संबंधों से ही बनी होती है।
उत्तर:
विदेश

2. शीतयुद्ध के दौरान चीन में ……………………. शासन की स्थापना हुई।
उत्तर:
कम्युनिस्ट

3. शीतयुद्ध के समय अमरीका द्वारा उत्तर अटलांटिक संधि संगठन का जवाब सोवियत संघ ने …………………….. नामक संधि संगठन बनाकर दिया।
उत्तर:
वारसा पैक्ट

4. ब्रिटेन ने स्वेज नहर के मामले को लेकर मिस्र पर ………………………….. में आक्रमण किया।
उत्तर:
1956

5. नेहरू की अगुवाई में भारत ने ……………………… के मार्च में ………………………. संबंध सम्मेलन का आयोजन किया।
उत्तर:
एशियाई

अतिलघूत्तरात्मक प्रश्न

प्रश्न 1.
भारत ने ताशकंद में किस संधि पर हस्ताक्षर किए?
उत्तर:
भारत ने ताशकंद में पाकिस्तान के साथ मैत्री संधि पर हस्ताक्षर किए।

प्रश्न 2.
भारत के पहले विदेश मंत्री कौन थे?
उत्तर:
पण्डित जवाहरलाल नेहरू।

प्रश्न 3.
भारत में एशियाई सम्मेलन का आयोजन कब किया गया?
उत्तर:
सन् 1947 में।

प्रश्न 4.
1971 के भारत-पाकिस्तान युद्ध में किस नये देश का जन्म हुआ?
उत्तर:
बांग्लादेश का।

JAC Class 12 Political Science Important Questions Chapter 4 भारत के विदेश संबंध

प्रश्न 5.
शिमला समझौते पर कब व किसने हस्ताक्षर किये?
उत्तर:
शिमला समझौते पर जुलाई, 1972 में भारत की प्रधानमंत्री इन्दिरा गाँधी तथा पाकिस्तान के प्रधानमंत्री जुल्फिकार अली भुट्टो ने हस्ताक्षर किये।

प्रश्न 6.
भारत द्वारा दूसरा परमाणु परीक्षण कब किया गया?
उत्तर:
11 व 13 मई, 1998 के बीच

प्रश्न 7.
1966 में ताशकंद समझौता किन दो राष्ट्रों के मध्य हुआ?
अथवा
ताशकन्द समझौता कब और किसके मध्य हुआ?
उत्तर:
10 जनवरी, 1966 को भारत और पाकिस्तान के मध्य।

प्रश्न 8.
चीन ने तिब्बत पर कब कब्जा किया?
उत्तर:
चीन ने तिब्बत पर 1950 में कब्जा किया।

प्रश्न 9.
चीन और भारत में पंचशील समझौता कब हुआ?
उत्तर:
सन् 1954 में।

प्रश्न 10.
एक राष्ट्र के रूप में भारत का जन्म कब हुआ था?
उत्तर:
एक राष्ट्र के रूप में भारत का जन्म विश्वयुद्ध की पृष्ठभूमि में हुआ था।

प्रश्न 11.
राष्ट्र शक्ति के तीन साधन कौन-कौनसे हैं?
उत्तर:
प्राकृतिक सम्पदा, धन, धन एवं जन-शक्ति।

JAC Class 12 Political Science Important Questions Chapter 4 भारत के विदेश संबंध

प्रश्न 12.
आजाद हिन्द फौज की स्थापना किसने की?
उत्तर:
सुभाषचन्द्र बोस ने।

प्रश्न 13.
विदेश नीति से क्या अभिप्राय है?
उत्तर:
प्रत्येक देश दूसरे देशों के साथ सम्बन्धों की स्थापना में एक विशेष प्रकार की नीति का प्रयोग करता है। जिसे विदेश नीति कहा जाता है।

प्रश्न 14.
भारतीय विदेश नीति के दो उद्देश्य लिखें।
उत्तर:

  1. क्षेत्रीय अखण्डता की रक्षा करना।
  2. अन्तर्राष्ट्रीय शान्ति और सुरक्षा कायम रखना और उसे प्रोत्साहन देना।

प्रश्न 15.
एशियन सम्बन्ध सम्मेलन कब और किसके नेतृत्व में हुआ?
उत्तर:
एशियन सम्बन्ध सम्मेलन मार्च, 1947 में पं. जवाहरलाल नेहरू के नेतृत्व में भारत में हुआ।

प्रश्न 16.
भारतीय विदेश नीति के संवैधानिक आधार बताइए।
उत्तर:
अनुच्छेद 51 के अनुसार राज्य को अन्तर्राष्ट्रीय झगड़ों को निपटाने के लिए मध्यस्थ का रास्ता अपनाने सम्बन्धी निर्देश दिये गये हैं।

प्रश्न 17.
तिब्बत के किस धार्मिक नेता ने भारत में कब शरण ली?
उत्तर:
तिब्बत के धार्मिक नेता दलाई लामा ने सीमा पार कर भारत में प्रवेश किया और 1959 में भारत में शरण माँगी।

प्रश्न 18.
शीतयुद्ध के दौरान अमरीका द्वारा कौन-सा संगठन बनाया गया?
उत्तर:
उत्तर अटलांटिक संधि संगठन (NATO)।.

JAC Class 12 Political Science Important Questions Chapter 4 भारत के विदेश संबंध

प्रश्न 19.
ब्रिटेन ने मिस्र पर कब और क्यों आक्रमण किया?
उत्तर:
ब्रिटेन ने मिस्र पर 1956 में स्वेज नहर के मामले को लेकर आक्रमण किया।

प्रश्न 20.
वारसा पैक्ट नामक संगठन किसके द्वारा बनाया गया?
उत्तर:
सोवियत संघ।

प्रश्न 21. भारत और चीन के बीच सबसे बड़ा मुद्दा क्या रहा है?
उत्तर:
भारत और चीन के बीच सबसे बड़ा मुद्दा सीमा विवाद का रहा है।

प्रश्न 22.
भारतीय विदेश नीति के किन्हीं दो सिद्धान्तों का उल्लेख कीजिये।
उत्तर:
भारतीय विदेश नीति के दो सिद्धान्त ये हैं।

  1. गुटनिरपेक्षता की नीति
  2. पंचशील सिद्धान्त।

प्रश्न 23.
गुटनिरपेक्षता का अर्थ बताइये।
उत्तर:
किसी गुट में शामिल न होते हुए, राष्ट्रीय हितों को ध्यान में रखकर अपनी स्वतन्त्र विदेश नीति का संचालन करना ही गुटनिरपेक्षता है।

प्रश्न 24.
नियोजित विकास की रणनीति में किस बात पर जोर दिया गया?
उत्तर:
आयात कम करने पर और संसाधन आधार तैयार करने पर।

प्रश्न 25.
बांडुंग सम्मेलन कब और कहाँ सम्पन्न हुआ?
उत्तर:
1955 में इंडोनेशिया के बांडुंग शहर में1

प्रश्न 26.
पंचशील के दो सिद्धान्त लिखिए।
उत्तर:

  1. एक-दूसरे के आन्तरिक मामलों में हस्तक्षेप न करना।
  2. अनाक्रमण।

प्रश्न 27.
WTO का पूरा नाम बताइये।
उत्तर:
वर्ल्ड ट्रेड ऑर्गनाइजेशन ( अन्तर्राष्ट्रीय व्यापार संगठन)।

प्रश्न 28.
गुटनिरपेक्षता की नीति के दो उद्देश्य बताइए।
उत्तर:

  1. स्वतन्त्र विदेश नीति का संचालन।
  2. विकासशील राष्ट्रों के आर्थिक विकास हेतु प्रयत्न करना।

प्रश्न 29.
सी. टी. बी. टी. का पूरा नाम बताइये।
उत्तर:
कॉम्प्रीहेन्सिव टेस्ट बैन ट्रीटी।

प्रश्न 30.
पंचशील सिद्धान्त का प्रतिपादन कब किया गया?
उत्तर:
पंचशील के सिद्धान्तों का प्रतिपादन 29 अप्रैल, 1954 को भारत और चीन के प्रधानमन्त्रियों ने तिब्बत के समझौते पर हस्ताक्षर करके किया।

प्रश्न 31.
मुजीबुर्रहमान की पार्टी का नाम क्या था?
उत्तर:
आवामी लीग।

JAC Class 12 Political Science Important Questions Chapter 4 भारत के विदेश संबंध

प्रश्न 32.
रक्षा – उत्पाद विभाग और रक्षा आपूर्ति विभाग की स्थापना कब हुई?
उत्तर:
रक्षा आपूर्ति विभाग – 1962
रक्षा आपूर्ति विभाग – 1965

प्रश्न 33.
चौथी पंचवर्षीय योजना कब शुरू की गई?
उत्तर:
1969

प्रश्न 34.
भारत ने परमाणु कार्यक्रम की शुरुआत किसके निर्देशन में की?
उत्तर:
होमी जहाँगीर भाभा।

प्रश्न 35.
अरब-इजरायल युद्ध कब हुआ?
उत्तर:
1973 में।

लघूत्तरात्मक प्रश्न

प्रश्न 1.
भारतीय विदेश नीति राष्ट्रीय हितों को सुरक्षित रखने में किस प्रकार सहायक सिद्ध हुई है?
उत्तर:
भारत की गुटनिरपेक्षता, दूसरे देशों से मैत्रीपूर्ण संबंध, जातीय भेदभाव का विरोध और संयुक्त राष्ट्र का समर्थन आदि विदेश नीति के सिद्धांत भारत के राष्ट्रीय हितों को सुरक्षित रखने में सहायक सिद्ध हुए हैं। भारत शुरुआत से ही शांतिप्रिय देश रहा है इसलिए भारत ने अपनी विदेश नीति को राष्ट्रीय हितों के सिद्धांत पर आधारित किया। अंतर्राष्ट्रीय क्षेत्र में भी भारत ने सबसे मित्रतापूर्ण व्यवहार रखा। वर्तमान समय में भी भारत के संबंध विश्व की महाशक्तियों एवं अपने लगभग सभी पड़ोसी देशों के साथ अच्छे हैं।

प्रश्न 2.
विकासशील देशों की विदेश नीति का लक्ष्य सीधा-सादा होता है। स्पष्ट कीजिए।
उत्तर;
जिस प्रकार किसी व्यक्ति या परिवार के व्यवहारों को अंदरूनी और बाहरी कारक निर्देशित करते हैं उसी ” तरह एक देश की विदेश नीति पर भी घरेलू और अंतर्राष्ट्रीय वातावरण का असर पड़ता है। विकासशील देशों के पास अंतर्राष्ट्रीय व्यवस्था के भीतर अपने सरोकारों को पूरा करने के लिए जरूरी संसाधनों का अभाव होता है। इसके कारण विकासशील देश बढ़े – चढ़े देशों की अपेक्षा सीधे-सादे लक्ष्यों को लेकर अपनी विदेश नीति तय करते हैं। ऐसे देश इस बात पर जोर देते हैं कि उनके पड़ोसी देशों में शांति कायम रहे और विकास भी होता रहे।

प्रश्न 3.
शिमला समझौता पर संक्षिप्त टिप्पणी लिखिए।
अथवा
शिमला समझौता क्या है?
उत्तर:
भारत-पाक युद्ध 1971 के बाद जुलाई, 1972 में शिमला में भारत की प्रधानमंत्री इन्दिरा गाँधी और पाकिस्तान के प्रधानमंत्री भुट्टो के बीच एक समझौता हुआ जिसे शिमला समझौता कहा जाता है।

प्रश्न 4.
पंचशील के सिद्धान्तों को स्पष्ट कीजिए।
उत्तर:
पंचशील के सिद्धान्तों का प्रतिपादन 29 अप्रैल, 1954 को भारत और चीन के प्रधानमन्त्रियों ने तिब्बत के सम्बन्ध में एक समझौता किया। ये सिद्धान्त हैं।

  1. सभी देश एक-दूसरे की प्रादेशिक अखण्डता का सम्मान करें।
  2. अनाक्रमण।
  3. एक-दूसरे के आन्तरिक मामलों में हस्तक्षेप न करना।
  4. परस्पर समानता तथा लाभ के आधार पर कार्य करना।
  5. शान्तिपूर्ण सहअस्तित्व।

प्रश्न 5.
अनुच्छेद 51 में वर्णित अन्तर्राष्ट्रीय शान्ति एवं सुरक्षा बढ़ाने वाले कोई दो नीति निदेशक तत्त्वों का वर्णन कीजिए।
उत्तर:
अनुच्छेद-51 में वर्णित अन्तर्राष्ट्रीय शान्ति एवं सुरक्षा बढ़ाने वाले दो नीति निदेशक तत्त्व ये हैं।

  1. अन्तर्राष्ट्रीय शान्ति और सुरक्षा में अभिवृद्धि करना।
  2. अन्तर्राष्ट्रीय विवादों को मध्यस्थता द्वारा निपटाने का प्रयास करना।

प्रश्न 6.
पण्डित नेहरू के अनुसार गुटनिरपेक्षता का क्या अर्थ है?
उत्तर:
पण्डित नेहरू के अनुसार गुटनिरपेक्षता नकारात्मक तटस्थता, अप्रगतिशील अथवा उपदेशात्मक नीति नहीं है । इसका अर्थ सकारात्मक है अर्थात् जो उचित और न्यायसंगत है उसकी सहायता एवं समर्थन करना तथा जो अनुचित एवं अन्यायपूर्ण है उसकी आलोचना एवं निन्दा करना है।

प्रश्न 7.
भारतीय विदेश नीति में साधनों की पवित्रता से क्या अर्थ है?
उत्तर:
भारतीय विदेश नीति में साधनों की पवित्रता से तात्पर्य है। अन्तर्राष्ट्रीय विवादों का समाधान करने में शान्तिपूर्ण तरीकों का समर्थन तथा हिंसात्मक एवं अनैतिक साधनों का विरोध करना । भारतीय विदेश नीति का यह तत्त्व अन्तर्राष्ट्रीय राजनीति को परस्पर घृणा तथा सन्देह की भावना से दूर रखना चाहता है।

JAC Class 12 Political Science Important Questions Chapter 4 भारत के विदेश संबंध

प्रश्न 8.
दूसरे युद्ध के पश्चात् विकासशील देशों ने क्या ध्यान में रखकर अपनी विदेश नीति बनाई?
उत्तर:
विकासशील देश आर्थिक और सुरक्षा की दृष्टि से ज्यादा ताकतवर देशों पर निर्भर होते हैं। इसलिए दूसरे विश्वयुद्ध के तुरंत बाद के दौर में अनेक विकासशील देशों ने ताकतवर देशों की मर्जी को ध्यान में रखकर अपनी विदेश नीति अपनाई क्योंकि इन देशों से इन्हें अनुदान अथवा कर्ज मिल रहा था।

प्रश्न 9.
विदेश मंत्री के रूप में नेहरू के योगदान का वर्णन कीजिए।
उत्तर:
विदेश मंत्री के रहते हुए प्रधानमंत्री जवाहरलाल नेहरू ने राष्ट्रीय एजेंडा तय करने में निर्णायक भूमिका निभाई। प्रधानमंत्री और विदेश मंत्री के रूप में 1946 से 1964 तक उन्होंने भारत की विदेश नीति की रचना और क्रियान्वयन पर गहरा प्रभाव डाला। इनकी विदेश नीति के तीन बड़े उद्देश्य थे। कठिन संघर्ष से प्राप्त संप्रभुता को बचाए रखना, क्षेत्रीय अखंडता को बनाए रखना और तेज रफ्तार से आर्थिक विकास करना।

प्रश्न 10.
गुटनिरपेक्ष आन्दोलन की स्थापना के लिए कौन-कौन उत्तरदायी थे?
उत्तर:
गुटनिरपेक्ष आन्दोलन के जन्मदाता के रूप में भारत का सबसे महत्त्वपूर्ण योगदान रहा। सर्वप्रथम नेहरूजी ने इस नीति को भारत के लिए उपयुक्त समझा। इसके पश्चात् 1955 के बांडुंग सम्मेलन के दौरान नासिर (मिस्र) एवं टीटो (यूगोस्लाविया) के साथ मिलकर इसे विश्व आन्दोलन बनाने पर सहमति प्रकट की।

प्रश्न 11.
गुट निरपेक्षता की नीति ने कम से कम दो तरह से भारत का प्रत्यक्ष रूप से हित साधन किया- स्पष्ट कीजिये। गुट निरपेक्ष नीति से भारत को मिलने वाले दो लाभ बताइये।
उत्तर:

  1. गुट निरपेक्ष नीति अपनाकर ही भारत शीत युद्ध काल में दोनों गुटों से सैनिक व आर्थिक सहायता प्राप्त करने में सफल रहा।
  2. इस नीति के कारण ही भारत को कश्मीर समस्या पर रूस का हमेशा समर्थन मिला।

प्रश्न 12.
1965 के भारत-पाकिस्तान युद्ध के क्या कारण थे?
उत्तर:
1965 के भारत-पाकिस्तान युद्ध के प्रमुख कारण निम्न थे।

  1. 1962 में चीन से हार जाने से पाकिस्तान ने भारत को कमजोर माना।
  2. 1964 में नेहरू की मृत्यु के बाद नए नेतृत्व को पाकिस्तान ने कमजोर माना।
  3. पाकिस्तान में सत्ता प्राप्ति की राजनीति।
  4. 1963-64 में कश्मीर में मुस्लिम विरोधी गतिविधियाँ पाकिस्तान की विजय में सहायक होंगी।

JAC Class 12 Political Science Important Questions Chapter 4 भारत के विदेश संबंध

प्रश्न 13.
विदेश नीति से संबंधित किन्हीं दो नीति निर्देशक सिद्धान्तों का उल्लेख कीजिये।
अथवा
भारतीय विदेश नीति के संवैधानिक सिद्धान्तों को सूचीबद्ध कीजिये।
अथवा
भारतीय संविधान के अनुच्छेद-51 में विदेश नीति के दिये गये संवैधानिक सिद्धान्तों का उल्लेख कीजिए।
उत्तर:

  1. अन्तर्राष्ट्रीय शान्ति और सुरक्षा में अभिवृद्धि।
  2. राष्ट्रों के बीच न्यायपूर्ण एवं सम्मानपूर्ण सम्बन्धों को बनाए रखना।
  3. अन्तर्राष्ट्रीय विवादों को मध्यस्थता द्वारा निपटाने का प्रयास करना।
  4. संगठित लोगों के परस्पर व्यवहारों में अन्तर्राष्ट्रीय विधि और संधियों के प्रति आदर की भावना रखना।

प्रश्न 14.
भारत की परमाणु नीति पर संक्षिप्त टिप्पणी लिखिए।
उत्तर:
भारत की परमाणु नीति- 18 अगस्त, 1999 को जारी की गई भारत की परमाणु नीति में शस्त्र नियंत्रण के सिद्धान्त को अपनाया गया है। भारत अपनी रक्षा के लिए परमाणु हथियार रखेगा, लेकिन उसने पहले परमाणु हमला नहीं करने की प्रतिबद्धता दर्शायी है।

प्रश्न 15.
भारतीय विदेश नीति में आया महत्त्वपूर्ण परिवर्तन बताओ।
उत्तर:
भारत की विदेश नीति में महत्त्वपूर्ण परिवर्तन पश्चिमी ब्लॉक की तरफ मैत्रीपूर्ण व्यवहार करना, परमाणु ब्लॉक में शामिल होना। 1990 के बाद से रूस का अन्तर्राष्ट्रीय महत्त्व कम हुआ इसी कारण भारत की विदेश नीति में अमरीका समर्थक रणनीतियाँ अपनाई गई हैं। इसके अतिरिक्त मौजूदा अन्तर्राष्ट्रीय परिवेश में सैन्य हितों के बजाय आर्थिक हितों पर जोर ज्यादा है। इसका असर भी भारत की विदेश नीति में अपनाए गए विकल्पों पर पड़ा है।

प्रश्न 16.
भारत-रूस ( सोवियत संघ ) 1971 की सन्धि के महत्त्व को स्पष्ट कीजिए।
उत्तर:
1971 में भारत: सोवियत संघ के बीच 20 वर्षीय मैत्री की सन्धि की गई थी। इस सन्धि के अधीन सोवियत संघ ने भारत की गुटनिरपेक्षता की नीति को स्वीकार किया तथा दोनों देशों ने किसी के विरुद्ध हुए बाह्य आक्रमण के समय परस्पर विचार-विमर्श करने की व्यवस्था की।

प्रश्न 17.
1950 के दशक में भारत-अमरीकी सम्बन्धों में खटास पैदा करने वाले दो कारणों का उल्लेख कीजिए।
उत्तर:

  1. पाकिस्तान अमरीकी नेतृत्व वाले सैन्य गठबन्धन में शामिल हो गया। इससे अमरीका तथा भारत के सम्बन्धों में खटास पैदा हो गई।
  2. अमरीका, सोवियत संघ से भारत की बढ़ती हुई दोस्ती को लेकर भी नाराज था।

प्रश्न 18.
प्रथम एफ्रो-एशियाई एकता सम्मेलन कहाँ हुआ? इसकी दो विशेषताएँ बताइये।
उत्तर:
प्रथम एफ्रो एशियाई एकता सम्मेलन इंडोनेशिया के एक बड़े शहर बांडुंग में 1955 में हुआ इसकी विशेषताएँ हैं।

  1. इस सम्मेलन में गुट निरपेक्ष आन्दोलन की नींव पड़ी।
  2. इस सम्मेलन में भाग लेने वाले देशों ने इण्डोनेशिया में नस्लवाद विशेषकर दक्षिण अफ्रीका में रंग-भेद का विरोध किया।

JAC Class 12 Political Science Important Questions Chapter 4 भारत के विदेश संबंध

प्रश्न 19.
1971 के भारत-पाकिस्तान युद्ध के कोई तीन राजनीतिक परिणाम बताइये।
उत्तर:

  1. भारतीय सेना के समक्ष पाकिस्तानी सेना ने 90,000 सैनिकों के साथ आत्मसमर्पण कर दिया।
  2. बांग्लादेश के रूप में एक स्वतन्त्र राज्य का उदय हुआ।
  3. 3 जुलाई, 1972 को इन्दिरा गाँधी और जुल्फिकार अली भुट्टो के बीच शिमला समझौते पर हस्ताक्षर हुए और अमन की बहाली हुई।

प्रश्न 20.
विदेश नीति के चार अनिवार्य कारक बताइए।
उत्तर:
विदेश नीति के चार प्रमुख अनिवार्य कारक ये हैं।

  1. राष्ट्रीय हित,
  2. राज्य की राजनीतिक स्थिति,
  3. पड़ोसी देशों के साथ सम्बन्ध,
  4. अन्तर्राष्ट्रीय राजनीतिक वातावरण।

प्रश्न 21.
भारतीय विदेश नीति के लक्ष्यों का वर्णन कीजिए।
उत्तर:

  1. राष्ट्रीय हित: भारतीय विदेश नीति का लक्ष्य राष्ट्रीय हितों की पूर्ति करना है जिसमें सामाजिक, राजनीतिक, आर्थिक व राष्ट्रीय सुरक्षा के क्षेत्र में उत्तरोत्तर विकास करना है।
  2. विश्व समस्याओं के प्रति दृष्टिकोण-इनमें प्रमुख रूप से विश्व शान्ति, राज्यों का सहअस्तित्व, राज्यों का आर्थिक विकास मानवाधिकार आदि शामिल हैं।

प्रश्न 22.
नेहरूजी की विदेश नीति की कोई दो विशेषताएँ लिखिये
अथवा
भारतीय विदेश नीति की प्रमुख विशेषताएँ बताइए।
उत्तर:

  1. गुटनिरपेक्षता की नीति का अनुसरण करना।
  2. राष्ट्रीय हितों की रक्षा करना।
  3. जाति, रंग, भेदभाव, उपनिवेशवाद, साम्राज्यवाद का विरोध करना।

प्रश्न 23.
एशियाई देशों के मामले में नेहरू के योगदान का वर्णन कीजिए।
उत्तर:
भारत के आकार, अवस्थिति और शक्ति: संभावना को भाँपकर नेहरू ने विश्व के मामलों, मुख्यतया एशियाई मामलों में भारत के लिए बड़ी भूमिका का स्वप्न देखा था। नेहरू के दौर में भारत ने एशियाई और अफ्रीका के नव-स्वतंत्र देशों के साथ संपर्क बनाए 1940 और 1950 के दशकों में नेहरू बड़े मुखर स्वर में एशियाई एकता की पैरोकारी करते रहे। नेहरू की अगुवाई में भारत ने 1947 के मार्च में एशियाई संबंध सम्मेलन का आयोजन किया। भारत ने इंडोनेशिया को डच औपनिवेशिक शासन से मुक्त कराने के लिए स्वतंत्रता संग्राम के समर्थन में अंतर्राष्ट्रीय सम्मेलन किया।

प्रश्न 24.
ताशकंद समझौते के कोई दो प्रावधान लिखें।
उत्तर:

  1. दोनों पक्षों ( भारत – पाकिस्तान) का यह प्रयास रहेगा कि संयुक्त राष्ट्र के घोषणा-पत्र के अनुसार दोनों में मधुर सम्बन्ध बनें।
  2. दोनों पक्ष इस बात पर सहमत थे कि दोनों देशों की सेनाएँ फरवरी, 1966 से पहले उस स्थान पर पहुँच जाएँ जहाँ 5 अगस्त, 1965 से पहले थीं।

JAC Class 12 Political Science Important Questions Chapter 4 भारत के विदेश संबंध

प्रश्न 25.
शिमला समझौते पर संक्षिप्त टिप्पणी लिखिए।
उत्तर:
1972 में भारत और पाकिस्तान के मध्य शिमला समझौता हुआ। इसकी प्रमुख शर्तें हैं।

  1. दोनों देश आपसी मतभेदों का शान्तिपूर्ण ढंग से समाधान करेंगे।
  2. दोनों देश एक-दूसरे की सीमा पर आक्रमण नहीं करेंगे।

प्रश्न 26.
भारत और चीन के मध्य तनाव के कोई दो कारण बताइए।
उत्तर:

  1. भारत और चीन में महत्त्वपूर्ण विवाद सीमा का विवाद है। चीन ने भारत की भूमि पर कब्जा कर रखा है।
  2. चीन का तिब्बत पर कब्जा और भारत का दलाईलामा को राजनीतिक शरण देना।

प्रश्न 27.
भारत व चीन के मध्य अच्छे सम्बन्ध बनाने हेतु दो सुझाव दीजिये।
उत्तर:

  1. दोनों पक्ष सीमा विवादों के निपटारे के लिए वार्ताएँ जारी रखें तथा सीमा क्षेत्र में शांति बनाए रखें।
  2. दोनों देशों को द्विपक्षीय व्यापार में वृद्धि करने का प्रयत्न करना चाहिए।

प्रश्न 28.
भारत और पाकिस्तान के बीच सम्बन्धों में तनाव के कोई दो कारण बताइए।
उत्तर:

  1. भारत-पाक सम्बन्धों में तनाव का महत्त्वपूर्ण कारण कश्मीर का मामला है।
  2. भारत-पाक के मध्य तनाव का अन्य कारण भारत में पाक समर्थित आतंकवाद है। पाकिस्तान पिछले कुछ वर्षों से कश्मीर के आतंकवादियों की सभी तरह से सहायता कर रहा है।

प्रश्न 29.
भारत द्वारा परमाणु नीति अपनाने के मुख्य कारण बताइ ।
उत्तर:

  1. भारत परमाणु नीति एवं परमाणु हथियार बनाकर दूसरे देशों के आक्रमण से बचने के लिए न्यूनतम – अवरोध की स्थिति प्राप्त करना चाहता है।
  2. भारत के दो पड़ोसी देशों चीन एवं पाकिस्तान के पास परमाणु हथियार हैं और इन दोनों देशों से भारत युद्ध भी लड़ चुका है।

प्रश्न 30.
भारत की परमाणु नीति की किन्हीं दो विशेषताओं का वर्णन कीजिए।
उत्तर:

  1. भारत परमाणु क्षेत्र में न्यूनतम प्रतिरोध की क्षमता प्राप्त करना चाहता है।
  2. भारत परमाणु हथियारों का प्रयोग पहले नहीं करेगा।

प्रश्न 31.
भारतीय विदेश नीति के चार निर्धारक तत्त्वों का विवेचन कीजिए।
उत्तर:
भारतीय विदेश नीति के निर्धारक तत्त्व : भारतीय विदेश नीति के प्रमुख निर्धारक तत्त्व निम्नलिखित हैं।

  1. भारत की विदेश नीति की आधारशिला उसके राष्ट्रीय हितों की सुरक्षा है।
  2. शान्तिपूर्ण सहअस्तित्व की भावना का विकास करना।
  3. पड़ोसी देशों के साथ मित्रतापूर्ण सम्बन्ध बनाये रखना।
  4. अन्तर्राष्ट्रीय विवादों को सुलझाने हेतु शान्तिपूर्ण साधनों के प्रयोग पर बल देना।

JAC Class 12 Political Science Important Questions Chapter 4 भारत के विदेश संबंध

प्रश्न 32.
आप नेहरूजी को विदेश नीति तय करने का एक अनिवार्य संकेतक क्यों मानते हैं? दो कारण बताइये।
उत्तर:

  1. नेहरूजी प्रधानमंत्री के साथ-साथ विदेश मंत्री भी थे। प्रधानमंत्री और विदेश मंत्री के रूप में उन्होंने भारत की विदेश नीति की रचना और क्रियान्वयन पर गहरा प्रभाव डाला।
  2. नेहरूजी ने देश की संप्रभुता को बचाए रखने, क्षेत्रीय अखंडता को बनाए रखने तथा तीव्र आर्थिक विकास की दृष्टि से गुटनिरपेक्षता की नीति अपनायी।

प्रश्न 33.
पण्डित नेहरू के काल में भारत की विदेश नीति की उपलब्धियों पर प्रकाश डालिए।
उत्तर:
पण्डित नेहरू के काल में भारत की विदेश नीति की उपलब्धियाँ: प्रधानमन्त्री नेहरू की विदेश नीति अत्यधिक आदर्शवादी और भावना प्रधान थी। इस नीति के चलते भारत ने दोनों गुटों से प्रशंसा तथा सहायता पाने में सफलता प्राप्त की। यह विश्व शान्ति बनाये रखने में अत्यधिक सफल रही। नेहरू की गुटनिरपेक्षता की नीति के कारण ही भारत-चीन युद्ध के समय उन्होंने रूस तथा अमेरिका दोनों देशों से सहायता प्राप्त की इस नीति के कारण भारत शान्तिदूत, गुटों का पुल, तटस्थ विश्व का नेता आदि माना जाने लगा।

प्रश्न 34.
पंचशील के पाँच सिद्धान्त क्या हैं?
अथवा
पंचशील के सिद्धान्त पर संक्षिप्त टिप्पणी लिखिए।
उत्तर:
पंचशील के सिद्धान्त: पंचशील के पाँच सिद्धान्त निम्नलिखित हैं।

  1. सभी राष्ट्र एक-दूसरे की प्रादेशिक अखण्डता और सम्प्रभुता का सम्मान करें।
  2. कोई राज्य दूसरे राज्य पर आक्रमण न करे और राष्ट्रीय सीमाओं का अतिक्रमण न करे।
  3. कोई राज्य किसी दूसरे राज्य के आन्तरिक मामलों में हस्तक्षेप न करे।
  4. प्रत्येक राज्य एक-दूसरे के साथ समानता का व्यवहार करे तथा पारस्परिक हित में सहयोग प्रदान करे।
  5. सभी राष्ट्र शान्तिपूर्ण सह-अस्तित्व के सिद्धान्त में विश्वास करें

प्रश्न 35.
भारतीय विदेश नीति के ऐसे कोई दो उदाहरण दीजिये जिनमें भारत ने स्वतंत्र दृष्टिकोण अपनाया है।
उत्तर:
निम्नलिखित दो अन्तर्राष्ट्रीय घटनाओं में भारत ने स्वतंत्र दृष्टिकोण अपनाया है।

  1. शीतयुद्ध के दौरान भारत न तो संयुक्त राज्य अमेरिका और न ही सोवियत संघ के खेमे में सम्मिलित हुआ तथा उसने गुटनिरपेक्ष आंदोलन को शुरू करने, समय-समय पर होने वाले सम्मेलनों में स्वेच्छा एवं पूर्ण निष्पक्षता से भाग लिया।
  2. भारत ने शांति व विकास के लिए परमाणु ऊर्जा व शक्ति के प्रयोग का समर्थन किया तथा निर्भय होकर सन् में परमाणु परीक्षण किया तथा उसे उचित बताया।

प्रश्न 36.
अन्तर्राष्ट्रीय घटनाओं के ऐसे कोई दो उदाहरण दीजिये जिनमें भारत ने स्वतंत्र दृष्टिकोण अपनाया है।
उत्तर:

  1. 1956 में जब ब्रिटेन ने स्वेज नहर के मामले को लेकर मिस्र पर आक्रमण किया तो भारत ने इस औपनिवेशिक हमले के विरुद्ध विश्वव्यापी विरोध की अगुवाई की।
  2. 1956 में ही जब सोवियत संघ ने हंगरी पर आक्रमण किया तो भारत ने सोवियत संघ के इस कदम की सार्वजनिक निंदा की।

JAC Class 12 Political Science Important Questions Chapter 4 भारत के विदेश संबंध

प्रश्न 37
भारतीय विदेश नीति में परिवर्तन के स्वरूप को स्पष्ट कीजिए।
उत्तर:
भारतीय विदेश नीति का परिवर्तित स्वरूप: भारतीय विदेश नीति के परिवर्तित स्वरूप की व्याख्या निम्न बिन्दुओं के अन्तर्गत की जा सकती है।

  1. सामरिक और तकनीकी ताकत हासिल करने, परमाणु परीक्षण करने और परमाणु अप्रसार सन्धि तथा सी. टी. बी. टी. पर हस्ताक्षर न करने की नीति को अपनाया।
  2. बांग्लादेश की स्वतन्त्रता के लिए श्रीलंका की सरकार के चाहने पर तथा मालदीव की सुरक्षा के लिए भारतीय सेनाओं को इन देशों में भेजा।
  3. व्यावहारिक कूटनीति को अपनाना, जैसे अफगानिस्तान पर रूसी कार्यवाही पर चुप रहना, नेपाल को चेतावनी देना, इजरायल से दौत्य सम्बन्ध स्थापित करना आदि।
  4. आकार, शक्ति, तकनीकी और सैनिक श्रेष्ठता आदि के आधार पर दक्षिण एशिया को भारतीय प्रभाव क्षेत्र बनाना।

प्रश्न 38.
बांडुंग सम्मेलन के बारे में आप क्या जानते हैं?
उत्तर:
इंडोनेशिया के एक शहर बांडुंग में एफ्रो-एशियाई सम्मेलन 1955 में हुआ । इसी सम्मेलन को बांडुंग सम्मेलन के नाम से जाना जाता है। अफ्रीका और एशिया के नव-स्वतंत्र देशों के साथ भारत के बढ़ते संपर्क का यह चरम बिंदु था। बांडुंग सम्मेलन में ही गुटनिरपेक्ष आंदोलन की नींव पड़ी।

प्रश्न 39.
पण्डित नेहरू की विदेश नीति की कोई चार विशेषताएँ बताइये।
उत्तर:
पण्डित नेहरू की विदेश नीति की विशेषताएँ पण्डित नेहरू की विदेश नीति की प्रमुख विशेषताएँ निम्नलिखित हैं।

  1. गुटनिरपेक्षता: नेहरू की विदेश नीति की सबसे प्रमुख विशेषता गुटनिरपेक्षता है। गुटनिरपेक्षता का अर्थ – किसी गुट में शामिल न होना और स्वतन्त्र नीति का अनुसरण करना।
  2. विश्व शान्ति और सुरक्षा की नीति: नेहरू की विदेश नीति का आधारभूत सिद्धान्त विश्व शान्ति और सुरक्षा बनाए रखना है।
  3. साम्राज्यवाद एवं उपनिवेशवाद का विरोध: नेहरू ने सदैव साम्राज्यवाद तथा उपनिवेशवाद का विरोध किया है।
  4. अन्य देशों के साथ मित्रतापूर्ण व्यवहार: नेहरू की विदेश नीति की एक अन्य विशेषता विश्व के सभी देशों से मित्रतापूर्ण सम्बन्ध बनाने का प्रयास करना रही।

प्रश्न 40.
भारत की पड़ोसी देशों के प्रति क्या नीति है? संक्षेप में बताइए।
उत्तर:
भारत की पड़ोसी देशों के प्रति नीति – भारत सदैव ही पड़ोसी देशों से मित्रवत् सम्बन्ध चाहता है। भारत का मानना है कि बिना मित्रतापूर्ण सम्बन्ध के कोई भी देश सामाजिक, राजनीतिक एवं आर्थिक विकास नहीं कर सकता। इसलिए भारत ने पाकिस्तान, चीन, बांग्लादेश, श्रीलंका, नेपाल, भूटान, मालदीव, म्यांमार इत्यादि पड़ोसी देशों से मधुर सम्बन्ध बनाए रखने के लिए समय-समय पर कई कदम उठाए हैं। उन्हीं महत्त्वपूर्ण कदमों में से एक सार्क की स्थापना है। इससे न केवल भारत के अन्य पड़ोसी देशों के साथ सम्बन्ध मधुर होंगे, बल्कि दक्षिण एशिया और अधिक विकास कर सकेगा। भारत की नीति यह है कि पड़ोसी देशों के साथ जो भी मतभेद हैं, उन्हें युद्ध से नहीं बल्कि बातचीत द्वारा हल किया जाना चाहिए।

प्रश्न 41.
1962 के भारत-चीन युद्ध के कारण बताइए।
उत्तर:
भारत-चीन युद्ध के कारण: 1962 के भारत-चीन युद्ध के प्रमुख कारण निम्नलिखित हैं।

  1. तिब्बत की समस्या: 1962 के भारत-चीन युद्ध की सबसे बड़ी समस्या तिब्बत की समस्या थी। चीन ने सदैव तिब्बत पर अपना दावा किया, जबकि भारत इस समस्या को तिब्बत वासियों की भावनाओं को ध्यान में रखकर सुलझाना चाहता था।
  2. मानचित्र से सम्बन्धित समस्या- भारत और चीन के बीच 1962 में हुए युद्ध का एक कारण दोनों देशों के बीच मानचित्र में रेखांकित भू-भाग था। चीन ने 1954 में प्रकाशित अपने मानचित्र में कुछ ऐसे भाग प्रदर्शित किये जो वास्तव में भारतीय भू-भाग में थे, अत: भारत ने इस पर चीन के साथ अपना विरोध दर्ज कराया।
  3. सीमा विवाद – भारत-चीन के बीच युद्ध का एक कारण सीमा विवाद भी था।

JAC Class 12 Political Science Important Questions Chapter 4 भारत के विदेश संबंध

प्रश्न 42.
ताशकन्द समझौता कब हुआ? इसके प्रमुख प्रावधान लिखिए।
उत्तर:
ताशकन्द समझौता: सितम्बर, 1965 में हुए भारत-पाक युद्ध के बाद 10 जनवरी, 1966 को ताशकंद में भारत के प्रधानमंत्री लालबहादुर शास्त्री तथा पाकिस्तान के राष्ट्रपति अयूब खां के बीच ताशकंद समझौता सम्पन्न हुआ। इस समझौते के प्रमुख प्रावधान निम्नलिखित थे।
संजीव पास बुक्स

  1. भारत एवं पाकिस्तान अच्छे पड़ोसियों की भाँति सम्बन्ध स्थापित करेंगे और विवादों को शान्तिपूर्ण ढंग से सुलझायेंगे।
  2. दोनों देश के सैनिक युद्ध से पूर्व की स्थिति में चले जायेंगे। दोनों युद्ध-विराम की शर्तों का पालन करेंगे।
  3. दोनों एक-दूसरे के आन्तरिक मामलों में हस्तक्षेप नहीं करेंगे।
  4. दोनों राजनयिक सम्बन्धों को पुनः सामान्य रूप से स्थापित करेंगे।
  5. दोनों आर्थिक एवं व्यापारिक सम्बन्धों को पुनः सामान्य रूप से स्थापित करेंगे।
  6. दोनों देश सन्धि की शर्तों का पालन करने के लिए सर्वोच्च स्तर पर आपस में मिलते रहेंगे।

प्रश्न 43.
बांग्लादेश युद्ध, 1971 पर संक्षिप्त टिप्पणी लिखिये।
उत्तर:
बांग्लादेश युद्ध, 1971 1970 में पाकिस्तान के हुए आम चुनाव के बाद पाकिस्तान की सेना ने 1971 में शेख मुजीब को गिरफ्तार कर लिया। इसके विरोध में पूर्वी पाकिस्तान की जनता ने अपने इलाके को पाकिस्तान से मुक्त कराने का संघर्ष छेड़ दिया। पाकिस्तानी शासन ने पूर्वी पाकिस्तान के लोगों पर जुल्म ढाना शुरू कर दिया । फलतः लगभग 80 लाख शरणार्थी पाकिस्तान से भाग कर भारत में शरण लिये हुए थे। महीनों राजनायिक तनाव और सैन्य तैनाती के बाद 1971 के दिसम्बर में भारत और पाकिस्तान के बीच युद्ध छिड़ गया। दस दिनों के अन्दर भारतीय सेना ने ढाका को तीन तरफ से घेर लिया और अपने 90000 सैनिकों के साथ पाकिस्तानी सेना को आत्मसमर्पण करना पड़ा। बांग्लादेश के रूप में एक स्वतंत्र राष्ट्र का उदय हुआ। भारतीय सेना ने एकतरफा युद्ध विराम कर दिया।

प्रश्न 44.
शिमला समझौता कब हुआ? इसके प्रमुख प्रावधान लिखिए।
उत्तर:
शिमला समझौता:
28 जून, 1972 को श्रीमती इन्दिरा गाँधी एवं जुल्फिकार अली भुट्टो के द्वारा शिमला में दोनों देशों के मध्य जो समझौता हुआ उसे शिमला समझौते के नाम से जाना जाता है। इस समझौते के निम्नलिखित प्रमुख प्रावधान थे।

  • दोनों देश सभी विवादों एवं समस्याओं के शान्तिपूर्ण समाधान के लिए सीधी वार्ता करेंगे।
  • दोनों एक-दूसरे के विरुद्ध दुष्प्रचार नहीं करेंगे।
  • दोनों देशों के सम्बन्धों को सामान्य बनाने के लिए
    1. संचार सम्बन्ध फिर से स्थापित करेंगे,
    2. आवागमन की सुविधाओं का विस्तार करेंगे।
    3. व्यापार एवं आर्थिक सहयोग स्थापित करेंगे,
    4. विज्ञान एवं सांस्कृतिक क्षेत्र में आदान-प्रदान करेंगे।
  • (4) स्थायी शान्ति स्थापित करने हेतु हर सम्भव प्रयास किये जाएँगे।
  • (5) भविष्य में दोनों सरकारों के अध्यक्ष मिलते रहेंगे।

प्रश्न 45.
वी. के. कृष्णमेनन पर संक्षिप्त टिप्पणी लिखिए।
उत्तर:
वी.के. कृष्णमेनन (1897-1974) भारतीय राजनयिक एवं मंत्री थे। 1939 से 1947 के समयकाल में ये इंग्लैंड की लेबर पार्टी में सक्रिय थे। आप इंग्लैंड में भारतीय उच्चायुक्त एवं बाद में संयुक्त राष्ट्र में भारतीय प्रतिनिधि मंडल के मुखिया थे। आप राज्यसभा के सांसद एवं बाद में लोकसभा सांसद बने। 1956 से संघ केबिनेट के सदस्य 1957 से रक्षा मंत्री का पद संभाला। आपने 1962 में भारत-चीन के युद्ध के बाद इस्तीफा दे दिया।

JAC Class 12 Political Science Important Questions Chapter 4 भारत के विदेश संबंध

प्रश्न 46.
चीन के साथ युद्ध का भारत पर क्या परिणाम हुआ?
उत्तर:
चीन के साथ युद्ध से भारत की छवि को देश और विदेश दोनों ही जगह धक्का लगा। इस संकट से उबरने के लिए भारत को अमरीका और ब्रिटेन से सैन्य मदद लेनी पड़ी। चीन युद्ध से भारतीय राष्ट्रीय स्वाभिमान को ठेस लगी परंतु राष्ट्र – भावना मजबूत हुई। नेहरू की छवि भी धूमिल हुई। चीन के इरादों को समय रहते न भाँप सकने और सैन्य तैयारी न कर पाने को लेकर नेहरू की पड़ी आलोचना हुई। पहली बार, उनकी सरकार के खिलाफ अविश्वास प्रस्ताव लाया गया। इसके तुरंत बाद, कांग्रेस ने कुछ महत्त्वपूर्ण चुनावों में हार का सामना किया। देश का राजनीतिक मानस बदलने लगा था।

प्रश्न 47.
भारत और पाकिस्तान के मध्य तनाव के कोई चार कारण बताइये।
उत्तर:
भारत और पाकिस्तान के मध्य तनाव के कारण- भारत और पाकिस्तान के मध्य तनाव के प्रमुख कारण निम्नलिखित हैं।

  1. कश्मीर समस्या: भारत एवं पाकिस्तान के मध्य तनाव का प्रमुख मुद्दा कश्मीर है।
  2. सियाचिन ग्लेशियर का मामला: पिछले कुछ समय से पाकिस्तान सैनिक कार्यवाही द्वारा सियाचिन पर कब्जा करने का प्रयास कर रहा है जिसे भारत के सैनिकों ने विफल कर दिया।
  3. आतंकवाद की समस्या: पाकिस्तान भारत में जेहाद के नाम पर आतंकवादी गतिविधियाँ फैला रहा है।
  4. आणविक हथियारों की होड़: भारत और पाकिस्तान के मध्य आणविक हथियारों की होड़ भी दोनों देशों के मध्य तनाव का मुख्य कारण माना जाता है।

प्रश्न 48.
करगिल संघर्ष के कारणों का उल्लेख कीजिये।
उत्तर:
करगिल संघर्ष के कारण: करगिल संघर्ष के प्रमुख कारण निम्नलिखित थे।

  1. पाकिस्तान को अमेरिका तथा चीन द्वारा अपनी अति गोपनीय कूटनीति विस्तार हेतु आर्थिक सहायता प्रदान
  2. पाकिस्तानी सेना प्रमुख द्वारा इस मामले को पाकिस्तानी प्रधानमंत्री को अंधेरे में रखना।
  3. पाकिस्तानी सेना द्वारा छद्म रूप से भारतीय नियंत्रण रेखा के कई ठिकानों, जैसे द्रास, माश्कोह, काकस तथा तालिक पर कब्जा कर लेना।

प्रश्न 49.
करगिल की लड़ाई पर संक्षिप्त टिप्पणी लिखिए।
अथवा
करगिल संकट पर एक संक्षिप्त नोट लिखिए।
उत्तर:
करगिल की लड़ाई: सन् 1999 के शुरूआती महीनों में भारतीय नियन्त्रण रेखा के कई ठिकानों जैसे द्रास, माश्कोह, काकसर और बतालिक पर अपने को मुजाहिदीन बताने वालों ने कब्जा कर लिया था। पाकिस्तानी सेना की इसमें मिली भगत भांप कर भारतीय सेना हरकत में आयी। इससे दोनों देशों के बीच संघर्ष छिड़ गया।

इसे करगिल की लड़ाई के नाम से जाना जाता है। 1999 के मई-जून में यह लड़ाई जारी रही। 26 जुलाई, 1999 तक भारत अपने अधिकतर क्षेत्रों पर पुनः अधिकार कर चुका था। करगिल की इस लड़ाई ने पूरे विश्व का ध्यान खींचा था क्योंकि इससे ठीक एक वर्ष पहले दोनों देश परमाणु हथियार बनाने की अपनी क्षमता का प्रदर्शन कर चुके थे।

प्रश्न 50.
तिब्बत का पठार भारत और चीन के तनाव का बड़ा मामला कैसे बना?
उत्तर:

  1. सन् 1950 में चीन ने तिब्बत पर नियंत्रण कर लिया। सन् 1958 में चीनी आधिपत्य के विरुद्ध तिब्बत में सशस्त्र विद्रोह हुआ जिसे चीनी सेनाओं ने दबा दिया। स्थिति को बिगड़ता देखकर तिब्बत के पारम्परिक नेता दलाई लामा ने सीमा पार कर भारत में प्रवेश किया तथा उसने 1959 में भारत से शरण मांगी। भारत ने दलाई लामा को शरण दे दी। चीन ने भारत के इस कदम का कड़ा विरोध किया।
  2. 1950 और 1960 के दशक में भारत के अनेक राजनीतिक दल तथा राजनेताओं ने तिब्बत की आजादी के प्रति अपना समर्थन जताया, जबकि चीन इसे अपना अभिन्न अंग मानता है। इन कारणों से तिब्बत भारत और चीन के बीच तनाव का बड़ा मामला बना।

JAC Class 12 Political Science Important Questions Chapter 4 भारत के विदेश संबंध

प्रश्न 51.
भारत-चीन युद्ध पर संक्षिप्त टिप्पणी लिखिए।
उत्तर:
भारत-चीन युद्ध-20 अक्टूबर, 1962 को चीन ने नेफा और लद्दाख की सीमाओं में घुस कर भारत पर सुनियोजित ढंग से बड़े पैमाने पर आक्रमण किया। नेफा क्षेत्र में भारत को पीछे हटना पड़ा और असम के मैदान में खतरा उत्पन्न हो गया। 24 नवम्बर, 1962 को चीन ने अपनी तरफ से युद्ध विराम की घोषणा कर दी और चीनी सेनाएँ 7 नवम्बर, 1959 की वास्तविक नियन्त्रण रेखा से 20 कि.मी. पीछे हट गईं। यद्यपि भारत की पर्याप्त भूमि पर उन्होंने अपना अधिकार कर लिया था।

चीन ने वार्ता का प्रस्ताव भी किया परन्तु भारत ने इस शर्त के साथ इसे अस्वीकार कर दिया कि जब तक चीनी सेनाएँ 8 सितम्बर, 1962 की स्थिति तक वापस नहीं लौट जायेंगी तब तक वार्ता नहीं हो सकती। इस युद्ध में भारत की पराजय से एशिया तथा विश्व में भारत की प्रतिष्ठा घटी और इस युद्ध के पीछे चीन का यह मूल उद्देश्य था, जिसमें सफल रहा।

प्रश्न 52.
भारत तथा बांग्लादेश के बीच सहयोग और असहमति के किसी एक-एक क्षेत्र का उल्लेख कीजिये।
उत्तर:

  1. भारत तथा बांग्लादेश के बीच सहयोग: भारत तथा बांग्लादेश के बीच 1972 में 25 वर्षीय मैत्री, सहयोग और शांति संधि हुई थी। इसके साथ ही दोनों देशों के मध्य व्यापार समझौता भी हुआ। फलस्वरूप दोनों देशों में 1. सहयोग और मित्रता बढ़ती गयी।
  2. भारत तथा बांग्लादेश के बीच असहयोग: बांग्लादेश से शरणार्थी और घुसपैठिये लगातार भारत आते रहते हैं। भारत में लाखों बांग्लादेशी किसी न किसी तरह से अनाधिकृत रूप से रह रहे हैं। इस मुद्दे पर दोनों में सहमति नहीं हो पा रही है।

प्रश्न 53.
भारत द्वारा परमाणु नीति एवं कार्यक्रम अपनाने के कोई चार कारण बताइये।
उत्तर:
भारत द्वारा परमाणु नीति एवं कार्यक्रम निर्धारण के कारण: भारत द्वारा परमाणु नीति एवं कार्यक्रम निर्धारण के प्रमुख कारण निम्नलिखित है।

  1. आत्मनिर्भर राष्ट्र बनना: भारत परमाणु नीति एवं परमाणु हथियार बनाकर एक आत्म-निर्भर राष्ट्र बनना चाहता है । विश्व के जिन देशों के पास भी हथियार हैं वे सभी आत्म-निर्भर राष्ट्र हैं।
  2. न्यूनतम अवरोध की स्थिति प्राप्त करना: भारत परमाणु नीति एवं परमाणु हथियार बनाकर दूसरे देशों के आक्रमण से बचने के लिए न्यूनतम अवरोध की स्थिति प्राप्त करना चाहता है।
  3. दो पड़ोसी देशों के पास परमाणु हथियार होना: भारत के लिए परमाणु नीति एवं हथियार बनाना इसलिए आवश्यक है क्योंकि भारत के दोनों पड़ोसी देशों चीन एवं पाकिस्तान के पास परमाणु हथियार हैं।
  4. परमाणु सम्पन्न राष्ट्रों की विभेदपूर्ण नीति: परमाणु सम्पन्न राष्ट्रों ने 1968 में परमाणु अप्रसार सन्धि (NPT) तथा 1996 में व्यापक परमाणु परीक्षण निषेध सन्धि (C. T. B. T.) को विभेदपूर्ण ढंग से लागू किया जिसके कारण भारत ने इस पर हस्ताक्षर नहीं किये।

प्रश्न 54.
भारत की नवीन परमाणु नीति का मसौदा क्या है?
उत्तर:
भारत की नवीन परमाणु नीति- 18 अगस्त, 1999 को भारत सरकार ने अपनी नवीन परमाणु नीति का एक मसौदा प्रकाशित किया है। इसकी प्रमुख विशेषताएँ निम्नलिखित हैं।

  1. इसमें शस्त्र नियन्त्रण के सिद्धान्त को अपनाया गया है।
  2. किसी भी देश पर भारत पहला हमला नहीं करेगा, परन्तु उस पर हमला किया गया तो उसका मुँहतोड़ जवाब देगा।
  3. प्रधानमन्त्री या प्रधानमन्त्री द्वारा नामांकित व्यक्ति परमाणु विस्फोट के लिए उत्तरदायी होगा।
  4. सी. टी. बी. टी. के प्रश्न को इस मसौदे से अलग रखा गया है।
  5. भारत विश्व को परमाणु शक्तिहीन बनाने की प्रतिबद्धता पर कायम रहेगा।
  6. परमाणु अथवा मिसाइल प्रौद्योगिकी के निर्यात पर कड़ा नियन्त्रण रखा जायेगा।

प्रश्न 55.
चीन के साथ भारत के सम्बन्धों को बेहतर बनाने के लिए आप क्या सुझाव देंगे?
उत्तर:
चीन के साथ भारत के सम्बन्धों को बेहतर बनाने के लिए हम निम्न सुझाव देंगे।

  1. दोनों पक्ष सीमा विवाद को निपटारे के लिए वार्ताएँ जारी रखें तथा सीमा क्षेत्र में शांति बनाए रखें।
  2. हमें चीन के साथ द्विपक्षीय व्यापार में वृद्धि करने का प्रयत्न करना चाहिए।
  3. अन्तर्राष्ट्रीय क्षेत्र में पर्यावरण प्रदूषण के प्रश्नों में दोनों देशों को मिलकर संयुक्त राष्ट्र संघ की संस्थाओं में अपना पक्ष रखना चाहिए क्योंकि दोनों देशों के हित समान हैं।
  4. हमें चीन के साथ सांस्कृतिक आदान-प्रदान को भी बढ़ावा देना चाहिए।

JAC Class 12 Political Science Important Questions Chapter 4 भारत के विदेश संबंध

प्रश्न 56.
1962 के भारत-चीन युद्ध के बाद के भारत-चीन संबंध पर संक्षिप्त टिप्पणी लिखिए।
उत्तर:
1962 के भारत-चीन युद्ध के बाद इन दोनों देशों के बीच संबंध सामान्य होने में दस साल लग गए। 1976 में दोनों देशों के बीच पूर्ण राजनयिक संबंध बहाल हो सके। शीर्ष नेता के तौर पर अटल बिहारी वाजपेयी (तब के विदेश मंत्री ) 1979 में चीन के दौरे पर गए। इसके बाद से चीन के साथ भारत संबंधों में ज्यादा जोर व्यापारिक मसलों पर रहा है।

प्रश्न 57.
कश्मीर मुद्दे पर संघर्ष के बावजूद भारत और पाकिस्तान की सरकारों के बीच सहयोग-संबंध कायम रहे। उदाहरण के साथ स्पष्ट कीजिए।
उत्तर:
कश्मीर मुद्दे पर संघर्ष के बावजूद भारत और पाकिस्तान की सरकारों के बीच सहयोग-संबंध कायम रहे इस कथन का सत्यापन निम्न उदाहरणों द्वारा किया जा सकता है।

  1. दोनों सरकारों ने मिल-जुल कर प्रयास किया कि बँटवारे के समय अपहृत महिलाओं को उनके परिवार के पास लौटाया जा सके।
  2. विश्व बैंक की मध्यस्थता से नदी जल में हिस्सेदारी का लंबा विवाद सुलझा लिया गया।
  3. नेहरू और जनरल अयूब खान ने सिंधु नदी जल संधि पर 1960 में हस्ताक्षर किए और भारत-पाक संबंधों में तनाव के बावजूद इस संधि पर ठीक-ठाक अमल होता रहा।

प्रश्न 58.
भारत ने सोवियत संघ के साथ 1971 में शांति और मित्रता की 20 वर्षीय संधि पर दस्तखत क्यों किये?
उत्तर:
पूर्वी पाकिस्तान की जनता ने अपने इलाके को पाकिस्तान से मुक्त कराने के लिए संघर्ष छेड़ दिया। भारत ने बांग्लादेश के ‘मुक्ति संग्राम’ को नैतिक समर्थन और भौतिक सहायता दी। ऐसे समय पर पाकिस्तान को अमरीका और चीन ने सहायता की। 1960 के दशक में अमरीका और चीन के बीच संबंधों को सामान्य करने की कोशिश चल रही थी, इससे एशिया में सत्ता- समीकरण नया रूप ले रहा था। अमरीकी राष्ट्रपति रिचर्ड निक्सन के सलाहकार हेनरी किसिंजर ने 1971 के जुलाई में पाकिस्तान होते हुए गुपचुप चीन का दौरा किया। अमरीका- पाकिस्तान-चीन की धुरी बनते देख भारत ने इसके जवाब में सोवियत संघ के साथ 1971 में शांति और मित्रता की एक 20 वर्षीय संधि पर दस्तखत किए। इस संधि से भारत को यह आश्वासन मिला कि हमला होने की सूरत में सोवियत संघ भारत की मदद करेगा ।

प्रश्न 59.
चीन के साथ हुए युद्ध ने भारत के नेताओं पर आंतरिक क्षेत्रीय नीतियों के हिसाब से क्या उल्लेखनीय प्रभाव डाला? संक्षेप में टिप्पणी लिखिए।
उत्तर:
चीन के साथ हुए युद्ध ने भारत के नेताओं को पूर्वोत्तर की डावांडोल स्थिति के प्रति सचेत किया। यह इलाका अत्यंत पिछड़ी दशा में था और अलग-थलग पड़ गया था। चीन युद्ध के तुरंत बाद इस इलाके को नयी तरतीब में ढालने की कोशिशें शुरू की गई। नागालैंड को प्रांत का दर्जा दिया गया। मणिपुर और त्रिपुरा हालांकि केन्द्र शासित प्रदेश थे लेकिन उन्हें अपनी विधानसभा के निर्वाचन का अधिकार मिला।

JAC Class 12 Political Science Important Questions Chapter 4 भारत के विदेश संबंध

प्रश्न 60.
1962 और 1965 के युद्धों का भारतीय रक्षा व्यय पर क्या प्रभाव पड़ा?
उत्तर:
आजादी के बाद भारत ने अपने सीमित संसाधनों के साथ नियोजित विकास की रणनीति के साथ शुरुआत की। पड़ोसी देशों के साथ संघर्ष के कारण पंचवर्षीय योजना पटरी से उतर गई। 1962 के बाद भारत को अपने सीमित संसाधनों को रक्षा क्षेत्र में लगाना पड़ा। भारत को अपने सैन्य ढाँचे का आधुनिकीकरण करना पड़ा। 1962 में रक्षा उत्पाद और 1965 में रक्षा आपूर्ति विभाग की स्थापना हुई। तीसरी पंचवर्षीय योजना पर असर पड़ा और इसके बाद लगातार तीन एक-वर्षीय योजना पर अमल हुआ। चौथी पंचवर्षीय योजना 1969 में ही शुरू हो सकी। युद्ध के बाद भारत का रक्षा-व्यय बहुत ज्यादा बढ़ गया।

निबन्धात्मक प्रश्न

प्रश्न 1.
भारत की विदेश नीति के मुख्य सिद्धान्तों का वर्णन कीजिए।
अथवा
भारतीय विदेश नीति की प्रमुख विशेषताओं को स्पष्ट कीजिये।
उत्तर:
भारतीय विदेश नीति की विशेषताएँ: भारतीय विदेश नीति की प्रमुख विशेषताएँ निम्नलिखित हैं।

  • अन्तर्राष्ट्रीय शान्ति एवं सुरक्षा में आस्था: भारत ने अन्तर्राष्ट्रीय शांति एवं सुरक्षा के लिए सदैव अपना सहयोग प्रदान किया है, चाहे वह कोरिया समस्या हो या इराक की समस्या।
  • असंलग्नता अथवा गुट निरपेक्षता की नीति: असंलग्नता का अभिप्राय है। किसी गुट (पंक्ति) से संलग्न नहीं होना यह गुटों से पृथक् रहते हुए एक स्वतंत्र विदेश नीति है। यह तटस्थ न होकर एक सक्रिय विदेश नीति है।
  • पंचशील के सिद्धान्त तथा शांतिपूर्ण सहअस्तित्व की नीति: पंचशील का अर्थ है। पाँच सिद्धान्त। ये पाँच सिद्धान्त अग्रलिखित हैं।
    1. परस्पर एक-दूसरे की भौगोलिक अखण्डता तथा संप्रभुता का सम्मान।
    2. एक-दूसरे पर आक्रमण नहीं करना।
    3. एक-दूसरे के आन्तरिक मामलों में हस्तक्षेप नहीं करना।
    4. परस्पर समानता तथा लाभ के आधार पर कार्य करना।
    5. शांतिपूर्ण सहअस्तित्व।
  • साम्राज्यवाद तथा उपनिवेशवाद का विरोध: भारत ने उपनिवेशवाद तथा साम्राज्यवाद के विरोध की नीति अपनाई। इसी नीति को दृष्टिगत रखते हुए भारत ने एशिया तथा अफ्रीकी देशों के स्वतन्त्रता आन्दोलनों को सक्रिय समर्थन प्रदान किया।
  • सभी राष्ट्रों के साथ मैत्रीपूर्ण सम्बन्धों की स्थापना: भारत अपना गुट बनाने के स्थान पर सभी देशों के साथ मैत्रीपूर्ण सम्बन्धों की स्थापना में विश्वास करता है।
  • निःशस्त्रीकरण का समर्थन: भारत ने सदैव ही निःशस्त्रीकरण का समर्थन किया है।
  • अन्तर्राष्ट्रीय कानूनों एवं संयुक्त राष्ट्र के प्रति आस्था: भारत ने सदैव अन्तर्राष्ट्रीय कानूनों का पालन किया है तथा संयुक्त राष्ट्र के प्रति आस्था व्यक्त की है।

प्रश्न 2.
वर्तमान में भारतीय गुटनिरपेक्षता की नीति का महत्त्व बताइए।
उत्तर:
भारतीय विदेश नीति में गुटनिरपेक्षता की नीति का महत्त्व: भारतीय विदेश नीति में गुटनिरपेक्षता के महत्त्व का विवेचन निम्नलिखित बिन्दुओं के अन्तर्गत किया जा सकता

  1. भारत गुटनिरपेक्षता की नीति अपनाकर ही शीत युद्ध काल में दोनों गुटों से सैनिक और आर्थिक सहायता बिना शर्त प्राप्त करने में सफल रहा।
  2. गुट निरपेक्ष नीति के कारण भारत को कश्मीर समस्या पर रूस का हमेशा समर्थन मिला जबकि पश्चिमी शक्तियाँ पाकिस्तान का समर्थन कर रही थीं।
  3. भारत की गुटनिरपेक्ष नीति ने उसे आत्म-निर्भरता का पाठ पढ़ाया है।
  4. भारत की गुटनिरपेक्ष नीति शीतयुद्ध काल में अमरीका और रूस दोनों के शासनाध्यक्षों की प्रशंसा की पात्र रही है।
  5. भारत की गुटनिरपेक्षता की नीति विश्व शांति में सहायक रही है। भारत ने शीत युद्ध की चरमावस्था में दोनों गुटों में सेतुबन्ध का कार्य किया और संकट की स्थिति को टालने का प्रयत्न किया।

इस प्रकार भारत की गुटनिरपेक्ष नीति अनेक कसौटियों पर कसी गई है। विश्व चाहे द्विध्रुवीय रहा हो या एकध्रुवीय, भारत चाहे अपने आर्थिक विकास के लिए पश्चिम या पूर्व से सहायता ले या सीमाओं की रक्षा के लिए पश्चिम से सैनिक अस्त्र-शस्त्र ले, द्विपक्षीय समझौतों के अन्तर्गत शान्ति सेनाएँ भेजे या मालदीव जैसी स्थितियों में तुरत-फुरत सक्रियता दिखाये, भारत के लिए असंलग्नता की नीति ही सर्वोत्तम है। इसी से उसके राष्ट्रीय हितों की सर्वोत्तम सुरक्षा हो सकती है तथा शान्ति स्थापित की जा सकती है।

प्रश्न 3.
गुटनिरपेक्ष आन्दोलन की समकालीन प्रासंगिकता बताइये।
उत्तर:
गुटनिरपेक्ष आन्दोलन की प्रासंगिकता: गुटनिरपेक्ष आन्दोलन की समकालीन प्रासंगिकता के पक्ष में निम्नलिखित तर्क दिये जा सकते हैं।

  1. नये राष्ट्रों की स्वतन्त्रता तथा विकास की दृष्टि से प्रासंगिक: नए स्वतन्त्र देशों की स्वतन्त्रता की रक्षा तथा आर्थिक और सामाजिक विकास हेतु उन्हें युद्धों से दूर रहने की दृष्टि से गुटनिरपेक्ष आन्दोलन आज भी प्रासंगिक बना हुआ है।
  2. विकासशील राष्ट्रों के बीच परस्पर आर्थिक एवं सांस्कृतिक सहयोग: वर्तमान समय में विकासशील देशों को शोषण से बचाने, उनमें पारस्परिक आर्थिक तथा तकनीकी सहयोग को बढ़ाने, अपनी समाचार एजेन्सियों का निर्माण करने के लिए गुटनिरपेक्ष आन्दोलन का औचित्य बना हुआ है।
  3. विश्व जनमत में सहायक: गुटनिरपेक्ष आन्दोलन की वर्तमान काल में विश्व जनमत के निर्माण के लिए अत्यधिक आवश्यकता है।
  4. बहुगुटीय विश्व: सोवियत संघ के विघटन के बाद आज विश्व बहुगुटीय बन रहा है। वर्तमान बहुध्रुवीय विश्व में गुटनिरपेक्ष आन्दोलन की महती प्रासंगिकता बनी हुई है।
  5. एशिया तथा अफ्रीका में विस्फोटक स्थिति: वर्तमान काल में एशिया तथा अफ्रीका के विभिन्न क्षेत्र तनाव केन्द्र हैं। ऐसी स्थिति में निर्गुट आन्दोलन इनकी समस्याओं का समाधान ढूँढ़ने में सहायक सिद्ध हो सकता है।
  6. महत्वपूर्ण उद्देश्यों की प्राप्ति शेष:
    • नई अन्तर्राष्ट्रीय अर्थव्यवस्था की स्थापना
    • संयुक्त राष्ट्र संघ का लोकतन्त्रीकरण
    • न्यायसंगत विश्व की स्थापना तथा
    • नि:शस्त्रीकरण आदि उद्देश्यों की प्राप्ति में इसकी प्रासंगिकता बनी हुई है।
  7. प्रदूषण एवं अन्तर्राष्ट्रीय आतंकवाद – प्रदूषण एवं अन्तर्राष्ट्रीय आतंकवाद के विरुद्ध एक संगठित दबाव पैदा करने की दृष्टि से गुटनिरपेक्ष आन्दोलन उपयोगी है।

JAC Class 12 Political Science Important Questions Chapter 4 भारत के विदेश संबंध

प्रश्न 4.
भारत और पाकिस्तान के मध्य तनाव – विवाद के प्रमुख कारण बताइए।
अथवा
भारत-पाक सम्बन्धों की समीक्षा कीजिये।
उत्तर:
भारत और पाकिस्तान के मध्य तनाव के कारण: अपने पड़ौसी देशों के प्रति मधुर सम्बन्ध रखने को उच्च प्राथमिकता देने की भारत की नीति के बावजूद अनेक कारणों से भारत-पाक सम्बन्ध में तनाव व कटुता बनी रही है। भारत और पाकिस्तान के मध्य तनाव के मुख्य कारण निम्नलिखित हैं।

  1. विभाजन से उत्पन्न अविश्वास: भारत विभाजन ने भारत और पाकिस्तान के मध्य शत्रुता उत्पन्न कर दी। यह ब्रिटिश शासकों की रणनीति थी।
  2. देशी रियासतों की समस्या तथा कश्मीर विवाद: कश्मीर के विलय का विवाद अभी भी दोनों देशों के मध्य व्याप्त है और यह निरन्तर दोनों देशों के बीच तनाव का मुख्य बिन्दु बना हुआ है।
  3. कच्छ के रन ( सरक्रीक) का प्रश्न 1947 में कच्छ की रियासत के भारत में विलय के साथ ही कच्छ का रन भी भारत का अंग बन गया था। लेकिन जुलाई, 1948 में पाकिस्तान ने यह प्रश्न उठाया कि कच्छ का रन चूँकि एक मृत समुद्री भाग है, अतः उसका मध्य भाग दोनों देशों की सीमा होना चाहिए। भारत ने उसके इस दावे को स्वीकार नहीं किया। दोनों देशों के मध्य इस विवाद को सुलझाने की दिशा में अनेक वार्ताएँ हुई हैं लेकिन अभी तक इनका निपटारा नहीं हो सका हैं।
  4. साम्प्रदायिक विभाजन की राजनीति: पाकिस्तानी राजनेता जान-बूझकर दोनों देशों के मध्य साम्प्रदायिक वैमनस्य बनाये रखना चाहते हैं। वे साम्प्रदायिक वैमनस्य को अन्तर्राष्ट्रीय सम्मेलनों और संगठनों में अभिव्यक्त करते रहे हैं।
  5. भारत के विरुद्ध पाक प्रायोजित आतंकवाद – भारत के विरुद्ध पाक-प्रायोजित आतंकवाद के कारण भी दोनों देशों के मध्य कटुता बनी हुई है।

प्रश्न 5.
भारत-पाक सम्बन्धों को सुधारने हेतु सुझाव दीजिए।
उत्तर:
भारत-पाक सम्बन्धों को सुधारने हेतु सुझाव: भारत-पाक सम्बन्धों में आयी कटुता को दूर करने तथा उनके बीच सम्बन्धों को सुधारने हेतु निम्न सुझाव दिये जा सकते हैं।

  1. आपसी बातचीत एवं समझौते की नीति- भारत और पाकिस्तान के मध्य विवादों को आपसी बातचीत और समझौते द्वारा दूर किया जा सकता है।
  2. दोनों देशों के मध्य विश्वास बहाली के उपाय किये जाने चाहिए – भारत और पाकिस्तान दोनों देशों के लोगों में आपसी विश्वास को मजबूत करने के लिए बसों, ट्रेनों की आवाजाही तथा व्यक्तिगत सम्पर्क आदि को बढ़ावा दिया जाना चाहिए।
  3. सांस्कृतिक आदान-प्रदान पर बल: दोनों देशों में धार्मिक भिन्नताओं के बावजूद सांस्कृतिक समानताएँ हैं। अतः दोनों देशों में समय-समय पर एक-दूसरे के धार्मिक उत्सवों एवं समारोहों का आयोजन किया जाना चाहिए जिससे दोनों देशों की जनता के बीच परस्पर भाईचारे एवं सद्भावना का विकास हो।
  4. आपसी मतभेदों के अन्तर्राष्ट्रीयकरण पर रोक- दोनों देशों को मतभेदों का समाधान करने के लिए अन्तर्राष्ट्रीय मंचों के स्थान पर द्विपक्षीय वार्ता एवं परस्पर सहयोग एवं समझौते की नीति का अनुसरण करना चाहिए।
  5. आतंकवादी गतिविधियों पर रोक लगायी जानी चाहिए: पाकिस्तान को चाहिए कि वह आतंकवादी गतिविधियों पर अंकुश लगाए ताकि वार्ता द्वारा समझौते का वातावरण बन सके।

प्रश्न 6.
भारत एवं चीन सम्बन्धों का वर्तमान संदर्भ में परीक्षण कीजिए।
उत्तर:
भारत-चीन सम्बन्ध: वर्तमान संदर्भ में भारत तथा चीन के सम्बन्धों का विवेचन निम्नलिखित बिन्दुओं के अन्तर्गत किया गया है।

  • सीमा-विवाद यथावत्: आज भी दोनों देशों के मध्य सीमा के प्रश्न पर व्यापक मतभेद हैं, तथापि दोनों पक्ष इसे सुलझाने के लिए वार्ता जारी रखे हुए हैं। वर्तमान में दोनों देश इस नीति का दृढ़ता से पालन कर रहे हैं कि जब तक सीमा विवाद का अन्तिम रूप से समाधान नहीं हो जाता है, दोनों पक्ष सीमा क्षेत्र में शांति बनाए रखेंगे।
  • आर्थिक तथा व्यापारिक क्षेत्रों में सहयोग की ओर बढ़ते कदम: 1993 में दोनों देशों के मध्य हुए एक व्यापारिक समझौते के बाद दोनों देशों के बीच सीमा – व्यापार पुनः प्रारम्भ हो गया है तथा यह निरन्तर बढ़ता जा रहा है। इसके अतिरिक्त दोनों देशों के बीच 50 से अधिक संयुक्त उद्यमों की स्थापना हुई है।
  • विश्व व्यापार संगठन की बैठकों में परस्पर सहयोग; दोनों पक्षों में यह भी सहमति हुई कि वे विश्व व्यापार संगठन की बैठकों में दोनों के हितों से जुड़े मुद्दों पर परस्पर सहयोग करेंगे।
  • वर्तमान समय में भारत-चीन के मध्य विवाद के प्रमुख मुद्दे: वर्तमान समय में भारत और चीन के मध्य विवाद के प्रमुख मुद्दे निम्न हैं।
    1. अनसुलझा सीमा विवाद
    2. घुसपैठ से घिरा अण्डमान-निकोबार
    3. चीन-पाकिस्तान में बढ़ती निकटता
    4. चीन का नया सामरिक गठजोड़
    5. तिब्बत पर कसता चीनी शिकंजा
    6. माओवाद से घिरता नेपाल
    7. चीन का साइबर आक्रमण
    8. भारतीय सीमा में घुसपैठ
    9. कश्मीरियों के लिए अलग से चीनी वीजा।

JAC Class 12 Political Science Important Questions Chapter 4 भारत के विदेश संबंध

प्रश्न 7.
भारत द्वारा परमाणु नीति अपनाने के मुख्य कारण बताइए।
उत्तर:
भारत द्वारा परमाणु नीति अपनाने के कारण: भारत ने सर्वप्रथम 1974 में एक तथा 1998 में पांच परमाणु परीक्षण करके विश्व को दिखला दिया कि भारत भी एक परमाणु सम्पन्न राष्ट्र है। भारत द्वारा परमाणु नीति एवं परमाणु हथियार को बनाने एवं रखने के पक्ष में निम्नलिखित तर्क दिये जा सकते हैं।

  1. आत्मनिर्भर राष्ट्र बनना: भारत परमाणु नीति एवं परमाणु हथियार बनाकर एक आत्मनिर्भर राष्ट्र बनना चाहता है। विश्व में जिन देशों के पास भी परमाणु हथियार हैं वे सभी आत्मनिर्भर राष्ट्र माने जाते हैं।
  2. प्रतिष्ठा प्राप्त करना: भारत परमाणु नीति एवं परमाणु हथियार बनाकर शक्तिशाली राष्ट्र बन विश्व में प्रतिष्ठा प्राप्त करना चाहता है।
  3. न्यूनतम अवरोध की स्थिति प्राप्त करना: भारत परमाणु नीति एवं परमाणु हथियार बनाकर दूसरे देशों के आक्रमण से बचने के लिए न्यूनतम अवरोध की स्थिति प्राप्त करना चाहता है।
  4. परमाणु सम्पन्न राष्ट्रों की भेदपूर्ण नीति- परमाणु सम्पन्न राष्ट्रों ने NPT-1996 तथा CTBT 1996 की भेदभावपूर्ण संधियों द्वारा अन्य राष्ट्रों को परमाणु सम्पन्न न बनने देने की नीति अपना रखी है। भारत ने इन पर हस्ताक्षर नहीं किये तथा परमाणु कार्यक्रम जारी रखा।
  5. भारत द्वारा लड़े गए युद्ध – भारत ने समय- समय पर 1962, 1965, 1971 एवं 1999 में युद्धों का सामना किया । युद्धों में होने वाली अधिक हानि से बचने के लिए भारत परमाणु हथियार प्राप्त करना चाहता है।
  6. दो पड़ोसी राष्ट्रों के पास परमाणु हथियार होना – भारत के लिए परमाणु नीति एवं परमाणु हथियार बनाने इसलिए भी आवश्यक हैं, क्योंकि भारत के दोनों पड़ोसी देशों चीन एवं पाकिस्तान के पास परमाणु हथियार हैं।

प्रश्न 8.
भारत की विदेश नीति में नेहरू की भूमिका को स्पष्ट कीजिये।
उत्तर:
भारत की विदेश नीति में नेहरू की भूमिका भारत के पहले प्रधानमंत्री जवाहर लाल नेहरू ने राष्ट्रीय एजेंडा तय करने में निर्णायक भूमिका निभायी। वे प्रधानमंत्री के साथ-साथ विदेश मंत्री भी थे। प्रधानमंत्री और विदेशमंत्री के रूप में 1946 से 1964 तक उन्होंने भारत की विदेश नीति की रचना और क्रियान्वयन पर गहरा प्रभाव डाला।
1. गुटनिरपेक्षता की नीति:
नेहरू की विदेश नीति के तीन बड़े उद्देश्य थे। कठिन संघर्ष से प्राप्त संप्रभुता को बचाए रखना, क्षेत्रीय अखण्डता को बनाए रखना तथा तेज रफ्तार से आर्थिक विकास करना। नेहरू इन उद्देश्यों को गुटनिरपेक्षता की नीति अपनाकर हासिल करना चाहते थे। इन दिनों में भारत में कुछ राजनैतिक दल, नेता तथा समूह ऐसे भी थे जिनका मानना था कि भारत को अमरीकी खेमे के साथ ज्यादा नजदीकी बढ़ानी चाहिए क्योंकि इस खेमे की प्रतिष्ठा लोकतंत्र के हिमायती के रूप में थी। लेकिन विदेश नीति को तैयार करने में नेहरू को खासी बढ़त हासिल थी।

2. सैनिक गठबन्धनों से दूर रहने की नीति:
स्वतंत्र भारत की विदेश नीति में शांतिपूर्ण विश्व का सपना था और इसके लिए भारत ने गुटनिरपेक्षता की नीति का पालन किया। भारत ने इसके लिए शीत युद्ध से उपजे तनाव को कम करने की कोशिश की और संयुक्त राष्ट्र संघ के शांति-अभियानों में अपनी सेना भेजी। भारत ने अमरीका और सोवियत संघ की अगुवाई वाले सैन्य गठबंधनों से अपने को दूर रखना चाहता था। दोनों खेमों के बीच भारत ने अन्तर्राष्ट्रीय मामलों पर स्वतंत्र रवैया अपनाया। उसे दोनों खेमों के देशों ने सहायता और अनुदान दिये।

3. एफ्रो-एशियायी एकता की नीति- भारत के आकार, अवस्थिति और शक्ति संभावना को भांपकर नेहरू ने विश्व के मामलों, विशेषकर एशियायी मामलों में भारत के लिए बड़ी भूमिका निभाने की नीति अपनायी। उन्होंने एशिया और अफ्रीका के नव-स्वतंत्र देशों से सम्पर्क बनाए तथा एशियायी एकता की पैरोकारी की। 1955 का बांडुंग में एफ्रो- एशियायी देशों का सम्मेलन हुआ जो अफ्रीका व एशिया के नव-स्वतंत्र देशों के साथ भारत के बढ़ते सम्पर्क का चरम बिन्दु था।

4. चीन के साथ शांति और संघर्ष:
नेहरू के नेतृत्व में भारत ने चीन के साथ अपने रिश्तों की शुरुआत दोस्ताना ढंग से की भारत ने सबसे पहले चीन की कम्युनिस्ट सरकार को मान्यता दी। शांतिपूर्ण सहअस्तित्व के पांच सिद्धान्तों यानी पंचशील की घोषणा नेहरू और चाऊ एन लाई ने संयुक्त रूप से 29 अप्रेल, 1954 में की। लेकिन चीन ने 1962 में भारत पर अचानक आक्रमण कर इस दोस्ताना रिश्ते को शत्रुता में बदल दिया तथा भारत के काफी बड़े भू-भाग पर कब्जा कर लिया। तब से लेकर अब तक दोनों देशों के बीच सीमा विवाद जारी है। यद्यपि चीन के सम्बन्ध में सरदार वल्लभ भाई पटेल ने आशंका व्यक्त की थी, लेकिन नेहरू ने इस आशंका को नजरअंदाज कर दिया था। एक विदेश नीति के मामले में नेहरू की एक भूल साबित हुई।

प्रश्न 9.
चीन के साथ भारत के युद्ध का भारत तथा भारत की राजनीति पर क्या प्रभाव हुआ? विस्तारपूर्वक समझाइए
उत्तर:

  1. चीन-युद्ध से भारत की छवि को देश और विदेश दोनों ही जगह धक्का लगा। इस संकट से निपटने के लिए भारत को अमरीका और ब्रिटेन से सैन्य सहायता माँगनी पड़ी।
  2. चीन- युद्ध से भारतीय राष्ट्रीय स्वाभिमान को चोट पहुँची लेकिन इसके साथ-साथ राष्ट्र भावना भी बलवती हुई। इस युद्ध के कारण नेहरू की छवि भी धूमिल हुई। चीन के इरादों को समय रहते न भाँप सकने और सैन्य तैयारी न कर पाने को लेकर नेहरू की आलोचना हुई ।
  3. नेहरू की सरकार के खिलाफ अविश्वास प्रस्ताव लाया गया। कुछ महत्त्वपूर्ण उप-चुनावों में कांग्रेस ने हार का सामना किया। देश का राजनीतिक मानस बदलने लगा था।
  4. भारत-चीन संघर्ष का असर विपक्षी दलों पर भी हुआ। इस युद्ध और चीन – सोवियत संघ के बीच बढ़ते मतभेद से भारतीय कम्युनिस्ट पार्टी के अंदर बड़ा बदलाव हुआ। भारतीय कम्युनिस्ट पार्टी 1964 में टूट गई। इस पार्टी के भीतर जो खेमा चीन का पक्षधर था उसने मार्क्सवादी भारतीय कम्युनिस्ट पार्टी बनाई। चीन युद्ध के क्रम में माकपा के कई नेताओं को चीन का पक्ष लेने के आरोप में गिरफ्तार किया गया।
  5. चीन के साथ हुए युद्ध ने भारत के नेताओं को पूर्वोत्तर की डाँवाडोल स्थिति के प्रति सचेत किया क्योंकि राष्ट्रीय एकता के लिहाज से यह इलाका चुनौतीपूर्ण था।
  6. चीन – र – युद्ध के बाद पूर्वोत्तर भारत के इलाकों को नयी तरतीब में ढालने की कोशिशें शुरू की गईं। नागालैंड को प्रांत का दर्जा दिया गया। मणिपुर और त्रिपुरा हालाँकि केन्द्र – शासित प्रदेश थे लेकिन उन्हें अपनी विधानसभा के निर्वाचन का अधिकार मिला। संजीव पास बुक्स।

JAC Class 12 Political Science Important Questions Chapter 4 भारत के विदेश संबंध

प्रश्न 10.
ऐतिहासिक रूप से तिब्बत भारत और चीन के बीच विवाद का एक बड़ा मसला रहा है। विस्तारपूर्वक स्पष्ट कीजिए।
उत्तर:
तिब्बत मध्य एशिया का मशहूर पठार है। अतीत में समय-समय पर चीन ने तिब्बत पर अपना प्रशासनिक अधिकार जताया और कई दफा तिब्बत आजाद भी हुआ। 1950 में चीन ने तिब्बत पर नियंत्रण कर लिया। तिब्बत के ज्यादातर लोगों ने चीनी कब्जे का विरोध किया। 1954 में भारत और चीन के बीच पंचशील समझौते पर हस्ताक्षर हुए तो इसके प्रावधानों में एक बात यह भी थी कि दोनों देश एक-दूसरे की क्षेत्रीय संप्रभुता का सम्मान करेंगे। चीन ने इसका यह अर्थ लगाया कि भारत तिब्बत पर चीन की दावेदारी को स्वीकार कर रहा है।

1965 में चीनी शासनाध्यक्ष जब भारत आए. उसे समय तिब्बत के धार्मिक नेता दलाई लामा भी भारत पहुँचे और उन्होंने तिब्बत की बिगड़ती स्थिति की जानकारी नेहरू को दी। चीन ने यह आश्वासन दिया कि तिब्बत को चीन के अन्य इलाकों से ज्यादा स्वायत्तता दी जाएगी। 1958 में चीनी आधिपत्य के विरुद्ध तिब्बत में सशस्त्र विद्रोह हुआ। इस विद्रोह को चीनी सेनाओं द्वारा दबा दिया गया। स्थिति को बिगड़ता देख तिब्बत के पारंपरिक नेता दलाई लामा ने सीमा पार कर भारत में प्रवेश किया और 1959 में भारत से शरण माँगी। भारत ने दलाई लामा को शरण दे दी। चीन ने भारत के इस कदम का कड़ा विरोध किया।

पिछले 50 सालों में बड़ी संख्या में तिब्बती जनता ने भारत और दुनिया के अन्य देशों में शरण ली है। भारत में तिब्बती शरणार्थियों की बड़ी-बड़ी बस्तियाँ हैं। हिमाचल प्रदेश के धर्मशाला में संभवतया तिब्बती शरणार्थियों की सबसे बड़ी बस्ती है। दलाई लामा ने भी भारत में धर्मशाला को अपना निवास स्थान बनाया है। 1950 और 1960 के दशक में भारत के अनेक राजनीतिक दल और राजनेताओं ने तिब्बत की आजादी के प्रति अपना समर्थन जताया। इन दलों में सोशलिस्ट पार्टी और जनसंघ शामिल हैं।

JAC Class 12 History Important Questions Chapter 12 औपनिवेशिक शहर : नगर-योजना, स्थापत्य

Jharkhand Board JAC Class 12 History Important Questions Chapter 12 औपनिवेशिक शहर : नगर-योजना, स्थापत्य Important Questions and Answers.

JAC Board Class 12 History Important Questions Chapter 12 औपनिवेशिक शहर : नगर-योजना, स्थापत्य

बहुविकल्पीय प्रश्न (Multiple Choice Questions)

1. 1857 के विद्रोह से पूर्व दिल्ली के कोतवाल थे-
(क) गंगाधर नेहरू
(ख) अरुण नेहरू
(ग) मोतीलाल नेहरू
(घ) जवाहरलाल नेहरू
उत्तर:
(क) गंगाधर नेहरू

2. अखिल भारतीय जनगणना का प्रयास भारत में प्रथम बार हुआ-
(क) 1772 में
(ख) 1872 में
(ग) 1786 में
(घ) 1881 में
उत्तर:
(ख) 1872 में

3. शाहजहाँनाबाद को बसाया था-
(क) अकबर ने
(ख) औरंगजेब ने
(ग) शाहजहाँ ने
(घ) हुमायूँ ने
उत्तर:
(ग) शाहजहाँ ने

4. ब्रिटिश काल में पहला हिल स्टेशन बना-
(क) शिमला
(ख) दार्जिलिंग
(ग) नैनीताल
(घ) मनाली
उत्तर:
(क) शिमला

5. आगरा, दिल्ली तथा लाहौर की एक सामान्य विशेषता थी-
(क) तीनों शहर मिली-जुली संस्कृति के लिए जाने जाते थे।
(ख) तीनों शहर सूफी सन्तों तथा भक्तों को खूब प्रिय थे।
(ग) तीनों शहर सोलहवीं और सत्रहवीं सदी में मुगल शासन के महत्त्वपूर्ण केन्द्र थे।
(घ) तीनों शहर मुस्लिम आबादी वाले थे।
उत्तर:
(ग) तीनों शहर सोलहवीं और सत्रहवीं सदी में मुगल शासन के महत्त्वपूर्ण केन्द्र थे।

JAC Class 12 History Important Questions Chapter 12 औपनिवेशिक शहर : नगर-योजना, स्थापत्य

6. भारत में रेलवे की शुरुआत हुई
(क) 1856 में
(ग) 1855 में
(ख) 1853 में
(घ) 1851 में
उत्तर:
(ख) 1853 में

7. कलकत्ता के स्थान पर दिल्ली को राजधानी बनाया गया-
(क) 1911 में
(ख) 1916 में
(ग) 1908 में
(घ) 1899 में
उत्तर:
(क) 1911 में

8. प्लासी का युद्ध लड़ा गया था-
(क) 1755 में
(ख) 1757 में
(ग) 1765 में
(घ) 1857 में
उत्तर:
(ख) 1757 में

9. दक्षिण भारत के दो शहरों मदुराई और कांचीपुरम की मुख्य विशेषता थी-
(क) ये दोनों नगर हमलावरों से सुरक्षित थे।
(ख) इनमें मुख्य केन्द्र मन्दिर तथा महत्त्वपूर्ण व्यापारिक केन्द्र होते थे।
(ग) इनमें विश्व के प्रमुख धर्मों के त्यौहार मनाए जाते थे।
(घ) समाज के लोग इन शहरों में साहित्यिक चर्चा किया करते थे।
उत्तर:
(ख) इनमें मुख्य केन्द्र मन्दिर तथा महत्त्वपूर्ण व्यापारिक केन्द्र होते थे।

10. औपनिवेशिक शहरों में प्राय: निम्न तीन शहर शामिल किए जाते थे-
(क) दिल्ली, बम्बई, कलकत्ता
(ख) दिल्ली, मद्रास, कलकत्ता
(ग) मद्रास, कलकत्ता, बम्बई
(घ) इनमें से कोई नहीं
उत्तर:
(ग) मद्रास, कलकत्ता, बम्बई

11. मद्रास, कलकत्ता एवं बम्बई तीनों शहरों की एक सामान्य विशेषता थी-
(क) तीनों व्यापारिक राजधानी थे
(ख) तीनों मूलतः मत्स्य ग्रहण एवं बुनाई के गाँव थे
(ग) तीनों शहरों को अंग्रेजों ने बसाया था
(घ) इनमें से कोई नहीं
उत्तर:
(ख) तीनों मूलतः मत्स्य ग्रहण एवं बुनाई के गाँव थे

12. मिर्जा गालिब थे-
(क) चित्रकार
(ग) कवि
(ख) शायर
(घ) फिल्मकार
उत्तर:
(ख) शायर

13. निम्न में से किस शहर का सम्बन्ध राइटर्स बिल्डिंग से है?
(क) बम्बई
(ग) कलकत्ता
(ख) दिल्ली
(घ) मद्रास
उत्तर:
(ग) कलकत्ता

14. कासल क्या था?
(क) एक दुर्ग
(ग) एक बस्ती
(ख) एक शहर
(घ) इनमें से कोई नहीं
उत्तर:
(क) एक दुर्ग

15.
विनोदिनी दासी कौन थीं?
(क) चित्रकार
(ख) रंगकर्मी
(ग) वास्तुकार
(घ) उपर्युक्त सभी
उत्तर:
(ख) रंगकर्मी

16. फोर्ट सेंट जार्ज कहाँ स्थित है?
(क) मद्रास
(ख) दिल्ली
(ग) कलकत्ता
(घ) बम्बई
उत्तर:
(क) मद्रास

17. किस भारतीय शहर का सम्बन्ध फोर्ट विलियम से है?
(क) दिल्ली
(ग) मद्रास
(ख) कलकत्ता
(घ) जयपुर
उत्तर:
(ख) कलकत्ता

JAC Class 12 History Important Questions Chapter 12 औपनिवेशिक शहर : नगर-योजना, स्थापत्य

18. नवशास्त्रीय स्थापत्य शैली का उदाहरण है-
(क) टाउन हॉल
(ख) लाल किला
(ग) विक्टोरिया टर्मिनल
(च) ताजमहल
उत्तर:
(क) टाउन हॉल

19. सात टापुओं का शहर है-
(क) बम्बई
(ग) मद्रास
(ख) कलकत्ता
(घ) दिल्ली
उत्तर:
(क) बम्बई

20. लॉटरी कमेटी का सम्बन्ध है-
(क) पुरातत्त्व व्यवस्था से
(ख) सैनिक व्यवस्था से
(ग) प्रशासनिक व्यवस्था से
(घ) नगर नियोजन से
उत्तर:
(घ) नगर नियोजन से

21. गारेर मठ क्या था?
(क) एक मैदान
(ग) एक बस्ती
(ख) एक शहर
(घ) एक पक्षी
उत्तर:
(क) एक मैदान

रिक्त स्थानों की पूर्ति कीजिए

1. 1853 में बम्बई से ………………….. “तक रेलवे लाइन बिछाई गई।
2. 1896 में बम्बई के ………………….. -होटल में पहली बार फिल्म दिखाई गई।
3. कलकत्ता की जगह दिल्ली को ………………….. में राजधानी बनाया गया।
4. 1857 में बम्बई में पहली स्पिनिंग और ………………….. मिल की स्थापना की गई।
5. ईस्ट इण्डिया कम्पनी द्वारा ………………… में कलकत्ता में सर्वोच्च न्यायालय की स्थापना की गई।
6. एशियाटिक सोसाइटी की स्थापना ………………….. में ………………….. की गई।
7. ………………… में बम्बई को ब्रिटेन के राजा ने कम्पनी को दे दिया।
8. ग्रामीण अंचल में एक छोटे नगर को ………………. कहा जाता है।
9. ………………….. कहा का तात्पर्य एक छोटे स्थायी बाजार है।
10. पुर्तगालियों ने 1510 में ……………… तथा डचों ने 1605 में ………………. में आधार स्थापित कर लिए थे।
11. मद्रास, बम्बई और कलकत्ता का आधुनिक नाम क्रमश: …………………. , …………………… और …………………. है।
12. प्लासी का युद्ध ……………. में हुआ था।
13. सर्वे ऑफ इण्डिया का गठन ……………. ने किया गया था।
14. कपडों जबकि …………… अपने स्टील उत्पादन के लिए प्रसिद्ध था।
15 ……………… की स्थापना गुरखा युद्ध के दौरान की गई थी।
उत्तरमाला –
1. ठाणे
2. वाटसन्स
3. 1911
4 बीविंग
5. 1773
6. सर विलियम जोन्स, 1784
7. 1661
8. कस्बा
9. गंज
10. पणजी, मछलीपट्टनम
11. चेन्नई, मुम्बई और कोलकाता
12. 1757
13. 1878
14. कानपुर, जमशेदपुर
15. शिमला।

अतिलघूत्तरात्मक प्रश्न-

प्रश्न 1.
प्लासी का युद्ध कब और किसके बीच हुआ?
उत्तर:
1757 ई. में प्लासी का युद्ध अंग्रेजों और बंगाल के नवाब सिराजुद्दौला के बीच हुआ।

प्रश्न 2.
गेटवे ऑफ इण्डिया कब और कहाँ बनाया गया?
उत्तर:
गेटवे ऑफ इण्डिया 1911 में बम्बई में बनाया

प्रश्न 3.
औपनिवेशिक भारत में स्थापित दो हिल स्टेशनों का उल्लेख कीजिए।
उत्तर:
(1) शिमला
(2) माउंट आबू

JAC Class 12 History Important Questions Chapter 12 औपनिवेशिक शहर : नगर-योजना, स्थापत्य

प्रश्न 4.
ईस्ट इण्डिया कम्पनी द्वारा अपनी बस्तियों की किलेबन्दी करने का प्रमुख उद्देश्य क्या था? उत्तर- यूरोपीय कम्पनियों के बीच प्रतिस्पर्द्धा के कारण सुरक्षा बनाये रखना।

प्रश्न 5.
अखिल भारतीय जनगणना का प्रथम प्रयास किस वर्ष में किया गया था?
उत्तर:
सन् 1872 ई. में

प्रश्न 6.
भारत के मद्रास, कलकत्ता और बम्बई तीनों शहर मूलत: किस प्रकार के गाँव थे?
उत्तर:
ये तीनों शहर मत्स्य ग्रहण और बुनाई के गाँव थे।

प्रश्न 7.
इंग्लिश ईस्ट इण्डिया कम्पनी के एजेन्ट मद्रास में किस सन् में बस गये?
उत्तर:
सन् 1639 में।

प्रश्न 8.
इंग्लिश ईस्ट इण्डिया कम्पनी ने मद्रास, कलकत्ता और बम्बई में किस प्रकार के कार्यालय स्थापित किये?
उत्तर:
व्यापारिक और प्रशासनिक कार्यालय।

प्रश्न 9.
भारत में मुगलकाल में शाही प्रशासन और सत्ता के तीन महत्त्वपूर्ण केन्द्रों के नाम लिखिए।
उत्तर:
(1) आगरा
(2) दिल्ली और
(3) लाहौर।

प्रश्न 10.
दक्षिण भारत के ऐसे दो नगरों के नाम लिखिए जिनमें मुख्य केन्द्र मन्दिर होता था।
उत्तर:
मदुरई और कांचीपुरम

प्रश्न 11.
18वीं सदी के अन्त तक स्थल आधारित साम्राज्यों का स्थान कैसे साम्राज्यों ने ले लिया?
उत्तर:
जल आधारित यूरोपीय साम्राज्यों ने।

प्रश्न 12.
भारत में रेलवे की शुरुआत कब हुई?
उत्तर:
1853 में भारत में रेलवे की शुरुआत हुई। प्रश्न 13, 19वीं सदी के मध्य तक भारत में कौनसे दो औद्योगिक शहर थे?
उत्तर:
(1) कानपुर,
(2) जमशेदपुर।

प्रश्न 14.
कलकत्ता में किन लोगों ने बाजारों के आस-पास ब्लैक टाउन में परम्परागत ढंग से दालानी मकान बनवाए?
उत्तर:
अमीर भारतीय एजेन्टों और बिचौलियों ने।

प्रश्न 15.
कलकत्ता में मजदूर वर्ग के लोग कहाँ रहते में।
उत्तर:
शहर के विभिन्न इलाकों की कच्ची झोंपड़ियों

प्रश्न 16.
1857 के विद्रोह के बाद अंग्रेजों के लिए किस नाम से नए शहरी इलाके विकसित किये गये?
उत्तर:
सिविल लाइन्स’ के नाम से

प्रश्न 17.
सिविल लाइन्स में किनको बसाया गया?
उत्तर:
केवल गोरों को।

प्रश्न 18.
पहला हिल स्टेशन कब और कौनसा बनाया गया?
उत्तर:
पहला हिल स्टेशन 1815-16 में शिमला में स्थापित किया गया।

प्रश्न 19.
ईस्ट इण्डिया कम्पनी ने कलकत्ता और बम्बई में अपने व्यापारिक केन्द्र कब स्थापित किये?
उत्तर:
(1) 1661 में बम्बई में
(2) 1690 में कलकत्ता में।

प्रश्न 20.
1800 ई. तक जनसंख्या की दृष्टि से कौनसे भारतीय शहर विशालतम शहर बन गए थे?
उत्तर:
(1) मद्रास
(2) कलकत्ता
(3) बम्बई।

प्रश्न 21.
भारत में दशकीय जनगणना कब से एक नियमित व्यवस्था बन गई थी?
उत्तर:
पहला हिल-स्टेशन 1815-16 में शिमला में स्थापित किया गया।

JAC Class 12 History Important Questions Chapter 12 औपनिवेशिक शहर : नगर-योजना, स्थापत्य

प्रश्न 22.
1900 से 1940 के मध्य 40 वर्षों की अवधि में भारतीय शहरी आबादी में कितनी वृद्धि हुई?
उत्तर:
दस प्रतिशत से बढ़कर लगभग 13 प्रतिशत हो गई।

प्रश्न 23
19वीं शताब्दी में भारत में कौनसे रेलवे नगर अस्तित्व में आए?
उत्तर;
जमालपुर, वाल्टेयर तथा बरेली।

प्रश्न 24.
अंग्रेजों ने अपनी किन बस्तियों की किलेबन्दी की थी?
उत्तर:
बम्बई, मद्रास और कलकत्ता की।

प्रश्न 25.
औपनिवेशिक काल के दो औद्योगिक शहरों के नाम लिखिए।
उत्तर:
(1) कानपुर
(2) जमशेदपुर।

प्रश्न 26.
चमड़े की चीजों तथा ऊनी और सूती वस्त्रों के निर्माण के लिए कौनसा औद्योगिक नगर प्रसिद्ध था?
उत्तर:
कानपुर। था?

प्रश्न 27.
जमशेदपुर किसके उत्पादन के लिए प्रसिद्ध
उत्तर:
स्टील उत्पादन के लिए।

प्रश्न 28.
राइटर्स बिल्डिंग कहाँ पर स्थित थी?
उत्तर:
कलकत्ता में।

प्रश्न 29.
इंग्लिश ईस्ट इण्डिया कम्पनी के एजेंट कलकत्ता में किस वर्ष बसे?
उत्तर:
1690 ई. में।

प्रश्न 30.
अंग्रेजों द्वारा स्थापित दो हिल स्टेशनों के नाम बताइये ये कब स्थापित किये गए?
उत्तर:
1818 में माउण्ट आबू तथा 1835 में दार्जिलिंग।

प्रश्न 31.
सेनेटोरियम के रूप में किनका विकास किया गया था?
उत्तर:
हिल स्टेशनों का।

प्रश्न 32.
कौनसा हिल स्टेशन भारतीय सेना के कमांडर इन चीफ (प्रधान सेनापति) का भी अधिकृत वास बन गया था?
उत्तर:
शिमला।

प्रश्न 33.
भारत के नये शहरों में यातायात के कौनसे साधन थे?
उत्तर:
घोड़ागाड़ी, ट्राम तथा बस।

प्रश्न 34.
भारतीय शहरों में किस नये वर्ग का प्रादुर्भाव हुआ?
उत्तर:
मध्य वर्ग’ का ।

प्रश्न 35.
भारतीय शहरों में किन लोगों की माँग बढ़ रही थी?
उत्तर:
वकीलों, डॉक्टरों, शिक्षकों, क्लकों, इंजीनियरों, लेखाकारों की।

प्रश्न 36.
आमार कथा (मेरी कहानी) की रचना किसने की थी?
उत्तर:
विनोदिनी दास ने

प्रश्न 37.
मद्रास में ‘व्हाइट टाउन’ का केन्द्र कौन
उत्तर:
फोर्ट सेंट जार्ज।

JAC Class 12 History Important Questions Chapter 12 औपनिवेशिक शहर : नगर-योजना, स्थापत्य

प्रश्न 38.
कलकत्ता को किन तीन गाँवों को मिलाकर बनाया गया था?
उत्तर:
(1) सुतानाती
(2) कोलकाता
(3) गोविन्दपुर।

प्रश्न 39.
कलकत्ता का किला क्या कहलाता था?
उत्तर:
फोर्ट विलियम।

प्रश्न 40.
‘स्वास्थ्यकर नगर’ और ‘अस्वास्थ्यकर नगर’ कौनसे होते थे?
उत्तर:
व्हाइट टाउन’ स्वास्थ्यकर तथा ‘ब्लैक टाउन’ अस्वास्थ्यकर कहलाते थे।

प्रश्न 41.
लार्ड वेलेजली भारत के गवर्नर जनरल कब बने?
उत्तर:
1798 ई. में।

प्रश्न 42.
लार्ड वेलेजली ने कलकत्ता में अपने लिए रहने के लिए जो महल बनवाया, वह क्या कहलाता था?
उत्तर:
गवर्नमेंट हाउस

प्रश्न 43.
ग्रामीण क्षेत्रों के लोग किस प्रकार जीवन- यापन करते थे?
उत्तर:
खेती, जंगलों में संग्रहण तथा पशुपालन द्वारा।

प्रश्न 44.
ग्रामीण क्षेत्रों से कस्बों तथा शहरों को अलग दिखाने वाली एक विशेषता बताइए।
उत्तर:
कस्बों तथा शहरों की किलेबन्दी की जाती थी, जबकि ग्रामीण क्षेत्रों की नहीं। –

प्रश्न 45.
जनगणना के आँकड़ों का एक महत्त्व बताइए। उत्तर- जनगणना शहरीकरण के इतिहास का अध्ययन करने का बहुमूल्य स्रोत है।

प्रश्न 46.
आरम्भ में अधिकांश लोग जनगणना को सन्देह की दृष्टि से क्यों देखते थे?
उत्तर:
क्योंकि लोगों का मानना था कि सरकार नए कर लागू करने के लिए जाँच करवा रही है।

प्रश्न 47.
बगीचा पर’ का सम्बन्ध किस औपनिवेशिक शहर से था?
उत्तर:
कलकत्ता।

प्रश्न 48.
प्रारम्भ में बम्बई कितने टापुओं का इलाका था?
उत्तर:
सात टापुओं का।

प्रश्न 49.
बम्बई में नव-शास्त्रीय (नव क्लासिकल शैली) में निर्मित दो इमारतों के नाम लिखिए।
उत्तर:
(1) बम्बई का टाउन हाल
(2) एल्फिंस्टन सर्कल।

प्रश्न 50.
बम्बई में नव-गॉथिक शैली में निर्मित चार इमारतों के नाम लिखिए।
उत्तर:
(1) बम्बई सचिवालय
(2) बम्बई विश्वविद्यालय
(3) उच्च न्यायालय
(4) बम्बई टर्मिनस रेलवे स्टेशन।

प्रश्न 51.
नव-गॉथिक शैली में निर्मित सर्वोत्कृष्ट इमारत कौनसी थी जो बम्बई में निर्मित थी?
उत्तर:
विक्टोरिया टर्मिनस रेलवे स्टेशन

JAC Class 12 History Important Questions Chapter 12 औपनिवेशिक शहर : नगर-योजना, स्थापत्य

प्रश्न 52.
मद्रास में इण्डो सारसेनिक स्थापत्य शैली में निर्मित भवन का नाम लिखिए।
उत्तर:
मद्रास ला कोर्टस।

प्रश्न 53.
बम्बई में निर्मित ‘गेटवे ऑफ इण्डिया’ किस शैली का प्रतीक है?
उत्तर:
गुजराती शैली का।

प्रश्न 54.
राजा जार्ज पंचम और उनकी पत्नी मेरी के स्वागत के लिए कौनसी इमारत बनाई गई थी और कब?
उत्तर:
(1) गेटवे ऑफ इण्डिया
(2) 1911 में।

प्रश्न 55.
भारत में सबसे पहली रेल किन शहरों के बीच चलाई गई और कब?
उत्तर:
(1) बम्बई से ठाणे तक
(2) 1853 में।

प्रश्न 56.
बम्बई, मद्रास और कलकत्ता विश्वविद्यालयों की स्थापना कब की गई?
उत्तर:
1857 में।

प्रश्न 57.
अंग्रेजों ने कलकत्ता की जगह दिल्ली को कब राजधानी बनाया?
उत्तर;
1911 में

प्रश्न 58.
फोर्ट विलियम पर टिप्पणी लिखिए। उत्तर- फोर्ट विलियम कलकत्ता का एक प्रसिद्ध दुर्ग था। इसका निर्माण ब्रिटिश ईस्ट इण्डिया कम्पनी ने करवाया था।

प्रश्न 59
6वीं तथा 17वीं सदी में मुगलों द्वारा बनाए गए शहर किन बातों के लिए प्रसिद्ध थे? उत्तर-ये शहर जनसंख्या के केन्द्रीकरण, अपने विशाल भवनों तथा शाही शोभा व समृद्धि के लिए प्रसिद्ध थे।

प्रश्न 60.
मुगल राजधानियों दिल्ली और आगरे के राजनीतिक प्रभुत्व की समाप्ति के बाद किन क्षेत्रीय राजधानियों का महत्त्व बढ़ गया था?
उत्तर- लखनऊ, हैदराबाद, सारंगपट्म, पूना, नागपुर, बड़ौदा, तंजौर

प्रश्न 61.
पेठ और पुरम में क्या अन्तर है?
उत्तर:
पेठ तमिल शब्द है जिसका अर्थ होता है बस्ती जबकि पुरम शब्द गाँव के लिए प्रयोग किया जाता है।

प्रश्न 62.
अंग्रेजों ने अपनी व्यापारिक गतिविधियों का केन्द्र सर्वप्रथम किसे बनाया था?
उत्तर:
सूरत।

प्रश्न 63.
कलकत्ता में अंग्रेजों की सत्ता की प्रतीक इमारत कौनसी थी?
उत्तर:
गवर्नमेंट हाऊस।

प्रश्न 64.
लॉटरी कमेटी का सम्बन्ध किस औपनिवेशिक शहर से था?
उत्तर:
कलकत्ता।

प्रश्न 65.
बम्बई सचिवालय का डिजाइन किसने बनाया था?
उत्तर:
एच. एस. टी. क्लेयर विलकिन्स ने।

प्रश्न 66.
मद्रास, कलकत्ता और बम्बई में कौनसे इलाके ब्रिटिश आबादी के रूप में जाने जाते थे?
उत्तर:
मद्रास, कलकत्ता और बम्बई में क्रमश: फोर्ट सेंट जार्ज, फोर्ट विलियम और फोर्ट इलाके ब्रिटिश आबादी के रूप में प्रसिद्ध थे।

प्रश्न 67.
आजकल बम्बई, मद्रास तथा कलकत्ता किन नामों से पुकारे जाते हैं?
उत्तर:
आजकल बम्बई, मद्रास तथा कलकत्ता क्रमशः मुम्बई, चेन्नई तथा कोलकाता नाम से पुकारे जाते हैं।

प्रश्न 68.
पुर्तगालियों तथा डचों ने कब और कहाँ अपने व्यापारिक केन्द्रों की स्थापना की?
उत्तर:
पुर्तगालियों ने 1510 में पणजी में तथा डचों ने 1605 में मछलीपट्नम में अपने व्यापारिक केन्द्र स्थापित किये।

प्रश्न 69.
अंग्रेजों तथा फ्रांसीसियों ने कब और कहाँ व्यापारिक केन्द्र स्थापित किये?
उत्तर:
अंग्रेजों ने 1639 में मद्रास में तथा फ्रांसीसियों ने 1673 में पांडिचेरी में अपने व्यापारिक केन्द्रों की स्थापना की।

JAC Class 12 History Important Questions Chapter 12 औपनिवेशिक शहर : नगर-योजना, स्थापत्य

प्रश्न 70.
भारत के एक आधुनिक औद्योगिक देश न बनने का क्या कारण था?
उत्तर:
इंग्लैण्ड की पक्षपातपूर्ण औपनिवेशिक नीति के कारण।

प्रश्न 71.
‘राइटर्स बिल्डिंग’ से आप क्या समझते हैं?
उत्तर:
राइटर्स बिल्डिंग कम्पनी का एक मुख्य प्रशासकीय कार्यालय था क्लर्क ‘राइटर्स’ कहलाते थे।

प्रश्न 72.
‘दि मार्बल पैलेस’ के बारे में आप क्या जानते हैं?
उत्तर:
किसी शहरी सम्भ्रान्त वर्ग के एक भारतीय परिवार ने कलकत्ता में दि मार्बल पैलेस नामक एक अत्यंत भव्य इमारत बनवाई थी।

प्रश्न 73.
अंग्रेजों के लिए ‘सिविल लाइन्स’ में किन इमारतों का निर्माण किया गया?
उत्तर:
चौड़ी सड़कों, बड़े बगीचों में बने बंगलों, बैरकों, परेड मैदानों, चर्च आदि का निर्माण किया गया।

प्रश्न 74.
चितपुर बाजार कहाँ स्थित था?
उत्तर:
चितपुर बाजार कलकत्ता में ब्लैक टाउन और व्हाइट टाउन की सीमा पर स्थित था।

प्रश्न 75.
ब्लैक टाउन में भारतीयों द्वारा बनवाये गए मन्दिर को अंग्रेज क्या कहते थे?
उत्तर:
ब्लैक पगौडा।

प्रश्न 76.
‘व्हाइट टाउन’ से क्या अभिप्राय है?
उत्तर:
जिस शहरी इलाके में अंग्रेज (गोरे लोग ) रहते थे, वह ‘व्हाइट टाउन’ कहलाता था।

प्रश्न 77.
‘ब्लैक टाउन’ किसे कहते थे?
उत्तर:
जिस शहरी इलाके में भारतीय (काले लोग ) रहते थे, वह ‘ब्लैक टाउन’ कहलाता था।

प्रश्न 78.
प्रसिद्ध उद्योगपति जमशेदजी टाटा ने बम्बई में किस शैली में किस होटल का निर्माण करवाया था?
उत्तर:
गुजराती शैली में ताजमहल होटल का।

प्रश्न 79.
कस्बा और गंज में क्या अन्तर था?
उत्तर:
कस्बा ग्रामीण अंचल में एक छोटे नगर को तथा गंज एक छोटे स्थानीय बाजार को कहा जाता था।

प्रश्न 80.
अंग्रेजों की नजर में ‘ब्लैक टाउन’ कैसे थे?
उत्तर:
अंग्रेजों की नजर में ‘ब्लैक टाउन’ अराजकता तथा हो-हल्ला के केन्द्र व गन्दगी और बीमारियों के स्त्रोत थे।

प्रश्न 81.
अंग्रेजों द्वारा हिल स्टेशनों की स्थापना किस उद्देश्य से की गई थी?
अथवा
अंग्रेजी शासन काल में पहाड़ी शहरों (हिल स्टेशनों) का विकास क्यों किया गया?
उत्तर:
हिल स्टेशन अंग्रेज सैनिकों को ठहराने, सीमाओं की चौकसी करने और शत्रु के विरुद्ध आक्रमण करने के लिए महत्त्वपूर्ण स्थान थे।

प्रश्न 82.
किस गवर्नर जनरल ने अपनी काउंसिल कहाँ स्थानान्तरित की थी और कब?
उत्तर:
1864 में गवर्नर जनरल जान लारेन्स ने अपनी काउंसिल शिमला में स्थानान्तरित की थी।

प्रश्न 83.
मद्रास के नये ‘ब्लैक टाउन’ से क्या अभिप्राय है?
उत्तर:
नये ‘ब्लैक टाउन’ में भारतीय लोग रहते थे। यहाँ आड़ी-टेढ़ी संकरी गलियों में अलग-अलग जातियों के मोहल्ले थे।

प्रश्न 84.
लार्ड वेलेजली ने शहर में मूलभूत सुविधाएँ प्रदान करने के लिए सरकार की जिम्मेदारियों का उल्लेख किसमें किया था?
उत्तर:
1803 में ‘कलकत्ता मिनट्स’ में

JAC Class 12 History Important Questions Chapter 12 औपनिवेशिक शहर : नगर-योजना, स्थापत्य

प्रश्न 85.
अंग्रेज ‘बस्ती’ का प्रयोग किस रूप में ‘करते थे?
उत्तर:
गरीबों की कच्ची झोंपड़ियों के रूप में।

प्रश्न 86.
बम्बई अन्तर्राष्ट्रीय व्यापार का केन्द्र क्यों था?
उत्तर:
एक प्रमुख बन्दरगाह होने के नाते।

प्रश्न 87.
19वीं शताब्दी के अन्त तक भारत का आधा निर्यात तथा आयात किस शहर से होता था?
उत्तर:
बम्बई से

प्रश्न 88.
अम्बई से व्यापार की एक महत्त्वपूर्ण वस्तु कौनसी थी?
उत्तर:
अफीम

प्रश्न 89.
व्यापार की उन्नति के कारण कौनसा भारतीय शहर ‘भारत का सरताज शहर’ कहलाता था?
उत्तर:
बम्बई।

प्रश्न 90.
एशियाटिक सोसायटी ऑफ बाम्बे’ का कार्यालय कहाँ है?
उत्तर:
बम्बई के टाउनहाल में।

प्रश्न 91.
बम्बई की उस इमारत का नाम लिखिए जो ग्रीको-रोमन स्थापत्य शैली से प्रभावित है।
उत्तर:
एल्फिंस्टन सर्कल।

प्रश्न 92.
औपनिवेशिक शहरों में इमारतें बनाने के लिए कौनसी स्थापत्य शैलियों का प्रयोग किया गया?
उत्तर:
(1) नवशास्त्रीय
(2) नव-गॉथिक शैली
(3) इण्डो-सारसेनिक शैली।

प्रश्न 93.
नव-गॉथिक स्थापत्य शैली की विशेषताएँ बताइये।
उत्तर:
ऊँची उठी हुई छलें, नोकदार मेहराबें, बारीक साज-सज्जा

प्रश्न 94.
इण्डो-सारसेनिक स्थापत्य शैली क्या थी?
उत्तर;
इण्डो-सारसेनिक स्थापत्य शैली में भारतीय और यूरोपीय दोनों शैलियों के तत्त्व थे।

प्रश्न 95.
चॉल से आप क्या समझते हैं?
उत्तर:
बम्बई में जगह की कमी एवं भीड़भाड़ के कारण एक विशेष प्रकार की इमारतें बनायी गयीं जिन्हें चॉल कहा गया।

प्रश्न 96.
स्थापत्य शैलियों से क्या पता चलता है? उत्तर-स्थापत्य शैलियों से अपने समय के सौन्दर्यात्मक आदशों एवं उनमें निहित विविधताओं का पता चलता है।

लघुत्तरात्मक प्रश्न-

प्रश्न 1.
18वीं शताब्दी के अन्त तक भारत में स्थल आधारित साम्राज्यों का स्थान जल आधारित शक्तिशाली यूरोपीय साम्राज्यों ने ले लिया।” व्याख्या कीजिए।
उत्तर:
18वीं शताब्दी के अन्त तक जल आधारित शक्तिशाली यूरोपीय साम्राज्यों ने प्रमुख स्थान प्राप्त कर लिया। अन्तर्राष्ट्रीय व्यापार, वाणिज्यवाद तथा पूँजीवाद की शक्तियाँ अब समाज के स्वरूप को निर्धारित करने लगीं। अब मद्रास, कोलकाता तथा मुम्बई जैसे औपनिवेशिक बन्दरगाह शहर नई आर्थिक राजधानियों के रूप में प्रकट हुए। ये औपनिवेशिक प्रशासन और सत्ता के केन्द्र भी बन गए। अब नये भवनों और संस्थानों का विकास हुआ।

प्रश्न 2.
तर्क सहित सिद्ध कीजिये कि औपनिवेशिक शहरों का सामाजिक जीवन वर्तमान शहरों में भी दिखाई पड़ता है?
उत्तर;
औपनिवेशिक शहरों की भाँति वर्तमान शहरों में भी घोड़गाड़ी, ट्रामों, बसों का यातायात के साधनों के रूप में प्रयोग किया जाता है। उनहाल सार्वजनिक पार्क, रंगशाला, सिनेमाहाल आदि लोगों के मिलने-जुलने के स्थान हैं। वर्तमान शहरों में ‘मध्य वर्ग’ का काफी प्रभाव है। वर्तमान शहरों में स्कूल, कॉलेज, विश्वविद्यालय, लाइब्रेरी आदि शिक्षा के केन्द्र बने हुए हैं। स्वियों में भी जागृति आई है।

प्रश्न 3.
‘चाल’ से आप क्या समझते हैं? इनकी विशेषता लिखिए।
उत्तर:
शहर में जगह की कमी और भीड़भाड़ के कारण बम्बई में एक खास तरह की इमारतें बनाई गई, जिन्हें ‘चाल’ का नाम दिया गया। ये इमारतें बहुमंजिला होती थीं, जिनमें एक-एक कमरे वाली आवासीय इकाइयाँ बनाई जाती थीं। इमारत के सारे कमरों के सामने एक खुला बरामदा या गलियारा होता था और बीच में दालान होता था। इस प्रकार की इमारतों में बहुत थोड़ी सी जगह में बहुत सारे परिवार रहते थे। सभी लोग एक-दूसरे के सुख- दुःख में भागीदार होते थे।

JAC Class 12 History Important Questions Chapter 12 औपनिवेशिक शहर : नगर-योजना, स्थापत्य

प्रश्न 4.
अंग्रेजों ने भारत में भवन निर्माण के लिए यूरोपीय शैली को किस कारण चुना? तर्क सहित लिखिए।
उत्तर:
(1) इसमें एक अनजान देश में जाना- पहचाना सा भू-दृश्य रचने की और उपनिवेश में घर जैसा अनुभव करने की अंग्रेजों की चाहत दिखाई पड़ती थी।
(2) अंग्रेजों को लगता था कि यूरोपीय शैली उनकी श्रेष्ठता, अधिकार और सत्ता का प्रतीक होगी।
(3) वे सोचते थे कि यूरोपीय ढंग की दिखने वाली इमारतों से औपनिवेशिक शासकों और भारतीय प्रजा के बीच फर्क और फासला साफ दिखने लगेगा।

प्रश्न 5.
अंग्रेजों ने मद्रास (चेन्नई) के आसपास अपना वर्चस्व कैसे स्थापित किया?
उत्तर:
1611 में अंग्रेजों ने मछलीपट्नम में अपनी फैक्ट्री स्थापित की। लेकिन जल्दी ही उन्होंने मद्रास को अपना केन्द्र बनाया, जिसका पट्टा वहाँ के जा ने 1639 में उन्हें दे दिया। राजा ने उन्हें उस स्थान की किलेबन्दी करने, सिक्के ढालने तथा प्रशासन की अनुमति दे दी। यहाँ पर अंग्रेजों ने अपनी फैक्ट्री के इर्द-गिर्द एक किला बनाया, जिसका नाम फोर्ट सेंट जार्ज रखा गया 1761 में फ्रांसीसियों की हार के बाद मद्रास और सुरक्षित हो गया।

प्रश्न 6.
दक्षिण भारत की परिस्थितियाँ अंग्रेजों के लिए अधिक अनुकूल थीं, क्यों? लिखिए।
उत्तर:
दक्षिण भारत में अंग्रेजों का मुकाबला किसी ताकतवर राज्य से नहीं हुआ 1665 में तालीकोटा के युद्ध के बाद विजयनगर साम्राज्य नष्ट-भ्रष्ट हो गया और उसके स्थान पर छोटे-छोटे राज्य स्थापित हो गये, जैसे- बीदर, बरार, गोलकुण्डा आदि। इन राज्यों को अंग्रेजों ने अपनी कूटनीति के द्वारा डरा-धमका कर या लालच देकर अपने अधीन कर लिया। केवल मैसूर के राज्य से ही उन्हें टक्कर लेनी पड़ी।

प्रश्न 7.
नक्शे हमें क्या बताते हैं और क्या छिपाते है?
उत्तर:
1878 में भारत में सर्वे ऑफ इण्डिया का गठन किया गया। उस समय के नक्शों से हमें काफी जानकारी उपलब्ध होती है, साथ ही हमें अंग्रेजों की भेदभावपूर्ण सोच का भी पता लग जाता है। उदाहरण के लिए, नक्शे में गरीबों की बस्तियों को चिह्नित नहीं किया गया। इसका अर्थ यह लगाया गया कि नक्शे में रिक्त स्थान अन्य योजनाओं के लिए उपलब्ध हैं। जब इन योजनाओं को शुरू किया गया तो गरीबों की बस्तियों को वहाँ से हटा दिया गया।

प्रश्न 8.
औपनिवेशिक काल में ग्रामीण इलाकों एवं कस्बों के चरित्र में अन्तर बताइए।
उत्तर:
ग्रामीण इलाकों एवं कस्बों के चरित्र में भिन्नता के मुख्य बिन्दु निम्नलिखित हैं-
(1) ग्रामीण इलाकों के लोग खेती, पशुपालन एवं जंगलों में संग्रहण द्वारा अपनी जीविका का निर्वाह करते हैं। इसके विपरीत कस्बों में शासक, व्यापारी, प्रशासक व शिल्पकार आदि रहते थे।
(2) कस्बों एवं शहरों की प्रायः किलेबन्दी की जाती थी। यह किलेबन्दी उन्हें ग्रामीण क्षेत्रों से अलग करती थी।
(3) कस्बों पर ग्रामीण जनता का प्रभुत्व रहता था। वे खेती से प्राप्त होने वाले करों एवं अधिशेष के आधार पर निर्भर रहते थे।

प्रश्न 9.
मध्यकालीन दक्षिण भारत के शहरों की मुख्य विशेषताओं का उल्लेख कीजिए।
उत्तर:
(1) दक्षिण भारत के नगर मदुरई तथा कांचीपुरम प्रमुख केन्द्र थे।
(2) दक्षिण भारत के अनेक नगरों में बन्दरगाह होते थे।
(3) ये व्यापार के मुख्य केन्द्रों के कारण विकसित हुए थे।
(4) दक्षिण भारत के शहरों में धार्मिक उत्सव अत्यधिक धूम-धाम के साथ मनाए जाते थे।

प्रश्न 10.
कस्बा एवं गंज के बारे में आप क्या जानते हैं?
उत्तर:
कस्बा – कस्बा ग्रामीण अंचल में एक छोटे नगर को कहा जाता है जो सामान्यतः स्थानीय विशिष्ट वस्तुओं का केन्द्र होता है।
गंज-गंज एक छोटे स्थायी बाजार को कहा जाता है। कस्बा और गंज दोनों कपड़ा, फल, सब्जी एवं दुग्ध उत्पादों से सम्बन्ध थे। ये विशिष्ट परिवारों एवं सेना के लिए सामग्री उपलब्ध करवाते थे।

प्रश्न 11.
अमेरिका के गृहयुद्ध और स्वेज नहर के खुलने का भारत की आर्थिक गतिविधियों पर क्या प्रभाव पड़ा?
अथवा
अमेरिकी गृह युद्ध ने भारत में ‘रैयत’ समुदाय के जीवन को किस प्रकार प्रभावित किया?
उत्तर:
सन् 1861 में अमेरिका में गृहयुद्ध शुरू होने के कारण वहाँ से कपास का निर्यात बन्द हो गया। इससे भारतीय कपास की माँग बढ़ी, जिसकी खेती मुख्य रूप से दक्कन में होती थी। 1869 में स्वेज नहर को व्यापार के लिए खोल दिया गया, जिससे विश्व अर्थव्यवस्था के साथ-साथ बम्बई की अर्थव्यवस्था भी मजबूत हुई। बम्बई की सरकार और भारतीय व्यापारियों ने बम्बई को ‘भारत का सरताज शहर’ घोषित कर दिया।

प्रश्न 12.
18वीं शताब्दी के प्रारम्भिक वर्षों में औपनिवेशिक सरकार ने नगरों के लिए मानचित्र तैयार करने पर ध्यान क्यों दिया ?
अथवा
औपनिवेशिक सरकार ने मानचित्र तैयार करने पर विशेष ध्यान क्यों दिया?
उत्तर:
औपनिवेशिक सरकार की मान्यता थी कि किसी शहर की बनावट और भूदृश्य को समझने के लिए मानचित्र आवश्यक होते हैं। इस जानकारी के आधार पर वे इलाके पर अधिक नियन्त्रण स्थापित कर सकते थे। शहरों के मानचित्रों से हमें उस स्थान पर पहाड़ियों, नदियों व हरियाली का पता चलता है। ये समस्त बातें रक्षा सम्बन्धी उद्देश्यों के लिए योजना तैयार करने में बड़ी उपयोगी सिद्ध होती हैं। मकानों की सघनता, सड़कों की स्थिति आदि से इलाके की व्यावसायिक सम्भावनाओं की जानकारी मिलती है।

JAC Class 12 History Important Questions Chapter 12 औपनिवेशिक शहर : नगर-योजना, स्थापत्य

प्रश्न 13.
जनगणना से प्राप्त आँकड़ों का क्या महत्त्व है?
उत्तर:
(1) ये आँकड़े शहरीकरण का अध्ययन करने के लिए एक बहुमूल्य स्रोत हैं।
(2) बीमारियों से होने वाली मृत्युओं की सारणियों, आयु, लिंग, जाति व व्यवसाय के अनुसार लोगों को गिनने की व्यवस्था से संख्याओं का एक विशाल भण्डार मिलता है।
(3) जनगणना के माध्यम से आबादी के बारे में सामाजिक जानकारियों को सुगम्य आँकड़ों में बदला जा सकता था।

प्रश्न 14.
औपनिवेशिक भारत में जनगणना सम्बन्धी भ्रमों का उल्लेख कीजिये ।
उत्तर:
(1) आबादी के विभिन्न वर्गों का वर्गीकरण करने के लिए अलग-अलग श्रेणियाँ बनाई गई थीं। कई बार यह वर्गीकरण अतार्किक होता था।
(2) लोग जनगणना आयुक्तों को गलत जवाब दे देते थे।
(3) प्राय: लोग स्वयं भी जनगणना के कार्य में सहायता देने से इनकार कर देते थे। ऊँची जाति के लोग अपने परिवार की स्त्रियों के बारे में जानकारी देने से संकोच करते थे बीमारियों से सम्बन्धित आँकड़ों को कठिन था।
(4) मृत्यु दर तथा एकत्रित करना बहुत

प्रश्न 15.
” अठारहवीं शताब्दी में औपनिवेशिक शहर अंग्रेजों की वाणिज्यिक संस्कृति को प्रतिबिम्बित करते थे।” व्याख्या कीजिये।
उत्तर:
अठारहवीं शताब्दी में राजनीतिक सत्ता और संरक्षण भारतीय शासकों के स्थान पर ईस्ट इण्डिया कम्पनी के व्यापारियों के हाथों में आ गई दुभाषिए, बिचौलिए, व्यापारी और माल आपूर्तिकर्ता के रूप में काम करने वाले भारतीयों का भी इन नये शहरों में एक महत्त्वपूर्ण स्थान था। नदी या समुद्र के किनारे आर्थिक गतिविधियों से गोदियों और घाटियों का विकास हुआ। समुद्र किनारे गोदाम, वाणिज्यिक कार्यालय, बीमा एजेंसियों, यातायात डिपो और बैंकिंग संस्थानों की स्थापना हुई।

प्रश्न 16.
1857 के विद्रोह के बाद अंग्रेज़ों ने शहरी इलाकों में ‘सिविल लाइन्स’ नामक इलाके क्यों विकसित किये?
उत्तर:
1857 के विद्रोह के बाद अंग्रेज शासकों ने अनुभव किया कि अंग्रेजों को भारतीयों (देशियों) के खतरे से दूर, अधिक सुरक्षित व पृथक् बस्तियों में रहना चाहिए। अत: उन्होंने पुराने कस्बों के चारों ओर चरागाहों और खेतों को साफ कर ‘सिविल लाइन्स’ नामक नये शहरी इलाके विकसित किये। ‘सिविल लाइन्स’ में केवल गोरे लोगों को बसाया गया। चौड़ी सड़कों, बड़े बगीचों में बने बंगलों, बैरकों, परेड मैदान, चर्च आदि से लैस छावनियाँ यूरोपीय लोगों के लिए सुरक्षित आश्रय स्थल थीं।

प्रश्न 17.
हिल स्टेशनों की अंग्रेजों के लिए क्या उपयोगिता थी?
उत्तर:
(1) हिल स्टेशन अंग्रेज सैनिकों को ठहराने, सीमाओं की चौकसी करने और शत्रु के विरुद्ध आक्रमण करने के लिए महत्त्वपूर्ण स्थान थे।
(2) हिल स्टेशनों की जलवायु अंग्रेजों के लिए स्वास्थ्यप्रद थी।
(3) यहाँ अंग्रेज सैनिक हैजा, मलेरिया आदि बीमारियों से मुक्त रह सकते थे।
(4) ये हिल स्टेशन सेनेटोरियम के रूप में भी विकसित किये गए थे। यहाँ सैनिकों को विश्राम करने एवं इलाज कराने के लिए भेजा जाता था।

प्रश्न 18.
“औपनिवेशिक शहरों में नये सामाजिक समूह बने तथा लोगों की पुरानी पहचानें महत्त्वपूर्ण नहीं रहीं।” व्याख्या कीजिये ।
अथवा
औपनिवेशिक शहरों में ‘मध्य वर्ग’ के विकास का वर्णन कीजिये।
उत्तर:
अठारहवीं शताब्दी में औपनिवेशिक शहरों में समस्त वर्गों के लोग आने लगे शहरों में क्लकों, डॉक्टरों, इन्जीनियरों वकीलों, शिक्षकों तथा लेखाकारों की माँग बढ़ती जा रही थी। परिणामस्वरूप शहरों में ‘मध्य वर्ग’ का विकास हुआ। मध्य वर्ग के लोग सुशिक्षित थे तथा इनकी स्कूल, कॉलेज, लाइब्रेरी तक अच्छी पहुँच थी। वे समाज और सरकार के बारे में समाचार-पत्रों, पत्रिकाओं और सार्वजनिक सभाओं में अपने विचार व्यक्त कर सकते थे।

प्रश्न 19.
इतिहासकारों को जनगणना जैसे स्रोतों का प्रयोग करते समय सावधानी क्यों रखनी चाहिए?
उत्तर:
इतिहासकारों को जनगणना जैसे स्रोतों का प्रयोग करते समय सावधानी इसलिए रखनी चाहिए क्योंकि जनगणना के आँकड़े भ्रामक भी हो सकते हैं। इन आँकड़ों का प्रयोग करने से पहले हमें इस बात को अच्छी तरह समझ लेना चाहिए कि आँकड़े किसने इकट्ठा किए हैं तथा उन्हें क्यों व कैसे इकट्ठा किया गया था।

प्रश्न 20.
अंग्रेजों ने बंगाल में अपने शासन के शुरू से ही नगर नियोजन का कार्यभार अपने हाथों में क्यों लिया?
उत्तर- अंग्रेज व्यापारी नवाब सिराजुद्दौला की सम्प्रभुता से असन्तुष्ट थे। उसने उनसे माल गोदाम के रूप में प्रयोग किये जाने वाला छोटा किला छीन लिया था। प्लासी के युद्ध में विजय के उपरान्त अंग्रेजों ने कलकत्ता में ऐसा किला बनाने का निश्चय किया जिस पर आसानी से आक्रमण न किया जा सके।

प्रश्न 21.
नवाब सिराजुद्दौला ने कलकत्ता नगर पर हमला क्यों किया?
उत्तर:
ईस्ट इण्डिया कम्पनी के व्यापारी नवाब सिराजुद्दौला की सम्प्रभुता पर लगातार सवाल उठा रहे थे। वे न तो कस्टम ड्यूटी चुकाना चाहते थे और न ही उनके द्वारा तय की गई कारोबार की शर्तों पर काम करना चाहते थे। इसलिए नवाब सिराजुद्दौला ने 1756 ई. में कलकत्ता पर हमला करके अंग्रेजों द्वारा बनाए गए किले पर अपना अधिकार कर लिया।

JAC Class 12 History Important Questions Chapter 12 औपनिवेशिक शहर : नगर-योजना, स्थापत्य

प्रश्न 22.
अठारहवीं शताब्दी में शहरों में स्त्रियों में आई जागरूकता का रूढ़िवादी लोग विरोध क्यों करने लगे?
उत्तर:
रूढ़िवादियों को भय था कि यदि स्त्रियाँ पढ़ लिख गई, तो वे संसार में क्रान्ति ला देंगी तथा सम्पूर्ण सामाजिक व्यवस्था का आधार खतरे में पड़ जायेगा। कुछ महिला सुधारक भी स्त्रियों को माँ और पत्नी की परम्परागत भूमिकाओं में ही देखना चाहते थे। उनका कहना था फि स्त्रियों को पर की चारदीवारी के भीतर ही रहना चाहिए। वे परम्परागत पितृसत्तात्मक रीति-रिवाजों, कानूनों को परिवर्तित करने के प्रयासों से असन्तुष्ट थे।

प्रश्न 23.
‘नव-गॉथिक शैली’ से आप क्या समझते हैं ?
उत्तर:
ऊँची उठी हुई छर्ने, नोकदार मेहराबें और बारीक साज-सज्जा ‘नव- गाँधिक शैली’ की विशेषताएँ थीं। बम्बई सचिवालय, बम्बई विश्वविद्यालय, बम्बई उच्च न्यायालय आदि भव्य इमारतें समुद्र किनारे इसी शैली में बनाई गई। यूनिवर्सिटी लाइब्रेरी के घंटाघर का निर्माण प्रेमचन्द रायचन्द के धन से किया गया था। इसका नाम उनकी माँ के नाम पर राजाबाई टावर रखा गया। परन्तु नव-गॉथिक शैली का सर्वोत्कृष्ट उदाहरण विक्टोरिया टर्मिनस रेलवे स्टेशन है।

प्रश्न 24.
नवशास्त्रीय या नियोक्लासिकल स्थापत्य कला पर एक संक्षिप्त टिप्पणी लिखिए।
उत्तर:
बड़े-बड़े स्तम्भों के पीछे रेखागणितीय संरचनाओं का निर्माण नवशास्त्रीय स्थापत्य शैली की विशेषता थी। यह शैली मूल रूप से प्राचीन रोम की भवन निर्माण शैली से निकली थी। 1833 में बम्बई का टाउन हाल इसी शैली के अनुसार बनाया गया था। 1860 के दशक में अनेक व्यावसायिक इमारतों के समूह को ‘एल्फिन्स्टन सर्कल’ कहा जाता था। बाद में इसका नाम बदलकर हार्निमान सर्कल रख दिया गया था। इसमें पहली मंजिल पर ढके हुए तोरणपथ का रचनात्मक ढंग से प्रयोग किया गया।

प्रश्न 25.
भारत में रेलवे की शुरुआत कब हुई और इसके क्या प्रभाव हुए?
अथवा
1853 में रेलवे के आरम्भ की शहरीकरण की प्रक्रिया में क्या भूमिका रही?
अथवा
1853 में रेलवे की स्थापना से किस प्रकार नगरों का भाग्य बदल गया? कोई दो परिवर्तन बताइये।
उत्तर:
(1) भारत में रेलवे की शुरुआत 1853 में हुई। अब आर्थिक गतिविधियों का केन्द्र परम्परागत शहरों से दूर जाने लगा क्योंकि ये शहर पुराने मार्गों और नदियों के निकट थे।
(2) प्रत्येक रेलवे स्टेशन कच्चे माल का संग्रह केन्द्र और आयातित माल का वितरण केन्द्र बन गया था।
(3) रेलवे नेटवर्क के विस्तार के साथ रेलवे वर्कशाप, रेलवे कालोनियों भी बनने लगीं और जमालपुर, बरेली और वाल्टेयर जैसे रेलवे नगर अस्तित्व में आए।

प्रश्न 26.
अंग्रेजों ने ब्लैक टाउनों में सफाई व्यवस्था पर ज्यादा ध्यान कब व क्यों दिया? स्पष्ट कीजिए।
उत्तर:
जब ब्लैक टाउन (काले इलाके में हैजा और प्लेग जैसी महामारियाँ फैली और हजारों लोग मौत का शिकार हुए तब अंग्रेज अफसरों को स्वच्छता व सार्वजनिक स्वास्थ्य के लिए ज्यादा कठोर कदम उठाने पड़े। उनको इस बात का डर था कि कहीं ये बीमारियाँ ब्लैक टाउन से ह्वाइट टाउन में भी न फैल जायें। 1960-70 के दशकों से साफ-सफाई के बारे में कड़े प्रशासकीय उपाय लागू किए गए और भारतीय शहरों में निर्माण गतिविधियों पर अंकुश लगाया गया।

प्रश्न 27.
आमार कथा (मेरी कहानी ) क्या है और किसके द्वारा लिखी गई ?
अथवा
विनोदिनी दास पर एक संक्षिप्त टिप्पणी लिखिए।
उत्तर:
विनोदिनी दास बंगाली रंगमंच की एक प्रसिद्ध अदाकारा थीं। ‘स्टार थियेटर’, कलकत्ता की स्थापना ( 1883 ) के पीछे उनका मुख्य हाथ था 1910 से 1913 के बीच उन्होंने ‘आमार कथा’ के नाम से किस्तों में अपनी आत्मकथा लिखी। वे एक जबरदस्त व्यक्तित्व वाली महिला थीं। उन्होंने समाज में औरतों की समस्याओं पर केन्द्रित कई भूमिकाएँ निभाई। वे अभिनेत्री, संस्था निर्मात्री और लेखिका के रूप में कई भूमिकाएँ एक साथ निभाती

प्रश्न 28.
अंग्रेजों ने जहाँ पर भी किले बनाए, उनके चारों ओर खुले मैदान क्यों छोड़े और इसके पीछे क्या दलील दी ?
उत्तर:
अंग्रेजों ने कलकत्ता में फोर्ट विलियम के इर्द- गिर्द एक विशाल जगह खाली छोड़ दी। खाली मैदान रखने का उद्देश्य यह था कि किले की ओर बढ़ने वाली शत्रु की सेना पर किले से बेरोक-टोक गोलीबारी की जा सके। जब अंग्रेजों को कलकत्ता में अपनी उपस्थिति स्थायी दिखाई देने लगी, तो वे फोर्ट से बाहर मैदान के किनारे पर भी आवासीय इमारतें बनाने लगे।

प्रश्न 29.
“कलकत्ता के लिए जो पैटर्न तैयार किया गया था, उसे बहुत सारे शहरों में दोहराया गया।” व्याख्या कीजिये।
उत्तर:
1857 के विद्रोह के बाद अंग्रेज विद्रोहियों के गढ़ों को अपने लिए सुरक्षित बनाने लगे। उन्होंने दिल्ली में लाल किले पर अपना कब्जा करके वहाँ अपनी सेना तैनात कर दी। उन्होंने किले के पास बनी इमारतों को साफ करके भारतीय मोहल्लों और किले के बीच काफी फासला बना दिया। इसके पीछे उन्होंने यह दलील दी कि अगर . कभी शहर के लोग फिरंगी-राज के खिलाफ खड़े हो जाएँ तो उन पर गोली चलाने के लिए खुली जगह जरूरी थी।

JAC Class 12 History Important Questions Chapter 12 औपनिवेशिक शहर : नगर-योजना, स्थापत्य

प्रश्न 30.
कलकत्ता नगर नियोजन में लाटरी कमेटी के कार्यों की वर्तमान संदर्भ में प्रासंगिकता स्पष्ट कीजिए।
अथवा
‘लाटरी कमेटी’ क्या थी? इसकी कार्यप्रणाली का वर्णन कीजिए।
उत्तर:
1817 में कलकत्ता में एक कमेटी बनाई गई जो सरकार की मदद से नगर नियोजन का कार्य करती थी। यह कमेटी नगर सुधार के लिए धन की व्यवस्था जनता के बीच लाटरी बेचकर करती थी, इसलिए इसका नाम ‘लाटरी कमेटी’ पड़ा। लाटरी कमेटी ने एक नक्शा बनवाया, जिससे कलकत्ता शहर की एक सम्पूर्ण तस्वीर सामने आ सके। कमेटी की प्रमुख गतिविधियों में शहर के हिन्दुस्तानी आबादी वाले हिस्से में सड़क निर्माण और नदी के किनारे से अवैध कब्जे हटाना शामिल था।

प्रश्न 31.
औपनिवेशिक काल में कस्बों का स्वरूप गाँवों से भिन्न था फिर भी इनके बीच की पृथकता अनिश्चित होती थी? स्पष्ट कीजिए।
उत्तर- औपनिवेशिक काल में लोग ग्रामीण इलाकों में खेती, जंगलों में संग्रहण या पशुपालन द्वारा जीवन निर्वाह करते थे। इसके विपरीत कस्बों में शिल्पकार, व्यापारी, प्रशासक एवं शासक रहते थे। कस्बों पर ग्रामीण जनता का प्रभुत्व रहता था तथा वे खेती से प्राप्त करों एवं अधिशेष के आधार पर फलते-फूलते थे प्रायः कस्बों व शहरों की किलेबन्दी की जाती थी जो उन्हें ग्रामीण क्षेत्रों से अलग करती थी फिर भी कस्बों एवं गाँवों के मध्य की पृथकता अनिश्चित होती थी।

किसान तीर्थयात्रा करने के लिए लम्बी दूरियाँ तय करते थे एवं कस्बों से होकर गुजरते थे। दूसरी ओर लोगों और माल का कस्बे से गाँवों की ओर गमन होता रहता था। व्यापारी और फेरीवाले कस्बों से माल गाँव ले जाकर बेचते थे इससे बाजारों का फैलाव और उपभोग की नई शैलियों का उदय होता था। इसके अतिरिक्त जब कस्बों पर आक्रमण होते थे तो लोग प्रायः ग्रामीण क्षेत्रों में शरण लेते थे।

प्रश्न 32.
अठारहवीं शताब्दी के मध्य से शहरों का रूप परिवर्तन क्यों एवं किस प्रकार हुआ?
उत्तर:
अठारहवीं शताब्दी के मध्य से शहरों के रूप परिवर्तन का एक नया चरण प्रारम्भ हुआ। व्यापारिक गतिविधियों के अन्य स्थानों पर केन्द्रित होने के कारण सत्रहवीं शताब्दी में विकसित हुए शहर- सूरत, मछलीपट्टनम व ढाका पतनोन्मुख हो गए। 1757 ई. में बंगाल के नवाब सिराजुद्दौला एवं अंग्रेजों के मध्य हुए प्लासी के बुद्ध में अंग्रेजों की जीत हुई। मद्रास, बम्बई व कलकत्ता ये शहर औपनिवेशिक प्रशासन एवं सत्ता के केन्द्र भी बन गये। नए भवनों एवं संस्थानों का उदय हुआ एवं शहरी केन्द्रों को नए तरीके से पूर्णतः व्यवस्थित किया गया। इन शहरों में नये नये रोजगारों का सृजन हुआ, जिससे लोग इन शहरों में बसने लगे। लगभग 1800 ई. तक ये शहर जनसंख्या की दृष्टि से भारत के विशाल शहर बन गए।

प्रश्न 33.
प्रारम्भिक वर्षों में औपनिवेशिक सरकार ने मानचित्र बनाने पर विशेष ध्यान क्यों दिया?
उत्तर:
प्रारम्भिक वर्षों में औपनिवेशिक सरकार ने निम्नलिखित कारणों से मानचित्र बनाने पर विशेष ध्यान दिया-
(i) सरकार का मानना था कि किसी स्थान की बनावट एवं भूदृश्य को समझने के लिए मानचित्र आवश्यक होते हैं। इस जानकारी के आधार पर वे शहरी प्रदेश पर नियन्त्रण बनाये रख सकते थे।

(ii) जब शहरों का विस्तार होने लगा तो न केवल उनके विकास की योजना तैयार करने के लिए बल्कि शहर को विकसित करने एवं अपनी सत्ता मजबूत बनाने के लिए भी मानचित्र बनाये जाने लगे।

(ii) शहरों के मानचित्रों से हमें उसकी पहाड़ियों, नदियों एवं हरियाली का पता चलता है। यह जानकारी रक्षा सम्बन्धी उद्देश्यों के लिए योजना बनाने में बहुत काम आती

प्रश्न 34.
किन सरकारी नीतियों ने भारतीयों के भीतर उपनिवेशवाद विरोधी और राष्ट्रवादी भावनाओं को बढ़ावा दिया?
उत्तर:
उन्नीसवीं सदी में शहरों में सरकारी दखलन्दाजी और सख्त हो गई। इस आधार पर और अधिक तेजी से झुग्गी-झोंपड़ियों को हटाना शुरू किया गया। दूसरे इलाकों की अपेक्षा ब्रिटिश आबादी वाले हिस्सों को तेजी से विकसित किया जाने लगा। ‘स्वास्थ्यकर’ और ‘अस्वास्थ्यकर’ के नए विभेद के कारण ‘हाइट’ और ‘ब्लैक’ टाउन वाले नस्ली विभाजन को और बल मिला। इन सरकारी नीतियों के विरुद्ध जनता के प्रतिरोध ने भारतीयों के भीतर उपनिवेशवाद विरोधी और राष्ट्रवादी भावनाओं को बढ़ावा दिया।

JAC Class 12 History Important Questions Chapter 12 औपनिवेशिक शहर : नगर-योजना, स्थापत्य

प्रश्न 35.
बम्बई स्थित होटल ताजमहल के बारे में आप क्या जानते हैं?
उत्तर:
बम्बई स्थित प्रसिद्ध होटल ताजमहल का निर्माण प्रसिद्ध उद्योगपति जमशेदजी टाटा ने करवाया था। यह परम्परागत गुजराती शैली में निर्मित है। यह इमारत न केवल भारतीय उद्यमशीलता का प्रतीक है अपितु अंग्रेजों के स्वामित्व एवं नियन्त्रण वाले नस्ली क्लबों और होटलों के लिए चुनौती भी थी।

प्रश्न 36.
फोर्ट सेण्ट जॉर्ज के विषय में आप क्या जानते हैं? क्या यह व्हाइट टाउन का केन्द्र बिन्दु था?
उत्तर:
मद्रास स्थित फोर्ट सेंट जॉर्ज औपनिवेशिक शासन का मुख्य केन्द्र था। यहाँ फोर्ट सेंट व्हाइट टाउन का केन्द्र था। यहाँ अधिकांशतः यूरोपीय रहते थे इसकी दीवारों तथा बुर्जों ने इसे एक खास प्रकार की घेराबन्दी का रूप दे दिया था। किले के भीतर रहने का निर्णय रंग तथा धर्म के आधार पर किया जाता था। भारतीयों के साथ कम्पनी के कर्मचारियों अथवा उनके परिवार के सदस्यों को विवाह की अनुमति नहीं थी। यूरोपीय ईसाई होने के कारण डच तथा पुर्तगालियों को वहाँ रहने की छूट थी। व्हाइट टाउन का केन्द्रबिन्दु इस किले का विकास गोरे विशेषकर अंग्रेजों की जरूरतों एवं सुविधाओं के अनुसार किया गया था।

प्रश्न 37.
लॉर्ड वेलेजली की नगर योजना के विषय में आप क्या जानते हैं?
उत्तर:
लॉर्ड वेलेजली 1798 ई. में बंगाल का गवर्नर जनरल बना। यह अपनी नगर योजना के लिये भी जाना जाता है। वेलेजली ने कलकत्ता में अपने लिये एक गवर्नमेण्ट
हाउस नाम का एक शानदार महल बनवाया था। यह भवन अंग्रेजी सत्ता का प्रतीक था। लॉर्ड वेलेजली कलकत्ता में आ जाने के पश्चात् यहाँ की भीड़-भाड़, अत्यधिक हरियाली, गंदे तालाब तथा साँध से परेशान हो गया। वेलेजली को इन सब तत्त्वों से चिढ़ थी तथा अंग्रेजों का यह भी विचार था कि भारत की उष्णकटिबन्धीय जलवायु बीमारियों तथा महामारियों के अधिक अनुकूल है अतः वेलेजली ने शहर को अधिक स्वास्थ्यपरक बनाने के लिए अधिक खुले स्थान रखने का निर्णय लिया। 1803 ई. में वेलेजली ने नगर नियोजन की आवश्यकता पर एक प्रशासकीय आदेश जारी किया। अतः कहा जा सकता है कि वेलेजली अपनी सहायक सन्धि के कारण जितना कुख्यात है उससे अधिक विख्यात वह स्वास्थ्यपरक नगर नियोजन के लिये है।

प्रश्न 38.
इमारतें और स्थापत्य शैलियाँ क्या बताती हैं?
उत्तर:
स्थापत्य शैलियों से अपने समय के सौन्दर्यात्मक आदर्शों और उनमें निहित विविधताओं का पता चलता है। इमारतें उन लोगों की सोच और नजर के बारे में भी बताती हैं जो उन्हें बना रहे थे इमारतों के जरिये सभी शासक अपनी ताकत को अभिव्यक्त करना चाहते थे। स्थापत्य शैलियों से केवल प्रचलित रुचियों का ही पता नहीं चलता, वे उनको बदलती भी हैं। वे नई शैलियों को लोकप्रियता प्रदान करती हैं और संस्कृति की रूपरेखा तय करती हैं।

प्रश्न 39.
‘बंगला’ भवन निर्माण शैली में किस बात का द्योतक है? इसकी रूपरेखा स्पष्ट कीजिए।
उत्तर:
अंग्रेजों ने अपनी जरूरतों के मुताबिक भवन निर्माण में भारतीय शैलियों को भी अपना लिया था। ‘बंगला’ इसका स्पष्ट उदाहरण है बंगला बम्बई और पूरे देश में सरकारी अफसरों के लिए बनाए जाने वाला भवन था। औपनिवेशिक बंगला एक बड़ी जमीन पर बना होता था। इसमें परम्परागत ढलवाँ छत होती थी और चारों तरफ बरामदा होता था। बंगले के परिसर में घरेलू नौकरों के लिए अलग से क्वार्टर होते थे।

प्रश्न 40.
पूर्व औपनिवेशिक काल के शहरी केन्द्रों की प्रमुख विशेषताएँ क्या थीं?
उत्तर:
(1) ये शहर जनसंख्या के केन्द्रीकरण, अपने विशाल भवनों तथा अपनी शाही शोभा और समृद्धि के लिए प्रसिद्ध थे।
(2) मनसबदार और जागीरदार सामान्यतः इन शहरों में अपने आवास रखते थे।
(3) इन शहरी केन्द्रों में सम्राट और कुलीन वर्ग की ‘उपस्थिति के कारण, यहाँ शिल्पकार, राजकोष, सम्राट का किलेबन्द महल होता था तथा नगर एक दीवार से घिरा होता था।
(4) नगरों के भीतर उद्यान, मस्जिदें, मन्दिर, मकबरे, महाविद्यालय, बाजार तथा सराय स्थित होती थीं।

प्रश्न 41.
ब्रिटिश भारत में निर्मित भवनों में किन- किन स्थापत्य शैलियों का सम्मिश्रण देखने को मिलता है?
उत्तर:
ब्रिटिश भारत में निर्मित भवनों में ‘नव-गॉधिक शैली’, ‘नवशास्त्रीय’ तथा ‘इण्डोसारसेनिक’ स्थापत्य शैलियों का सम्मिश्रण देखने को मिलता है। ऊँची उठी हुई छलें, नोकदार मेहरावें तथा बारीक साज-सज्जा नव-गॉथिक शैली की विशेषताएँ हैं। बड़े- बड़े स्तम्भों के पीछे रेखागणितीय संरचनाओं का निर्माण नवशास्त्रीय शैली की विशेषताएँ हैं, इण्डोसारसेनिक शैली गुम्बदों, छतरियों, मेहराबों से प्रभावित थी।

प्रश्न 42.
लॉटरी कमेटी क्या थी? इसके अन्तर्गत कलकत्ता के नगर नियोजन के लिए कौन-कौनसे कदम उठाए गए?
उत्तर:
गवर्नर जनरल लार्ड वेलेजली के पश्चात् नगर नियोजन का कार्य सरकार की सहायता से लॉटरी कमेटी ने जारी रखा। लॉटरी कमेटी का यह नाम इसलिए पड़ा कि यह कमेटी नगर सुधार के लिए पैसे की व्यवस्था जनता के बीच लॉटरी बेचकर करती थी।
लॉटरी कमेटी द्वारा नगर नियोजन के लिए उठाए गए कदम –
(i) लॉटरी कमेटी ने कलकत्ता शहर का नया मानचित्र बनाया ताकि कलकत्ता को नया रूप दिया जा सके।
(ii) लॉटरी कमेटी की प्रमुख गतिविधियों में शहर में हिन्दुस्तानी जनसंख्या वाले भाग में सड़कें बनवाना एवं नदी किनारे से अवैध कब्जे हटाना सम्मिलित था।
(iii) कलकत्ता शहर के भारतीय हिस्से को साफ- सुथरा बनाने के लिए कमेटी ने बहुत सी झोंपड़ियों को साफ कर दिया एवं गरीब मजदूरों को वहाँ से बाहर निकाल दिया। उन्हें कलकत्ता के बाहरी किनारे पर निवास हेतु जगह दी गई।

प्रश्न 43.
कौन-कौनसी सरकारी नीतियों ने कलकत्ता में भारतीयों के भीतर उपनिवेशवाद विरोधी एवं राष्ट्रवादी भावनाओं को बढ़ावा दिया?
उत्तर:
19वीं शताब्दी के प्रारम्भ के साथ ही कलकत्ता शहर में सरकारी हस्तक्षेप बहुत अधिक सख्त हो चुका था। वित्त पोषण (फण्डिंग) सहित नगर नियोजन के समस्त आयामों को अंग्रेज सरकार ने अपने हाथों में ले लिया। इस आधार पर और तीव्र गति से शुग्गी-झोंपड़ियों को हटाना प्रारम्भ किया गया। दूसरे क्षेत्रों की अपेक्षा ब्रिटिश आबादी वाले हिस्सों का तेजी से विकास किया जाने लगा। स्वास्थ्यकर एवं अस्वास्थ्यकर के नये विभेद के कारण व्हाइट और ब्लैक टाउन वाले नस्ली विभाजन को और बल मिला। कलकत्ता नगर निगम में मौजूद भारतीय प्रतिनिधियों ने शहर के यूरोपीय आबादी वाले क्षेत्रों के विकास पर आवश्यकता से अधिक ध्यान दिये जाने की आलोचना की। इन सरकारी नीतियों के विरुद्ध जनता के प्रतिरोध ने कलकत्ता में भारतीयों के भीतर उपनिवेशवाद विरोधी एवं राष्ट्रवादी भावनाओं को बढ़ावा दिया।

JAC Class 12 History Important Questions Chapter 12 औपनिवेशिक शहर : नगर-योजना, स्थापत्य

प्रश्न 44.
बम्बई का वाणिज्यिक शहर के रूप में किस प्रकार विकास हुआ? टिप्पणी लिखिए।
उत्तर:
19वीं शताब्दी के अन्त तक भारत का आधार आयात तथा निर्यात वाणिज्य शहर बम्बई से होता था। इस समय व्यापार की मुख्य वस्तु अफीम तथा नील थी। यहाँ से ईस्ट इण्डिया कम्पनी चीन को अफीम का निर्यात किया करती थी। इस व्यापार से शुद्ध भारतीय पूँजीपति वर्ग का निर्माण हुआ। पारसी, मारवाड़ी, कोंकणी, मुसलमान, गुजराती, ईरानी, आर्मेनियाई, यहूदी, बोहरे तथा बनिये इत्यादि यहाँ के मुख्य व्यापारी वर्ग से सम्बन्धित थे 1869 ई. में स्वेज नहर को व्यापार के लिये खोला गया था इससे बम्बई के व्यापारिक सम्बन्ध शेष विश्व के साथ अत्यधिक मजबूत हुए। 19वीं शताब्दी के उत्तरार्द्ध तक बम्बई में भारतीय व्यापारी कॉटन मिल जैसे नवीन उद्योगों में अत्यधिक धन का निवेश कर रहे थे।

प्रश्न 45
औपनिवेशिक शहरों में स्त्रियों के सामाजिक जीवन में आए परिवर्तनों का उल्लेख कीजिए।
उत्तर:
औपनिवेशिक शहरों में महिलाओं के लिए नए अक्सर थे। वे पत्र-पत्रिकाओं, आत्मकथाओं तथा पुस्तकों के माध्यम से स्वयं को अभिव्यक्त कर रही थीं। सार्वजनिक स्थानों पर महिलाओं की उपस्थिति बढ़ रही थी। वे नौकरानी, फैक्ट्री मजदूर, शिक्षिका, रंगकर्मी और फिल्म कलाकार के रूप में शहरों के नये व्यवसायों में प्रविष्ट होने लगीं। परन्तु घर से निकलकर सार्वजनिक स्थानों में जाने वाली महिलाओं का सम्मान नहीं था।

प्रश्न 46.
मद्रास का कौनसा क्षेत्र व्हाइट टाउन कां केन्द्रक बन गया था?
उत्तर:
मद्रास में स्थित फोर्ट सेन्ट जार्ज व्हाइट टाउन .का केन्द्रक बन गया था। यहाँ अधिकतर यूरोपीय लोग रहते ‘थे। किले के अन्दर रहने का निर्णय रंग और धर्म के आधार पर किया जाता था। कम्पनी के लोगों को भारतीयों के साथ विवाह करने की अनुमति नहीं थी। संख्या की दृष्टि से कम होते हुए भी यूरोपीय लोग शासक थे और मद्रास शहर का विकास शहर में रहने वाले गोरे लोगों की आवश्यकता को ध्यान में रखकर किया जा रहा था।

प्रश्न 47.
मद्रास में स्थित ब्लैक टाउन का वर्णन कीजिये।
उत्तर:
मद्रास में ब्लैक टाउन किले के बाहर स्थित था। यहाँ आबादी को सीधी पंक्तियों में बसाया गया था। कुछ समय बाद उत्तर की दिशा में दूर जाकर एक नवा ब्लैक टाउन बसाया गया। इसमें भारतीय बुनकरों, कारीगरों, बिचौलियों, दुभाषियों को रखा गया। इसमें भारतीय लोग रहते थे। यहाँ मंदिर और बाजार के आस-पास आवासीय मकान बनाए गए थे। शहर के बीच से गुजरने वाली आड़ी-टेड़ी संकरी गलियों में अलग-अलग जातियों के मोहल्ले थे।

प्रश्न 48.
कोलकाता में नगर नियोजन को क्यों प्रोत्साहन दिया गया?
उत्तर:
कोलकाता में 1817 में हैजा फैल गया तथा 1896 में प्लेग ने शहर को अपनी चपेट में ले लिया। इस स्थिति में अंग्रेजों ने लोलकाता में नगर नियोजन पर बल दिया। घनी आबादी के इलाकों को अस्वच्छ माना जाता था। इसलिए कामकाजी लोगों की झोंपड़ियों तथा वस्तियों को वहाँ से हटा दिया गया। मजदूरों, फेरीवालों कारागरों और बेरोजगारों को दूर वाले इलाकों में ढकेल दिया गया। आग लगने की आशंका को ध्यान में रखते हुए फँस की झोंपड़ियों को अवैध घोषित कर दिया गया।

प्रश्न 49.
मुम्बई में यूरोपीय शैली की इमारतों का निर्माण क्यों किया गया?
उत्तर:
(1) यूरोपीय शैली की इमारतों में भारत जैसे अपरिचित देश में जाना पहिचाना सा भू-दृश्य रचने की और उपनिवेश में भी पर जैसा महसूस करने की अंग्रेजों की आकांक्षा प्रकट होती थी।
(2) अंग्रेजों की मान्यता थी कि यूरोपीय शैली उनकी श्रेष्ठता, अधिकार और सत्ता का प्रतीक थी
(3) वे सोचते थे कि यूरोपीय शैली में निर्मित इमारतों से अंग्रेज शासकों और भारतीय लोगों के बीच अन्तर साफ दिखाई देगा।

निबन्धात्मक प्रश्न-

प्रश्न 1.
औपनिवेशिक भारत के प्रमुख बन्दरगाहों का वर्णन कीजिये।
उत्तर:
औपनिवेशिक भारत के प्रमुख बन्दरगाह
(1) बन्दरगाहों की किलेबन्दी-अठारहवीं शताब्दी तक भारत में मद्रास, कलकत्ता तथा बम्बई महत्त्वपूर्ण बन्दरगाह बन चुके थे। यहाँ जो बस्तियों बसों, वे चीजों के संग्रह के लिए बड़ी उपयोगी सिद्ध हुई ईस्ट इण्डिया कम्पनी ने इन बस्तियों में अपने कारखाने अर्थात् वाणिज्यिक कार्यालय स्थापित किये। कम्पनी ने सुरक्षा के उद्देश्य से इन बस्तियों की किलाबन्दी की। मद्रास में फोर्ट सेन्ट जार्ज, कलकत्ता में फोर्ट विलियम और बम्बई में फोर्ट नामक किले बनाये गए।

(2) यूरोपीयों और भारतीयों के लिए अलग-अलग बस्तियों की व्यवस्था-यूरोपीय व्यापारी किलों के अन्दर रहते थे, जबकि भारतीय व्यापारी, कारीगर, कामगार आदि इन किलों के बाहर अलग बस्तियों में रहते थे। जिस बस्ती में यूरोपीय लोग रहते थे वह ‘व्हाइट टाउन’ (गोरा शहर ) तथा जिस बस्ती में भारतीय लोग रहते थे, वह ‘ब्लैक टाउन’ (काला शहर) के नाम से पुकारे जाते थे।

(3 ) देहाती एवं दूरस्थ इलाकों का बन्दरगाहों से जुड़ना-उन्नीसवीं शताब्दी के मध्य में रेलवे के विकास ने इन शहरों को शेष भारत से जोड़ दिया। परिणामस्वरूप ऐसे देहाती तथा दूरस्थ इलाके भी इन बन्दरगाहों से जुड़ गए जहाँ से कच्चा माल तथा मजदूर आते थे।

(4) कारखानों की स्थापना- कलकत्ता, बम्बई तथा मद्रास में कारखानों की स्थापना करना भी आसान था। इसका कारण यह था कि कच्चा माल निर्यात के लिए इन शहरों में आता था तथा यहाँ सस्ता श्रम उपलब्ध था। 1850 के दशक के बाद भारतीय व्यापारियों और उद्यमियों ने बम्बई में सूती कपड़े की मिलें स्थापित कीं। कलकत्ता के बाहरी इलाके में यूरोपियन लोगों ने जूट मिलों की स्थापना की। परन्तु इन शहरों की अर्थव्यवस्था मुख्य रूप से फैक्टरी उत्पादन पर आधारित नहीं थी।

JAC Class 12 History Important Questions Chapter 12 औपनिवेशिक शहर : नगर-योजना, स्थापत्य

(5) भारत का औद्योगिक देश न बन पाना 19वीं शताब्दी में भारत में केवल कानपुर तथा जमशेदपुर ही औद्योगिक शहर थे। कानपुर में चमड़े की चीजें तथा ऊनी और सूती कपड़े बनते थे, जबकि जमशेदपुर स्टील उत्पादन के लिए प्रसिद्ध था।
भारत कभी भी एक आधुनिक औद्योगिक देश नहीं बन पाया क्योंकि ब्रिटिश सरकार की पक्षपातपूर्ण नीति ने भारत के औद्योगिक विकास को कभी प्रोत्साहन नहीं दिया। यद्यपि कलकत्ता, बम्बई तथा मद्रास बड़े शहरों के रूप में तो विख्यात हुए, परन्तु इससे औपनिवेशिक भारत की सम्पूर्ण अर्थव्यवस्था में कोई क्रान्तिकारी वृद्धि नहीं हुई।

प्रश्न 2.
सन् 1800 के पश्चात् हमारे देश में शहरीकरण की गति धीमी रही। इसके लिए उत्तरदायी कारणों की व्याख्या कीजिए।
उत्तर:
शहरीकरण- ग्रामीण एवं कृषि करने वाले लोगों का गाँवों से शहरों की ओर अच्छे रोजगार अथवा काम की तलाश में पलायन करना या सामान्य गमनागमन शहरीकरण कहलाता है। यह कस्बों एवं शहरों में कुल जनसंख्या के बढ़ते हुए आनुपातिक सन्तुलन को भी इंगित करता है। कुल जनसंख्या में शहरी जनसंख्या के अनुपात से शहरीकरण के विकास की गति का मापन होता है 1800 ई. के पश्चात् हमारे देश में शहरीकरण की गति धीमी रही। सम्पूर्ण उन्नीसवीं और बीसवीं शताब्दी के पहले दो दशकों तक देश की कुल जनसंख्या में शहरी जनसंख्या का अनुपात लगभग स्थिर रहा। इसमें केवल 3 प्रतिशत को ही बढ़ोत्तरी हुई। सन् 1900 से 1940 के मध्य शहरी जनसंख्या 10 प्रतिशत बढ़कर 13 प्रतिशत हो गई।

शहरीकरण की गति के स्थिर रहने के पीछे निम्नलिखित कारण उत्तरदायी थे-
1. छोटे कस्बों के पास आर्थिक रूप से विकसित होने के पर्याप्त अवसर नहीं थे परन्तु दूसरी तरफ कलकत्ता, बम्बई और मद्रास का विस्तार तेजी से हुआ।
2. कलकता, अम्बई व मद्रास औपनिवेशिक अर्थव्यवस्था के केन्द्र होने के कारण भारतीय सूती कपड़े जैसे निर्यात होने वाले उत्पादों के संग्रहण केन्द्र थे। लेकिन इंग्लैण्ड में हुई औद्योगिक क्रान्ति के बाद इस प्रवाह की दिशा परिवर्तित हो गई। भारत से अब तैयार माल की अपेक्षा कच्चे माल का निर्यात होने लगा।

3. 1853 ई. में औपनिवेशिक सरकार ने भारत में रेलवे की शुरुआत की, इसने भारतीय शहरों को पूर्ण रूप से परिवर्तित कर दिया। प्रत्येक रेलवे स्टेशन कच्चे माल के संग्रह और आयातित वस्तुओं के वितरण का केन्द्र बन गया। उदाहरण के लिए गंगा के किनारे स्थित मिर्जापुर दक्कन से आने वाली कंपास एवं सूती वस्त्रों के संग्रह का केन्द्र था जो बम्बई तक जाने वाली रेलवे लाइन के निर्माण के पश्चात् अपनी पुरानी पहचान को खोने लगा था। भारत में रेलवे नेटवर्क के विस्तार के पश्चात् रेलवे वर्कशॉप्स और रेलवे कॉलोनियों की स्थापना होना भी प्रारम्भ हो गया। फलस्वरूप जमालपुर, बरेली व वाल्टेयर जैसे रेलवे नगरों का जन्म हुआ।

प्रश्न 3.
अंग्रेजों ने हिल स्टेशनों की स्थापना क्यों की थी?
अथवा
हिल स्टेशनों की अंग्रेजों के लिए क्या उपयोगिता थी?
उत्तर:
हिल स्टेशनों की स्थापना का प्रमुख उद्देश्य हिल स्टेशनों की स्थापना और बसावट का सम्बन्ध सबसे पहले ब्रिटिश सेना की जरूरतों से था। सिमला (शिमला) की स्थापना गुरखा युद्ध (1815-16) के दौरान, माउंट आबू की स्थापना अंग्रेज-मराठा युद्ध (1818) के कारण की गई तथा दार्जिलिंग को 1835 में सिक्किम के राजाओं से छीना गया। ये हिल स्टेशन फौजियों को ठहराने, सरहद की चौकसी करने और दुश्मन के खिलाफ हमला बोलने की दृष्टि से महत्त्वपूर्ण स्थान थे।

हिल स्टेशनों की उपयोगिता-यूरोपियनों के लिए हिल स्टेशन अग्रलिखित कारणों से उपयोगी थे-
(1) स्वास्थ्यवर्द्धक तथा ठण्डी जलवायु-भारतीय पहाड़ों की मृदु और ठण्डी जलवायु को फायदे की चीज माना जाता था, खासतौर से अंग्रेज गर्मियों के मौसम को बीमारी पैदा करने वाला मानते थे। उन्हें गर्मियों के कारण हैजा व मलेरिया की सबसे ज्यादा आशंका रहती थी।

(2) सेना की सुरक्षा – सेना की भारी भरकम मौजूदगी के कारण ये स्थान पहाड़ियों में एक छावनी के रूप में बदल गये। हिल स्टेशनों को सेनीटोरियम के रूप में भी विकसित किया गया, जहाँ सिपाही विश्राम करने व इलाज कराने के लिए जाते थे।

(3) यूरोप की जलवायु से मिलती-जुलती जलवायु हिल स्टेशनों की जलवायु यूरोप की ठण्डी जलवायु से मिलती-जुलती थी, इसलिए नए शासकों को वहाँ की जलवायु बहुत पसन्द थी 1864 में वायसराय जॉन लारेंस ने अधिकृत रूप से अपनी काउंसिल शिमला में स्थापित कर दी और इस प्रकार गर्मी में राजधानियाँ बदलने के सिलसिले पर रोक लगा दी। शिमला भारतीय सेना के कमाण्डर इन चीफ का भी अधिकृत आवास बन गया।

(4) अंग्रेजों व यूरोपियन के लिए आदर्श स्थान -हिल स्टेशन ऐसे अंग्रेजों व यूरोपियन के लिए भी आदर्श स्थान थे। अलग-अलग मकानों के बाद एक-दूसरे से सटे विला और बागों के बीच कॉटेज बनाए जाते थे। एंग्लिकन चर्च और शैक्षणिक संस्थान आंग्ल आदर्शों का प्रतिनिधित्व करते थे। सामाजिक दावत, चाय बैठक, पिकनिक, रात्रिभोज मेले, रेस और रंगमंच जैसी घटनाओं के रूप में यूरोपियों का सामाजिक जीवन भी एक खास किस्म का था।

(5) पर्वतीय सैरगाहें रेलवे के आने से पर्वतीय सैरगाहें बहुत तरह के लोगों की पहुँच में आ गई। उच्च व मध्यम वर्गीय लोग, महाराजा, वकील और व्यापारी सैर- सपाटे के लिए वहाँ जाने लये।

(6) औपनिवेशिक अर्थव्यवस्था के लिए महत्त्वपूर्ण हिल स्टेशन औपनिवेशिक अर्थव्यवस्था के लिए भी महत्त्वपूर्ण केन्द्र थे। पास के इलाकों में चाय और काफी के बागानों की स्थापना से मैदानी इलाकों से बड़ी संख्या में मजदूर वहाँ रोजगार हेतु आने लगे ।

प्रश्न 4.
कलकत्ता (वर्तमान में कोलकाता) शहर के नगर नियोजन पर एक लघु निबन्ध लिखिए।
अथवा
कलकत्ता नगर के विकास में अंग्रेजों की भूमिका का वर्णन कीजिये।
अथवा
औपनिवेशिक काल में कलकत्ता में नगर नियोजन के इतिहास की रूपरेखा प्रस्तुत कीजिये ।
उत्तर:
कलकत्ता में नगर नियोजन का विकास क्रम कलकत्ता में नगर नियोजन के विकास क्रम को अग्रलिखित बिन्दुओं के अन्तर्गत स्पष्ट किया गया है-
(1) फोर्ट विलियम और मैदान का निर्माण तथा मैदान के किनारे आवासीय इमारतें बनाना- कलकत्ता को- सुतानाती, कोलकाता और गोविन्दपुर- इन तीन गाँवों को मिलाकर बनाया गया था। कम्पनी ने गोविन्दपुर गाँव की जमीन को साफ करने के लिए वहाँ के व्यापारियों और बुनकरों को हटवा दिया। फोर्ट विलियम के इर्द-गिर्द एक विशाल खाली जगह छोड़ दी गई, जिसे स्थानीय लोग मैदान या ‘गारेर मठ’ कहने लगे।

(2) गवर्नमेंट हाउस का निर्माण- 1798 में गवर्नर जनरल लार्ड वेलेजली ने कलकत्ता में अपने लिए ‘गवर्नमेंट हाउस’ के नाम से एक महल बनवाया। यह इमारत अंग्रेजी सत्ता का प्रतीक थी।

JAC Class 12 History Important Questions Chapter 12 औपनिवेशिक शहर : नगर-योजना, स्थापत्य

(3) जन स्वास्थ्य की दृष्टि से नगर नियोजन की आवश्यकता पर बल वेलेजली ने हिन्दुस्तानी आबादी वाले भीड़भाड़ भरे गन्दे तालाबों और निकासी की खस्ता हालत को देखते हुए यह माना कि ऐसी अस्वास्थ्यकर बस्तियों से बीमारियाँ फैलती हैं। फलतः वेलेजली ने 1803 में नगर नियोजन की आवश्यकता पर एक प्रशासकीय आदेश जारी किया। बहुत सारे बाजारों, घाटों, कब्रिस्तानों और चर्मशोधन इकाइयों को हटा दिया गया। इस प्रकार ‘जन-स्वास्थ्य’ शहरों की सफाई और नगर नियोजन परियोजनाओं का मुख्य विचार बन गया।

(4) लॉटरी कमेटी द्वारा नगर नियोजन कार्य को गति प्रदान करना – वेलेजली के जाने के बाद नगर नियोजन का काम सरकार की मदद से लॉटरी कमेटी ने जारी रखा। लॉटरी कमेटी ने शहर का एक नक्शा बनवाया, जिससे कलकत्ता की पूरी तस्वीर सामने आ सके। इसके अतिरिक्त कमेटी ने शहर के हिन्दुस्तानी हिस्से में सड़क निर्माण कराया, नदी किनारे से ‘अवैध कब्जे हटाये तथा बहुत सारी झोंपड़ियों को साफ कर दिया और गरीब लोगों को वहाँ से हटाकर कलकत्ता के बाहरी किनारे पर जगह दी गई।

(5) महामारी की आशंका से नगर नियोजन कार्य में तीव्र गति – अगले कुछ दशकों में प्लेग, हैजा आदि महामारियों की आशंका से नगर नियोजन की अवधारणा को बल मिला। कलकत्ता को और अधिक स्वास्थ्यकर बनाने के लिए कामकाजी लोगों की झोंपड़ियों या बस्तियों को तेजी से हटाया गया और यहाँ के गरीब मजदूर वाशिंदों को पुनः दूर वाले इलाकों में ढकेल दिया गया। फूँस की झोंपड़ियों को अवैध घोषित कर दिया गया।

(6) नगर नियोजन के सारे आयामों को सरकार द्वारा अपने हाथ में लेना 19वीं सदी में और ज्यादा तेजी से झुग्गियों को हटाया गया तथा दूसरे इलाकों की कीमत पर ब्रिटिश आबादी वाले हिस्सों को तेजी से विकसित किया गया।

(7) ह्वाइट और ब्लैक टाउन वाले नस्ली विभाजन को बढ़ावा-‘ स्वास्थ्यकर’ और ‘अस्वास्थ्यकर’ के नये विभेद के सहारे ‘डाइट’ और ‘ब्लैक’ टाउन वाले नस्ली विभाजन को और बल मिला तथा शहर के यूरोपीय आबादी वाले इलाकों के विकास पर आवश्यकता से अधिक ध्यान दिया गया।

प्रश्न 5.
तर्क सहित सिद्ध कीजिए कि औपनिवेशिक शहरों का सामाजिक जीवन वर्तमान शहरों में भी दिखाई पड़ता है।
उत्तर:
औपनिवेशिक शहरों का सामाजिक जीवन वर्तमान शहरों में भी दिखाई देता है। इस बात के समर्थन में निम्नलिखित तर्क प्रस्तुत हैं-
(i) वर्गभेद – औपनिवेशिक शहरों में वर्गभेद स्पष्ट दिखाई देता था। एक ओर व्हाइट टाउन थे जहाँ गोरे लोग ही रह सकते थे दूसरी ओर ब्लैक टाउन में मात्र भारतीय ही रहते थे। व्हाइट टाउन में समस्त सुविधाएँ थीं, वहीं ब्लैक टाउन आवश्यक सुविधाओं एवं स्वच्छता से विहीन थे। यही स्थिति वर्तमान शहरों में देखने को मिल रही है। धनवान लोगों की कालोनियों में पर्याप्त नागरिक सुविधाएँ व स्वच्छता देखने को मिलती है।

(ii) यातायात के साधनों का विकास औपनिवेशिक शहरों में यातायात के साधनों का पर्याप्त विकास हुआ जिस कारण लोग शहर के केन्द्र से दूर जाकर भी बस सकते थे। वर्तमान शहरों में भी यही स्थिति देखने को मिलती है। लोग यातायात के साधनों के विकास के कारण शहर के केन्द्र से दूर जाकर बस रहे हैं।

(ii) मनोरंजन तथा मिलने-जुलने के नये सार्वजनिक स्थल औपनिवेशिक शहरों की तरह वर्तमान शहरों में टाउन हॉल सार्वजनिक पार्क, रंगशाला एवं सिनेमा हॉल जैसे सार्वजनिक स्थानों का निर्माण हुआ है, जिससे शहरों में लोगों को मिलने-जुलने तथा मनोरंजन के नये अवसर मिलने लगे हैं। शहरों में नये सामाजिक समूह बने हैं।

(iv) मध्यम वर्ग का विस्तार औपनिवेशिक शहरों की तरह वर्तमान में सभी वर्गों के लोग शहरों में आने लगे हैं। क्लकों, शिक्षकों, वकीलों, डॉक्टरों, इंजीनियरों, एकाउण्टेन्ट्स की माँग बढ़ने लगी है परिणामस्वरूप मध्यम वर्ग बढ़ता जा रहा है। उनके पास स्कूल, कॉलेज तथा लाइब्रेरी जैसे नये शिक्षा केन्द्रों तक अच्छी पहुँच है।

(v) महिलाओं की स्थिति में परिवर्तन औपनिवेशिक शहरों में महिलाओं के लिए नये अवसर थे। वर्तमान शहरों में भी महिलाओं के लिए नये अवसर मौजूद हैं। आज महिलाएँ घर की चारदीवारी से निकलकर समाचार पत्र, पत्र-पत्रिकाओं और पुस्तकों के द्वारा स्वयं को अभिव्यक्त कर रही हैं।

(vi) मेहनतकश, गरीबों अथवा कामगारों में वृद्धि- औपनिवेशिक शहरों की तरह वर्तमान शहरों में भी ग्रामीण क्षेत्रों से लोग रोजगार की तलाश में लगातार शहरों की ओर आ रहे हैं।

प्रश्न 6.
मद्रास शहर के नगर नियोजन की विवेचना कीजिए।
अथवा
मद्रास में बसावट और पृथक्करण पर एक लघु निबन्ध लिखिए।
उत्तर:
स्थापना- अंग्रेज व्यापारियों ने 1639 में एक व्यापारिक चौकी मद्रासपट्म में स्थापित की फ्रेंच इंस्ट इण्डिया कम्पनी के साथ प्रतिद्वंद्विता के कारण (1746- 63) अंग्रेजों को मद्रास की किलेबन्दी करनी पड़ी।
(1 ) फोर्ट सेंट जार्ज एवं हाइट टाउन का निर्माण- अंग्रेजों ने मद्रास में अपना किला बनाया जो ‘फोर्ट सेन्ट जार्ज’ कहलाता था। इसमें ज्यादातर यूरोपीय रहते थे। मद्रास शहर का विकास शहर में रहने वाले थोड़े से गोरों की जरूरतों और सुविधाओं के हिसाब से किया जा रहा।

(2) ब्लैक टाउन का विकास ब्लैक टाउन को किले के बाहर बसाया गया। इस आबादी को भी सीधी कतारों में बसाया गया जो कि औपनिवेशिक नगरों की खास विशेषता थी नये ब्लैक टाउन में बुनकरों, कारीगरों, बिचौलियों और दुभाषियों को बसाया गया, जो कम्पनी के व्यापार में महत्त्वपूर्ण भूमिका निभाते थे। वहाँ मन्दिर और बाजार के आसपास रिहायशी मकान बनाए गए। आड़ी-टेढ़ी गलियों में अलग- अलग जातियों के मोहल्ले थे। चिन्ताद्रीपेठ इलाका केवल बुनकरों के लिए था। वाशरमेन पेठ में रंगसाज और धोबी रहते थे। दुबाश एजेन्ट और व्यापारी के रूप में कार्य करते थे, इसमें सम्पन्न लोग थे। ये लोग भारतीयों एवं यूरोपियनों के बीच मध्यस्थ की भूमिका निभाते थे।

(3) स्थानीय वाशिंदे व नौकरी-शुरू में कम्पनी में कार्य करने वालों में लगभग सारे वेल्लालार होते थे। वह एक स्थानीय ग्रामीण जाति थी। ब्राह्मण भी इसी तरह के पदों के लिए जोर लगाने लगे। तेलुगू कोमाटी समुदाय एक शक्तिशाली व्यावसायिक समूह था पेरियार और बनियार गरीब कामगार वर्ग था माइलापुर और ट्रिप्लीकेन हिन्दू धार्मिक केन्द्र थे ट्रिप्लीकेन में बड़ी संख्या में मुस्लिम आबादी रहती थी सानधोम और वहाँ का चर्च रोमन कैथोलिक समुदाय का केन्द्र था।

(4) यूरोपीय लोगों द्वारा गार्डन हाउसेज का निर्माण- जैसे-जैसे अंग्रेजी सत्ता मजबूत होती गई, यूरोपीय निवासी किले के बाहर जाकर गार्डन हाउसेज का निर्माण करने लगे।

(5) सम्पन्न भारतीयों द्वारा उपशेहरी इलाकों का निर्माण धीरे-धीरे सम्पन्न भारतीय भी अंग्रेजों की तरह रहने लगे थे। परिणामस्वरूप मद्रास के इर्द-गिर्द स्थित गाँवों की जगह बहुत सारे नये उपशहरी इलाकों ने ले ली।

(6) गरीब लोगों का काम के नजदीक वाले गाँवों में निवास- गरीब लोग अपने काम की जगह से नजदीक पड़ने वाले गाँवों में बस गये। बढ़ते शहरीकरण के कारण इन गाँवों के बीच वाले इलाके शहर के भीतर आ गए। इस तरह मद्रास एक अर्द्धग्रामीण-सा शहर दिखने लगा।

प्रश्न 7.
बम्बई एक आधुनिक नगर और भारत की वाणिज्यिक राजधानी कैसे बना? इस पर प्रकाश डालिए।
अथवा
बम्बई शहर के नगर नियोजन की विशेषताओं की विवेचना कीजिए।
अथवा
मुम्बई का उदाहरण देते हुए स्पष्ट कीजिये कि अंग्रेजों ने नगर नियोजन के माध्यम से अपने औपनिवेशिक सपने को कैसे पूरा किया?
अथवा
बम्बई नगर नियोजन तथा भवन निर्माण के मुख्य चरणों की व्याख्या कीजिये मुख्य रूप से नवशास्त्रीय ( नियोक्लासिकल) शैली में बनी इमारतों का उल्लेख कीजिये।
उत्तर:
प्रारम्भ में बम्बई सात टापुओं का इलाका था। जैसे-जैसे आबादी बढ़ी, इन टापुओं को एक-दूसरे से जोड़ दिया गया और इन टापुओं के जुड़ने पर एक विशाल शहर अस्तित्व में आया।

(1) औपनिवेशिक भारत की वाणिज्यिक राजधानी अम्बई औपनिवेशिक भारत की वाणिज्यिक राजधानी थी। पश्चिमी तट पर एक प्रमुख बन्दरगाह होने के नाते यह अन्तर्राष्ट्रीय व्यापार का केन्द्र था। 19वीं सदी के अन्त तक भारत का आधा आयात-निर्यात बम्बई से ही होता था। इस व्यापार की एक महत्त्वपूर्ण वस्तु अफीम थी। अफीम के व्यापार में ईस्ट इण्डिया कम्पनी के साथ भारतीय व्यापारी और बिचौलियों की भी प्रमुख भूमिका थी। इससे भारतीय पूँजीपति वर्ग का विकास हुआ। बाद में अमेरिकी गृहयुद्ध के कारण भारतीय कपास की बढ़ती माँग और बढ़ती कीमतों तथा स्वेज नहर के खुलने से बम्बई का वाणिज्यिक विकास हुआ और बम्बई को भारत का ‘सरताज शहर’ घोषित किया गया।

(2) विशाल इमारतें तथा उनका स्थापत्य अंग्रेजों ने अनेक भव्य और विशाल इमारतों का निर्माण कराया। इनकी स्थापत्य शैली यूरोपीय शैली पर आधारित थी। धीरे-धीरे भारतीयों ने भी यूरोपीय स्थापत्य शैली को अपना लिया। अंग्रेजों ने अनेक बंगले बनवाये।
(i) अंग्रेजों ने नव-गॉथिक शैली में इमारतों का निर्माण कराया। विक्टोरिया टर्मिनस रेलवे स्टेशन इस शैलीका बेहतरीन उदाहरण है। इसके अतिरिक्त बम्बई विश्वविद्यालय, सचिवालय और उच्च न्यायालय जैसी भव्य इमारतों का निर्माण कराया गया।
(ii) नवशास्त्रीय शैली में ‘एल्फिंस्टन सर्कल’ का निर्माण किया गया।
(iii) बीसवीं सदी के प्रारम्भ में भारतीय और यूरोपीय 4 शैली को मिलाकर एक नयी शैली का विकास हुआ जो ‘इण्डोसारसेनिक’ कहलाती थी यह शैली गुम्बदों, छतरियों, जालियों और मेहराबों से प्रभावित थी गुजराती शैली में बने गेट वे ऑफ इण्डिया तथा होटल ताज प्रमुख इमारतें हैं।

JAC Class 12 History Important Questions Chapter 12 औपनिवेशिक शहर : नगर-योजना, स्थापत्य

(3) चाल – बम्बई शहर में जगह की कमी तथा भीड़-भाड़ की वजह से विशेष प्रकार की इमारतें भी बनाई गई, जिन्हें ‘चाल’ कहा जाता था। ये बहुमंजिला इमारतें होती थीं, जिनमें एक-एक कमरेवाली आवासीय इकाइयाँ बनाई जाती थीं।

प्रश्न 8.
औपनिवेशिक काल में भारत में सार्वजनिक भवनों के निर्माण के लिए कौन-कौनसी स्थापत्य शैलियों का प्रयोग किया गया?
अथवा
ब्रिटिश काल में इण्डो-सारसेनिक स्थापत्य कला की दो विशेषताएँ बताइये।
उत्तर:
सार्वजनिक भवनों के लिए स्थापत्य शैलियाँ औपनिवेशिक काल में सार्वजनिक भवनों के लिए मोटे तौर पर तीन स्थापत्य शैलियों का प्रयोग किया गया।
(1) नवशास्त्रीय या नियोक्लासिकल शैली-बड़े- बड़े स्तम्भों के पीछे रेखागणितीय संरचनाओं का निर्माण इस शैली की विशेषता थी। भारत में ब्रिटिश साम्राज्य के लिए उसे खासतौर से अनुकूल माना जाता था। अंग्रेजों को लगता था कि जिस शैली में शाही रोम की भव्यता दिखाई देती थी, उसे शाही भारत के वैभव की अभिव्यक्ति के लिए भी प्रयोग किया जा सकता है क्योंकि यह शैली मूल रूप से प्राचीन रोम की भवन निर्माण शैली से निकली थी और इसे यूरोपीय पुनर्जागरण के दौरान पुनर्जीवित, संशोधित और लोकप्रिय किया गया था।

इस स्थापत्य शैली के भूमध्यसागरीय उद्गम के कारण उसे उष्णकटिबंधीय मौसम के अनुकूल भी माना गया। 1833 में बम्बई का ‘टाउन हॉल’ इसी शैली के अनुसार बनाया गया था। 1860 में व्यावसायिक इमारतों के समूह को ‘एल्फिंस्टन सर्कल’ कहा जाता था। यह इमारत इटली की इमारतों से प्रेरित थी। इसमें पहली मंजिल पर ढँके हुए तोरण पथ का रचनात्मक ढंग से इस्तेमाल किया गया। दुकानदारों व पैदल चलने वालों को तेज धूप और बरसात से बचाने के लिए यह सुधार काफी उपयोगी था ।

(2) नव-गॉथिक शैली- एक अन्य स्थापत्य शैली जिसका काफी प्रयोग किया गया, वह थी- नव- गाँधिक शैली ऊँची उठी हुई छरों, नोकदार मेहराबें और बारीक साज-सज्जा इस शैली की खासियत थी। इस शैली की इमारतों का जन्म गिरजाघरों से हुआ था, जो मध्यकाल में यूरोप में काफी बनाए गए थे। 19वीं सदी के मध्य इंग्लैण्ड में इसे दुबारा अपनाया गया बम्बई सचिवालय, बम्बई विश्वविद्यालय और उच्च न्यायालय की इमारतें इसी शैली में बनाई गई नव-गॉथिक शैली का सबसे बेहतरीन उदाहरण ‘विक्टोरिया टर्मिनस’ है, जो ग्रेट इंडियन पेनिन्स्युलर रेलवे कम्पनी का स्टेशन और मुख्यालय हुआ करता था।

(3) इंडो-सारासेनिक शैली 20वीं सदी के प्रारम्भ में एक नयी मिश्रित स्थापत्य शैली विकसित हुई, जिसमें भारतीय और यूरोपीय, दोनों तरह की शैलियों के तत्व थे। इस शैली को ‘इंडो-सारासेनिक शैली’ का नाम दिया गया था। 1911 में बना ‘गेट वे ऑफ इण्डिया’ परम्परागत गुजराती शैली का प्रसिद्ध उदाहरण था उद्योगपति जमशेदजी टाटा ने इसी शैली में ‘ताजमहल होटल’ बनवाया था। बम्बई के ज्यादातर ‘भारतीय’ इलाकों में सजावट एवं भवन निर्माण और साज-सज्जा में इसी शैली का बोलबाला था।

प्रश्न 9.
औपनिवेशिक भारत की स्थापत्य शैलियों से किन तथ्यों के बारे में जानकारी मिलती है?
उत्तर:
औपनिवेशिक भारत की स्थापत्य शैलियों से विभिन्न तथ्यों की जानकारी
औपनिवेशिक भारत की स्थापत्य शैलियों से निम्नलिखित तथ्यों के बारे में जानकारी मिलती है-
(1) सौन्दर्यात्मक आदर्शों एवं उनमें निहित विविधताओं का बोध होना स्थापत्य शैलियों से अपने समय के सौन्दर्यात्मक आदर्शों और उनमें निहित विविधताओं का पता चलता है।

(2) भवन-निर्माताओं के दृष्टिकोण की प्रतीक ये इमारतें उन लोगों की सोच और दृष्टिकोण के बारे में भी बताती हैं, जो उन्हें बना रहे थे इमारतों के जरिये शासक अपनी ताकत का इजहार करना चाहते थे तथा इन इमारतों से उस समय की सत्ता को किस रूप में देखा जा रहा था, इसका भी ज्ञान होता है। इस प्रकार एक विशिष्ट युग की स्थापत्य शैली को देखकर हम यह समझ सकते हैं कि उस समय सत्ता को किस तरह देखा जा रहा था।

(3) प्रचलित रुचियों की जानकारी एवं नवीन शैलियों को लोकप्रियता प्रदान करना- स्थापत्य शैलियों से प्रचलित रुचियों का तो पता लगता ही है, साथ ही संजीव पास बुक्स यह भी पता लगता है कि इन शैलियों ने उन रुचियों को बदलने में भी महत्त्वपूर्ण भूमिका निभाई है तथा संस्कृति की रूपरेखा तय की है, जैसे बहुत सारे भारतीय भी यूरोपीय स्थापत्य शैलियों को आधुनिकता व सभ्यता का प्रतीक मानते हुए उन्हें अपनाने लगे थे तथा बहुतों ने उनके आधुनिक तत्वों को स्थानीय परम्पराओं के तत्वों में समाहित कर दिया। उन्नीसवीं शताब्दी के अन्त से हमें औपनिवेशिक आदर्शों से भिन्न क्षेत्रीय एवं राष्ट्रीय अभिरुचियों को परिभाषित करने के प्रयास दिखाई देते हैं। इस तरह स्थापत्य शैलियों को देखकर हम इस बात को भी समझ सकते हैं कि शाही और राष्ट्रीय तथा राष्ट्रीय और क्षेत्रीय के बीच सांस्कृतिक टकराव और राजनीतिक खींचतान किस तरह शक्ल ले रही थी।

प्रश्न 10.
औपनिवेशिक शासन किस प्रकार बेहिसाब आँकड़ों और जानकारियों के संग्रह पर आधारित था?
अथवा
औपनिवेशिक शहरों के अध्ययन में सहायक तत्त्वों की विवेचना कीजिये।
उत्तर:
औपनिवेशिक शासन में संग्रहित आँकड़े व जानकारियाँ ( औपनिवेशिक शहरों के अध्ययन में सहायक तत्त्व )
औपनिवेशिक शासन बेहिसाब आँकड़ों और जानकारियों के संग्रह पर आधारित था। यथा-
(1) व्यापारिक गतिविधियों का विस्तृत ब्यौरा रखना-अंग्रेजों ने अपने व्यावसायिक मामलों को चलाने के लिए व्यापारिक गतिविधियों का विस्तृत ब्यौरा रखा था। (2) शहरों का नियमित सर्वेक्षण तथा उनके सांख्यिकीय आँकड़े एकत्रित करना – वे बढ़ते शहरों में जीवन की गति और दिशा पर नजर रखने के लिए नियमित रूप से सर्वेक्षण करते थे। वे सांख्यिकीय आँकड़े इकट्ठा करते थे और विभिन्न प्रकार की सरकारी रिपोर्ट प्रकाशित करते थे।

(3) मानचित्र तैयार करना- प्रारम्भिक वर्षों में ही औपनिवेशिक सरकार ने मानचित्र तैयार करने पर ध्यान दिया। उसका मानना था कि किसी जगह की बनावट और भू-दृश्य को समझने के लिए नक्शे जरूरी होते हैं। इस जानकारी के सहारे वे इलाके पर ज्यादा बेहतर नियंत्रण कायम कर सकते थे। जब शहर बढ़ने लगे तो न केवल उनकी विकास की योजना तैयार करने के लिए बल्कि व्यवसाय को विकसित करने और अपनी सत्ता को मजबूत करने के लिए भी नक्शे बनाये जाने लगे। शहरों के नक्शों से हमें उस स्थान पर पहाड़ियों, नदियों व हरियाली का पता चलता है। ये सारी चीजें रक्षा सम्बन्धी उद्देश्यों के लिए योजना तैयार करने में बहुत काम आती हैं। इसके अतिरिक्त पाटों की जगहों, मकानों की समनता तथा गुणवत्ता तथा सड़कों की स्थिति आदि से इलाके की व्यावसायिक सम्भावनाओं का पता लगाने और कराधान की रणनीति बनाने में मदद मिलती थी।

(4) नगर निगमों की गतिविधियों से उत्पन्न रिकाईस – 19वीं सदी में शहरों के रख-रखाव के लिए पैसा इकट्ठा करने के लिए आंशिक लोक प्रतिनिधित्व से लैस नगर निगम जैसी संस्था की स्थापना की गई। नगर- निगमों की गतिविधियों से नए तरह के रिकार्ड्स पैदा हुए, जिन्हें नगरपालिका रिकार्ड रूम में सम्भाल कर रखा जाने
लगा।

(5) जनगणना आँकड़े शहरों के फैलाव पर नजर रखने के लिए नियमित रूप से लोगों की गिनती की जाती थी। 19वीं सदी के मध्य तक विभिन्न क्षेत्रों में कई जगह स्थानीय स्तर पर जनगणना की जा चुकी थी। अखिल भारतीय जनगणना का पहला प्रयास 1872 में किया गया। इसके बाद, 1881 से हर दस साल में जनगणना एक नियमित व्यवस्था बन गई। भारत में शहरीकरण का अध्ययन करने के लिए जनगणना से निकले आँकड़े एक बहुमूल्य स्रोत हैं। इस प्रकार जनगणना, नगरपालिका जैसे संस्थानों के सर्वेक्षण, मानचित्रों और अन्य रिकार्डों के सहारे औपनिवेशिक शहरों का पुराने शहरों के मुकाबले ज्यादा विस्तार से अध्ययन किया जा सकता है।

JAC Class 12 History Important Questions Chapter 12 औपनिवेशिक शहर : नगर-योजना, स्थापत्य

प्रश्न 11.
औपनिवेशिक काल में भारत में जनगणना की प्रक्रिया में क्या भ्रम थे? जनगणनाओं के सावधानी से अध्ययन करने पर क्या दिलचस्प रुझान सामने आते हैं? उत्तर- औपनिवेशिक भारत में जनगणना से सम्बन्धित भ्रम यद्यपि भारत में शहरीकरण का अध्ययन करने के लिए जनगणना से निकले आँकड़े एक बहुमूल्य स्रोत हैं, तथापि इस प्रक्रिया में भी कई भ्रम थे यथा-

(1) वर्गीकरण सम्बन्धी भ्रम- जनगणना आयुक्तों ने आबादी के विभिन्न तबकों का वर्गीकरण करने के लिए अलग-अलग श्रेणियाँ बना दी थीं। कई बार यह वर्गीकरण निहायत अतार्किक होता था और लोगों की परिवर्तनशील तथा परस्पर काटती पहचानों को पूरी तरह नहीं पकड़ पाता था।

(2) स्वयं लोगों द्वारा सहयोग न करना- प्राय: लोग स्वयं भी इस प्रक्रिया में सहयोग करने से इनकार कर देते थे या जनगणना आयुक्तों को गलत जवाब दे देते थे। काफी समय तक वे जनगणना कार्यों को सन्देह की दृष्टि से देखते रहे। उन्हें लगता था कि सरकार नये टैक्स लागू करने के लिए जाँच करवा रही है।

(3) औरतों के बारे में जानकारी देने में हिचकिचाना – ऊँची जाति के लोग अपने घर की औरतों के बारे में पूरी जानकारी देने से हिचकिचाते थे। महिलाओं से अपेक्षा की जाती थी कि वे घर के भीतरी हिस्से में दुनिया से कट कर रहें। उनके बारे में सार्वजनिक जाँच को सही नहीं माना जाता था।

(4) पहचान सम्बन्धी दावों का भ्रामक होना- जनगणना अधिकारियों ने यह भी पाया कि बहुत सारे लोग ऐसी पहचानों का दावा करते थे जो ऊँची हैसियत की मानी जाती थीं। उदाहरण के लिए, शहरों में ऐसे लोग भी थे जो फेरी लगाते थे या काम न होने पर मजदूरी करने लगते थे। इस तरह के बहुत सारे लोग जनगणना कर्मचारियों के सामने स्वयं को प्रायः व्यापारी बताते थे क्योंकि उन्हें मजदूर की तुलना में व्यापारी ज्यादा सम्मानप्रद लगता था।

(5) मृत्युदर और बीमारियों से सम्बन्धित आँकड़ों को इकट्ठा करना लगभग असम्भव था मृत्यु दर और बीमारियों से सम्बन्धित आंकड़ों को इकट्ठा करना भी लगभग असम्भव था। बीमार पड़ने की जानकारी भी लोग प्रायः नहीं देते थे। बहुत बार इलाज भी गैर लाइसेंसी डॉक्टरों से करा लिया जाता था। ऐसे में बीमारी या मौत की घटनाओं का सटीक हिसाब लगाना सम्भव नहीं था। जनगणनाओं का सावधानी से अध्ययन करने पर कुछ दिलचस्प रुझान जनगणनाओं का सावधानी से अध्ययन करने पर निम्नलिखित दिलचस्प रुझान सामने आते हैं-

(1) सन् 1800 के बाद हमारे देश में शहरीकरण की रफ्तार धीमी रही। पूरी 19वीं सदी और 20वीं सदी के पहले दो दशकों तक देश की कुल आबादी में शहरी आबादी का हिस्सा बहुत मामूली तथा स्थिर रहा। यह लगभग 10 प्रतिशत रहा।
(2) 1900 से 1940 के बीच 40 सालों के दरमियान शहरी आबादी 10 प्रतिशत से बढ़कर 13 प्रतिशत हो गई थी।
(3) नए व्यावसायिक एवं प्रशासनिक केन्द्रों के रूप में बम्बई, मद्रास और कलकत्ता शहर पनपे लेकिन दूसरे तत्कालीन शहर कमजोर भी हुए।

प्रश्न 5.
” भारत छोड़ो आन्दोलन ब्रिटिश शासन के खिलाफ गाँधीजी का तीसरा बड़ा आन्दोलन था।” व्याख्या कीजिए।
उत्तर:
क्रिप्स मिशन की विफलता के पश्चात् महात्मा गाँधी ने ब्रिटिश शासन के खिलाफ अपना तीसरा बड़ा आन्दोलन छेड़ने का फैसला किया। अगस्त, 1942 ई. में शुरू किए गए इस आन्दोलन को ‘अंग्रेज भारत छोड़ो’ के नाम से जाना गया। भारत छोड़ो आन्दोलन प्रारम्भ करने के कारण-
(i) अंग्रेजों की साम्राज्यवादी नीति- सितम्बर, 1939 में द्वितीय विश्व युद्ध प्रारम्भ हो गया। महात्मा गाँधी व जवाहरलाल नेहरू दोनों ही हिटलर व नात्सियों के आलोचक थे। तदनुरूप उन्होंने फैसला किया कि यदि अंग्रेज बुद्ध समाप्त होने के पश्चात् भारत को स्वतन्त्रता देने पर सहमत हों तो कांग्रेस उनके युद्ध प्रयासों में सहायता दे सकती है। ब्रिटिश सरकार ने कांग्रेस के इस प्रस्ताव को खारिज कर दिया। इस घटनाक्रम ने अंग्रेजी साम्राज्यवादी नीति के विरुद्ध आन्दोलन प्रारम्भ करने हेतु प्रोत्साहित किया।

(ii) क्रिप्स मिशन की असफलता द्वितीय विश्व युद्ध में कांग्रेस व गाँधीजी का समर्थन प्राप्त करने के लिए तत्कालीन ब्रिटिश प्रधानमन्त्री विंस्टन चर्चिल ने अपने एक मन्त्री सर स्टेफर्ड क्रिप्स को भारत भेजा। क्रिप्स के साथ वार्ता में कांग्रेस ने इस बात पर जोर दिया कि यदि धुरी शक्तियों से भारत की रक्षा के लिए ब्रिटिश शासन कांग्रेस का समर्थन चाहता है तो वायसराय को सबसे पहले अपनी कार्यकारी परिषद् में किसी भारतीय को एक रक्षा सदस्य के रूप में नियुक्त करना चाहिए। इसी बात पर वार्ता टूट गयी। क्रिप्स मिशन की विफलता के पश्चात् गाँधीजी ने अंग्रेजों भारत छोड़ो आन्दोलन प्रारम्भ करने का फैसला किया। भारत छोड़ो आन्दोलन का प्रारम्भ-9 अगस्त, 1942 ई. को गाँधीजी के नेतृत्व में भारत छोड़ो आन्दोलन प्रारम्भ हो गया।

अंग्रेजों ने इस आन्दोलन को दबाने के लिए बड़ी कठोरता से काम लिया। कांग्रेस को अवैध घोषित कर दिया गया तथा सभाओं, जुलूसों व समाचार-पत्रों पर कठोर प्रतिबन्ध लगा दिए गए। इसके बावजूद देशभर के युवा कार्यकर्ता हड़तालों एवं तोड़फोड़ की कार्यवाहियों के माध्यम से आन्दोलन चलाते रहे। कांग्रेस में जयप्रकाश नारायण जैसे समाजवादी सदस्य भूमिगत होकर अपनी गतिविधियों को चलाते रहे। आन्दोलन का अन्त-अंग्रेजों ने भारत छोड़ो आन्दोलन के प्रति कठोर रवैया अपनाया फिर भी इस विद्रोह का दमन करने में साल भर से अधिक समय लग गया।

JAC Class 12 History Important Questions Chapter 9 शासक और इतिवृत्त : मुगल दरबार

Jharkhand Board JAC Class 12 History Important Questions Chapter 9 शासक और इतिवृत्त : मुगल दरबार Important Questions and Answers.

JAC Board Class 12 History Important Questions Chapter 9 शासक और इतिवृत्त : मुगल दरबार

बहुविकल्पीय प्रश्न (Multiple Choice Questions)

1. ‘मुगल’ की उत्पत्ति किस शब्द से हुई है –
(अ) महान
(स) मुसलमान
(ब) मंगोल
(द) मोगली
उत्तर:
(ब) मंगोल

2. किस मुगल सम्राट को शेरशाह से पराजित होकर ईरान भागना पड़ा?
(अ) बाबर
(ब) अकबर
(स) हुमायूँ
(द) जहाँगीर
उत्तर:
(स) हुमायूँ

3. मुगल साम्राज्य का संस्थापक था –
(अ) बाबर
(ब) हुमायूँ
(स) अकबर
(द) जहाँगीर
उत्तर:
(अ) बाबर

JAC Class 12 History Important Questions Chapter 9 शासक और इतिवृत्त : मुगल दरबार

4. मुगल राजवंश का अन्तिम सम्राट कौन था?
(अ) मुअज्जम
(ब) फर्रुखसियर
(स) शाह आलम
(द) बहादुरशाह जफर द्वितीय
उत्तर:
(द) बहादुरशाह जफर द्वितीय

5. महाभारत का फारसी में अनुवाद किसके रूप में हुआ –
(अ) अकबरनामा
(ब) शाहीनामा
(स) एमनामा
(द) बाबरनामा
उत्तर:
(स) एमनामा

6. फारसी के हिन्दवी के साथ पारस्परिक सम्पर्क से किस नई भाषा का जन्म हुआ?
(ब) उर्दू
(द) संस्कृत
(अ) अरबी
(स) तुर्की
उत्तर:
(ब) उर्दू

7. नस्तलिक शैली के सबसे अच्छे सुलेखकों में से एक था –
(अ) रजा खाँ
(ब) इब्राहीम खाँ
(स) मन्सूर अली
(द) मुहम्मद हुसैन
उत्तर:
(द) मुहम्मद हुसैन

JAC Class 12 History Important Questions Chapter 9 शासक और इतिवृत्त : मुगल दरबार

8. एशियाटिक सोसाइटी आफ बंगाल की स्थापना किसके द्वारा की गई?
(अ) जॉन मार्शल
(ब) मैकाले
(स) विलियम जोन्स
(द) कनिंघम
उत्तर:
(स) विलियम जोन्स

9. एशियाटिक सोसाइटी ऑफ बंगाल की स्थापना की गई –
(अ) 1784 ई.
(ब) 1794 ई.
(स) 1884 ई.
(द) 1795 ई.
उत्तर:
(अ) 1784 ई.

10. अकबर ने जजिया कर कब समाप्त कर दिया?
(अ) 1573
(ब) 1564
(द) 1579
(स) 1604
उत्तर:
(ब) 1564

11. मुगल पितृ पक्ष से निम्न में से किस तुर्की शासक के वंशज थे?
(अ) तिमूर
(ब) चंगेज खाँ
(स) मंगोल
(द) उपर्युक्त सभी
उत्तर:
(अ) तिमूर

12. रज्मनामा निम्नलिखित में से किस ग्रन्थ का फारसी अनुवाद है-
(अ) रामायण
(ब) लीलावती
(स) पंचतन्त्र
(द) महाभारत
उत्तर:
(द) महाभारत

13. नस्तलिक का सम्बन्ध निम्नलिखित में से किससे है?
(अ) एक ग्रन्थ
(ब) एक कविता
(स) सुलेखन शैली
(द) इनमें से कोई नहीं
उत्तर:
(स) सुलेखन शैली

14. निम्न में से किस मुगल दरबारी इतिहासकार ने चित्रकारी को एक जादुई कला के रूप में वर्णित किया है?
(अ) मुहम्मद हुसैन
(ब) अबुल फजल
(स) अब्दुल हमीद लाहौरी
(द) अब्दुल हसन
उत्तर:
(ब) अबुल फजल

JAC Class 12 History Important Questions Chapter 9 शासक और इतिवृत्त : मुगल दरबार

15. 1560 के दशक में अकबर ने किस इमारत का निर्माण करवाया?
(अ) आगरा का किला
(ब) ताजमहल
(स) फतेहप्हपुर सीकरी
(द) सिकन्दर
उत्तर:
(अ) आगरा का किला

रिक्त स्थानों की पूर्ति कीजिए :
1. जंगल बुक …………… द्वारा लिखी गई है जिससे युवा नायक …………….. का नाम व्युत्पन्न हुआ है।
2. हुमायूँ ने सूरों को …………… में पराजित किया था।
3. अकबर का उत्तराधिकारी …………….. हुआ।
4. …………… मुगल वंश का अन्तिम मुगल शासक था।
5. अकबर, शाहजहाँ आलमगीर की कहानियों पर आधारित इतिवृत्तों के शीर्षक क्रमशः ……………. और …………. है।
6. चगताई तुर्क स्वयं को …………….. के सबसे बड़े पुत्र का वंशज मानते थे।
उत्तर:
1. रुडयार्ड किपलिंग, मोगली
2. 1555
3. जहाँगीर
4. बहादुरशाह जफर द्वितीय
5. अकबरनामा, शाहजहाँनामा, आलमगीरनामा
6. चंगेज खाँ।

अतिलघूत्तरात्मक प्रश्न

प्रश्न 1.
मुगल शासकों में सबसे महान कौन था?
उत्तर:
मुगल शासकों में अकबर सबसे महान था।

प्रश्न 2.
‘सिजदा तथा जमींबोसी ‘ का क्या अर्थ है?
उत्तर:
‘सिजदा’ का अर्थ बादशाह के सामने दंडवत लेटना तथा ‘जमींबोसी’ का अर्थ जमीन चूमना था।

प्रश्न 3.
कौन-सा किला सफावियों और मुगलों के बीच झगड़े का कारण रहा था?
उत्तर:
कन्धार का किला।

प्रश्न 4.
बादशाहनामा के दरबारी दृश्य में अंकित शान्तिपूर्वक चिपटकर बैठे हुए शेर एवं बकरी क्या संदेश देते हैं
उत्तर:
मुगल- राज्य में सबल तथा निर्बल परस्पर सद्भाव से रहते थे।

प्रश्न 5.
मुगल दरबार में प्रचलित अभिवादन के दो तरीकों का उल्लेख कीजिए।
उत्तर:
(1) सिजदा अथवा दण्डवत लेटना
(2) जमीं बरेसी (जमीन चूमना)।

प्रश्न 6.
फतेहपुर सीकरी का बुलन्द दरवाजा क्यों और किसने बनवाया?
उत्तर:
गुजरात में मुगल विजय की बाद में अकबर ने बुलन्द दरवाजा बनवाया।

JAC Class 12 History Important Questions Chapter 9 शासक और इतिवृत्त : मुगल दरबार

प्रश्न 7.
मुगलकाल में पांडुलिपि रचना का मुख्य केन्द्र क्या कहलाता था?
उत्तर:
शाही किताबखाना।

प्रश्न 8.
‘मुगल’ नाम किस शब्द से व्युत्पन्न हुआ है?
उत्तर:
मंगोल।

प्रश्न 9.
मुगल किस मूल के वंशज थे और उनकी मातृ भाषा क्या थी?
उत्तर:
(1) चगताई तुर्क
(2) तुर्की भाषा।

प्रश्न 10.
मुगल शासक अपने को तैमूरी क्यों कहते थी?
उत्तर:
क्योंकि पितृ पक्ष से वे वंशज थे। तुर्की शासक तिमूर के

प्रश्न 11.
बावर मातृपक्ष से किसका सम्बन्धी था?
उत्तर:
चंगेज खाँ का।

प्रश्न 12.
मुगल साम्राज्य का संस्थापक कौन था?
उत्तर:
जहीरुद्दीन बाबर।

प्रश्न 13.
अकबर के तीन योग्य उत्तराधिकारियों के नाम लिखिए।
उत्तर:
(1) जहाँगीर
(2) शाहजहाँ
(3) औरंगजेब।

प्रश्न 14.
अकबर, शाहजहाँ, आलमगीर (औरंगजेब ) की कहानियों पर आधारित इतिवृत्तों के शीर्षक बताइये।
उत्तर:
(1) अकबरनामा
(2) शाहजहाँनामा
(3) आलमगीरनामा।

प्रश्न 15.
संस्कृत के दो ग्रन्थों के नाम लिखिए जिनका फारसी में अनुवाद करवाया गया।
उत्तर:
(1) रामायण
(2) महाभारत

प्रश्न 16.
चुगताई तुर्क कौन थे?
उत्तर:
मुगल

JAC Class 12 History Important Questions Chapter 9 शासक और इतिवृत्त : मुगल दरबार

प्रश्न 17.
अकबर की पसंदीदा सुलेखन शैली कौनसी
उत्तर:
नस्तलिक।

प्रश्न 18.
अबुल फसल के अनुसार चित्रकारी का क्या महत्त्व था?
उत्तर:
चित्रकारी निर्जीव वस्तुओं में भी प्राण फूंक सकती थी।

प्रश्न 19.
ईरान से मुगल दरबार में आने वाले दो प्रसिद्ध चित्रकारों के नाम लिखिए।
उत्तर:
(1) मीर सैय्यद अली
(2) अब्दुस समद।

प्रश्न 20.
दो महत्त्वपूर्ण चित्रित मुगल इतिहासों के नाम लिखिए।
उत्तर:
(1) अकबरनामा
(2) बादशाहनामा।

प्रश्न 21.
एशियाटिक सोसाइटी ऑफ बंगाल की स्थापना का क्या उद्देश्य था?
उत्तर:
भारतीय पांडुलिपियों का सम्पादन, प्रकाशन और अनुवाद करना।

प्रश्न 22.
अबुल फसल के अनुसार बादशाह अपनी प्रजा के किन-किन सत्वों की रक्षा करते थे?
उत्तर:
मुगल सम्राट अपनी प्रजा के निम्न सत्त्वों की रक्षा करते थे-
(1) जीवन (जन),
(2) धन (माल),
(3) सम्मान (नामस)
(4) विश्वास (दीन)।

प्रश्न 23.
इन सवों की रक्षा के बदले में बादशाह अपनी प्रजा से क्या माँग करता था?
उत्तर:
(1) आज्ञापालन
(2) संसाधनों में हिस्से की

प्रश्न 24.
जजिया क्या था?
उत्तर:
जजिया एक कर था, जो गैर-मुसलमानों पर लगाया जाता था।

प्रश्न 25.
अकबर ने फतेहपुर सीकरी से राजधानी को कहाँ स्थानान्तरित किया और कब?
उत्तर:
1585 में अकबर ने राजधानी को लाहौर स्थानान्तरित किया।

प्रश्न 26.
मुगलकाल में दरबारियों के द्वारा बादशाह के अभिवादन के उच्चतम तरीके क्या थे?
उत्तर:
(1) सिजदा अथवा दण्डवत् लेटना
(2) चार तस्लीम
(3) जर्मी बोसी (जमीन चूमना )।

JAC Class 12 History Important Questions Chapter 9 शासक और इतिवृत्त : मुगल दरबार

प्रश्न 27.
मुगल दरबार को किन चार अवसरों पर खूब सजाया जाता था?
उत्तर:
(1) सिंहासनारोहण की वर्षगांठ
(2) ईद
(3) शबे बरात
(4) होली। प्रश्न

प्रश्न 28.
औरंगजेब ने जयसिंह तथा जसवन्त सिंह
को किसकी पदवी प्रदान की थी?
उत्तर:
मिर्जा राजा।

प्रश्न 29.
मुगल सम्राट द्वारा दिए जाने वाले उपहारों (पुरस्कारों) का उल्लेख कीजिए।
उत्तर:
(1) सम्मान का जामा (खिल्लत)
(2) सरप्या
(3) रत्नजड़ित आभूषण।

प्रश्न 30.
शाहजहाँ की कौनसी पुत्रियों को ऊंचे शाही मनसबदारों के समान वार्षिक आय होती थी?
उत्तर-
(1) जहाँआरा
(2) रोशनआरा।

प्रश्न 31.
मुगल साम्राज्य के आरम्भ से ही कौन से अभिजात अकबर की शाही सेवा में उपस्थित थे?
उत्तर:
तूरानी और ईरानी अभिजात।

प्रश्न 32.
मुगलकाल में सभी सरकारी अधिकारियों के दर्जे और पदों में दो प्रकार के संख्या-विषयक पद कौनसे होते थे?
उत्तर:
(1) जात
(2) सवार

प्रश्न 33.
केन्द्रीय सरकार के दो महत्त्वपूर्ण मंत्रियों के नामोल्लेख कीजिए।
उत्तर:
(1) दीवाने आला (वित्तमंत्री)
(2) सद्र- उस- सुदूर (मदद-ए-माशा अथवा अनुदान का मन्त्री)

प्रश्न 34.
मुगलकाल में प्रान्तीय शासन के तीन प्रमुख अधिकारियों के नामोल्लेख कीजिए।
उत्तर:
(1) दीवान
(2) बख्शी
(3) सूबेदार।

JAC Class 12 History Important Questions Chapter 9 शासक और इतिवृत्त : मुगल दरबार

प्रश्न 35.
मुगलकालीन परगनों के तीन अधिकारियों के नामोल्लेख कीजिए।
उत्तर:
(1) कानूनगो (राजस्व आलेख का रखवाला)
(2) चौधरी (राजस्व संग्रह का प्रभारी)
(3) काजी।

प्रश्न 36.
ईरानी सेना ने मुगलों के किस दुर्ग पर और कब अधिकार किया?
उत्तर:
1622 में कन्धार के दुर्ग पर

प्रश्न 37.
पहला जेसुइट शिष्टमण्डल फतेहपुर सीकरी के मुगल दरबार में कब पहुँचा?
उत्तर:
1580 ई. में

प्रश्न 38.
अकबर ने इबादतखाने का निर्माण कहाँ करवाया था?
उत्तर:
फतेहपुरी सीकरी में।

प्रश्न 39.
मुन्तखाब उत तवारीख’ की रचना किसने की थी?
उत्तर:
अब्दुल कादिर बदायूँनी ने

प्रश्न 40.
अकबर द्वारा निर्मित बुलन्द दरवाजा कहाँ स्थित है?
उत्तर:
फतेहपुर सीकरी में।

प्रश्न 41.
फतेहपुर सीकरी में शेख सलीम चिश्ती का मकबरा किस मुगल सम्राट ने बनवाया था?
उत्तर:
अकबर ने।

प्रश्न 42
आलमगीर की पदवी किसने धारण की
उत्तर:
औरंगजेब।

प्रश्न 43.
फारसी के हिन्दवी के साथ पारस्परिक सम्पर्क से किस भाषा की उत्पत्ति हुई?
उत्तर:
उर्दू।

प्रश्न 44.
‘बाबरनामा’ क्या है?
उत्तर:
‘बाबरनामा’ में बाबर के संस्मरण संकलित हैं।

JAC Class 12 History Important Questions Chapter 9 शासक और इतिवृत्त : मुगल दरबार

प्रश्न 45.
फारसी को मुगल दरबार की भाषा बनाने वाला सम्राट कौन था?
उत्तर:
अकबर।

प्रश्न 46.
अकबर के शासनकाल में ‘महाभारत’ का -फारसी में किस नाम से अनुवाद हुआ?
उत्तर:
‘रक्मनामा’।

प्रश्न 47.
अबुल फजल ने चित्रकारी को किसकी संज्ञा दी है?
था।
उत्तर:
‘जादुई कला’ की।

प्रश्न 48.
बिहजाद कौन था?
उत्तर:
बिहजाद मुगलकाल का एक प्रसिद्ध चित्रकार

प्रश्न 49.
अकबर ने धार्मिक क्षेत्र में कौनसी नीति अपनाई थी?
उत्तर:
सुलह-ए-कुल।

प्रश्न 50.
‘सुलह-ए-कुल’ का क्या अर्थ है?
उत्तर:
पूर्ण शान्ति।

प्रश्न 51.
अकबर के पश्चात् किस मुगल सम्राट ने हिन्दुओं पर जजिया कर पुनः लगा दिया?
उत्तर:
औरंगजेब ने।

प्रश्न 52.
पीड़ित लोगों को निष्पक्षतापूर्वक न्याय दिलाने के लिए अपने महल में न्याय की जंजीर लगाने वाला मुगल सम्राट कौन था?
उत्तर:
जहाँगीर।

प्रश्न 53.
झरोखा दर्शन की प्रथा किस मुगल सम्राट ने शुरू की थी?
उत्तर:
अकबर ने

प्रश्न 54.
कोर्निश का क्या अर्थ है?
उत्तर:
कोर्निश दरबार में औपचारिक अभिवादन का एक तरीका था।

प्रश्न 55.
शाहजहाँ ने कब और किसे अपनी राजधानी बनाया?
उत्तर:
1648 में शाहजहाँ ने शाहजहाँनाबाद को अपनी राजधानी बनाया।

JAC Class 12 History Important Questions Chapter 9 शासक और इतिवृत्त : मुगल दरबार

प्रश्न 56.
शाहजहाँ द्वारा निर्मित तीन इमारतों के नाम लिखिए।
उत्तर:
(1) लाल किला (दिल्ली)
(2) जामा मस्जिद (दिल्ली)
(3) ताजमहल (आगरा)।

प्रश्न 57.
हरम शब्द का क्या अर्थ है?
उत्तर:
‘पवित्र स्थान’ मुगलों का अन्तःपुर हरम कहलाता था।

प्रश्न 58.
मुगल परिवार में ‘अगाचा’ कौन होती थीं?
उत्तर:
मुगल सम्राटों की उप-पत्नियाँ।

प्रश्न 59.
‘हुमायूँनामा’ की रचना किसने की थी?
उत्तर:
गुलबदन ने।

प्रश्न 60.
अकबर की शाही सेवा में नियुक्त होने वाला प्रथम व्यक्ति कौनसा राजपूत मुखिया था?
उत्तर:
भारमल कछवाहा।

प्रश्न 61.
अकबर के साथ सर्वप्रथम किस राजपूत नरेश ने अपनी पुत्री का विवाह किया था?
उत्तर:
आम्बेर के राजा भारमल कछवाहा ने।

प्रश्न 62.
टोडरमल कौन था?
उत्तर:
टोडरमल अकबर का वित्तमन्त्री था

प्रश्न 63.
मीरबख्शी कौन था?
उत्तर:
यह सैन्य विभाग का मन्त्री था।

प्रश्न 64.
मुगलों के केन्द्रीय शासन के दो प्रमुख मन्त्रियों का उल्लेख कीजिये।
उत्तर:
(1) दीवान-ए-आला
(2) सद्र-उस- सुदूर

JAC Class 12 History Important Questions Chapter 9 शासक और इतिवृत्त : मुगल दरबार

प्रश्न 65.
मुगलकाल में प्रान्त का प्रमुख क्या कहलाता
उत्तर:
सूबेदार

प्रश्न 66.
मुगल काल में प्रत्येक सूबा किनमें विभाजित
उत्तर:
सरकारों में।

प्रश्न 67.
अकबर ने कन्धार के दुर्ग पर कब अधिकार स्थापित किया था?
उत्तर:
1595 ई. में

प्रश्न 68.
ईरानियों और मुगलों के बीच विवाद का मुख्य कारण क्या था?
उत्तर:
कन्धार पर अधिकार करना।

प्रश्न 69.
मुगलकाल में सिजदा के महत्व का उल्लेख कीजिये।
उत्तर:
‘अकबरनामा’ का लेखक अबुल फसल तथा ‘बादशाहनामा’ का लेखक अब्दुल हमीद लाहौरी था।

प्रश्न 79.
अबुल फजल के अनुसार सुलह-ए-कुल क्या था?
उत्तर:
अबुल फसल सुलह-ए-कुल के आदर्श को प्रबुद्ध शासन की आधारशिला मानता था।

प्रश्न 80.
सुलह-ए-कुल की नीति के अन्तर्गत अकबर द्वारा किए गए दो उपायों का उल्लेख कीजिए।
उत्तर:
अकबर ने 1563 में तीर्थयात्रा कर तथा 1564 में जजिया कर समाप्त कर दिया।

प्रश्न 81.
सुलह-ए-कुल का क्या आदर्श था?
उत्तर:
साम्राज्य के सभी धार्मिक तथा नृजातीय समूहों के लोगों के बीच शान्ति, सद्भावना और एकता बनाये रखना।

प्रश्न 82.
राज्यारोहण के बाद जहाँगीर ने पहला आदेश क्या दिया था?
उत्तर:
न्याय की जंजीर को लगाने का ताकि पीड़ित व्यक्ति जंजीर हिलाकर जहाँगीर का ध्यान आकर्षित कर सकें।

प्रश्न 83.
अकबर द्वारा बनाई गई राजधानियों के नाम लिखिए।
उत्तर:
(1) आगरा
(2) फतेहपुरी सीकरी
(3) लाहौर।

प्रश्न 84.
अकबर ने कब और किसे अपनी राजधानी बनाया?
उत्तर:
1570 के दशक में अकबर ने फतेहपुरी सीकरी को अपनी राजधानी बनाया।

JAC Class 12 History Important Questions Chapter 9 शासक और इतिवृत्त : मुगल दरबार

प्रश्न 85.
झरोखा दर्शन की प्रथा का क्या उद्देश्य था?
उत्तर:
जन विश्वास के रूप में शाही सत्ता की स्वीकृति को और विस्तार देना।

प्रश्न 86.
दीवान-ए-आम से क्या अभिप्राय है?
उत्तर:
दीवान-ए-आम सार्वजनिक सभा भवन था जहाँ राज्य के प्रमुख अधिकारी अपनी रिपोर्ट और प्रार्थना-पत्र प्रस्तुत करते थे।

प्रश्न 87.
‘दीवान-ए-खास’ क्या था?
उत्तर:
मुगल सम्राट दीवान-ए-खास नामक भवन में राज्य के वरिष्ठ मन्त्रियों से गोपनीय मुद्दों पर चर्चा करते थे। प्रश्न 88. मुगल सम्राट वर्ष में कौन-से तीन मुख्य त्यौहार मनाते थे?
उत्तर:
(1) सूर्य वर्ष के अनुसार सम्राट का जन्म दिन
(2) चन्द्र वर्ष के अनुसार सम्राट का जन्म दिन तथा
(3) फारसी नव वर्ष (नौ रोज)।

प्रश्न 89.
मुगल सम्राटों द्वारा अपने अमीरों को दी जाने वाली प्रमुख पदवियों का उल्लेख कीजिए।
उत्तर:
प्रश्न 90.
(1) आसफ खाँ
(2) मिर्जा राजा।मुगल परिवार में बेगम और अगहा महिलाएँ कौन होती थीं?
उत्तर:
(1) शाही परिवारों से आने वाली महिलाएँ ‘बेगम’ कहलाती थीं।
(2) कुलीन परिवार में जन्म न लेने वाली महिलाएँ ‘अगहा’ कहलाती थीं।

JAC Class 12 History Important Questions Chapter 9 शासक और इतिवृत्त : मुगल दरबार

प्रश्न 91.
गुलबदन बेगम कौन थी?
उत्तर:
गुलबदन बेगम बाबर की पुत्री, हुमायूँ की बहिन तथा अकबर की बुआ थी।

प्रश्न 92.
अभिजात कौन थे?
उत्तर:
मुगल राज्य के अधिकारियों का दल अभिजात- वर्ग कहलाता था।

प्रश्न 93.
आप ‘जात’ व ‘सवार’ से क्या समझते हैं? स्पष्ट करें।
उत्तर:
‘जात’ मनसबदार के पद, वेतन का सूचक था तथा ‘सवार’ यह सूचित करता था कि उसे कितने घुड़सवार रखने थे।

प्रश्न 94.
मीरबख्शी का क्या कार्य था?
उत्तर:
वह शाही सेवा में नियुक्ति तथा पदोन्नति के सभी उम्मीदवारों को मुगल सम्राट के सामने प्रस्तुत करता था।

प्रश्न 95.
‘तैनात ए-रकांब’ से आप क्या समझते हैं?
उत्तर:
यह अभिजातों का ऐसा सुरक्षित दल था जिसे किसी भी प्रान्त या सैन्य अभियान में प्रतिनियुक्त किया जा सकता था।

प्रश्न 96.
दरबारी लेखक या वाकिया नवीस कौन थे?
उत्तर:
दरबारी लेखक दरबार में प्रस्तुत किये जाने वाले सभी प्रार्थना-पत्रों, दस्तावेजों तथा शासकीय आदेशों का आलेख तैयार करते थे।

JAC Class 12 History Important Questions Chapter 9 शासक और इतिवृत्त : मुगल दरबार

प्रश्न 97.
नस्तलिक शैली से क्या अभिप्राय है?
उत्तर:
नस्तलिक शैली एक ऐसी शैली थी जिसे लम्बे सपाट प्रवाही ढंग से लिखा जाता था।

लघुत्तरात्मक प्रश्न-

प्रश्न 1.
जलालुद्दीन अकबर को मुगल बादशाहों में महानतम क्यों माना जाता है?
उत्तर:
जलालुद्दीन अकबर को (1556-1605) को मुगल बादशाहों में महानतम माना जाता है क्योंकि उसने न केवल मुगल साम्राज्य का विस्तार ही किया, बल्कि इसे अपने समय का विशालतम दृढ़तम तथा सबसे समृद्ध राज्य भी बना दिया। अकबर ने हिन्दूकुश पर्वत तक अपने साम्राज्य की सीमाओं का विस्तार किया। उसने ईरान के सफावियों और तूरान (मध्य एशिया) के उजबेकों की विस्तारवादी योजनाओं पर अंकुश लगाए रखा। उसने 1595 में कन्धार पर अधिकार कर लिया।

प्रश्न 2.
सुलेखन का क्या महत्त्व था?
उत्तर:
सुलेखन अर्थात् हाथ से लिखने की कला अत्यन्त महत्त्वपूर्ण कौशल मानी जाती थी। इसका प्रयोग भिन्न- भिन्न शैलियों में होता था ‘नस्तलिक’ अकबर की मनपसन्द शैली थी। यह एक ऐसी तरल शैली थी जिसे लम्बे सपाट प्रवाही ढंग से लिखा जाता था। इसे 5 से 10 मिलीमीटर की नोक वाले छटे हुए सरकंडे के टुकड़े से स्याही में डुबोकर लिखा जाता था।

प्रश्न 3.
मुगल पांडुलिपियों में चित्रों के समावेश का क्या महत्त्व था?
उत्तर:
(1) चित्र पुस्तक के सौन्दर्य में वृद्धि करते हैं।
(2) जहाँ लिखित माध्यम से राजा और राजा की शक्ति के विषय में जो बात कही न जा सके, उन्हें चित्रों के माध्यम से व्यक्त किया जा सकता था।
(3) अबुल फजल के अनुसार चित्रकला एक जादुई कला थी। उसकी राय में चित्रकला किसी निर्जीव वस्तु में भी प्राण फूँक सकती थी।

प्रश्न 4.
“अबुल फजल ने न्यायपूर्ण सम्प्रभुता को एक सामाजिक अनुबन्ध के रूप में परिभाषित किया है।” व्याख्या कीजिए।
उत्तर:
अबुल फजल के अनुसार बादशाह अपनी प्रजा के चार सत्वों की रक्षा करता है-
(1) जीवन (जन),
(2) धन (मूल्य),
(3) सम्मान (नामस),
(4) विश्वास (दीन)।
इसके बदले में वह आज्ञापालन तथा संसाधनों में हिस्से की माँग करता है। केवल न्यायपूर्ण सम्प्रभु ही शक्ति और दैवीय मार्गदर्शन के साथ इस सामाजिक अनुबन्ध का सम्मान कर पाते थे।

प्रश्न 5.
मुगल काल में न्याय के विचार को दृश्य रूप में किन प्रतीकों के माध्यम से निरूपित किया गया? इसका क्या उद्देश्य था?
उत्तर:
मुगल राजतंत्र में न्याय के विचार को सर्वोत्तम सद्गुण माना गया। कलाकारों द्वारा प्रयुक्त सर्वाधिक मनपसन्द प्रतीकों में से एक था. एक-दूसरे के साथ चिपटकर शान्तिपूर्वक बैठे हुए शेर और बकरी अथवा शेर और गाय इसका उद्देश्य राज्य को एक ऐसे क्षेत्र के रूप में दर्शाना था जहाँ दुर्बल तथा सबल सभी परस्पर सद्भाव से रह सकते थे।

JAC Class 12 History Important Questions Chapter 9 शासक और इतिवृत्त : मुगल दरबार

प्रश्न 6.
अकबर ने फतेहपुर सीकरी को अपनी राजधानी बनाने का निश्चय क्यों किया?
उत्तर:
अकबर ने फतेहपुर सीकरी को अपनी राजधानी बनाने का निश्चय किया। इसका कारण यह था कि फतेहपुर | सीकरी अजमेर को जाने वाली सीधी सड़क पर स्थित थी,
जहाँ शेख मुइनुद्दीन चिश्ती की दरगाह उस समय तक एक महत्त्वपूर्ण तीर्थ स्थल बन चुकी थी। मुगल बादशाहों के चिश्ती सिलसिले के सूफियों के साथ अच्छे सम्बन्ध थे।

प्रश्न 7.
राज्यारोहण के बाद जहाँगीर द्वारा दिए गए पहले आदेश का उल्लेख कीजिए।
उत्तर:
राज्यारोहण के बाद जहाँगीर ने जो पहला आदेश दिया, वह न्याय की जंजीर लगाने का था। इसका कारण यह था कि यदि न्याय करने वाले अधिकारी न्याय करने में देरी करें अथवा वे न्याय चाहने वाले लोगों के विषय में मिथ्याचार करें तो उत्पीड़ित व्यक्ति इस जंजीर को हिलाकर बादशाह का ध्यान आकृष्ट कर सकता था। इस जंजीर को शुद्ध सोने से बनाया गया था। यह 30 गज लम्बी थी तथा इसमें 60 पटयाँ लगी हुई थीं।

प्रश्न 8.
शाहजहाँ द्वारा किसे शाही राजधानी बनाया गया? उसका संक्षिप्त उल्लेख कीजिए।
उत्तर:
1648 में शाहजहाँ ने शाहजहांनाबाद को नई शाही राजधानी बनाया। दिल्ली के प्राचीन रिहायशी नगर में शाहजहाँनाबाद एक नई और शाही आबादी थी। यहाँ लाल किला, जामा मस्जिद, चांदनी चौक के बाजार की वृक्षवीथि और अभिजात वर्ग के बड़े-बड़े घर थे। शाहजहाँनाबाद विशाल एवं भव्य मुगल राजतंत्र की औपचारिक कल्पना को व्यक्त करता था।

प्रश्न 9.
‘कोर्निश’ से क्या अभिप्राय है?
उत्तर:
मुगल काल में ‘कोर्निश’ औपचारिक अभिवादन का एक ऐसा तरीका था, जिसमें दरबारी दाएँ हाथ की तलहधी को ललाट पर रखकर आगे की ओर सिर झुकाते थे। यह इस बात का प्रतीक था कि कोर्निश करने वाला व्यक्ति अपने इन्द्रिय और मन के स्थल को हाथ लगाते हुए शुककर विनम्रतापूर्वक शाही दरबार में अपने को प्रस्तुत कर रहा था।

प्रश्न 10.
मुगल अभिजात वर्गों के बीच हैसियत को निर्धारित करने वाले नियमों का उल्लेख कीजिए।
उत्तर:
दरबार में अभिजात वर्ग के किसी व्यक्ति की हैसियत इस बात से निर्धारित होती थी कि वह बादशाह के कितना पास और दूर बैठा है। किसी भी दरबारी को बादशाह द्वारा दिया गया स्थान बादशाही की दृष्टि में उसकी महत्ता का प्रतीक था। बादशाह को किए गए अभिवादन के तरीके से पदानुक्रम में उस व्यक्ति की हैसियत का पता चलता था।

प्रश्न 11.
भारत में मुगल राजवंश का अन्त कैसे हुआ?
उत्तर:
1707 ई. में औरंगजेब की मृत्यु के पश्चात् मुगल राजवंश की शक्ति घट गई। अतः विशाल मुगल साम्राज्य के स्थान पर कई क्षेत्रीय शक्तियाँ उभर आई, फिर भी सांकेतिक रूप से मुगल शासक की प्रतिष्ठा बनी रही। 1857 ई. में इस वंश के अन्तिम शासक बहादुरशाह जफर द्वितीय को अंग्रेजों ने उखाड़ फेंका इस प्रकार मुगल राजवंश का अन्त हो गया।

प्रश्न 12.
मुगल बादशाहों द्वारा तैयार करवाए गए इतिवृत्तों के कोई दो उद्देश्य लिखिए।
उत्तर:
मुगल बादशाहों द्वारा तैयार करवाए गए इतिवृत्तों के प्रमुख दो उद्देश्य निम्नलिखित थे.
(i) मुगल साम्राज्य के अन्तर्गत आने वाले समस्त लोगों के समक्ष मुगल राज्य को एक प्रबुद्ध राज्य के रूप में चित्रित करना।
(ii) मुगल शासन का विरोध करने वाले लोगों को यह बताना कि उनके सभी विरोधों का असफल होना निश्चित है।

प्रश्न 13.
न्याय की जंजीर किस बादशाह द्वारा लगवाई गई?
उत्तर:
न्याय की जंजीर जहाँगीर नामक बादशाह के द्वारा लगवाई गयी जो अपनी न्यायप्रियता के लिए बहुत प्रसिद्ध था जहाँगीर ने सिंहासन पर आरूढ़ होने के बाद सबसे पहला आदेश न्याय की जंजीर लगाने के सम्बन्ध में दिया। कोई भी व्यक्ति जिसे प्रशासन से न्याय न मिले, वह सीधे इस जंजीर को हिलाकर सम्बद्ध घण्टे को बजाकर न्याय की फरियाद बादशाह से कर सकता था। इस जंजीर को विशुद्ध रूप से सोने से बनाया गया था यह 30 गज लम्बी थी तथा इसमें 60 घण्टियाँ लगी हुई थीं।

JAC Class 12 History Important Questions Chapter 9 शासक और इतिवृत्त : मुगल दरबार

प्रश्न 14.
मुगल काल में अभिवादन का ‘चार तसलीम’ तरीका क्या था?
उत्तर:
मुगल काल में अभिवादन का ‘चार तसलीम’ तरीका दाएँ हाथ को जमीन पर रखने से शुरू होता था। इसमें तलहथी ऊपर की ओर होती थी। इसके बाद हाथ को धीरे-धीरे उठाते हुए व्यक्ति खड़ा होता था तथा तलहथी को सिर के ऊपर रखता था ऐसी तसलीम चार बार की जाती थी। तसलीम का शाब्दिक अर्थ आत्मनिवेदन है।

प्रश्न 15.
मुगल दरबार में राजनयिक दूत सम्बन्धी नयाचारों का उल्लेख कीजिए।
उत्तर:
मुगल बादशाह के समक्ष प्रस्तुत होने वाले राजदूत से यह अपेक्षा की जाती थी कि वह अभिवादन के मान्य रूपों में से एक या तो बहुत झुककर अथवा जमीन को चूमकर अथवा फारसी रिवाज के अनुसार छाती के सामने हाथ बाँधकर तरीके से अभिवादन करेगा। अंग्रेज राजदूत टामस रो ने यूरोपीय रिवाज के अनुसार जहाँगीर के सामने केवल झुककर अभिवादन किया और बैठने के लिए कुर्सी का आग्रह किया।

प्रश्न 16.
शब-ए-बारात’ पर संक्षिप्त टिप्पणी लिखिए।
उत्तर:
शब-ए-बारात हिजरी कैलेंडर के आठवें महीने अर्थात् चौदहवें सावन को पड़ने वाली पूर्ण चन्द्ररात्रि है। भारतीय उपमहाद्वीप में प्रार्थनाओं और आतिशबाजियों के खेल द्वारा इस दिन को मनाया जाता है। ऐसा माना जाता है कि इस रात मुसलमानों के लिए आगे आने वाले वर्ष का भाग्य निर्धारित होता है और पाप माफ कर दिए जाते हैं।

प्रश्न 17.
सम्मान का जामा’ तथा ‘सरप्पा’ नामक पुरस्कार क्या थे?
उत्तर:
योग्य व्यक्तियों को मुगल सम्राटों द्वारा पुरस्कार दिए जाते थे। इन पुरस्कारों में ‘सम्मान का जामा’ (खिल्लत) भी शामिल था जिसे पहले कभी न कभी बादशाह द्वारा पहना गया हुआ होता था इसलिए यह समझा जाता था कि वह बादशाह के आशीर्वाद का प्रतीक था ‘सरप्पा’ (सर से पाँव तक) एक अन्य उपहार था इस उपहार के तीन हिस्से हुआ करते थे- जामा, पगड़ी और पटका।

JAC Class 12 History Important Questions Chapter 9 शासक और इतिवृत्त : मुगल दरबार

प्रश्न 18.
‘शाही परिवार’ से क्या अभिप्राय है?
उत्तर:
‘हरम’ शब्द का प्रयोग प्राय: मुगलों की घरेलू दुनिया की ओर संकेत करता है। यह शब्द फारसी से निकला है जिसका तात्पर्य है पवित्र स्थान’ मुगल परिवार (शाही) में बादशाह की पत्नियों और उपपत्नियाँ, उसके निकटस्थ और दूर के सम्बन्धी (माता, सौतेली व उपमाताएँ. बहन, पुत्री, बहू, चाची-मौसी, बच्चे आदि) व महिला परिचारिकाएँ तथा गुलाम होते थे।

प्रश्न 19.
‘जात’ और ‘सवार’ मनसबदार से क्या अभिप्राय है?
उत्तर:
मुगल काल में सभी सरकारी अधिकारियों के दर्जे और पदों में दो तरह के संख्या-विषयक पद होते थे। ‘जात’ शाही पदानुक्रम में अधिकारी ( मनसबदार) के पद और वेतन का सूचक था और ‘सवार’ यह सूचित करता था कि उससे सेवा में कितने घुड़सवार रखना अपेक्षित था। सत्रहवीं शताब्दी में 1000 या उससे ऊपर जात वाले मनसबदार अभिजात ‘उमरा’ कहलाते थे।
हैं?

प्रश्न 20.
‘तैनात ए रकाब से आप क्या समझते
उत्तर:
भुगल दरबार में नियुक्त ‘तैनात ए रकाब ‘ अभिजातों का एक ऐसा सुरक्षित दल था, जिसे किसी भी प्रान्त या सैन्य अभियान में प्रतिनियुक्त किया जा सकता था। ये प्रतिदिन दो बार सुबह व शाम को सार्वजनिक सभा- भवन में बादशाह के प्रति आत्मनिवेदन करने के लिए उपस्थित होते थे। वे दिन-रात बादशाह और उसके घराने की सुरक्षा का दायित्व भी वहन करते थे।

प्रश्न 21.
अब्दुल कादिर बदायूँनी द्वारा ‘हरम में होम’ का वर्णन किस प्रकार किया है?
उत्तर:
‘अब्दुल कादिर बदायूँनी ने लिखा है कि युवावस्था के आरम्भ से ही, मुगल सम्राट अकबर अपनी पत्नियों अर्थात् भारत के राजाओं की पुत्रियों के सम्मान में ‘हरम में होम’ का आयोजन कर रहे थे। यह ऐसी धर्मक्रिया थी जो अग्नि पूजा (आतिश-परस्ती) से उत्पन्न हुई थी परन्तु अपने 25वें शासन वर्ष ( 1578) के नए वर्ष पर उसने सार्वजनिक रूप से सूर्य और अग्नि को दंडवत् प्रणाम किया।

प्रश्न 22.
मुगल सम्राट अपने राजवंश का इतिहास क्यों लिखवाना चाहते थे?
उत्तर:
मुगल सम्राटों की यह मान्यता थी कि उन्हें एक विशाल एवं विजातीय जनता पर शासन के लिए स्वयं ईश्वर ने नियुक्त किया है। ऐसा दृष्टिकोण सदैव महत्त्वपूर्ण बना रहा। इस दृष्टिकोण के प्रचार-प्रसार का एक तरीका राजवंशीय इतिहास लिखना लिखवाना था। अतः मुगल सम्राटों ने दरबारी इतिहासकारों को विवरण लिखने का कार्य सौंपा। इन विवरणों में मुगल सम्राटों के समय की घटनाओं का लेखा-जोखा प्रस्तुत किया गया।

प्रश्न 23.
मुगल इतिवृत्त / इतिहास के महत्त्व पर प्रकाश डालिए।
उत्तर:
मुगल इतिवृत्त घटनाओं का काल क्रमानुसार विवरण प्रस्तुत करते हैं। मुगलों का इतिहास लिखने के इच्छुक विद्वानों के लिए ये इतिवृत्त अनिवार्य स्रोत हैं। एक ओर तो ये इतिवृत्त मुगल साम्राज्य की संस्थाओं के बारे में तथ्यात्मक सूचनाएँ प्रदान करते थे, तो दूसरी ओर ये उन उद्देश्यों का प्रसार करते थे जिन्हें मुगल सम्राट अपने क्षेत्र में लागू करना चाहते थे। ये इतिवृत्त यह प्रकट करते हैं कि शाही विचारधाराएँ कैसे रची तथा प्रसारित की जाती थीं।

JAC Class 12 History Important Questions Chapter 9 शासक और इतिवृत्त : मुगल दरबार

प्रश्न 24.
खिल्लत, सरण्या, पद्म मुरस्सा आदि उपहारों से आप क्या समझते हैं?
उत्तर:
मुगल सम्राटों द्वारा योग्य व्यक्तियों को पदवियाँ और पुरस्कार प्रदान कर सम्मानित किया जाता था खिल्लत पुरस्कार में बादशाह अपने द्वारा पहना हुआ जामा उपहारस्वरूप देता था। इसे बादशाह के आशीर्वाद का प्रतीकं समझा जाता था सरप्पा, सिर से पाँव तक का उपहार होता था। इसमें जामा, पगड़ी और पटका शामिल होते थे। बादशाह द्वारा उपहार में दिए गए कमल की मंजरियों वाले रत्नजड़ित गहनों के सेट को पद्म मुरस्सा कहा जाता था।

प्रश्न 25.
मुगल वास्तुकला में जहाँआरा के योगदान पर संक्षिप्त टिप्पणी लिखिए।
उत्तर:
जहाँआरा मुगल बादशाह शाहजहाँ की पुत्री थी। उसने शाहजहाँ की नई राजधानी शाहजहाँनाबाद (दिल्ली) की कई वास्तुकलात्मक परियोजनाओं में भाग लिया। इनमें से आँगन एवं बाग के साथ एक दोमंजिली भव्य कारवाँ सराय थी। शाहजहाँनाबाद के मुख्य केन्द्र चाँदनी चौक की रूपरेखा भी जहाँआरा द्वारा बनाई गई थी।

प्रश्न 26.
सरकारी अधिकारियों के दर्जे जात, मनसबदार, उमरा आदि से आपका क्या तात्पर्य सवार, है?
उत्तर:
सरकारी अधिकारियों के दर्जे और पदों में दो तरह के ओहदे होते थे। यह ओहदे संख्या विषयक होते थे। शाही पदानुक्रम में जात अधिकारी जिसे मनसबदार कहा जाता था, के पद और वेतन को सूचित करता था और ‘सवार’ से यह सूचित होता था कि वह कितने घुड़सवार रख सकता था। 1000 या उससे ऊपर जात वाले मनसबदार ‘उमरा’ जो कि अमीर का बहुवचन है, कहे जाते थे।

प्रश्न 27.
मीर बख्शी कौन था? मीर बख्शी तथा उसका कार्यालय क्या कार्य करता था?
उत्तर:
मुगलकाल में मीर बख्शी सर्वोच्च वेतनदाता था। मीर बख्शी खुले दरबार में बादशाह के दायीं ओर खड़ा रहकर नियुक्ति तथा पोदन्नति पाने वाले उम्मीदवारों को प्रस्तुत करता था उसका कार्यालय उसकी मुहर एवं हस्ताक्षर के साथ-साथ बादशाह की मुहर तथा हस्ताक्षर वाले आदेश भी तैयार करता था।

प्रश्न 28.
मुगल कौन थे?
उत्तर:
‘मुगल’ शब्द की उत्पत्ति ‘मंगोल’ से हुई है। मुगल शासकों ने अपने को ‘तैमूरी’ कहा क्योंकि पितृपक्ष से वे तुर्की शासक तिमूर के वंशज थे प्रथम मुगल शासक बाबर मातृपक्ष से चंगेजखों का सम्बन्धी था। वह तुर्की भाषा बोलता था। मुगल चगताई मूल के थे तथा तुर्की उनकी मातृभाषा थी। सोलहवीं शताब्दी में यूरोपियों ने मुगल राजवंश के लिए मुगल शब्द का प्रयोग किया था उस समय से ही इस शब्द का निरन्तर प्रयोग होता रहा है।

प्रश्न 29.
” मुगल बादशाहों द्वारा तैयार करवाए गए इतिवृत्त साम्राज्य और उसके दरबार के अध्ययन के महत्त्वपूर्ण स्त्रोत हैं।” व्याख्या कीजिए।
अथवा
इतिवृत्त किस प्रकार मुगल सम्राटों के विषय में महत्त्वपूर्ण जानकारी प्रदान करते हैं?
उत्तर:
मुंगल सम्राटों द्वारा तैयार करवाए गए इतिवृत्त साम्राज्य और उसके दरबार के अध्ययन के महत्त्वपूर्ण खोत हैं। ये इतिवृत्त मुगल साम्राज्य के अन्तर्गत आने वाले सभी लोगों के सामने मुगल- राज्य को एक प्रबुद्ध राज्य के रूप में दर्शाने के उद्देश्य से लिखे गए थे। इसी प्रकार उनका उद्देश्य मुगल शासन का विरोध करने वाले लोगों को यह भी बताना था कि उनके सभी विरोधों का असफल होना निश्चित है। मुगल सम्राट यह भी चाहते थे कि भावी पीढ़ियों के लिए उनके शासन के विवरण उपलब्ध रहे।

JAC Class 12 History Important Questions Chapter 9 शासक और इतिवृत्त : मुगल दरबार

प्रश्न 30.
“मुगल इतिवृत्तों के लेखक निरपवाद रूप से दरबारी ही रहे।” व्याख्या कीजिये।
उत्तर:
मुगल इतिवृत्तों में शासक पर केन्द्रित घटनाओं, शासक के परिवार दरबार व अभिजात वर्ग, बुद्ध और प्रशासनिक व्यवस्थाओं का समावेश था। अकबर, शाहजहाँ तथा आलमगीर (औरंगजेब) के शासन काल की घटनाओं पर आधारित इतिवृत्तों के शीर्षक ‘अकबरनामा’, ‘शाहजहाँनामा’ तथा ‘आलमगीरनामा’ यह संकेत देते हैं कि इनके लेखकों की दृष्टि में साम्राज्य एवं दरबार का इतिहास और बादशाह का इतिहास एक ही था।

प्रश्न 31.
अकबर ने फारसी को दरबार की मुख्य भाषा क्यों बनाया?
उत्तर:
अकबर ने सोच समझकर फारसी को दरबार की मुख्य भाषा बनाया। सम्भवतया ईरान के साथ सांस्कृतिक और बौद्धिक सम्पर्कों के साथ-साथ मुगल दरबार में पद पाने को इच्छुक ईरानी और मध्य एशियाई प्रवासियों ने अकबर को फारसी भाषा को अपनाए जाने के लिए प्रेरित किया होगा। फारसी को दरबार की भाषा का ऊंचा स्थान दिया गया तथा उन लोगों को शक्ति तथा प्रतिष्ठा प्रदान की गई, जिनका इस भाषा पर अच्छा नियंत्रण था।

प्रश्न 32.
मुगल पाण्डुलिपियों में चित्रों को किस प्रकार संलग्न किया जाता था?
उत्तर:
मुगल पांडुलिपियों की रचना में चित्रकारों का भी महत्त्वपूर्ण योगदान था। किसी मुगल सम्राट के शासन की घटनाओं का विवरण देने वाले इतिहासों में लिखित पाठ के साथ ही उन घटनाओं को चित्रों के माध्यम से दृश्य रूप में भी वर्णित किया जाता था जब किसी पुस्तक में घटनाओं अथवा विषयों को दृश्य रूप में वर्णित किया जाना होता था, तो सुलेखक उसके आस-पास के पृष्ठों को खाली छोड़ देते थे। चित्रकार शब्दों में वर्णित विषय को अलग से चित्र के रूप में उतारकर वहाँ संलग्न कर देते थे।

प्रश्न 33.
मुगल पाण्डुलिपियों में चित्रों को संलग्न करने का क्या महत्त्व था?
उत्तर:
पाण्डुलिपियों में चित्रों का महत्त्व-
(1) चित्र पुस्तक के सौन्दर्य में वृद्धि करते थे।
(2) राजा और उसकी शक्ति के बारे में जो बात लिखित रूप से नहीं कही जा सकती थी, उसे चित्र व्यक्त कर देते थे।
(3) अबुल फजल ने चित्रकारी को एक ‘जादुई कला’ की संज्ञा दी है उसके अनुसार चित्रकला किसी निर्जीव वस्तु को भी सजीव रूप प्रदान कर सकती है।

प्रश्न 34.
मुगल सम्राट तथा उसके दरबार का चित्रण करने वाले चित्रों की रचना को लेकर शासकों और उलमा के बीच तनाव की स्थिति क्यों बनी रहती थी?
उत्तर:
उलमा रूढ़िवादी मुसलमान वर्ग के प्रतिनिधि थे। उन्होंने कुरान के साथ-साथ हदीस में प्रतिष्ठापित मानव रूपों के चित्रण पर इस्लामी प्रतिबन्ध का आह्वान किया। हदीस में एक प्रसंग में पैगम्बर साहब को प्राकृतिक तरीके से जीवित रूपों के चित्रण की मनाही करते हुए उल्लिखित किया गया है। इसका कारण यह है कि ऐसा करने से यह लगता था कि कलाकार रचना की शक्ति को अपने हाथ में लेने का प्रयास कर रहा है। यह केवल ईश्वर का ही कार्य था।

JAC Class 12 History Important Questions Chapter 9 शासक और इतिवृत्त : मुगल दरबार

प्रश्न 35.
मुगल शासनकाल की पांडुलिपियों में चित्रों को किस प्रकार संलग्न किया जाता था?
उत्तर:
मुगल शासनकाल की पांडुलिपियों की रचनाओं में चित्रकारों का भी बहुत महत्त्वपूर्ण योगदान था। किसी मुगल बादशाह के शासन की घटनाओं का विवरण देने वाले इतिहासों में लिखित पाठ के साथ-साथ उन घटनाओं को चित्रों के माध्यम से दृश्य रूप में दर्शाया जाता था। जब किसी पुस्तक में घटनाओं अथवा विषयों को दृश्य रूप में व्यक्त किया जाना होता था तो सुलेखक उसके आसपास के पृष्ठों को खाली छोड़ देते थे। चित्रकार शब्दों में वर्णित विषय को अलग से चित्रों में उतारकर वहाँ संलग्न कर देते थे। ये लघु चित्र होते थे जिन्हें पांडुलिपि के पृष्ठों पर आसानी से लगाया और देखा जा सकता था।

प्रश्न 36.
दैवीय प्रकाश से आप क्या समझते हैं? दरबारी इतिहासकारों ने इसकी व्याख्या कैसे की?
उत्तर:
मुगल दरबार के इतिहासकारों ने मुगल सम्राटों का महिमामण्डन करने के लिए उन्हें इस प्रकार वर्णित किया कि वे ईश्वर से सीधे दैवीय शक्ति या दैवीय प्रकाश प्राप्त करते थे। इतिहासकारों ने इसके लिए एक दंतकथा का वर्णन किया है। इस दंतकथा में वर्णन है कि मंगोल रानी अलानकुआ अपने शिविर में विश्राम करते हुए सूर्य की किरण द्वारा गर्भवती हुई। उससे उत्पन्न सन्तान दैवीय प्रकाश से युक्त थी। प्रसिद्ध ईरानी सूफी शिहाबुद्दीन सुहरावर्दी के अनुसार देवीय प्रकाश से युक्त राजा अपनी प्रजा के लिए आध्यात्मिक मार्गदर्शन का स्रोत बन जाता था। अबुल फजल ने ईश्वर से उद्भूत प्रकाश को ग्रहण करने वाली वस्तुओं में मुगल राजत्व को सबसे शीर्ष स्थान पर रखा।

प्रश्न 37.
मुगल किताबखानों में रचित पाण्डुलिपियों की रचना में किन-किन लोगों का योगदान होता था?
अथवा
पांडुलिपियों की रचना में विविध प्रकार के कार्य करने वाले लोगों का वर्णन कीजिये।
उत्तर:
पांडुलिपियों की रचना में विविध प्रकार के कार्य करने वाले अनेक लोग शामिल थे। कागज बनाने बालों की पांडुलिपि के पन्ने तैयार करने, सुलेखकों की पाठ की नकल तैयार करने, कोफ्तगरों की पृष्ठों को चमकाने के लिए, चित्रकारों की पाठ से दृश्यों को चित्रित करने के लिए और जिल्दसाजों की प्रत्येक पन्ने को इकट्ठा कर उसे अलंकृत आवरण में बैठाने के लिए आवश्यकता होती थी। तैयार पांडुलिपि को एक बहुमूल्य वस्तु के रूप में देखा जाता था।

प्रश्न 38.
अबुल फजल पर एक संक्षिप्त टिप्पणी लिखिए।
उत्तर:
अबुल फजल अकबर का एक प्रसिद्ध दरबारी इतिहासकार था। अबुलफजल का पालन-पोषण मुगल राजधानी आगरा में हुआ था। वह अरबी, फारसी, यूनानी दर्शन और सूफीवाद का प्रकाण्ड विद्वान था। वह एक प्रभावशाली विवादी तथा स्वतंत्र चिन्तक था। उसने सदैव रूढ़िवादी उलमा के विचारों का विरोध किया। अबुल फजल के इन गुणों से अकबर बढ़ प्रभावित हुआ। उसने अबुलफजल को अपने सलाहकार और अपनी नीतियों के प्रवक्ता के रूप में बहुत उपयुक्त पाया।

प्रश्न 39.
‘अकबरनामा’ पर एक संक्षिप्त टिप्पणी लिखिए।
उत्तर:
अबुलफजल द्वारा रचित अकबरनामा तीन जिल्दों में विभाजित है जिनमें से प्रथम दो इतिहास हैं। तीसरी जिल्द ‘आइन-ए-अकबरी’ है पहली जिल्द में आदम से | लेकर अकबर के जीवन के एक खगोलीय कालचक्र तक (30 वर्ष) का मानव जाति का इतिहास है। दूसरी जिल्द अकबर के 46वें शासन वर्ष (1601 ई.) पर समाप्त होती है ‘अकबरनामा’ में राजनीतिक घटनाओं, साम्राज्य के भौगोलिक, सामाजिक, प्रशासनिक तथा सांस्कृतिक आदि पक्षों का विस्तृत विवरण मिलता है।

JAC Class 12 History Important Questions Chapter 9 शासक और इतिवृत्त : मुगल दरबार

प्रश्न 40.
‘बादशाहनामा’ का संक्षिप्त परिचय दीजिए।
उत्तर:
‘बादशाहनामा’ का लेखक अब्दुल हमीद लाहौरी था। ‘बादशाहनामा’ सरकारी इतिहास है। यह तीन जिल्दों में विभाजित है। प्रत्येक जिल्द दस चन्द्र वर्षों का विवरण देती है। लाहौरी ने शाहजहाँ के शासन (1627-47) के पहले दो दशकों पर पहला व दूसरा दफ्तर लिखा। इन जिल्दों में बाद में शाहजहाँ के वजीर सादुल्लाखाँ ने सुधार किया। वृद्धावस्था की कमजोरियों के कारण लाहौरी तीसरे दशक के बारे में नहीं लिख सका जिसे बाद में इतिहासकार वारिस ने लिखा।

प्रश्न 41.
औपनिवेशिक काल में ऐतिहासिक पांडुलिपियों को किस प्रकार संरक्षित किया गया?
उत्तर:
1784 ई. में सर विलियम जोन्स द्वारा स्थापित ‘एशियाटिक सोसाइटी आफ बंगाल ने अनेक भारतीय पांडुलिपियों के सम्पादन, प्रकाशन और अनुवाद का दायित्व अपने ऊपर ले लिया। ‘अकबरनामा’ तथा ‘बादशाहनामा’ के सम्पादित पाठान्तर सर्वप्रथम एशियाटिक सोसाइटी द्वारा उन्नीसवीं शताब्दी में प्रकाशित किए गए। बीसवीं शताब्दी के प्रारम्भ में हेनरी बेवरिज ने ‘अकबरनामा’ का अंग्रेजी में अनुवाद किया। ‘बादशाहनामा’ के केवल कुछ ही अंशों का अभी तक अंग्रेजी में अनुवाद हुआ है।

प्रश्न 42.
” मुगल सम्राटों में दैवीय प्रकाश सम्प्रेषित होता था। व्याख्या कीजिए।
उत्तर:
दरबारी इतिहासकारों ने कई साक्ष्यों के आधार पर यह दर्शाया कि मुगल सम्राटों को सीधे ईश्वर से शक्ति मिली थी। अबुल फजल ने ईश्वरीय प्रकाश को ग्रहण करने वाली वस्तुओं में मुगल राजत्व को सबसे ऊँचे स्थान पर रखा। प्रसिद्ध ईरानी सूफी शिहाबुद्दीन सुहरावर्दी के विचार के अनुसार यह दैवीय प्रकाश राजा में सम्प्रेषित होता था जिसके बाद राजा अपनी प्रजा के लिए आध्यात्मिक मार्गदर्शन का स्रोत बन जाता था।

प्रश्न 43.
“सुलह-ए-कुल एकीकरण का एक स्रोत था।” विवेचना कीजिए।
अथवा
सुलहकुल का आदर्श क्या था? इसे किस प्रकार लागू किया गया?
उत्तर:
अबुल फसल ‘सुलह-ए-कुल’ (पूर्ण शान्ति ) के आदर्श को प्रबुद्ध शासन का आधार बताता है। यद्यपि ‘सुलह-ए-कुल’ में सभी धर्मों तथा मतों को अभिव्यक्ति की स्वतन्त्रता थी, किन्तु उसकी एक शर्त थी कि वे राज्य- सत्ता को क्षति नहीं पहुँचाएंगे अथवा आपस में नहीं लड़ेंगे। ईरानी तूरानी, अफगानी राजपूत, दक्खनी आदि अभिजातों को पद और पुरस्कार दिए गए। 1563 में अकबर ने तीर्थयात्रा कर तथा 1564 में जजिया कर को समाप्त कर दिया।

प्रश्न 44.
अबुल फजल ने प्रभुसत्ता को एक सामाजिक अनुबन्ध के रूप में परिभाषित क्यों किया? उत्तर- अबुल फजल ने लिखा है कि बादशाह अपनी प्रजा के चार सत्त्वों की रक्षा करता है –

  1. जीवन (जन),
  2. धन (माल),
  3. सम्मान (नामस) तथा
  4. विश्वास ( दोन )।

इसके बदले में वह आज्ञापालन तथा संसाधनों में हिस्से की माँग करता है। केवल न्यायपूर्ण सम्प्रभु ही शक्ति और दैवीय मार्गदर्शन के साथ इस अनुबन्ध का सम्मान कर पाते थे। न्याय के विचार को मुगल राजतन्त्र में सर्वोत्तम सद्गुण माना गया। न्याय के विचार को चित्रों में अनेक प्रतीकों के रूप में दर्शाया गया।

प्रश्न 45.
मुगल दरबार में अभिजात वर्ग के बीच उनकी हैसियत दर्शाने वाले नियमों का वर्णन कीजिए।
उत्तर:
दरबार में किसी अभिजात की हैसियत इस बात से निर्धारित होती थी कि वह बादशाह के कितने पास और दूर बैठा है। किसी भी दरबारी को बादशाह द्वारा दिया गया स्थान बादशाह की दृष्टि में उसकी महत्ता का द्योतक था दरबारी समाज में सामाजिक नियन्त्रण का व्यवहार दरबार में मान्य सम्बोधन, शिष्टाचार तथा बोलने के नियमों द्वारा होता था। शिष्टाचार का उल्लंघन करने वालों को तुरन्त ही दंडित किया जाता था।

JAC Class 12 History Important Questions Chapter 9 शासक और इतिवृत्त : मुगल दरबार

प्रश्न 46.
राजवंशीय इतिहास लेखन, इतिवृत्त से आप क्या समझते हैं?
उत्तर:
मुगलवंशीय साम्राज्य के शासकों ने अपने उद्देश्यों के प्रचार प्रसार एवं राजवंशीय इतिहास के संकलन हेतु दरबारी इतिहासकारों को इतिवृत्तों के लेखन का कार्य सौंपा। आधुनिक इतिहासकारों ने इस शैली को इतिवृत्ति इतिहास के नाम से सम्बोधित किया। इतिवृत्तों द्वारा हमें मुगल राज्य की घटनाओं का कालक्रम के अनुसार क्रमिक विवरण प्राप्त होता है इतिवृत्त मुगल राज्य की संस्थाओं के प्रामाणिक स्त्रोत थे: जिन्हें इनके लेखकों द्वारा परिश्रम से एकत्र और वर्गीकृत किया था। इतिवृत्तों के द्वारा इस बात की जानकारी प्राप्त होती है कि शाही विचारधाराएँ कैसे प्रसारित की जाती हैं।

प्रश्न 47.
मुगल दरबार में राजनयिक दूतों से सम्बन्धित न्याचारों का उल्लेख कीजिए।
उत्तर:
मुगल दरबार में बादशाह के सामने प्रस्तुत होने वाले राजदूत से यह अपेक्षा की जाती थी कि वह अभिवादन के मान्य रूपों में से एक या तो बहुत झुक कर अथवा जमीन को चूम कर अथवा फारसी रिवाज के अनुसार छाती के सामने हाथ बाँध कर- तरीके से अभिवादन करेगा। जेम्स प्रथम के अंग्रेज दूत टामसरो ने यूरोपीय रिवाज के अनुसार जहाँगीर के सामने केवल झुककर अभिवादन किया और फिर बैठने के लिए कुर्सी का आग्रह कर दरबार को आश्चर्यचकित कर दिया।

प्रश्न 48.
झरोखा दर्शन की प्रथा का उल्लेख कीजिए।
उत्तर:
मुगल काल में बादशाह अपने दिन की शुरूआत सूर्योदय के समय कुछ व्यक्तिगत धार्मिक प्रार्थनाओं से करता था। इसके बाद वह पूर्व की ओर मुँह किए एक छोटे छज्जे अर्थात् शरोखे में आता था। इसके नीचे लोगों की भारी भीड़ बादशाह की एक झलक पाने के लिए प्रतीक्षा करती थी। झरोखा दर्शन की प्रथा अकबर द्वारा शुरू की गई थी। इस प्रथा का उद्देश्य जन विश्वास के रूप में शाही सत्ता की स्वीकृति का और विस्तार करना था।

प्रश्न 49.
अबुल फजल के अनुसार बादशाह अपनी प्रजा का एक परिपूर्ण रक्षक है। स्पष्ट कीजिए।
उत्तर:
अबुल फजल के अनुसार बादशाह अपनी प्रजा का परिपूर्ण रक्षक है। वह प्रजा के जीवन, धन, सम्मान तथा विश्वास की रक्षा करता है जीवन, धन, सम्मान तथा विश्वास की रक्षा के बदले वह प्रजा से आज्ञापालन और राज्य के संसाधनों में कर के रूप में उचित हिस्सेदारी की आशा करता है। न्यायपूर्ण बादशाह ही शक्ति और दैवीय मार्गदर्शन के साथ इस सामाजिक अनुबन्ध का दृढ़तापूर्वक पालन कर सकने में समर्थ थे। मुगल सम्राटों के इन न्यायपूर्ण विचारों का तत्कालीन चित्रकारों द्वारा बादशाहनामा आदि ग्रन्थों में प्रतीकात्मक रूप से सचित्र निरूपण किया गया।

प्रश्न 50.
अबुल फजल ने अकबर के दरबार का किस प्रकार वर्णन किया है?
उत्तर:
अबुल फजल के अनुसार जब भी मुगल- सम्राट अकबर दरबार लगाते थे तो एक विशाल ढोल पीटा जाता था और इसके साथ-साथ अल्लाह का गुणगान होता था। बादशाह के पुत्र, पौत्र, दरबारी आदि दरबार में उपस्थित होते थे और कोर्निश (औपचारिक अभिवादन का एक तरीका ) कर अपने स्थान पर खड़े रहते थे प्रसिद्ध विद्वान तथा विशिष्ट कलाओं में निपुण लोग आदर व्यक्त करते थे तथा न्यायिक अधिकारी अपनी रिपोर्ट प्रस्तुत करते थे बादशाह सभी मामलों को सन्तोषजनक तरीके से निपटाते थे।

JAC Class 12 History Important Questions Chapter 9 शासक और इतिवृत्त : मुगल दरबार

प्रश्न 51.
शाहजहाँ के ‘रत्नजड़ित सिंहासन’ (तख्त- ए-मुरस्सा) पर एक संक्षिप्त टिप्पणी लिखिए।
उत्तर:
रत्नजड़ित सिंहासन की भव्य संरचना में एक छतरी थी जो द्वादशकोणीच स्तम्भों पर टिकी हुई थी। इसकी ऊँचाई सीढ़ियों से गुम्बद तक लगभग 10 फीट थी। बादशाह ने यह आदेश दिया कि 86 लाख रुपये के रत्न तथा बहुमूल्य पत्थर तथा चौदह लाख रुपये के मूल्य के एक लाख तोला सोने से इसे सुसज्जित किया जाना चाहिए। सिंहासन की साज-सज्जा में सात वर्ष लग गए। इसकी सजावट में प्रयुक्त हुए बहुमूल्य पत्थरों में रुबी था जिसका मूल्य एक लाख रुपये था।

प्रश्न 52.
मुगल दरबार में मनाए जाने वाले विशिष्ट उत्सवों का वर्णन कीजिए।
उत्तर:
सिंहासनारोहण की वर्षगाँठ, ईद, शब-ए-बारात तथा होली जैसे विशिष्ट अवसरों पर दरबार का वातावरण सजीव हो उठता था। सजे हुए डिब्बों में रखी सुगंधित मोमबत्तियाँ और महल की दीवारों पर लटक रहे रंग-बिरंगे बन्दनवार आने वाले लोगों को अत्यधिक प्रभावित करते थे। मुगल सम्राट वर्ष में तीन मुख्य त्यौहार मनाया करते थे-सूर्यवर्ष और चन्द्रवर्ष के अनुसार सम्राट का जन्म दिन और वसन्तागमन पर फारसी नववर्ष नौरोज

प्रश्न 53.
मुगल काल में शाही परिवारों में विवाहों के आयोजन का वर्णन कीजिए।
उत्तर:
मुगल काल में शाही परिवारों में विवाहों का आयोजन काफी खर्चीला होता था 1633 ई. में शाहजहाँ के ज्येष्ठ पुत्र दाराशिकोह का विवाह राजकुमार परवेज की पुत्री नादिरा से हुआ था। विवाह के उपहारों के प्रदर्शन की व्यवस्था दीवान-ए-आम में की गई थी। बादशाह तथा हरम की स्वियों दोपहर में इसे देखने के लिए आई। हिनाबन्दी (मेंहदी लगाना) की रस्म दीवान-ए-खास में अदा की गई। दरबार में उपस्थित लोगों के बीच पान, इलायची तथा मेवे बांटे गए।

प्रश्न 54.
शाहजहाँ के तख्त-ए-मुरस्सा के बारे में बादशाहनामा में क्या वर्णन किया गया है?
उत्तर:
आहरा महल के सार्वजनिक सभा भवन में रखे हुए शाहजहाँ के रत्नजड़ित सिंहासन के बारे में बादशाहनामा में निम्नलिखित प्रकार से वर्णन किया गया है-
सिंहासन की भव्य संरचना में एक छतरी है जो द्वादशकोणीय स्तम्भों पर टिकी हुई है। इसकी ऊँचाई सीढ़ियों से गुंबद तक पाँच क्यूबिट (लगभग 10 फुट) है। अपने राज्यारोहण के समय महामहिम ने यह आदेश दिया कि 86 लाख रुपए के रत्न तथा बहुमूल्य पत्थर और एक लाख तोला सोना जिसकी कीमत चौदह लाख रुपए है, से इसे सजाया जाना चाहिए। सिंहासन की साज-सज्जा में सात वर्ष लग गए। इसकी सजावट में प्रयुक्त हुए बहुमूल्य पत्थरों में रूबी था जिसकी कीमत एक लाख रुपए थी। इस रूबी पर महान बादशाह तिमूर साहिब-ए-किरान, मिर्जा शाहरुख, मिर्जा उलुग बेग और शाह अब्बास के साथ-साथ अकबर, जहाँगीर और स्वयं महामहिम के नाम अंकित थे।

JAC Class 12 History Important Questions Chapter 9 शासक और इतिवृत्त : मुगल दरबार

प्रश्न 55.
शाहजहाँ के ज्येष्ठ पुत्र दाराशिकोह के विवाह के आयोजन का वर्णन कीजिए।
उत्तर:
बादशाहनामा में दाराशिकोह के विवाह के आयोजन का वर्णन किया गया है। शाही परिवारों में विवाहों का आयोजन अत्यन्त धूम-धाम से किया जाता था। इसमें काफी धनराशि व्यय की जाती थी। 1633 ई. में दाराशिकोह और नादिरा, जो कि राजकुमार परवेज की पुत्री थी, के विवाह की व्यवस्था राजकुमारी जहाँआरा और दिवंगत महारानी मुमताज महल की प्रमुख नौकरानी सती उननिसाखानुम द्वारा की गई दीवान-ए-आम में उपहारों के प्रदर्शन की भव्य व्यवस्था की गई थी। हिनाबन्दी दीवाने खास में की गई। दरबार में उपस्थित व्यक्तियों के बीच पान, इलायची और मेवे बांटे गए। विवाह के इस आयोजन में तत्कालीन समय में 32 लाख रु. खर्च हुए। इनमें 6 लाख रु. शाही खजाने से, 16 लाख रु. जहाँआरा तथा शेष धनराशि दुल्हन की माँ के द्वारा दिए गए थे।

प्रश्न 56
मुगल परिवार में बेगमों, अगहा एवं अगाचा स्त्रियों की स्थिति का विवेचन कीजिए।
उत्तर:
मुगल परिवार में शाही परिवारों से आने वाली स्त्रियां बेगमें कहलाती थीं अगहा स्वियों ये थीं जिनका जन्म कुलीन परिवार में नहीं हुआ था। दहेज (मेहर) के रूप में विपुल धन नकद और बहुमूल्य वस्तुएँ लेने के बाद विवाह करके आई बेगमों को अपने पतियों से स्वाभाविक रूप से ‘अगहाओं’ की तुलना में अधिक ऊँचा दर्जा और सम्मान प्राप्त था। उपपत्नियों (अगाचा) की स्थिति सबसे निम्न थी। इन सभी को नकद मासिक भत्ता तथा अपने- अपने दर्जे के हिसाब से उपहार मिलते थे।

प्रश्न 57.
“राजपूत कुलों एवं मुगलों, दोनों के लिए विवाह राजनीतिक सम्बन्ध बनाने व मैत्री सम्बन्ध स्थापित करने का एक तरीका था।” व्याख्या कीजिए।
उत्तर:
विवाह में पुत्री को पेंटस्वरूप दिए जाने के साथ प्रायः एक क्षेत्र भी उपहार में दिया जाता थे। इससे विभिन्न शासक वर्गों के बीच पदानुक्रमिक सम्बन्धों की निरन्तरता निश्चित हो जाती थी इस प्रकार के विवाह और उनके फलस्वरूप विकसित सम्बन्धों के कारण ही मुगल शासक बन्धुता के एक व्यापक तन्त्र का निर्माण कर सके। इसके फलस्वरूप उनके महत्त्वपूर्ण वर्गों से घनिष्ठ सम्बन्ध स्थापित हुए और उन्हें एक विशाल साम्राज्य को सुरक्षित रखने में सहायता मिली।

JAC Class 12 History Important Questions Chapter 9 शासक और इतिवृत्त : मुगल दरबार

प्रश्न 58.
गुलबदन बेगम पर एक संक्षिप्त टिप्पणी लिखिए।
उत्तर:
गुलबदन बेगम बाबर की पुत्री, हुमायूँ की बहिन तथा अकबर की बुआ थी। उसने ‘हुमायूँनामा’ नामक पुस्तक लिखी जिससे हमें मुगलों की घरेलू दुनिया की एक झलक मिलती है। गुलबदन तुर्की तथा फारसी में धाराप्रवाह लिख सकती थी। गुलबदन ने जो लिखा वह मुगल बादशाहों की प्रशस्ति नहीं थी, बल्कि उसने राजाओं और राजकुमारों के बीच चलने वाले संघर्षों और तनावों का उल्लेख किया है। उसने परिवार की स्वियों द्वारा कुछ झगड़ों को सुलझाने का भी उल्लेख किया है।

प्रश्न 59.
“योग्य व्यक्तियों को पदवियाँ देना मुगल राजतन्त्र का एक महत्त्वपूर्ण पक्ष था।” विवेचना कीजिए।
अथवा
मुगल सम्राटों द्वारा अभिजात वर्ग के लोगों को प्रदान की जाने वाली पदवियों एवं पुरस्कारों का वर्णन कीजिए। उत्तर- ‘आसफ खाँ’ की पदवी उच्चतम मन्त्रियों को दी जाती थी। औरंगजेब ने अपने दो उच्चपदस्थ अभिजात जयसिंह और जसवन्त सिंह को ‘मिर्जा राजा’ की पदवी प्रदान की। योग्यता के आधार पर पदवियाँ अर्जित की जा सकती थीं कुछ मनसबदार धन देकर भी पदवियाँ प्राप्त करने का प्रयास करते थे। अन्य पुरस्कारों में सम्मान का ‘जामा’ (खिल्लत) भी शामिल था ‘सरप्पा’ एक अन्य उपहार था। इस उपहार के तीन भाग हुआ करते थे
(1) जामा
(2) पगड़ी तथा
(3) पटका।

प्रश्न 60.
दरबारियों एवं राजदूतों द्वारा मुगल बादशाहों को दिए जाने वाले उपहारों का वर्णन कीजिए।
उत्तर:
कोई भी दरबारी बादशाह के पास खाली हाथ नहीं जाता था। वह या तो नज के रूप में थोड़ा धन या पेशकश के रूप में मोटी रकम बादशाह की सेवा में प्रस्तुत करता था। राजनयिक सम्बन्धों में उपहारों को सम्मान और आदर का प्रतीक माना जाता था। राजदूत प्रतिद्वन्द्वी राजनीतिक शक्तियों के बीच सन्धि और सम्बन्धों के द्वारा समझौता करवाने का महत्त्वपूर्ण कार्य करते थे। ऐसी परिस्थितियों में उपहारों की महत्त्वपूर्ण प्रतीकात्मक भूमिका होती थी।

प्रश्न 61.
‘जात’ और ‘सवार’ मनसब से क्या अभिप्राय है?
उत्तर:
मुगलकाल में सभी सरकारी अधिकारियों के दर्जे और पर्दों में दो प्रकार के संख्या-विषयक पद होते थे-
(1) जात तथा
(2) सवार ‘जात’ शाही पदानुक्रम में अधिकारी (मनसबदार) के पद और वेतन का सूचक था। ‘सवार’ यह सूचित करता था कि उससे सेवा में कितने घुड़सवार रखना अपेक्षित था। सत्रहवीं शताब्दी में 1000 या उससे ऊपर जात वाले मनसबदार अभिजात ‘उमरा’ कहलाते थे।

JAC Class 12 History Important Questions Chapter 9 शासक और इतिवृत्त : मुगल दरबार

प्रश्न 62.
अभिजात वर्ग की सैनिक और असैनिक
कार्यों में क्या भूमिका थी?
उत्तर:
(1) अभिजात सैन्य अभियानों में अपनी सेनाओं के साथ भाग लेते थे।
(2) वे प्रान्तों में साम्राज्य के प्रशासनिक अधिकारियों के रूप में भी कार्य करते थे।
(3) प्रत्येक सैन्य कमांडर घुड़सवारों को भर्ती करता था। उन्हें हथियारों आदि से लैस करता था और उन्हें प्रशिक्षण देता था।
(4) घुड़सवार सेना मुगल सेना का अपरिहार्य अंग थी। घुड़सवार सैनिक उत्कृष्ट श्रेणी के घोड़े रखते थे।

प्रश्न 63,
“अभिजात वर्ग के सदस्यों के लिए शाही सेवा शक्ति, धन तथा उच्चतम प्रतिष्ठा प्राप्त करने का एक माध्यम थी।” स्पष्ट कीजिए।
उत्तर:
शाही सेवा में आने का इच्छुक व्यक्ति एक अभिजात के माध्यम से प्रार्थना-पत्र देता था जो बादशाह के सामने तजवीज प्रस्तुत करता था यदि प्रार्थी को सुयोग्य माना जाता था, तो उसे मनसब प्रदान किया जाता था। मीरबख्शी (उच्चतम वेतनदाता) खुले दरबार में बादशाह के दाएँ ओर खड़ा होता था तथा नियुक्ति और पदोन्नति के सभी उम्मीदवारों को प्रस्तुत करता था। उसका कार्यालय उसकी मुहर व हस्ताक्षर के साथ-साथ बादशाह की मुहर व हस्ताक्षर वाले आदेश तैयार करता था।

प्रश्न 64.
मुगलों के केन्द्रीय शासन पर एक संक्षिप्त टिप्पणी लिखिए।
उत्तर:
सम्राट अपने साम्राज्य का सर्वोच्च अधिकारी होता था साम्राज्य की समस्त शक्तियाँ उसी के हाथों में केन्द्रित थीं। शासन- कार्यों में सहायता व सलाह देने के लिए एक मन्त्रिपरिषद् होती थी। मीरबख्शी, दीवान-ए-आला तथा सद्र- उस सुदूर प्रमुख मन्त्री थे मीरबख्शी नियुक्ति और पदोन्नति के सभी उम्मीदवारों को प्रस्तुत करता था दीवान-ए-आला आय-व्यय का हिसाब रखता था तथा सद्र-उस-सुदूर अनुदान का मन्त्री और स्थानीय न्यायाधीशों की नियुक्ति का प्रभारी था।

प्रश्न 65.
“सटीक और विस्तृत आलेख तैयार करना मुगल प्रशासन के लिए मुख्य रूप से महत्त्वपूर्ण था। ” विवेचना कीजिए।
अथवा
मुगल दरबार की गतिविधियों से सम्बन्धित सूचनाएँ किस प्रकार लेखबद्ध की जाती थीं?
उत्तर:
मौरबख्शी दरबारी लेखकों (वाकिया नवीसों) के समूह का निरीक्षण करता था। ये लेखक ही दरबार में प्रस्तुत किए जाने वाले सभी अर्जियों एवं दस्तावेजों तथा सभी शासकीय आदेशों (फरमानों) का आलेख तैयार करते थे। इसके अतिरिक्त, अभिजातों और क्षेत्रीय शासकों के प्रतिनिधि (वकील), दरबार की बैठकों (पहर) की तिथि और समय के साथ ‘उच्च दरबार से समाचार’ (अखबारात- ए-दरबार-ए-मुअल्ला) शीर्षक के अन्तर्गत दरबार की सभी कार्यवाहियों का विवरण तैयार करते थे।

JAC Class 12 History Important Questions Chapter 9 शासक और इतिवृत्त : मुगल दरबार

प्रश्न 66.
अखबारात में मुगल दरबार से सम्बन्धित कौनसी सूचनाएँ होती थीं?
उत्तर:
अखबारात में हर प्रकार की सूचनाएँ होती थीं जैसे दरबार में उपस्थिति, पदों और पदवियों का दान, राजनयिक शिष्टमण्डलों, ग्रहण किए गए उपहारों अथवा किसी अधिकारी के स्वास्थ्य के बारे में बादशाह द्वारा की गई पूछताछ राजाओं और अभिजातों के सार्वजनिक और व्यक्तिगत जीवन का इतिहास लिखने के लिए ये सूचनाएँ बहुत उपयोगी होती थीं।

प्रश्न 67.
मुगल काल में साम्राज्य में समाचार वृत्तान्त और महत्त्वपूर्ण शासकीय दस्तावेज भेजे जाने की प्रक्रिया का वर्णन कीजिए।
उत्तर:
मुगल काल में समाचार वृत्तांत और महत्त्वपूर्ण शासकीय दस्तावेज शाही ठाक के द्वारा मुगल साम्राज्य में एक कोने से दूसरे कोने तक भेजे जाते थे। बांस के डिब्बों में लपेटकर रखे कागजों को लेकर हरकारों (कसीद अथवा पथमार) के दल दिन-रात दौड़ते रहते थे। काफी दूर स्थित प्रान्तीय राजधानियों से भी समाचार वृत्तान्त बादशाह को कुछ ही दिनों में मिल जाया करते थे। राजधानी से बाहर नियुक्त उच्च अधिकारी सूचनाएँ सन्देश वाहकों के द्वारा भेजते थे।

प्रश्न 68.
मुगलकालीन प्रान्तीय शासन का वर्णन कीजिए।
उत्तर:
मुगल साम्राज्य अनेक प्रान्तों में विभाजित था। प्रान्तीय शासन के संचालन के लिए सूबेदार, दीवान, बख्शी, सद्र आदि अधिकारी होते थे। प्रान्तीय शासन का प्रमुख गवर्नर (सूबेदार) होता था जो सीधा बादशाह को प्रतिवेदन प्रस्तुत करता था। प्रत्येक सूबा कई सरकारों में बंटा हुआ था। इन क्षेत्रों में फौजदारों को विशाल घुड़सवार सेना और तोपचियों के साथ नियुक्त किया जाता था। परगनों (उप- जिला) के प्रबन्ध के लिए कानूनगो, चौधरी तथा काजी नियुक्त किये जाते थे।

प्रश्न 69.
कन्धार सफावियों और मुगलों के बीच झगड़े की जड़ क्यों था? व्याख्या कीजिए।
उत्तर:
कन्धार ईरान के सफावियों एवं मुगलों के बीच झगड़े की जड़ था क्योंकि ईरान का शासक शाह अब्बास कन्धार पर अधिकार स्थापित करना चाहता था। 1613 में जहाँगीर ने शाह अब्बास के दरबार में कन्धार को मुगलों के आधिपत्य में रहने देने की वकालत करने के लिए एक राजनयिक दूत भेजा, परन्तु यह शिष्टमण्डल सफल नहीं हुआ। 1622 में एक ईरानी सेना ने कन्धार पर आक्रमण कर मुगल सेना को पराजित कर दिया और कन्धार के दुर्ग पर टि अधिकार कर लिया।

प्रश्न 70.
आटोमन साम्राज्य के साथ मुगलों के सम्बन्धों की विवेचना कीजिए।
उत्तर:
आरोमन साम्राज्य के साथ मुगलों ने अपने सम्बन्ध इस बात को ध्यान में रखकर बनाए कि वे आटोमन- नियन्त्रण वाले क्षेत्रों में व्यापारियों व तीर्थयात्रियों के स्वतन्त्र आवागमन को बनाये रखवा सकें मुगल बादशाह प्राय: धर्म एवं वाणिज्य के मुद्दों को मिलाने का प्रयास करता था। वह लाल सागर के बन्दरगाह अदन और मोरवा को बहुमूल्य वस्तुओं के निर्यात को प्रोत्साहन देता था और इनकी विक्री से प्राप्त आय को उस क्षेत्र के धर्म-स्थलों व फकीरों में दान में बाँट देता था।

JAC Class 12 History Important Questions Chapter 9 शासक और इतिवृत्त : मुगल दरबार

प्रश्न 71.
जेसुइट धर्म प्रचारकों के साथ मुगलों के सम्बन्धों की विवेचना कीजिए।
उत्तर:
अकबर ईसाई धर्म के विषय में जानने को बहुत उत्सुक था। उसके निमन्त्रण पर अनेक जेसुइट शिष्टमण्डल मुगल दरबार में आए। अकबर ने इन जेसुइट लोगों से ईसाई धर्म के विषय में वार्तालाप किया और उनसे बड़ा प्रभावित हुआ। जेसुइट लोगों को सार्वजनिक सभाओं में अकबर के सिंहासन के पास जगह दी जाती थी। वे उसके साथ अभियानों में जाते, उसके बच्चों को शिक्षा देते तथा उसके फुरसत के समय में वे प्रायः उसके साथ होते थे।

प्रश्न 72.
अकबर के धार्मिक विचारों के विकास का वर्णन कीजिए।
अथवा
अकबर की धार्मिक नीति का वर्णन कीजिये। उत्तर- 1575 में अकबर ने फतेहपुर सीकरी में इबादतखाने का निर्माण करवाया। उसने यहाँ इस्लाम, हिन्दू धर्म, ईसाई धर्म, जैन धर्म, पारसी धर्म के विद्वानों को आमन्त्रित किया। अकबर के धार्मिक विचार विभिन्न धर्मों व सम्प्रदायों के विद्वानों से प्रश्न पूछने और उनके धर्म-सिद्धान्तों के बारे में जानकारी एकत्रित करने से विकसित हुए। धीरे- धीरे वह धर्मों को समझने के रूढ़िवादी तरीकों से दूर प्रकाश और सूर्य पर केन्द्रित दैवीय उपासना की ओर बढ़ा।

निबन्धात्मक प्रश्न-

प्रश्न 1.
मुगल साम्राज्य के उत्कर्ष पर एक निबन्ध लिखिए।
अथवा
मुगल साम्राज्य के उत्थान व पतन का वर्णन कीजिए। उत्तर- मुगलों की उत्पत्ति ‘मुगल’ शब्द की उत्पत्ति मंगोल से हुई है। मुगलों ने स्वयं को ‘तैमूरी’ कहा क्योंकि पितृपक्ष से वे तुर्की शासक तिमूर के वंशज थे प्रथम मुगल शासक बाबर मातृपक्ष से चंगेज खाँ का सम्बन्धी था। उस समय से ही इस शब्द का निरन्तर प्रयोग होता रहा है।
मुगल साम्राज्य का उत्कर्ष
(1) जहीरुद्दीन बाबर जहीरुद्दीन बाबर मुगल- साम्राज्य का संस्थापक था उसे उसके प्रतिद्वन्द्वी उजबेकों ने उसके अपने देश फरगना से भगा दिया था। उसने सबसे पहले काबुल पर अधिकार किया। इसके बाद 1526 में अपने दल के सदस्यों की आवश्यकताएं पूरी करने के लिए क्षेत्रों और संसाधनों की खोज में वह भारतीय उपमहाद्वीप में और आगे की ओर बढ़ा। 1530 में बाबर की मृत्यु हो गई।

(2) हुमायूँ बाबर की मृत्यु के बाद 1530 में हुमायूँ गद्दी पर बैठा। उसने अपने शासनकाल (1530-40.1555- 56) में साम्राज्य की सीमाओं का विस्तार किया। परन्तु वह अफगान नेता शेरशाह सूरी से पराजित हो गया। उसे विवश होकर भारत से भागना पड़ा। उसे ईरान के सफावी शासक के दरबार में शरण लेनी पड़ी 1555 में हुमायूँ ने अफगानों को पराजित कर दिया। परन्तु एक वर्ष बाद ही उसकी मृत्यु हो गई।

(3) जलालुद्दीन अकबर हुमायूँ की मृत्यु के बाद उसका पुत्र जलालुद्दीन अकबर (1556-1605) गद्दी पर बैठा। वह मुगल राजवंश का सबसे महान शासक माना जाता है। अकबर ने हिन्दुकुश पर्वत तक अपने साम्राज्य की सीमाओं का विस्तार किया। उसने ईरान के सफ़ावियों और तूरान (मध्य एशिया) के उजबेकों की विस्तारवादी योजनाओं पर अंकुश लगाए रखा। 1605 में अकबर की मृत्यु हो गई।

JAC Class 12 History Important Questions Chapter 9 शासक और इतिवृत्त : मुगल दरबार

(4) अकबर के उत्तराधिकारी अकबर के बाद जहाँगीर (1605-27), शाहजहाँ (162858) तथा औरंगजेब (1658-1707) गद्दी पर बैठे ये तीनों ही बहुत योग्य उत्तराधिकारी सिद्ध हुए। इनके अधीन क्षेत्रीय विस्तार जारी रहा, यद्यपि इस विस्तार की गति काफी धीमी रही। उन्होंने शासन के विविध यन्त्रों को बनाए रखा और उन्हें सुदृढ़ किया।

(5) शाही संस्थाओं के ढाँचे का निर्माण इनके अन्तर्गत प्रशासन और कराधान के प्रभावशाली तरीके सम्मिलित थे। मुगलों द्वारा शुरू की गई राजनीतिक व्यवस्था सैन्य शक्ति और उपमहाद्वीप की भिन्न-भिन्न परम्पराओं को समायोजित करने की बुद्धिमत्तापूर्ण नीति पर आधारित थी।

(6) मुगल साम्राज्य का पतन 1707 में औरंगजेब की मृत्यु हो गई। उसकी मृत्यु के पश्चात् मुगलं साम्राज्य 1 का विघटन शुरू हो गया। फिर भी सांकेतिक रूप से मुगल साम्राज्य की प्रतिष्ठा बनी रही। 1857 में मुगल वंश के | अन्तिम शासक बहादुर शाह जफर द्वितीय को अंग्रेजों ने उखाड़ फेंका।

प्रश्न 2.
अबुल फजल के मुगल साम्राज्य में योगदान के विषय में आप क्या जानते हैं? विस्तार से समझाइए।
उत्तर:
अबुल फजल अकबर का एक अति महत्त्वपूर्ण दरबारी था। उसने अकबरनामा जैसे ग्रन्थों की रचना की थी। अबुल फजल का पालन-पोषण मुगल राजधानी आगरा हुआ था। वह अरबी तथा फारसी का अच्छा ज्ञाता था: साथ ही वह यूनानी दर्शन और सूफीवाद में भी रुचि रखता था अबुल फजल एक प्रभावशाली विचारक तथा तर्कशास्त्री था। उसने दकियानूसी उलेमाओं के विचारों का पूर्ण विरोध किया, जिसके कारण अकबर उससे अत्यधिक प्रभावित हुआ अकबर ने उसे अपने सलाहकार तथा अपनी नीतियों के प्रवक्ता के रूप में उपयुक्त पाया। अबुल फजल ने दरबारी इतिहासकार के रूप में अकबर के शासन से जुड़े विचारों को न केवल आकार दिया अपितु उनको स्पष्ट रूप से व्यक्त भी किया।

मुगल साम्राज्य में अबुल फजल का योगदान- अबुल फजल का महत्त्वपूर्ण योगदान अकबर के समय की ऐतिहासिक घटनाओं के वास्तविक विवरणों, शासकीय दस्तावेजों तथा जानकार व्यक्तियों के मौखिक प्रमाणों जैसे विभिन्न प्रकार के साक्ष्यों पर आधारित, अकबरनामा ग्रन्थ की रचना था। इस ग्रन्थ की रचना उसने 1589 ई. में आरम्भ की तथा इसे पूर्ण करने में उसे लगभग 13 वर्ष लगे। अबुल फजल का अकबरनामा तीन जिल्दों में है जिसमें प्रथम दो भाग इतिहास से सम्बन्धित हैं तथा तृतीय भाग आइन-ए-अकबरी है। प्रथम जिल्द में आदम से लेकर अकबर के जीवन के एक खगोलीय कालचक्र तक का मानव जाति का इतिहास है। द्वितीय जिल्द अकबर के 46वें शासन वर्ष पर खत्म होती है।

अकबरनामा का लेखन राजनीतिक रूप से महत्त्वपूर्ण पटनाओं का विवरण देने के उद्देश्य से किया गया। अकबर के साम्राज्य की तिथियों तथा समय के साथ होने वाले परिवर्तनों के उल्लेख के बिना ही भौगोलिक, सामाजिक, प्रशासनिक तथा सांस्कृतिक सभी पक्षों का विवरण प्रस्तुत करने के अभिन्न तरीके से भी इसका लेखन हुआ। अबुल फजल की आइन-ए-अकबरी में मुगल साम्राज्य को हिन्दुओं, जैनों, बौद्धों तथा मुसलमानों की भिन्न-भिन्न जनसंख्या वाले तथा एक मिश्रित संस्कृति वाले साम्राज्य के रूप में प्रस्तुत किया गया है। इससे भारतीय- फारसी शैली को दरबार में संरक्षण प्राप्त हुआ।

प्रश्न 3.
मुगलकाल में फारसी भाषा के विकास का वर्णन कीजिए।
अथवा
मुगल सम्राटों ने फारसी भाषा को महत्त्व क्यों प्रदान किया?
उत्तर:
मुगलकाल में फारसी भाषा का विकास मुगलों के दरबारी इतिहास फारसी भाषा में लिखे गए थे। दिल्ली के सुल्तानों के काल में उत्तर भारतीय भाषाओं के साथ-साथ फारसी दरवार तथा साहित्यिक रचनाओं की भाषा के रूप में खूब विकसित हुई। चूंकि मुगल चग़ताई मूल के थे, अतः तुर्की उनकी मातृभाषा थी प्रथम मुगल सम्राट बाबर ने कविताएँ और अपने संस्मरण तुर्की भाषा में ही लिखे थे।
(1) फारसी को दरबार की मुख्य भाषा बनाना- मुगल सम्राट अकबर ने फारसी को दरबार की मुख्य भाषा बनाया।

सम्भवतः ईरान के साथ सांस्कृतिक और बौद्धिक सम्पर्कों के साथ-साथ मुगल दरबार में पद पाने के इच्छुक ईरानी और मध्य एशियाई प्रवासियों ने बादशाह को फारसी भाषा को अपनाए जाने के लिए प्रेरित किया होगा। फारसी को दरबार की भाषा का ऊंचा स्थान दिया गया तथा उन लोगों को “शक्ति एवं प्रतिष्ठा प्रदान की गई जिनका इस भाषा पर अच्छा नियन्त्रण था। सम्राट शाही परिवार के लोग और दरबार के विशिष्ट सदस्य फारसी भाषा बोलते थे। कुछ और आगे, यह सभी स्तरों के प्रशासन की भाषा बन गई, जिससे लेखाकारों, लिपिकों तथा अन्य अधिकारियों ने भी इसे सीख लिया।

JAC Class 12 History Important Questions Chapter 9 शासक और इतिवृत्त : मुगल दरबार

(2) अन्य भाषाओं को प्रभावित करना- जहाँ-जहाँ फारसी प्रत्यक्ष प्रयोग में नहीं आती थी, वहाँ भी राजस्थानी, मराठी और यहाँ तक कि तमिल में शासकीय लेखकों की भाषा को फारसी की शब्दावली और मुहावरों ने व्यापक रूप से प्रभावित किया। चूंकि सोलहवीं तथा सत्रहवीं शताब्दियों के दौरान फारसी का प्रयोग करने वाले लोग उपमहाद्वीप के भिन्न-भिन्न क्षेत्रों से आए थे और वे अन्य भारतीय भाषाएँ भी बोलते थे, इसलिए स्थानीय मुहावरों को शामिल करने से फारसी का भी भारतीयकरण हो गया था। फारसी के हिन्दवी के साथ पारस्परिक सम्पर्क से उर्दू के रूप में एक नई भाषा का उद्भव हुआ।

(3) फारसी में अनुवाद ‘ अकबरनामा’ और ‘बादशाहनामा’ जैसे मुगल इतिहास फारसी में लिखे गए थे, जबकि बाबर के संस्मरणों का ‘बाबरनामा’ के नाम से तुर्की से फारसी में अनुवाद किया गया था। मुगल बादशाहों ने ‘महाभारत’ और ‘रामायण’ जैसे संस्कृत ग्रन्थों का फारसी में अनुवाद करवाया। ‘महाभारत’ का अनुवाद ‘रज्मनामा’ (युद्धों की पुस्तक) के रूप में हुआ।

प्रश्न 4.
“मुगलकाल में मानव रूपों का चित्रण निरन्तर तनाव का कारण था।” उपर्युक्त तर्क देकर इस कथन का औचित्य निर्धारित कीजिए।
उत्तर:
मुगलकाल में मानव रूपों का चित्रण निरन्तर तनाव का कारण था। इस कथन के पक्ष में निम्नलिखित तर्क प्रस्तुत हैं-
(i) मुगलकाल में बादशाह, उसके दरबार एवं उसमें भाग लेने वाले लोगों का चित्रण करने वाले चित्रों की रचना को लेकर शासकों और मुसलमानों, रूढ़िवादी वर्ग के प्रतिनिधियों अर्थात् उलेमाओं के मध्य लगातार तनाव बना रहा। उलेमाओं ने कुरान के साथ-सात हदीस जिसमें पैगम्बर मुहम्मद के जीवन से एक ऐसा ही प्रसंग वर्णित है, में प्रतिष्ठित मानव रूपों के चित्रण पर इस्लामी प्रतिबन्ध का आह्वान किया।

(ii) इस प्रसंग में पैगम्बर मुहम्मद को प्राकृतिक तरीके से जीवित रूपों के चित्रण की मनाही करते हुए उल्लिखित किया गया है। क्योंकि ऐसा करने में लगता है कि कलाकार रचना की शक्ति को अपने हाथ में लेने का प्रयास कर रहा है। यह एक ऐसा कार्य था, जो केवल ईश्वर का ही था।

(iii) समय के साथ-साथ शरिया की व्याख्याओं में भी परिवर्तन आया। विभिन्न सामाजिक समूहों ने इस्लामी परम्परा के ढाँचे की अलग-अलग तरीकों से व्याख्या की। प्रायः प्रत्येक समूह ने इस परम्परा की ऐसी व्याख्या प्रतिपादित की, जो उनकी राजनीतिक आवश्यकताओं में सबसे अधिक समानता रखती थी। जिन शताब्दियों के दौरान सामान्य निर्माण हो रहा था। उस समय कई चित्रकार स्रोतों के शासकों ने नियमित रूप से कलाकारों को उनके चित्र एवं उनके राज्य के जीवन के दृश्य चित्रित करने के लिए नियुक्त किया। मुगलकालीन भारत में ईरान से कई चित्रकार आए: जैसे—मीर सैयद अली एवं अब्दुल समद।

(iv) बादशाह एवं रूढ़िवादी मुसलमान विचारधारा के प्रवक्ताओं के मध्य मानवों के दृश्य निरूपण पर मुगल दरबार में तनाव बना हुआ था। अकबर का दरबारी इतिहासकार अबुल फजल बादशाह को यह कहते हुए उद्धृत करता है” कई लोग ऐसे हैं जो चित्रकला को नापसन्द करते हैं। पर मैं ऐसे व्यक्तियों को नापसन्द करता हूँ। मुझे ऐसा लगता है कि कलाकार के पास ख़ुदा को पहचानने का बेजोड़ तरीका है। क्योंकि कहीं न कहीं उसे ये महसूस होता है कि खुदा की रचना को वह जीवन नहीं दे सकता।”

(v) 17वीं शताब्दी में मुगल चित्रकारों ने बादशाहों को प्रभामण्डल के साथ चित्रित करना प्रारम्भ कर दिया। ईश्वर के प्रकाश के प्रतीक के रूप में इन प्रभामण्डलों को उन्होंने ईसा और वर्जिन मैरी के यूरोपीय चित्रों में देखा था।

प्रश्न 5.
मुगल पांडुलिपियों की रचना में चित्रकारों की भूमिका का वर्णन कीजिए। चित्रों के महत्त्व को स्पष्ट कीजिए। चित्रों की रचना को लेकर तनाव क्यों बना रहा? उत्तर- मुगल पांडुलिपियों की रचना में चित्रकारों की भूमिका मुगल पांडुलिपियों की रचना में चित्रकारों की महत्वपूर्ण भूमिका थी। किसी मुगल सम्राट के शासन की घटनाओं का विवरण देने वाले इतिहासों में लिखित पाठ के साथ ही उन घटनाओं को चित्रों के माध्यम से दृश्य रूप में भी वर्णित किया जाता था।
किसी
(1) चित्रों का महत्त्व –

  • सौन्दर्य में वृद्धि चित्र पुस्तक के सौन्दर्य में वृद्धि करते थे।
  • चित्रों के माध्यम से राजा की शक्ति को व्यक्त करना- लिखित माध्यम से राजा और राजा की शक्ति विषय में जो बात नहीं कही जा सकी हों, उन्हें चित्रों के माध्यम से व्यक्त कर दिया जाता था। के
  • निर्जीव वस्तुओं को सजीव बनाना इतिहासकार अबुल फजल ने चित्रकारी को एक ‘जादुई कला’ की संज्ञा दी है। उसके अनुसार चित्रकला निर्जीव वस्तुओं में भी प्राण फूंक सकती है।

(2) चित्रों की रचना को लेकर तनाव-मुगल सम्राट तथा उसके दरबार का चित्रण करने वाले चित्रों की रचना को लेकर शासकों और उलमा के बीच तनाव की स्थिति बनी रही। उलमा रूढ़िवादी मुसलमान वर्ग के प्रतिनिधि थे। उन्होंने कुरान के साथ-साथ हदीस में प्रतिष्ठापित मानव रूपों के चित्रण पर इस्लामी प्रतिबन्ध का आह्वान किया।

(3) नियमित रूप से कलाकारों को नियुक्त करना- कई एशियाई शासकों ने अपने तथा अपने राज्य के जीवन के दृश्य चित्रित करने के लिए नियमित रूप से कलाकारों को नियुक्त किया। उदाहरण के लिए, ईरान के सफावी राजाओं ने अपने दरबार की कार्यशालाओं में प्रशिक्षित एवं उत्कृष्ट कलाकारों को संरक्षण प्रदान किया। विहजाद जैसे चित्रकारों ने सफ़ावी दरबार की सांस्कृतिक प्रसिद्धि को चारों ओर फैलाने में महत्त्वपूर्ण योगदान दिया।

(4) ईरानी कलाकारों का मुगलकालीन भारत में आगमन – मुगलकाल में ईरानी कलाकार भी भारत में आए। ईरान के प्रसिद्ध चित्रकार मीर सैयद अली तथा अब्दुस समद बादशाह हुमायूँ के साथ दिल्ली आए। कुछ अन्य कलाकार संरक्षण और प्रतिष्ठा के अवसरों की तलाश में भारत पहुँचे। परन्तु बादशाह और रूढ़िवादी मुस्लिम विचारधारा के प्रवक्ताओं के बीच जीवधारियों के चित्र बनाने पर मुगल दरबार में तनाव की स्थिति बनी रही।

JAC Class 12 History Important Questions Chapter 9 शासक और इतिवृत्त : मुगल दरबार

प्रश्न 6.
अबुल फजल चित्रकला को महत्त्वपूर्ण क्यों मानता था? उसने अकबरकालीन चित्रकला की किन विशेषताओं का वर्णन किया है?
उत्तर:
चित्रकला का महत्त्व अबुल फसल निम्नलिखित आधारों पर चित्रकला को महत्त्वपूर्ण मानता था-
(1) चित्रकला एक जादुई कला इतिहासकार अबुल फजल ने चित्रकला को एक ‘जादुई कला’ की संज्ञा दी है उसके अनुसार चित्रकला किसी भी निर्जीव वस्तु में प्राण फूँक सकती है।

(2) मुगल सम्राटों की चित्रकला में रुचि अबुल फसल के अनुसार मुगल सम्राट अकबर ने अपनी युवावस्था के प्रारम्भिक दिनों से ही चित्रकला में अपनी अभिरुचि व्यक्त की थी। वह चित्रकला को अध्ययन और मनोरंजन दोनों का ही साधन मानते थे तथा इस कला को हर सम्भव प्रोत्साहन देते थे। मुगल पांडुलिपियों में चित्रों के अंकन के लिए चित्रकारों की एक बड़ी संख्या नियुक्त की गई थी। हर सप्ताह शाही कार्यालय के अनेक निरीक्षक और लिपिक बादशाह के सामने प्रत्येक कलाकार का कार्य प्रस्तुत करते थे। इन कृतियों के अवलोकन के बाद बादशाह प्रदर्शित उत्कृष्टता के आधार पर चित्रकारों को पुरस्कृत करते थे तथा उनके मासिक वेतन में वृद्धि करते थे।

(3) अकबरकालीन चित्रकला की विशेषताएँ- अकबरकालीन चित्रकला की विशेषताएँ निम्नलिखित थीं-

  • उत्कृष्ट चित्रकारों का उपलब्ध होना चित्रकारों को प्रोत्साहन दिए जाने के परिणामस्वरूप अकबर के समय में उत्कृष्ट चित्रकार मिलने लगे थे। बिहजाद जैसे चित्रकारों की सर्वोत्तम कलाकृतियों को तो उन यूरोपीय चित्रकारों की उत्कृष्ट कलाकृतियों के समक्ष रखा जा सकता था जिन्होंने विश्व में अत्यधिक ख्याति प्राप्त कर ली थी।
  • विवरण की सूक्ष्मता, परिपूर्णता तथा प्रस्तुतीकरण की निर्भीकता विवरण की सूक्ष्मता, परिपूर्णता तथा प्रस्तुतीकरण की निर्भीकता जो अकबरकालीन चित्रों में दिखाई पड़ती थी, वह अतुलनीय थी यहाँ तक कि निर्जीव वस्तुएँ भी सजीव प्रतीत होती थीं।
  • हिन्दू चित्रकारों की दक्षता अकबर के प्रोत्साहन के परिणामस्वरूप उसके दरबार में सौ से भी अधिक चित्रकार थे, जो इस कला की साधना में लगे हुए थे। हिन्दू कलाकारों के लिए यह बात विशेष तौर पर सही थी। उनके चित्र वस्तुओं की हमारी परिकल्पना से परे थे। वस्तुतः सम्पूर्ण विश्व में कुछ लोग ही उनकी तुलना कर सकते थे।

प्रश्न 7.
सोलहवीं और सत्रहवीं शताब्दी के दौरान मुगलों की राजधानियाँ तीव्रता से स्थानान्तरित होने लगीं। | व्याख्या कीजिए।
उत्तर:
राजधानी नगर मुगल साम्राज्य का केन्द्रीय स्थल होता था दरबार का आयोजन इसी केन्द्रीय स्थल राजधानी के नगर में किया जाता था मुगलों की राजधानियाँ सोलहवीं और सत्रहवीं शताब्दी में तीव्रता से एक नगर से दूसरे नगर में स्थानान्तरित होने लग। बाबर, जिसने कि मुगल साम्राज्य की स्थापना की थी, ने इब्राहीम लोदी के द्वारा स्थापित राजधानी आगरा पर अधिकार कर लिया था। अपने शासन काल के चार वर्षों में अपने दरबार भिन्न-भिन्न स्थानों पर लगाए। 1560 ई. में अकबर ने आगरा में किले का निर्माण करवाया, जिसे आस-पास की खदानों से लाए गए लाल बलुआ पत्थर से निर्मित किया गया था।

(1) फतेहपुर सीकरी – अकबर ने 1570 ई. के दशक में अपनी नई राजधानी फतेहपुर सीकरी में बनाने का निर्णय लिया। इसका कारण सम्भवतः यह माना जाता है कि अकबर प्रसिद्ध सूफी संत शेख मुईनुद्दीन चिश्ती का मुरीद था जिनकी दरगाह अजमेर में है। फतेहपुर सीकरी अजमेर जाने वाली सीधी सड़क पर स्थित था। अकबर ने फतेहपुर सीकरी में जुम्मा मस्जिद के निकट ही शेख सलीम चिश्ती के लिए संगमरमर के मकबरे का निर्माण भी करवाया। अकबर ने फतेहपुर सीकरी में ही एक विशाल मेहराबी प्रवेश द्वार: जिसे बुलन्द दरवाजा कहा जाता है, का निर्माण भी करवाया।

(2) लाहौर अकबर ने 1585 ई. में अपनी राजधानी को लाहौर स्थानान्तरित कर दिया। लाहौर में राजधानी के स्थानान्तरण का कारण उत्तरी-पश्चिमी सीमा पर प्रभावशाली नियन्त्रण स्थापित करना था।

(3) शाहजहांनाबाद में राजधानी की स्थापना- शाहजहाँ के पास भवन निर्माण के लिए व्यापक धन राशि का प्रबन्ध था। शाहजहाँ ने दिल्ली के प्राचीन रिहायशी नगर में शाहजहाँनाबाद के नाम से नई राजधानी का निर्माण किया। शाहजहांनाबाद एक नई शाही आबादी के रूप में विकसित हुआ। मुख्य बात तो यह है कि शाहजहाँ की पुत्री जहाँआरा ने इस नई राजधानी की कई वास्तुकलात्मक संरचनाओं की रूपरेखा निर्मित की: जिनमें चाँदनी चौक और कारवाँ सराय नामक दोमंजिली इमारत प्रमुख थी।

प्रश्न 8.
अकबरनामा के ऐतिहासिक महत्त्व का विवेचन कीजिए।
अथवा
‘अकबरनामा’ पर एक लघु निबन्ध लिखिए।
अथवा
‘अबुल फजल’ और ‘अकबरनामा’ का परिचय दीजिए।
उत्तर:
अबुल फजल का परिचयं अबुल फसल मुगल सम्राट अकबर का दरबारी इतिहासकार था। अबुल फजल का पालन पोषण मुगल राजधानी आगरा में हुआ था वह अरबी, फारसी, यूनानी दर्शन और सूफीवाद का प्रकाण्ड विद्वान् था। वह एक प्रभावशाली तर्कशील व्यक्ति तथा स्वतन्त्र चिन्तक था। उसने आजीवन रूढ़िवादी उलमा के विचारों का विरोध किया। अबुल फसल के इन गुणों से अकबर बहुत प्रभावित हुआ। दरबारी इतिहासकार के रूप में अबुल फजल ने अकबर के शासन से जुड़े विचारों को न केवल आकार प्रदान किया, बल्कि उनको स्पष्ट रूप से व्यक्त भी किया।

JAC Class 12 History Important Questions Chapter 9 शासक और इतिवृत्त : मुगल दरबार

‘अकबरनामा’
‘अकबरनामा’ महत्त्वपूर्ण चित्रित मुगल इतिहासों में सर्वाधिक प्रसिद्ध है। इसमें लगभग 150 पूरे अथवा दोहरे पृष्ठों पर लड़ाई, घेराबन्दी, शिकार, भवन-निर्माण, दरबारी दृश्य आदि के चित्र हैं। अबुल फजल ने 1589 में शुरू कर ‘ अकबरनामा’ पर तेरह वर्षों तक कार्य किया और इस दौरान उसने कई बार इसके प्रारूप में सुधार किया। यह इतिहास घटनाओं के वास्तविक विवरणों, शासकीय दस्तावेजों तथा जानकार व्यक्तियों के मौखिक प्रमाणों पर आधारित है।
(1) तीन जिल्दों में विभक्त’ अकबरनामा’ को तीन जिल्दों में विभाजित किया गया है, जिनमें से प्रथम दो जिल्दें इतिहास हैं। तीसरी जिल्द ‘आइन-ए-अकबरी’ है।

  • पहली जिल्द पहली जिल्द में आदम से लेकर अकबर के जीवन के एक खगोलीय कालचक्र तक (30 वर्ष) का मानव जाति का इतिहास है।
  • दूसरी जिल्द दूसरी जिल्द अकबर के 46 वें शासन वर्ष (1601 ई.) पर समाप्त होती है। 1602 में राजकुमार सलीम ने अबुल फजल की हत्या का एक षड्यन्त्र रचा जिसके अनुसार सलीम के सहापराधी बीरसिंह बुन्देला ने ‘अबुल फसल की हत्या कर दी।
  • तीसरी जिल्द-तीसरी जिल्द ‘आइन-ए-अकबरी’ है। इसमें मुगल साम्राज्य को हिन्दुओं, जैनियों, बौद्धों और मुसलमानों की भिन्न-भिन्न आबादी वाले तथा एक मिश्रित संस्कृति वाले साम्राज्य के रूप में प्रस्तुत किया गया है।

(2) ‘अकबरनामा’ का ऐतिहासिक महत्त्व इसमें तिथियों और समय के साथ होने वाले परिवर्तनों के उल्लेख के बिना ही अकबर साम्राज्य के भौगोलिक, सामाजिक, संजीव पास बुक्स प्रशासनिक और सांस्कृतिक सभी पक्षों का विवरण प्रस्तुत किया गया है।

(3) अबुल फजल की भाषा अबुल फजल की भाषा बहुत अलंकृत थी इस भारतीय फारसी शैली को मुगल दरबार में संरक्षण प्राप्त हुआ।

प्रश्न 9.
मुगल साम्राज्य के सूचना तन्त्र का वर्णन कीजिए।
उत्तर:
मुगल साम्राज्य में व्यापक सूचना तन्त्र विकसित था साम्राज्य से सम्बन्धित सभी क्रिया-कलापों का व्यापक आलेखन किया जाता था। सभी महत्त्वपूर्ण सूचनाओं को दर्ज किया जाता था। लेखन के लिए दरबारी लेखकों, जिन्हें वाकया नवीस कहा जाता था की नियुक्ति की गयी थी। मीर बख्शी इन लेखकों के कार्यों का निरीक्षण करता था। वाकया नवीस ही दरबार में प्रस्तुत की जाने वाली अर्जियों, दस्तावेजों और शासकीय आदेशों को लिपिबद्ध करते थे।

इसके अतिरिक्त अखबारात ए- दरबारी )- मुअल्ला उच्च दरबार से समाचार शीर्षक के अन्तर्गत ओभजातों और क्षेत्रीय शासकों के प्रतिनिधि दरबार की सभी कार्यवाहियों का विवरण तैयार करते थे। इन अखवारों में सभी प्रकार की सूचनाओं: जैसे- दरबार में उपस्थित लोगों का विवरण, पदों और पदवियों का दान, राजनयिक शिष्टमण्डलों से जुड़ी हुई सूचनाएँ, ग्रहण तथा दान किए गए उपहारों आदि का दिनांक और समय के साथ लेखन किया जाता था। यह सूचनाएँ राजाओं के सार्वजनिक और व्यक्तिगत जीवन का इतिहास लिखने के लिए महत्त्वपूर्ण स्रोत होती हैं।

सूचनाओं को एक स्थान से दूसरे स्थान पर भेजने के लिए पूरा साम्राज्य एक तन्त्र से जुड़ा हुआ था। समाचार वृत्तान्त और महत्त्वपूर्ण शासकीय दस्तावेज डाक के जरिए बाँस के डिब्बे में लपेटकर हरकारों के दलों द्वारा भेजे जाते थे। हरकारों के यह दल क्रमबद्ध रूप से दिन-रात दौड़ते रहते थे। इस प्रकार बादशाह को राजधानी से काफी दूर स्थित प्रान्तों के समाचार कुछ ही दिन में मिल जाते थे। प्रान्तों में राजधानी के अभिजातों के प्रतिनिधि दरवार द्वारा माँगी गई जानकारी को एकत्र कर सन्देशवाहकों के जरिए अपने अधिकारी के पास भेज दिया करते थे। विस्तृत मुगल साम्राज्य के सुचारु संचालन हेतु यह सूचना तन्त्र अति आवश्यक था।

JAC Class 12 History Important Questions Chapter 9 शासक और इतिवृत्त : मुगल दरबार

प्रश्न 10.
‘सुलह-ए-कुल’ के सिद्धान्त की विवेचना कीजिए। इसे किस प्रकार लागू किया गया?
उत्तर:
‘सुलह-ए-कुल’ के सिद्धान्त मुगल इतिवृत्त मुगल साम्राज्य को हिन्दुओं, जैनों, जरतुश्तियों और मुसलमानों के अनेक नृजातीय और धार्मिक समुदायों के समूह के रूप में प्रस्तुत करते हैं। शान्ति और स्थायित्व के स्त्रोत के रूप में मुगल सम्राट सभी धार्मिक और नृजातीय समूहों से ऊपर होता था वह इनके बीच मध्यस्थता करता था तथा यह सुनिश्चित करता था कि न्याय और शान्ति बनी रहे।
(1) प्रबुद्ध शासन का आधार अबुल फजल सुलाह- ए-कुल (पूर्ण शान्ति) के आदर्श को प्रबुद्ध शासन का आधार बताता है।
(2) अभिव्यक्ति की स्वतन्त्रता-‘सुलह-ए-कुल’ में सभी धर्मों और मतों को अभिव्यक्ति की स्वतन्त्रता थी।
(3) राज्य सत्ता को क्षति नहीं पहुँचाना-किन्तु उसकी एक शर्त थी कि वे राज्य सत्ता को क्षति नहीं पहुँचायेंगे अथवा आपस में नहीं लड़ेंगे।

सुलह-ए-कुल के आदर्श को लागू करना-
(1) अभिजात वर्ग को पद और पुरस्कार देना- सुलह-ए-कुल का आदर्श राज्य नीतियों के द्वारा लागू किया गया। मुगलों के अधीन अभिजात वर्ग में ईरानी, तूरानी, अफगानी राजपूत, दक्खनी सभी सम्मिलित थे। इन सब को पद और पुरस्कार दिए गए जो पूरी तरह से राजा के प्रति उनकी सेवा और निष्ठा पर आधारित थे।

(2) तीर्थयात्रा कर तथा जजिया समाप्त करना- 1563 में अकबर ने तीर्थयात्रा कर तथा 1564 में जजिया को समाप्त कर दिया क्योंकि ये दोनों कर धार्मिक पक्षपात पर आधारित थे।

(3) प्रशासन में सुलह-ए-कुल के नियमों का अनुपालन करवाना साम्राज्य के अधिकारियों को आदेश दिए गए कि ये प्रशासन में सुलह-ए-कुल’ के नियमों का अनुपालन करवाएं।

(4) सहिष्णुतापूर्ण धार्मिक नीति- अधिकांश मुगल बादशाहों ने सहिष्णुतापूर्ण धार्मिक नीति अपनाई। उन्होंने उपासना स्थलों के निर्माण व रख-रखाव के लिए अनुदान दिए। यहाँ तक कि युद्ध के दौरान जब मन्दिर नष्ट कर दिए जाते थे, तो बाद में सम्राटों द्वारा उनकी मरम्मत के लिए अनुदान जारी किए जाते थे। ऐसा हमें शाहजहाँ और औरंगजेब के शासन में पता चलता है, यद्यपि औरंगजेब के शासन काल में गैर-मुसलमान प्रजा पर जजिया फिर से लगा दिया गया।

प्रश्न 11.
मुगलकालीन राजधानी नगरों का वर्णन कीजिए।
उत्तर:
मुगलकालीन राजधानी नगर मुगल राजधानी नगर मुगल साम्राज्य का हृदय स्थल था। राजधानी नगर में ही दरबार लगता था। सोलहवीं और सत्रहवीं शताब्दियों के दौरान मुगलों की राजधानी तेजी से स्थानान्तरित होने लगीं। 1560 के दशक में अकबर ने आगरा के किले का निर्माण करवाया। इसे लाल बलुआ पत्थर से बनाया गया था। यह बलुआ पत्थर आस-पास के क्षेत्रों की खदानों से लाया गया था।

(1) फतेहपुर सीकरी – 1570 के दशक में अकबर ने फतेहपुर सीकरी में एक नई राजधानी बनाने का निर्णय लिया। मुगल सम्राटों के चिश्ती सिलसिले के सूफियों के साथ घनिष्ठ सम्बन्ध स्थापित हो गए थे। अकबर ने फतेहपुर सीकरी में जुम्मा मस्जिद के बगल में ही शेख सलीम चिश्ती के लिए सफेद संगमरमर का एक मकबरा बनवाया। उसने एक विशाल मेहराबी प्रवेशद्वार अथवा बुलन्द दरवाजा भी बनवाया जिसका उद्देश्य वहाँ आने वाले लोगों को गुजरात में मुगल विजय की याद दिलाना था।

(2) लाहौर 1585 ई. में उत्तर-पश्चिमी सीमा पर प्रभावशाली नियन्त्रण स्थापित करने के लिए राजधानी को लाहौर स्थानान्तरित कर दिया गया। इस प्रकार तेरह वर्षों तक अकबर ने इस सीमा पर गहरी चौकसी बनाए रखी।

JAC Class 12 History Important Questions Chapter 9 शासक और इतिवृत्त : मुगल दरबार

(3) शाहजहाँ की भवन निर्माण में गहरी रुचि- शाहजहाँ की भवन निर्माण में गहरी रुचि थी। उसने विवेकपूर्ण राजकोषीय नीतियों को आगे बढ़ाया तथा भवन निर्माण की अपनी अभिरुचि की पूर्ति के लिए पर्याप्त धन इकट्ठा कर लिया। राजकीय संस्कृतियों में भवन निर्माण कार्य राजवंशीय सत्ता, धन तथा प्रतिष्ठा का सर्वाधिक ठोस प्रतीक था मुसलमान शासकों के संदर्भ में इसे धर्मनिष्ठा के एक कार्य के रूप में भी देखा जाता था।

(4) नई राजधानी शाहजहाँनाबाद की स्थापना- 1648 में शाहजहाँ के शासनकाल में दरबार, सेना व राजसी खानदान आगरा से नई निर्मित शाही राजधानी शाहजहाँनाबाद (दिल्ली) चले गए। दिल्ली के प्राचीन रिहायशी नगर में स्थित शहजहांनाबाद एक नई और शाही आबादी थी। यहाँ लालकिला, जामा मस्जिद, चाँदनी चौक के बाजार की वृक्ष-वीथि और अभिजात वर्ग के बड़े-बड़े पर थे। शाहजहाँ द्वारा स्थापित यह नया शहर विशाल एवं भव्य मुगल राजतंत्र का प्रतीक था।

प्रश्न 12.
“मुगल दरबार का केन्द्र बिन्दु राजसिंहासन अथवा तख्त था जिसने सम्प्रभु के कार्यों को भौतिक स्वरूप प्रदान किया था।” व्याख्या कीजिए।
अथवा
मुगल दरबार में अभिजात वर्गों के बीच हैसियत को निर्धारित करने वाले नियमों, अभिवादन के तरीकों एवं दरबार में मनाए जाने वाले त्यौहारों का वर्णन कीजिए।
उत्तर:
मुगल दरबार बादशाह पर केन्द्रित दरबार की भौतिक व्यवस्था ने बादशाह के अस्तित्व को समाज के हृदय के रूप में दर्शाया। दीर्घकाल से भारत में राजतन्त्र का प्रतीक रही छतरी, बादशाह की चमक को सूर्य की चमक से अलग करने वाली मानी जाती थी।
(1) अभिजात वर्गों के बीच हैसियत को निर्धारित करने वाले नियम- मुगल इतिवृत्तों में मुगल अभिजात वर्गों के बीच हैसियत को निर्धारित करने वाले नियमों का वर्णन किया गया है।

दरबार में किसी दरबारी की हैसियत इस बात से निर्धारित होती थी कि वह बादशाह के कितना पास और दूर बैठा है। एक बार जब बादशाह सिंहासन पर बैठ जाता था तो किसी भी दरबारी को अपने स्थान से कहीं और जाने की अनुमति नहीं थी और न ही कोई अनुमति के बिना दरबार से बाहर जा सकता था।

(2) बादशाह को किए गए अभिवादन के तरीके से अभिजात वर्ग की हैसियत का बोध होना-मुगल दरबार में बादशाह को किए गए अभिवादन के तरीके से अभिजात- वर्ग के व्यक्ति की हैसियत का बोध होता था। जिस व्यक्ति के सामने अधिक झुक कर अभिवादन किया जाता था, उस व्यक्ति की हैसियत अधिक ऊँची मानी जाती थी। अभिवादन का उच्चतम रूप ‘सिजदा’ या दंडवत् लेटना था।

(3) राजनयिक दूतों से सम्बन्धित नयाचार- मुगल- सम्राट के सामने प्रस्तुत होने वाले राजदूत से यह अपेक्षा की जाती थी कि वह अभिवादन के मान्य रूपों में से एक या तो बहुत झुककर अथवा जमीन को चूमकर अथवा फारसी रिवाज के अनुसार छाती के सामने हाथ बाँधकर तरीके से अभिवादन करेगा।

(4) झरोखा दर्शन – मुगल बादशाह अपने दिन की शुरुआत सूर्योदय के समय कुछ व्यक्तिगत धार्मिक प्रार्थनाओं से करता था। इसके बाद वह पूर्व की ओर मुँह किए एक छोटे छज्जे अर्थात् झरोखे में आता था। इसके नीचे लोगों को एक भारी भीड़ बादशाह की एक झलक पाने के लिए प्रतीक्षा करती थी। इस प्रथा का उद्देश्य जन विश्वास के रूप में शाही सत्ता की स्वीकृति का और विस्तार करना था।

(5) दीवान-ए-आम एवं दीवान-ए-खास-झरोखे में एक घंटा बिताने के बाद बादशाह सार्वजनिक सभा- भवन (दीवान-ए-आम) में आता था। वहाँ राज्य के अधिकारी रिपोर्ट तथा प्रार्थना-पत्र प्रस्तुत करते थे दो घंटे बाद बादशाह दीवान-ए-खास में निजी सभाएँ करता था तथा गोपनीय मामलों पर चर्चा करता था।

(6) दरबार का जीवन्त वातावरण सिंहासनारोहण की वर्षगाँठ, ईद, शब-ए-बारात तथा होली जैसे कुछ विशिष्ट अवसरों पर दरबार का वातावरण जीवन्त हो उठता था सुसज्जित डिब्बों में रखी सुगंधित मोमबत्तियाँ और महल की दीवारों पर लटक रहे रंग-बिरंगे बंदनवार आने वाले लोगों को आश्चर्यजनक रूप से प्रभावित करते थे।

JAC Class 12 History Important Questions Chapter 9 शासक और इतिवृत्त : मुगल दरबार

(7) विवाह मुगल काल में शाही परिवारों में विवाहों का आयोजन काफी खर्चीला होता था। 1633 ई. में शाहजहाँ के ज्येष्ठ पुत्र दारा शिकोह का विवाह राजकुमार परवेज की पुत्री नादिरा से हुआ। विवाह के उपहारों के प्रदर्शन की व्यवस्था ‘दीवान-ए-आम’ में की गई थी। हिनाबन्दी (मेहंदी लगाना) की रस्म दीवान-ए-खास में अदा की गई। दरबार में उपस्थित लोगों के बीच पान, इलायची तथा मेवे वाटे गए। इस विवाह पर कुल 32 लाख रुपये खर्च हुए थे।

प्रश्न 13.
मुगल सम्राटों द्वारा अभिजात वर्ग को प्रदान की जाने वाली पदवियों और उपहारों का वर्णन कीजिए। दरबारियों और राजदूतों द्वारा बादशाहों को उपहार देने का क्या महत्त्व था?
उत्तर:
मुगल सम्राटों द्वारा अभिजात वर्ग को प्रदान की जाने वाली पदवियाँ तथा उपहार
राज्याभिषेक के समय अथवा किसी शत्रु पर विजय के. बाद मुगल बादशाह विशाल पदवियाँ ग्रहण करते थे। जब उद्घोषक (नकीब) इन पदवियों की घोषणा करते थे, तो दरबार में आश्चर्य का वातावरण बन जाता था मुगल सिक्कों पर राजसी नयाचार के साथ सत्तारूढ़ बादशाह की पूरी पदवी होती थी।

(1) अभिजात वर्ग को पदवियाँ, पुरस्कार एवं उपहार देना योग्य व्यक्तियों को पदवियाँ देना मुगल- राजतन्त्र का एक महत्त्वपूर्ण पक्ष था दरबारी पदानुक्रम में किसी व्यक्ति की उन्नति को उसके द्वारा धारण की जाने वाली पदवियों से जाना जा सकता था। ‘आसफखों की पदवी उच्चतम मन्त्रियों में से एक को दी जाती थी। औरंगजेब ने अपने दो उच्च पदस्थ अभिजातों-जयसिंह और जसवन्त सिंह को ‘मिज राजा’ की पदवी प्रदान की। योग्यता के आधार पर पदवियाँ या तो अर्जित की जा सकती थीं अथवा इन्हें प्राप्त करने के लिए धन दिया जा सकता था। मीरखान नामक एक मनसबदार ने अपने नाम में अलिफ अर्थात् ‘अ’ अक्षर लगाकर उसे अमीरखान करने के लिए औरंगजेब को एक लाख रुपये देने का प्रस्ताव किया।

( 2) ‘ सम्मान का जामा’ (खिल्लत) – अन्य पुरस्कारों में सम्मान का जामा ( खिल्लत) भी शामिल था जिसे पहले कभी-न-कभी बादशाह द्वारा पहना गया होता था। इसलिए यह समझा जाता था कि वह बादशाह के आशीर्वाद का प्रतीक है।

(3) सरप्पा (सर से पाँव तक )- बादशाह द्वारा अभिजातों को प्रदान किये जाने वाले एक अन्य उपहार

‘सरप्पा’ (सर से पाँव तक) था इस उपहार के तीन भाग हुआ करते थे-
(1) जामा
(2) पगड़ी तथा
(3) पटका।
(4) रत्नजड़ित आभूषण बादशाह द्वारा अभिजात- वर्ग के लोगों को प्रायः रत्नजड़ित आभूषण भी उपहार के रूप में दिए जाते थे। विशिष्ट परिस्थितियों में बादशाह कमल की मंजरियों वाला रत्नजड़ित गहनों का सेट (पद्ममुरस्सा) भी उपहार में प्रदान करता था।

(5) दरबारियों एवं राजदूतों द्वारा बादशाह की सेवा में उपहार प्राप्त करना कोई भी दरबारी बादशाह के पास कभी खाली हाथ नहीं जाता था। वह या तो नज़ के रूप में थोड़ा धन या पेशकश के रूप में मोटी रकम बादशाह की सेवा में प्रस्तुत करता था। राजनयिक सम्बन्धों में उपहारों को सम्मान और आदर का प्रतीक माना जाता था। राजदूत प्रतिद्वन्द्वी राजनीतिक शक्तियों के बीच सन्धि और सम्बन्धों के द्वारा समझौता करवाने के महत्त्वपूर्ण कार्य करते थे। ऐसी परिस्थितियों में उपहारों की महत्त्वपूर्ण प्रतीकात्मक भूमिका होती थी। अंग्रेज राजदूत सर टामस रो ने आसफ खाँ को एक अंगूठी भेंट की थी, परन्तु वह उसे केवल इसलिए वापस कर दी गई कि वह मात्र चार सौ रुपये मूल्य की थी।

JAC Class 12 History Important Questions Chapter 9 शासक और इतिवृत्त : मुगल दरबार

प्रश्न 14.
“मुगल साम्राज्य में शाही परिवार की स्त्रियों द्वारा निभाई गई भूमिका अत्यन्त महत्त्वपूर्ण थी। ” विवेचना कीजिए।
उत्तर:
(1) मुगल परिवार मुगल परिवार में बादशाह की पत्नियाँ और उपपत्नियाँ, उसके नजदीकी और दूर के सम्बन्धी (माता, सौतेली व उपमाताएँ, बहन, पुत्री, बहू, चाची-मौसी, बच्चे आदि) महिला परिचारिकाएँ तथा गुलाम होते थे। शासक वर्ग में बहुविवाह प्रथा व्यापक रूप से प्रचलित थी।

(2) अगहा तथा अगाचा मुगल परिवार में शाही परिवारों से आने वाली स्त्रियों (बेगमों) और अन्य स्वियों (अगहा) जिनका जन्म कुलीन परिवार में नहीं हुआ था, में अन्तर रखा जाता था। दहेज (मेहर) के रूप में विपुल नकद और बहुमूल्य वस्तुएँ लाने वाली बेगमों को अपने पतियों से स्वाभाविक रूप से ‘अगहाओं’ की तुलना में अधिक ऊंचा दर्जा और सम्मान मिलता था।

(3) वंश आधारित पारिवारिक ढाँचे का स्थायी नहीं होना – वंश आधारित पारिवारिक ढाँचा पूरी तरह स्थायी नहीं था।

(4) गुलाम- पत्नियों के अतिरिक्त मुगल परिवार में अनेक महिला तथा पुरुष गुलाम होते थे वे साधारण से साधारण कार्य से लेकर कौशल, निपुणता तथा बुद्धिमत्ता के अलग-अलग कार्य करते थे गुलाम हिजड़े (ख्वाजासर) परिवार के अन्दर और बाहर के जीवन में रक्षक, नौकर तथा व्यापार में रुचि रखने वाली महिलाओं के एजेन्ट होते थे।

(5) शाही परिवार की स्त्रियों का वित्तीय संसाधनों पर नियंत्रण – नूरजहाँ के बाद मुगल रानियों और राजकुमारियों ने महत्त्वपूर्ण वित्तीय स्रोतों पर नियंत्रण रखना शुरू कर दिया। शाहजहाँ की पुत्रियों जहाँआरा तथा रोशनआरा को ऊंचे शाही मनसबदारों के समान वार्षिक आय होती थी। इसके अतिरिक्त जहाँआरा को सूरत के बन्दरगाह नगर से राजस्व प्राप्त होता था।

(6) निर्माण कार्यों में भाग लेना मुगल परिवार की महत्त्वपूर्ण स्वियों ने इमारतों व बागों का निर्माण भी करवाया। जहाँआरा ने शाहजहाँ की नई राजधानी शाहजहांनाबाद (दिल्ली) की अनेक वास्तुकलात्मक परियोजनाओं में भाग लिया। इनमें से एक दो मंजिली भव्य इमारत कारवां सराय थी, जिसमें एक आँगन व बाग भी था। शाहजहाँनाबाद के मुख्य केन्द्र चाँदनी चौक की रूपरेखा भी जहाँआरा द्वारा बनाई गई थी।

(7) लेखन कार्य में योगदान गुलबदन बेगम एक उच्च कोटि की लेखिका थी वह बाबर की पुत्री हुमायूँ की बहिन तथा अकबर की बुआ थी वह स्वयं तुर्की तथा फारसी में धारा प्रवाह लिख सकती थी। उसने ‘हुमायूँनामा’ नामक पुस्तक लिखी जिससे हमें मुगलों की घरेलू दुनिया की एक झलक मिलती है।

प्रश्न 15.
कन्धार, आटोमन साम्राज्य तथा जैसुइट धर्म प्रचारकों के साथ मुगल सम्राटों के सम्बन्धों का विवेचन कीजिए।
उत्तर:
(1) कन्धार के साथ मुगल सम्राटों के सम्बन्ध-कन्धार ईरान के सफावियों और मुगलों के बीच झगड़े की जड़ था। कन्धार का किला नगर आरम्भ में हुमायूँ के आधिकार में था। 1595 में अकबर ने कन्धार पर पुनः अधिकार कर लिया था। यद्यपि ईरान के शासक शाह अब्बास ने मुगलों के साथ अपने राजनयिक सम्बन्ध बनाए रखे तथापि कन्धार पर वह अपना दावा करता रहा।

1613 में जहाँगीर ने शाह अब्बास के दरबार में कन्धार को मुगलों के आधिपत्य में रहने देने की वकालत करने के लिए एक राजनयिक दूत भेजा, परन्तु यह शिष्टमण्डल अपने उद्देश्यों में सफल नहीं हुआ। 1622 में एक ईरानी सेना ने कन्धार पर घेरा डाल दिया। यद्यपि मुगल सेना ने ईरानियों का मुकाबला किया, परन्तु उसे पराजय का मुंह देखना पड़ा। इस प्रकार कन्धार के किले तथा नगर पर ईरानियों का अधिकार हो गया।

(2) आटोमन साम्राज्य के साथ मुगलों के सम्बन्ध- आटोमन साम्राज्य के साथ मुगलों ने अपने सम्बन्ध इस बात को ध्यान में रखकर बनाए कि वे आटोमन नियन्त्रण वाले क्षेत्रों में व्यापारियों व तीर्थयात्रियों के स्वतंत्र आवागमन को बनाये रखवा सकें। इस क्षेत्र के साथ अपने सम्बन्धों में मुगल बादशाह, प्राय: धर्म एवं वाणिज्य के मुद्दों को मिलाने का प्रयास करता था। वह लाल सागर के बन्दरगाह अदन और मोरवा को बहुमूल्य वस्तुओं के निर्यात को प्रोत्साहन देता था और इनकी बिक्री से प्राप्त आय को उस क्षेत्र के धर्मस्थलों व फकीरों में दान में बाँट देता था।

JAC Class 12 History Important Questions Chapter 9 शासक और इतिवृत्त : मुगल दरबार

(3) जैसुइट धर्म प्रचारकों के साथ मुगलों के सम्बन्ध-पन्द्रहवीं शताब्दी के अन्त में पुर्तगाली व्यापारियों ने तटीय नगरों में व्यापारिक केन्द्रों का जाल स्थापित किया। पुर्तगाल का सम्राट भी ‘सोसाइटी ऑफ जीसस’ (जेसुइट ) के धर्म प्रचारकों की सहायता से ईसाई धर्म का प्रचार-प्रसार करना चाहता था। सोलहवीं शताब्दी के दौरान भारत आने वाले जेसुइट शिष्टमण्डल व्यापार और साम्राज्य निर्माण की इस प्रकिया के हिस्सा थे।

अकबर ईसाई धर्म के विषय में जानने को बहुत उत्सुक था। उसने जेसुइट पादरियों को आमन्त्रित करने के लिए एक दूत मंडल गोवा भेजा। पहला जेसुइट शिष्टमंडल फतेहपुर सीकरी के मुगल दरबार में 1580 में पहुँचा और वह वहाँ लगभग दो वर्ष रहा। इन जेसुइट लोगों ने ईसाई धर्म के विषय में अकबर से वार्तालाप किया और इसके सद्गुणों के विषय में उलमा से उनका वाद-विवाद हुआ।

लाहौर के मुगल दरबार में दो और शिष्टमण्डल 1591 और 1595 में भेजे गए। जेसुइट विवरण व्यक्तिगत अनुभवों पर आधारित हैं और वे मुगल सम्राट अकबर के चरित्र और सोच पर गहरा प्रकाश डालते हैं। सार्वजनिक सभाओं में जेसुइट लोगों को अकबर के सिंहासन के काफी निकट स्थान दिया जाता था। वे उसके साथ अभियानों में जाते, उसके बच्चों को शिक्षा देते तथा उसके फुरसत के समय में वे प्रायः उसके साथ होते थे। जेसुइट विवरण मुगलकाल के राज्य अधिकारियों और सामान्य जन-जीवन के विषय में फारसी इतिहासों में दी गई सूचना की पुष्टि करते हैं।

प्रश्न 16.
मुगलों के प्रान्तीय प्रशासन का वर्णन कीजिए।
उत्तर:
मुगलों का प्रान्तीय प्रशासन शासन को सुचारू रूप से चलाने के लिए मुगल- सम्राटों ने साम्राज्य को अनेक प्रान्तों में विभक्त कर दिया था प्रान्तीय शासन के प्रमुख अधिकारी निम्नलिखित थे-

  1. सूबेदार – प्रान्त का सर्वोच्च अधिकारी सूबेदार कहलाता था। सूबेदार की नियुक्ति सम्राट के द्वारा की जाती थी। वह सीधा मुगल सम्राट को प्रतिवेदन प्रस्तुत करता था।
  2. दीवान दीवान प्रान्तीय वित्त विभाग का अध्यक्ष होता था। वह प्रान्त की आय-व्यय की देख-रेख करता था।
  3. बख्शी सैनिकों की भर्ती, उनकी आवश्यकताओं एवं साज-सज्जा की पूर्ति, रसद की व्यवस्था उसके कार्य थे।
  4. सद्र सद्र का प्रमुख कार्य लोगों के नैतिक चरित्र की देखभाल करना, दान-पुण्य तथा इस्लाम के कानूनों के पालन की व्यवस्था करवाना था।
  5. काजी- काजी प्रान्त का प्रमुख न्यायिक अधिकारी होता था।

सरकार अथवा जिला प्रबन्ध-मुगलकाल में प्रान्तों को सरकारों अथवा जिलों में बाँटा गया था। सरकार का प्रबन्ध करने के लिए निम्नलिखित अधिकारी नियुक्त किए गए थे-
(1) फौजदार यह सरकार अथवा जिले का प्रमुख अधिकारी होता था। विद्रोहियों का दमन करना, अपने क्षेत्र में शान्ति एवं व्यवस्था बनाए रखना आदि उसके प्रमुख कार्य थे। उसके अधीन एक विशाल घुड़सवार सेना तथा तोपची होती थे।

(2) अमलगुजार यह सरकार का राजस्व अधिकारी परगने का प्रबन्ध- प्रत्येक सरकार अनेक परगनों में विभक्त था। परगनों का प्रबन्ध करने के लिए निम्नलिखित अधिकारी होते थे

  • कानूनगो ( राजस्व अभिलेख का रखवाला)
  • चौधरी (राजस्व संग्रह का प्रभारी) तथा
  • काजी (न्याय का अधिकारी)।

JAC Class 12 History Important Questions Chapter 8 किसान, ज़मींदार और राज्य : कृषि समाज और मुगल साम्राज्य

Jharkhand Board JAC Class 12 History Important Questions Chapter 8 किसान, ज़मींदार और राज्य : कृषि समाज और मुगल साम्राज्य Important Questions and Answers.

JAC Board Class 12 History Important Questions Chapter 8 किसान, ज़मींदार और राज्य : कृषि समाज और मुगल साम्राज्य

बहुविकल्पीय प्रश्न (Multiple Choice Questions)

1. सोलहवीं तथा सत्रहवीं शताब्दियों के कृषि इतिहास की जानकारी देने वाला प्रमुख ऐतिहासिक ग्रन्थ था –
(अ) अकबरनामा
(स) बादशाहनामा
(ब) आइन-ए-अकबरी
(द) बाबरनामा
उत्तर:
(ब) आइन-ए-अकबरी

2. बाबरनामा’ का रचयिता था –
(अ) हुमायूँ
(स) बाबर
(ब) अकबर
(द) जहाँगीर
उत्तर:
(स) बाबर

3. पंजाब में शाह नहर की मरम्मत किसके शासनकाल में करवाई गई –
(अ) बाबर
(ब) हुमायूँ
(स) अकबर
(द) शाहजहाँ
उत्तर:
(द) शाहजहाँ

4. तम्बाकू का प्रसार सर्वप्रथम भारत के किस भाग में हुआ?
(अ) उत्तर भारत
(ब) दक्षिण भारत
(स) पूर्वी भारत
(द) पूर्वोत्तर भारत
उत्तर:
(ब) दक्षिण भारत

5. किस मुगल सम्राट ने तम्बाकू के धूम्रपान पर प्रतिबन्ध लगा दिया था?
(अ) अकबर
(स) जहाँगीर
(ब) बाबर
(द) शाहजहाँ
उत्तर:
(स) जहाँगीर

6. कौनसी फसल नकदी फसल ( जिन्स-ए-कामिल) कहलाती थी –
(अ) कपास
(ब) गेहूँ
(स) जौ
(द) चना
उत्तर:
(अ) कपास

JAC Class 12 History Important Questions Chapter 8 किसान, ज़मींदार और राज्य : कृषि समाज और मुगल साम्राज्य

7. अफ्रीका और स्पेन से किसकी फसल भारत पहुँची ?
(अ) गेहूँ
(ब) बाजरा
(स) मक्का
(द) चना
उत्तर:
(स) मक्का

8. सत्रहवीं शताब्दी में लिखी गई एक पुस्तक मारवाड़ में में किसकी चर्चा किसानों के रूप में की गई है?
(अ) वैश्यों
(ब) राजपूतों
(स) ब्राह्मणों
(द) योद्धाओं
उत्तर:
(ब) राजपूतों

9. गांव की पंचायत का मुखिया कहलाता था –
(अ) फौजदार
(ब) ग्राम-प्रधान
(स) चौकीदार
(द) मुकद्दम (मंडल)
उत्तर:
(द) मुकद्दम (मंडल)

10. खुदकाश्त तथा पाहिकारत कौन थे?
(अ) किसान
(ब) सैनिक
(स) जमींदार
(द) अधिकारी
उत्तर:
(अ) किसान

JAC Class 12 History Important Questions Chapter 8 किसान, ज़मींदार और राज्य : कृषि समाज और मुगल साम्राज्य

11. मिरासदार कौन थे?
(अ) महाराष्ट्र प्रांत के धनी अथवा सम्पन्न कृषक
(ब) राजस्थान के धनी व्यापारी तथा कृषक
(स) राजस्थान में अधिकारियों का एक समूह
(द) उपर्युक्त में से कोई नहीं
उत्तर:
(अ) महाराष्ट्र प्रांत के धनी अथवा सम्पन्न कृषक

12. वह भूमि जिसे कभी खाली नहीं छोड़ा जाता था, कहलाती थी –
(अ) परौती
(ब) चचर
(स) उत्तम
(द) पोलज
उत्तर:
(द) पोलज

13. मनसबदार कौन थे?
(अ) मुगल अधिकारी
(ब) मुगल जमींदार
(स) मुगल दरबारी
(द) इनमें से कोई नहीं
उत्तर:
(अ) मुगल अधिकारी

14. इटली का यात्री जोवनी कारेरी कब भारत आया ?
(अ) 1560 ई.
(ब) 1690 ई.
(स) 1590 ई.
(द) 1670 ई.
उत्तर:
(ब) 1690 ई.

15. आइन-ए-अकबरी में कुल कितने भाग हैं?
(अ) दो
(ब) तीन
(स) चार
(द) पाँच
उत्तर:
(द) पाँच

रिक्त स्थानों की पूर्ति कीजिए

1. 1526 ई. में ……………. को पानीपत के युद्ध में हराकर बाबर पहला मुगल बादशाह बना।
2. मुगल काल के भारतीय फारसी स्रोत किसान के लिए रैयत या …………… शब्द का उपयोग करते थे।
3. सामूहिक ग्रामीण समुदाय के तीन घटक …………… और …………….. थे।
4. पंचायत का सरदार एक मुखिया होता था जिसे ………………. या मण्डल कहते थे।
5. मुद्रा की फेरबदल करने वालों को …………….. कहा जाता था।
6. जोवान्नी कारेरी ………………. का मुसाफिर था जो लगभग ……………. ई. में भारत से होकर गुजरा था।
उत्तर:
1. इब्राहिम लोदी
2. मुजरियान
3. खेतिहर किसान, पंचायत, गाँव का मुखिया
4. मुकद्दम
5. सर्राफ
6. इटली, 1690

JAC Class 12 History Important Questions Chapter 8 किसान, ज़मींदार और राज्य : कृषि समाज और मुगल साम्राज्य

अतिलघूत्तरात्मक प्रश्न

प्रश्न 1.
जिन्स-ए-कामिल’ फसल के बारे में बताइये।
उत्तर:
‘जिन्स-ए-कामिल’ सर्वोत्तम फसलें थीं जैसे कपास और गन्ने की फसलें।

प्रश्न 2.
आइन-ए-अकबरी किसके द्वारा लिखी गई ?
उत्तर:
अबुल फ़सल।

प्रश्न 3.
सोलहवीं और सत्रहवीं सदी में राज्य द्वारा जंगलों में घुसपैठ के क्या कारण थे ?
उत्तर:
(1) सेना के लिए सभी प्राप्त करना
(2) शिकार अभियान द्वारा न्याय करना।

प्रश्न 4.
सत्रहवीं शताब्दी के स्रोत भारत में कितने प्रकार के किसानों का उल्लेख करते हैं?
उत्तर:
दो प्रकार के किसानों की –
(1) खुद काश्त
(2) पाहि काश्त

प्रश्न 5.
अबुल फ़सल द्वारा रचित पुस्तक का नाम लिखिए।
उत्तर:
‘आइन-ए-अकबरी’।

प्रश्न 6.
मुगलकाल में गाँव की पंचायत के मुखिया को किस नाम से जाना जाता था?
उत्तर:
मुकद्दम या मंडल।

प्रश्न 7.
मुगलकाल के भारतीय फारसी स्रोत किसान के लिए किन शब्दों का प्रयोग करते थे?
उत्तर:
(1) रैयत
(2) मुजरियान
(3) किसान
(4) आसामी

प्रश्न 8.
खुद काश्त किसान कौन थे?
उत्तर:
खुद काश्त किसान उन्हीं गाँवों में रहते थे, जिनमें उनकी जमीनें थीं।

प्रश्न 9.
लगभग सोलहवीं सत्रहवीं शताब्दियों में कृषि के निरन्तर विस्तार होने के तीन कारक लिखिए।
उत्तर:

  • जमीन की प्रचुरता
  • मजदूरों की उपलब्धता
  • किसानों की गतिशीलता.

प्रश्न 10.
इस काल में सबसे अधिक उगाई जाने वाली तीन प्रमुख फसलों का उल्लेख कीजिए।
उत्तर:
(1) चावल
(2) गेहूँ
(3) ज्यार

JAC Class 12 History Important Questions Chapter 8 किसान, ज़मींदार और राज्य : कृषि समाज और मुगल साम्राज्य

प्रश्न 11.
चावल, गेहूँ तथा ज्वार की फसलें सबसे अधिक उगाई जाने का क्या कारण था?
उत्तर:
इस काल में खेती का प्राथमिक उद्देश्य लोगों का पेट भरना था।

प्रश्न 12.
मौसम के किन दो मुख्य चक्रों के दौरान खेती की जाती थी?
उत्तर;
(1) खरीफ (पतझड़ में) तथा
(2) रबी (बसन्त में)।

प्रश्न 13.
16वीं – 17वीं शताब्दी के दौरान भारत में कितने प्रतिशत लोग गाँवों में रहते थे?
उत्तर:
85 प्रतिशत।

प्रश्न 14.
भारत में कपास का उत्पादन कहाँ होता था?
उत्तर:
मध्य भारत तथा दक्कनी पठार में।

प्रश्न 15.
दो नकदी फसलों के नाम बताइये।
उत्तर:
(1) तिलहन (जैसे-सरसों) तथा
(2) दलहन

प्रश्न 16.
सत्रहवीं शताब्दी में दुनिया के विभिन्न भागों से भारत में पहुंचने वाली चार फसलों के नाम लिखिए।
उत्तर:

  • मक्का
  • टमाटर
  • आलू
  • मिर्च

प्रश्न 17.
सामूहिक ग्रामीण समुदाय के तीन घटकों का उल्लेख कीजिए।
उत्तर:
(1) खेतिहर किसान
(2) पंचायत
(3) गाँव का मुखिया (मुकद्दम या मंडल)।

प्रश्न 18.
1600 से 1700 के बीच भारत की जनसंख्या में कितनी वृद्धि हुई?
उत्तर:
लगभग 5 करोड़ की।

प्रश्न 19.
ग्राम की पंचायत में कौन लोग होते थे?
उत्तर:
ग्राम के बुजुर्ग।

प्रश्न 20.
ग्राम की पंचायत का मुखिया कौन होता
उत्तर:
मुकद्दम या मंडल।

JAC Class 12 History Important Questions Chapter 8 किसान, ज़मींदार और राज्य : कृषि समाज और मुगल साम्राज्य

प्रश्न 21.
पंचायत के मुखिया का प्रमुख कार्य क्या
उत्तर:
गांव की आमदनी एवं खर्चे का हिसाब-किताब अपनी निगरानी में बनवाना

प्रश्न 22.
इस कार्य में मुकद्दम की कौन राजकीय कर्मचारी सहायता करता था ?
उत्तर:
पटवारी

प्रश्न 23.
पंचायतों के दो अधिकारों का उल्लेख कीजिए।
उत्तर:
(1) जुर्माना लगाना
(2) दोषी व्यक्ति को समुदाय से निष्कासित करना।

प्रश्न 24.
पंचायत के निर्णय के विरुद्ध किसान विरोध का कौनसा अधिक उम्र रास्ता अपनाते थे? उत्तर- गाँव छोड़कर भाग जाना।

प्रश्न 25
उन दो प्रान्तों के नाम लिखिए जहाँ महिलाओं को जमींदारी उत्तराधिकार में मिलती थी जिसे बेचने व गिरवी रखने के लिए वे स्वतंत्र थीं।
उत्तर:
(1) पंजाब
(2) बंगाल

प्रश्न 26.
जंगल के तीन उत्पादों का उल्लेख कीजिए, जिनकी बहुत माँग थी।
उत्तर:
(1) शहद
(2) मधुमोम
(3) लाख

प्रश्न 27.
‘मिल्कियत’ का अर्थ स्पष्ट कीजिए।
उत्तर:
जमींदारों की विस्तृत व्यक्तिगत जमीन ‘मिल्कियत’ कहलाती थी।

प्रश्न 28.
जमींदारों की समृद्धि का क्या कारण था?
उत्तर:
जमींदारों की समृद्धि का कारण था उनकी विस्तृत व्यक्तिगत जमीन।

प्रश्न 29.
जमींदारों की शक्ति के दो स्रोत बताइये।
उत्तर:
(1) जमींदारों द्वारा राज्य की ओर से कर वसूल करना
(2) सैनिक संसाधन।

JAC Class 12 History Important Questions Chapter 8 किसान, ज़मींदार और राज्य : कृषि समाज और मुगल साम्राज्य

प्रश्न 30.
भू-राजस्व के प्रबन्ध के दो चरण कौन- कौन से थे?
उत्तर:
(1) कर निर्धारण
(2) वास्तविक वसूली।

प्रश्न 31.
अमील गुजार कौन था?
उत्तर:
अमील गुजार राजस्व वसूली करने वाला अधिकारी था।

प्रश्न 32.
अकबर ने भूमि का किन चार भागों में वर्गीकरण किया?
उत्तर:

  • पोलज
  • परौती
  • चचर
  • अंजर

प्रश्न 33.
मध्यकालीन भारत में कौन-कौनसी फसलें सबसे अधिक उगाई जाती थीं?
उत्तर:
गेहूं, चावल, ज्वार इत्यादि।

प्रश्न 34.
भारत के किस भाग में सबसे पहले तम्बाकू की खेती की जाती थी?
उत्तर:
दक्षिण भारत में।

प्रश्न 35.
मुगलकाल में खेतिहर समाज की बुनियादी इकाई क्या थी?
उत्तर:
गाँव

प्रश्न 36.
मुगल राज्य किसने को ‘जिन्स-ए-कामिल’ फसलों की खेती करने के लिए प्रोत्साहन क्यों देते थे?
उत्तर:
क्योंकि इन फसलों से राज्य को अधिक कर मिलता था।

प्रश्न 37.
चीनी के उत्पादन के लिए कौनसा प्रान्त प्रसिद्ध था?
उत्तर:
बंगाल।

प्रश्न 38.
19वीं शताब्दी के कुछ अंग्रेज अधिकारियों ने गाँवों को किसकी संज्ञा दी थी?
उत्तर:
‘छोटे गणराज्य’।

प्रश्न 39.
आइन-ए-दहसाला किसने जारी किया?
उत्तर:
अकबरअकबर ने

JAC Class 12 History Important Questions Chapter 8 किसान, ज़मींदार और राज्य : कृषि समाज और मुगल साम्राज्य

प्रश्न 40.
मुगल साम्राज्य की वित्तीय व्यवस्था की देख-रेख करने वाला कौनसा दफ्तर था?
उत्तर;
दीवान का दफ्तर।

प्रश्न 41.
अकबर के समय में वह जमीन क्या कहलाती थी, जिसे कभी खाली नहीं छोड़ा जाता था?
उत्तर:
पोलज

प्रश्न 42.
अकबर के शासन काल में वह जमीन क्या कहलाती थी, जिस पर कुछ दिनों के लिए खेती रोक दी जाती थी?
उत्तर:
परौती।

प्रश्न 43.
आइन-ए-अकबरी’ के अनुसार भू-राजस्व वसूल करने के लिए कौनसी प्रणालियाँ अपनाई जाती थीं?
उत्तर:
(1) कणाकृत
(2) बटाई
(3) खेत बटाई।

प्रश्न 44.
आइन-ए-अकबरी’ को कब पूरा किया गया?
उत्तर:
1598 ई. में

प्रश्न 45.
अकबरनामा’ की कितनी जिल्दों में रचना
की गई?
उत्तर:
तीन जिल्दों में।

प्रश्न 46.
‘अकबरनामा’ का रचयिता कौन था?
उत्तर:
अबुल फजल।

प्रश्न 47
रैयत कौन थे?
उत्तर:
मुगल काल के भारतीय फारसी खोत किसान के लिए आमतौर पर रैयत या मुजरियान शब्द का इस्तेमाल करते थे। था।

प्रश्न 48.
‘परगना’ से क्या अभिप्राय है?
उत्तर:
परगना मुगल प्रान्तों में एक प्रशासनिक प्रमंडल

JAC Class 12 History Important Questions Chapter 8 किसान, ज़मींदार और राज्य : कृषि समाज और मुगल साम्राज्य

प्रश्न 49.
ग्राम पंचायत (मुगलकाल) के मुखिया के दो कार्यों का वर्णन कीजिये।
उत्तर:
(1) गाँव के आय-व्यय का हिसाब-किताब तैयार करवाना
(2) जाति की अवहेलना को रोकने के लिए लोगों के आवरण पर नजर रखना।

प्रश्न 50.
पाहि काश्त किसान कौन थे?
उत्तर:
पाहि कारण किसान के खेतिहर थे, जो दूसरे गाँव से ठेके पर खेती करने आते थे।

प्रश्न 51.
बाबरनामा’ के अनुसार खेतों की सिंचाई साधन कौनसे थे?
गई ?
उत्तर:
(1) रहट के द्वारा
(2) बाल्टियों से।

प्रश्न 52.
भारत में नई दुनिया से कौनसी फसलें लाई
उत्तर:
टमाटर, आलू, मिर्च, अनानास, पपीता

प्रश्न 53.
पंचायत का प्रमुख कार्य क्या था?
उत्तर:
गाँव में रहने वाले अलग-अलग समुदायों के लोगों को अपनी जाति की सीमाओं के अन्दर रखना।

प्रश्न 54.
राजस्थान की जाति पंचायतों के दो अधिकारों का उल्लेख कीजिये।
उत्तर:

  • दीवानी के झगड़ों का निपटारा करना
  • जमीन से जुड़े दावेदारियों के झगड़े सुलझाना।

प्रश्न 55.
पंचायत के मुखिया का चुनाव किस प्रकार किया जाता था?
उत्तर:
(1) गाँव के बुजुर्गों की आम सहमति से और
(2) उसे इसकी स्वीकृति जमींदार से लेनी होती थी।

प्रश्न 56.
खेतिहर किन ग्रामीण दस्तकारियों में संलग्न रहते थे?
उत्तर:
रंगरेजी, कपड़े पर छपाई, मिट्टी के बर्तनों को पकाना, खेती के औजार बनाना।

प्रश्न 57.
सोलहवीं तथा सत्रहवीं सदी के प्रारम्भ में कृषि इतिहास को जानने के दो स्त्रोतों का उल्लेख करो।
उत्तर:
(1) आइन-ए-अकबरी
(2) ईस्ट इण्डिया कम्पनी के बहुत सारे दस्तावेज

JAC Class 12 History Important Questions Chapter 8 किसान, ज़मींदार और राज्य : कृषि समाज और मुगल साम्राज्य

प्रश्न 58.
गाँव के प्रमुख दस्तकारों का उल्लेख कीजिये।
उत्तर:
कुम्हार, लुहार, बढ़ई, नाई, सुनार

प्रश्न 59.
कौनसी दस्तकारियों के काम उत्पादन के लिए महिलाओं के श्रम पर निर्भर थे?
उत्तर:
सूत कातना, बर्तन बनाने के लिए मिट्टी साफ करना और गूंधना, कपड़ों पर कढ़ाई करना।

प्रश्न 60.
‘पेशकश का क्या अर्थ है?
उत्तर:
पेशकरा’ मुगल राज्य के द्वारा ली जाने वाली एक प्रकार की भेंट थी। था?

प्रश्न 61.
मुगल राज्य के लिए जंगल कैसा भू-भाग
उत्तर;
मुगल राज्य के अनुसार जंगल बदमाशों, विद्रोहियों आदि को शरण देने वाला अड्डा था।

प्रश्न 62.
जमींदार कौन थे?
उत्तर;
जमींदार अपनी जमीन के स्वामी होते थे उन्हें ग्रामीण समाज में उच्च प्रतिष्ठा प्राप्त थी।

प्रश्न 63.
जमींदारों की उच्च स्थिति के दो कारण लिखिए।
उत्तर:
(1) उनकी जाति
(2) जमींदारों के द्वारा राज्य को दी जाने वाली कुछ विशिष्ट सेवाएँ।

प्रश्न 64.
‘जमा’ और ‘हासिल’ में भेद कीजिये।
उत्तर:
‘जमा निर्धारित रकम थी, जबकि ‘हासिल’ वास्तव में वसूल की गई रकम थी।

प्रश्न 65.
जमींदारी पुख्ता करने के दो तरीकों का उल्लेख कीजिए।
उत्तर:
(1) नयी जमीनों को बसाकर
(2) अधिकारों के हस्तान्तरण के द्वारा।

प्रश्न 66.
अमीन कौन था?
उत्तर:
अमीन एक मुगल अधिकारी था, जिसका काम यह सुनिश्चित करना था कि राजकीय कानूनों का पालन हो रहा है।

प्रश्न 67.
मनसबदारी व्यवस्था क्या थी?
उत्तर:
मनसबदारी मुगल प्रशासनिक व्यवस्था के शीर्ष पर एक सैनिक नौकरशाही तत्व था, जिसे मनसबदारी व्यवस्था कहते हैं।

JAC Class 12 History Important Questions Chapter 8 किसान, ज़मींदार और राज्य : कृषि समाज और मुगल साम्राज्य

प्रश्न 68.
अकबर के समय का प्रसिद्ध इतिहासकार कौन था? उसके द्वारा रचित दो ग्रन्थों के नाम लिखिए।
उत्तर:
(1) अकबर के समय का प्रसिद्ध इतिहासकार अबुल फजल था।
(2) अबुल फजल ने ‘अकबरनामा’ तथा आइन-ए-अकबरी’ की रचना की।

प्रश्न 69.
चंडीमंगल नामक बंगाली कविता के रचयिता कौन हैं ?
उत्तर:
सोलहवीं सदी में मुकुंदराम चक्रवर्ती ने चंडीमंगल नामक कविता लिखी थी।

प्रश्न 70.
पेशकश क्या होती थी?
उत्तर:
पेशकश मुगल राज्य के द्वारा की जाने वाली एक तरह की भेंट थी।

प्रश्न 71.
गाँवों में कौनसी पंचायतें होती थीं?
उत्तर:
(1) ग्राम पंचायत
(2) जाति पंचायत ।

प्रश्न 72.
किस प्रांत में चावल की 50 किस्में पैदा की
जाती थीं?
उत्तर:
बंगाल में।

प्रश्न 73
गाँवों में पाई जाने वाली दो सामाजिक असमानताओं का उल्लेख कीजिये।
उत्तर:
(1) सम्पति की व्यक्तिगत मिल्कियत होती थी।
(2) जाति और सामाजिक लिंग के नाम पर समाज में गहरी विषमताएँ थीं।

प्रश्न 74.
शाह नहर कहाँ है?
उत्तर:
पंजाब में।

प्रश्न 75.
अकबर द्वारा अमील गुजार को क्या आदेश दिए गए थे?
उत्तर:
इस बात की व्यवस्था करना कि खेतिहर नकद भुगतान करे और वहीं फसलों में भुगतान का विकल्प भी खुला रहे।

JAC Class 12 History Important Questions Chapter 8 किसान, ज़मींदार और राज्य : कृषि समाज और मुगल साम्राज्य

प्रश्न 76
जमींदारी को पुख्ता करने के क्या तरीके थे?
उत्तर:
(1) नई जमीनों को बतकर
(2) अधिकारों के हस्तान्तरण के द्वारा
(3) राज्य के आदेश से
(4) जमीनों को खरीद कर

प्रश्न 77.
वाणिज्यिक खेती का प्रभाव जंगलवासियाँ के जीवन पर कैसे पढ़ता था?
उत्तर:
(1) शहद, मधु, मोम, लाक की अत्यधिक माँग होना
(2) हाथियों को पकड़ना और बेचना
(3) व्यापार के अन्तर्गत वस्तुओं की अदला-बदली।

प्रश्न 78.
जंगली (जंगलवासी) कौन थे?
उत्तर:
जिन लोगों का गुजारा जंगल के उत्पादों, शिकार और स्थानान्तरित खेती से होता था, वे जंगली (जंगलवासी) कहलाते थे।

लघुत्तरात्मक प्रश्न

प्रश्न 1.
खेतिहर समाज के बारे में आप क्या जानते हैं?
उत्तर:
खेतिहर समाज की मूल इकाई गाँव थी जिसमें किसान रहते थे। किसान वर्ष भर भिन्न-भिन्न मौसमें में फसल की पैदावार से जुड़े समस्त कार्य करते थे। इन कार्यों में जमीन की जुताई, बीज बोना और फसल पकने पर उसे काटना आदि सम्मिलित थे। इसके अतिरिक्त वे उन वस्तुओं के उत्पादन में भी शामिल थे जो कृषि आधारित थीं, जैसे कि शक्कर, तेल इत्यादि ।

प्रश्न 2.
सत्रहवीं शताब्दी के खोत किन दो प्रकार के किसानों का उल्लेख करते हैं?
उत्तर- सत्रहवीं शताब्दी के स्रोत निम्नलिखित दो प्रकार के किसानों का उल्लेख करते हैं- (1) खुद काश्त तथा (2) पाहि कार खुद काश्त किसान उन्हीं गाँवों में रहते थे जिनमें उनकी जमीन थी। पाहि काश्त वे किसान थे जो दूसरे गाँवों से ठेके पर खेती करने आते थे। कुछ लोग अपनी इच्छा से भी पाहि-कारत बनते थे, जैसे दूसरे गाँव में करों की शर्तें उत्तम मिलने पर कुछ लोग अकाल या भुखमरी के बाद आर्थिक परेशानी से बाध्य होकर भी पाहि कार किसान बनते थे।

प्रश्न 3.
मुगल काल में कृषि की समृद्धि से भारत में आबादी में किस प्रकार बढ़ोतरी हुई?
उत्तर:
कृषि उत्पादन में अपनाए गए विविध और लचीले तरीकों के परिणामस्वरूप भारत में आवादी धीरे-धीरे बढ़ने लगी। आर्थिक इतिहासकारों की गणना के अनुसार समय- समय पर होने वाली भुखमरी और महामारी के बावजूद 1600 से 1700 के बीच भारत की आबादी लगभग 5 करोड़ बढ़ गई। 200 वर्षों में यह लगभग 33 प्रतिशत बढ़ोतरी थी।

JAC Class 12 History Important Questions Chapter 8 किसान, ज़मींदार और राज्य : कृषि समाज और मुगल साम्राज्य

प्रश्न 4.
सोलहवीं सत्रहवीं शताब्दी का ग्रामीण समाज किन-किन रिश्तों के आधार पर निर्मित था ?
उत्तर:
सोलहवीं सत्रहवीं शताब्दी का ग्रामीण समाज छोटे किसानों एवं धनिक जमींदारों दोनों से निर्मित था। ये दोनों ही कृषि उत्पादन से जुड़े थे और फसल में हिस्सों के दावेदार थे। इससे उनके मध्य सहयोग, प्रतियोगिता एवं संघर्ष के रिश्ते निर्मित हुए कृषि से जुड़े इन समस्त रिश्तों से ग्रामीण समाज बनता था।

प्रश्न 5.
मुगल साम्राज्य के अधिकारी ग्रामीण समाज को नियन्त्रण में रखने का प्रयास क्यों करते थे?
उत्तर:
मुगल साम्राज्य अपनी आय का एक बड़ा भाग कृषि उत्पादन से प्राप्त करता था इसलिए राजस्व निर्धारित करने वाले, राजस्व की वसूली करने वाले एवं राजस्व का विवरण रखने वाले अधिकारी ग्रामीण समाज को नियन्त्रण में रखने का पूरा प्रयास करते थे। वे चाहते थे कि खेती की नियमित जुताई हो एवं राज्य को उपज से अपने हिस्से का कर समय पर मिल जाए।

प्रश्न 6.
ग्रामीण भारत के खेतिहर समाज पर संक्षेप में तीन पंक्तियाँ लिखिए।
उत्तर;
खेतिहर समाज की मूल इकाई गाँव थी; जिमसें किसान रहते थे। किसान वर्ष भर भिन्न-भिन्न मौसमों में फसल की पैदावार से जुड़े समस्त कार्य करते थे। इन कार्यों में जमीन की जुताई, बीज बोना एवं फसल पकने पर उसकी कटाई करना आदि कार्य सम्मिलित थे। इसके अतिरिक्त वे उन वस्तुओं के उत्पादन में भी सम्मिलित थे जो कृषि आधारित थीं जैसे कि शक्कर, तेल आदि ।

प्रश्न 7.
” सत्रहवीं सदी में दुनिया के अलग-अलग भागों से कई नई फसलें भारतीय उपमहाद्वीप पहुंचीं।” स्पष्ट कीजिए।
उत्तर:
सत्रहवीं सदी में दुनिया के विभिन्न भागों से कई नई फसलें भारतीय उपमहाद्वीप पहुंचीं। उदाहरणार्थ, मक्का भारत में अफ्रीका और स्पेन से पहुँचा और सत्रहवीं सदी तक इसकी गिनती पश्चिम भारत की मुख्य फसलों में होने लगी। टमाटर, आलू और मिर्च जैसी सब्जियाँ नई दुनिया से लाई गई। इसी प्रकार अनानास तथा पपीता जैसे फल भी नई दुनिया से आए।

प्रश्न 8.
मुगल काल में गाँव की पंचायत का गठन किस प्रकार होता था?
उत्तर:
गाँव की पंचायत बुजुर्गों की सभा होती थी। प्रायः वे गाँव के महत्त्वपूर्ण लोग हुआ करते थे जिनके पास अपनी सम्पत्ति के पैतृक अधिकार होते थे। जिन गाँवों में कई जातियों के लोग रहते थे, वहाँ प्रायः पंचायत में भी विविधता पाई जाती थी यह एक ऐसा अल्पतंत्र था, जिसमें गाँव के अलग-अलग सम्प्रदायों एवं जातियों का प्रतिनिधित्व होता था। पंचायत का निर्णय गाँव में सबको मानना पड़ता था।

प्रश्न 9.
गाँव की पंचायत का जाति सम्बन्धी प्रमुख काम क्या था?
उत्तर:
गांव की पंचायत का जाति सम्बन्धी प्रमुख काम यह सुनिश्चित करना था कि गाँव में रहने वाले अलग-अलग सम्प्रदायों के लोग अपनी जाति की सीमाओं के अन्दर रहें। पूर्वी भारत में सभी विवाह मंडल की उपस्थिति में होते थे। दूसरे शब्दों में ” जाति की अवहेलना के लिए” लोगों के आचरण पर नजर रखनां गाँव की पंचायत के मुखिया की एक महत्त्वपूर्ण जिम्मेदारी थी।

JAC Class 12 History Important Questions Chapter 8 किसान, ज़मींदार और राज्य : कृषि समाज और मुगल साम्राज्य

प्रश्न 10.
गाँव की पंचायतों को कौनसे न्याय सम्बन्धी अधिकार प्राप्त थे?
उत्तर:
गाँव की पंचायतों को दोषियों पर जुर्माना लगाने और समुदाय से निष्कासित करने जैसे दंड देने के अधिकार प्राप्त थे। समुदाय से निष्कासित करना एक कड़ा कदम था, जो एक सीमित समय के लिए लागू किया जाता था। इसके अन्तर्गत दण्डित व्यक्ति को (निर्धारित समय के लिए) गाँव छोड़ना पड़ता था। इस अवधि में वह अपनी जाति तथा व्यवसाय से हाथ धो बैठता था। इन नीतियों का उद्देश्य जातिगत परम्पराओं की अवहेलना को रोकना था।

प्रश्न 11.
सत्रहवीं सदी में गाँवों में मुद्रा के प्रचलन के बारे में फ्रांसीसी यात्री ज्याँ बैप्टिस्ट तैवर्नियर ने क्या लिखा है?
उत्तर:
सत्रहवीं सदी में भारत की यात्रा करने वाले फ्रांसीसी यात्री ज्याँ बैप्टिस्ट तैवर्नियर ने गाँवों में मुद्रा के प्रचलन के बारे में लिखा है कि, “भारत में वे गाँव बहुत ही छोटे कहे जायेंगे, जिनमें मुद्रा की फेरबदल करने वाले हाँ ये लोग सराफ कहलाते थे। एक बैंकर की भाँति सराफ हवाला भुगतान करते थे और अपनी इच्छा के अनुसार पैसे के मुकाबले रुपयों की कीमत तथा कौड़ियों के मुकाबले पैसों की कीमत बढ़ा देते थे।”

प्रश्न 12.
सोलहवीं तथा सत्रहवीं शताब्दी में भारत द में जंगलों के प्रसार का वर्णन कीजिए।
उत्तर:
सोलहवीं तथा सत्रहवीं शताब्दी में भारत में जमीन के विशाल भाग जंगल या शादियों से घिरे हुए थे। के ऐसे प्रदेश झारखंड सहित सम्पूर्ण पूर्वी भारत, मध्य भारत, उत्तरी क्षेत्र (जिसमें भारत-नेपाल की सीमावर्ती क्षेत्र की ि तराई शामिल है), दक्षिण भारत का पश्चिमी घाट और प दक्कन के पठारों में फैले हुए थे। समसामयिक स्रोतों से प्राप्त जानकारी के आधार पर यह अनुमान लगाया जा सकता है कि यह भारत में जंगलों के फैलाव का औसत लगभग 40 प्रतिशत था।

प्रश्न 13.
जंगलवासियों पर कौनसे नए सांस्कृतिक प्रभाव पड़े?
उत्तर:
जंगलवासियों पर नए सांस्कृतिक प्रभाव पड़े। कुछ इतिहासकारों की यह मान्यता है कि नए बसे इलाकों के खेतिहर समुदायों ने जिस प्रकार से धीरे-धीरे इस्लाम को अपनाया, उसमें सूफी सन्तों (पीर) ने एक महत्त्वपूर्ण भूमिका निभाई थी। इस प्रकार जंगली प्रदेशों में नए सांस्कृतिक प्रभावों के विस्तार से शुरूआत हुई। प्रश्न 14. जमींदार कौन थे?
उत्तर- जमींदारों की आय तो खेती से आती थी, परन्तु ये कृषि उत्पादन में सीधे भागीदारी नहीं करते थे। वे अपनी जमीन के मालिक होते थे। उन्हें ग्रामीण समाज में ऊँची स्थिति के कारण कुछ विशेष सामाजिक और आर्थिक सुविधाएँ प्राप्त थीं। जमींदारों की ऊँची स्थिति के दो कारण थे-
(1) उनकी जाति तथा
(2) उनके द्वारा राज्य को कुछ विशेष प्रकार की सेवाएं देना।

प्रश्न 15.
जमींदारों की समृद्धि का क्या कारण था?
उत्तर:
जमींदारों की विस्तृत व्यक्तिगत जमीन उनकी समृद्धि का कारण था उनकी व्यक्तिगत जमीन मिल्कियत कहलाती थी अर्थात् सम्पत्ति मिल्कियत जमीन पर जमींदार के निजी प्रयोग के लिए खेती होती थी। प्रायः इन जमीनों पर दिहाड़ी मजदूर या पराधीन मजदूर कार्य करते थे जमींदार अपनी इच्छानुसार इन जमीनों को बेच सकते थे, किसी और के नाम कर सकते थे या उन्हें गिरवी रख सकते थे।

प्रश्न 16.
जमींदारी को पुख्ता करने की प्रक्रिया का वर्णन कीजिए।
उत्तर:
जमींदारी को पुख्ता करने की प्रक्रिया धीमी थी। इसके निम्नलिखित तरीके थे—

  • नयी जमीनों को बसा कर
  • अधिकारों के हस्तान्तरण के द्वारा
  • राज्य के आदेश से
  • या फिर खरीदकर इन प्रक्रियाओं के द्वारा अपेक्षाकृत ‘निचली जातियों के लोग भी जमींदारों के दर्जे में शामिल हो सकते थे, क्योंकि इस काल में जमींदारी धड़ल्ले से खरीदी और बेची जाती थी।

प्रश्न 17.
1665 में औरंगजेब ने जमा निर्धारित करने के लिए अपने राजस्व अधिकारियों को क्या आदेश दिया?
उत्तर:
1665 में औरंगजेब ने जमा निर्धारित करने के लिए अपने राजस्व अधिकारियों को वह आदेश दिया कि वे परगनाओं के अमीनों को निर्देश दें कि वे प्रत्येक गाँव, प्रत्येक किसान (आसामीवार) के बारे में खेती की वर्तमान स्थितियों का पता करें बारीकी से उनकी जाँच करने के बाद सरकार के वित्तीय हितों व किसानों के कल्याण को ध्यान में रखते हुए जमा निर्धारित करें।

JAC Class 12 History Important Questions Chapter 8 किसान, ज़मींदार और राज्य : कृषि समाज और मुगल साम्राज्य

प्रश्न 18.
मुगल काल में कृषि की समृद्धि और भारत की आबादी की बढ़ोत्तरी में क्या सम्बन्ध था?
उत्तर:
मुगलकाल में शासकों की दूरदर्शिता द्वारा किसानों के हितों एवं कृषि उत्पादन के लिए विविध तकनीकों के प्रयोग के कारण कृषि आधारित समृद्धि में व्यापक वृद्धि हुई। इसके परिणामस्वरूप आबादी धीरे-धीरे बढ़ने लगी। आर्थिक इतिहासकारों की गणना के अनुसार 1600 से 1700 के बीच समय-समय पर होने वाली भुखमरी और महावारी के उपरान्त भी भारत की आबादी लगभग 5 करोड़ बढ़ गई। 200 वर्षों में आबादी की यह बढ़ोत्तरी करीब 33 प्रतिशत थी।

प्रश्न 19
सत्रहवीं शताब्दी में दुनिया के अलग- अलग हिस्सों से कई नई फसलें भारतीय उपमहाद्वीप पहुँची।” कथन को सोदाहरण स्पष्ट कीजिए।
अथवा
नई फसलों जैसे मक्का, ज्वार, आलू आदि का भारत में प्रवेश कैसे हुआ?
उत्तर:
सत्रहवीं शताब्दी में दुनिया के अलग-अलग हिस्सों से व्यापारिक आवागमन में वृद्धि के कारण कई नई फसलें भारतीय उपमहाद्वीप पहुँचीं। उदाहरण के रूप में मक्का भारत में अफ्रीका व स्पेन से पहुँची और सत्रहवीं शताब्दी तक इसकी गिनती पश्चिम भारत की मुख्य फसलों में होने लगी। टमाटर, आलू व मिर्च जैसी सब्जियाँ नई दुनिया से लाई गई। इसी प्रकार अनन्नास एवं पपीता भी नई दुनिया से आए।

प्रश्न 20.
जजमानी व्यवस्था क्या थी?
उत्तर:
18वीं शताब्दी में बंगाल में जमींदार लोहारों, बढ़ई व सुनारों जैसे ग्रामीण दस्तकारों को उनकी सेवाओं के बदले दैनिक भत्ता तथा खाने के लिए नकदी देते थे। इस व्यवस्था को जजमानी व्यवस्था कहा जाता था। यद्यपि यह प्रथा सोलहवीं व सत्रहवीं शताब्दी में अधिक प्रचलित नहीं थी।

प्रश्न 21.
मुगलकाल में गाँवों और शहरों के मध्य व्यापार से गाँवों में नकदी लेन-देन होना प्रारम्भ हुआ, सोदाहरण कथन को स्पष्ट कीजिए।
उत्तर:
मुगल काल में गाँवों और शहरों के मध्य व्यापार के कारण गाँवों में भी नकदी लेन-देन होने लगा। मुगलों के केन्द्रीय प्रदेशों से भी कर की गणना और वसूली नकद में दी जाती थी जो दस्तकार निर्यात के लिए उत्पादन करते थे, उन्हें उनकी मजदूरी अथवा पूर्ण भुगतान नकद में ही किया जाता था। इसी प्रकार कपास, रेशम या नील जैसी व्यापारिक फसलें उत्पन्न करने वाले किसानों का भुगतान भी नकदी में ही होता है।

JAC Class 12 History Important Questions Chapter 8 किसान, ज़मींदार और राज्य : कृषि समाज और मुगल साम्राज्य

प्रश्न 22.
आइन-ए-अकबरी’ में किन मसलों पर विस्तार से चर्चा की गई है?
अथवा
‘आइन-ए-अकबरी’ पर टिप्पणी लिखिए।
उत्तर:
आइन-ए-अकबरी’ में अनेक मसलों पर विस्तार से चर्चा की गई है; जैसे दरबार, प्रशासन और सेना का गठन, राजस्व के स्रोत और अकबरी साम्राज्य के प्रान्तों का भूगोल, लोगों के साहित्यिक, सांस्कृतिक एवं धार्मिक रिवाज अकबर की सरकार के समस्त विभागों और प्रान्तों (सूखों) के बारे में जानकारी दी गई है। ‘आइन’ में इन सूबों के बारे में जटिल और आँकड़ेबद्ध सूचनाएँ बड़ी बारीकी से दी गई हैं।

प्रश्न 23.
‘आइन’ कितने भागों (दफ्तरों) में विभक्त है? उनका सक्षिप्त उल्लेख कीजिए।
उत्तर- ‘आइन’ पाँच भागों (दफ्तरों में विभक्त है-
(1) मंजिल आबादी, शाही घर-परिवार और उसके रख- रखाव से सम्बन्ध रखती है
(2) सिपह- आबादी- सैनिक व नागरिक प्रशासन और नौकरों की व्यवस्था के बारे में है।
(3) मुल्क आबादी में साम्राज्य व प्रान्तों के वित्तीय पहलुओं तथा राजस्व की दरों के आँकड़े वर्णित हैं तथा बारह प्रान्तों का वर्णन है। चौथे और पाँचवें भाग भारत के लोगों के धार्मिक, साहित्यिक और सांस्कृतिक रीति-रिवाजों से सम्बन्ध रखते हैं।

प्रश्न 24.
अकबर के समय भूमि का वर्गीकरण किस प्रकार किया गया था?
अथवा
चाचर और बंजर भूमि की परिभाषा लिखिए।
उत्तर:

  • पोलज पोलज भूमि में एक के बाद एक हर फसल की वार्षिक खेती होती थी जिसे कभी खाली नहीं छोड़ा जाता था।
  • परीती परौती जमीन पर कुछ दिनों के लिए खेती रोक दी जाती थी ताकि वह अपनी खोई हुई उर्वरा शक्ति वापस पा सके।
  • चचर पचर जमीन तीन या चार वर्षों तक खाली रहती थी।
  • बंजर – बंजर जमीन वह थी जिस पर पाँच या उससे अधिक वर्षों से खेती नहीं की जाती थी।

प्रश्न 25.
कणकुत प्रणाली क्या थी?
उत्तर:
अकबर के समय कनकृत प्रणाली राजस्व निर्धारण की एक प्रणाली थी। हिन्दी में ‘कण’ का अर्थ है ‘अनाज’ और कुंत का अर्थ है अनुमान’। इसके अनुसार फसल को अलग-अलग पुलिन्दों में काटा जाता था-
(1) अच्छा
(2) मध्यम
(3) खराब इस प्रकार सन्देह दूर किया जाना चाहिए। प्रायः अनुमान से किया गया जमीन का आकलन भी पर्याप्त रूप से सही परिणाम देता था।

JAC Class 12 History Important Questions Chapter 8 किसान, ज़मींदार और राज्य : कृषि समाज और मुगल साम्राज्य

प्रश्न 26.
भारत में सोलहवीं तथा सत्रहवीं सदी के ग्रामीण समाज की क्या स्थिति थी?
उत्तर:
सोलहवीं तथा सत्रहवीं सदी में भारत में लगभग 85 प्रतिशत लोग गाँवों में रहते थे छोटे खेतिहर तथा भूमिहर जमींदार दोनों ही कृषि उत्पादन से जुड़े हुए थे और दोनों ही फसल के हिस्सों के दावेदार थे। इससे उनके बीच सहयोग, प्रतियोगिता तथा संघर्ष के सम्बन्ध बने खेती से जुड़े इन समस्त सम्बन्धों के ताने-बाने से गाँव का समाज बनता था। मुगल-राज्य अपनी आय का बहुत बड़ा भाग कृषि उत्पादन से वसूल करता था राजस्व अधिकारी ग्रामीण समाज पर नियन्त्रण रखते थे

प्रश्न 27.
खेतिहर समाज का परिचय दीजिए। कृषि समाज की भौगोलिक विविधताओं का भी उल्लेख कीजिए।
उत्तर:
खेतिहर समाज की मूल इकाई गाँव थी, जिसमें किसान रहते थे किसान वर्ष भर फसल की पैदावार से जुड़े समस्त कार्य करते थे। इन कार्यों में जमीन की जुताई, बीज बोना और फसल पकने पर उसे काटना आदि सम्मिलित थे। इसके अतिरिक्त वे उन वस्तुओं के उत्पादन में भी शामिल थे, जो कृषि आधारित थीं जैसे शक्कर, तेल इत्यादि । परन्तु सूखी भूमि के विशाल हिस्सों से लेकर पहाड़ियों वाले क्षेत्र में उस प्रकार की खेती नहीं हो सकती थी जैसी कि अधिक उपजाऊ जमीनों पर।

प्रश्न 28.
सोलहवीं तथा सत्रहवीं शताब्दी का कृषि- इतिहास लिखने के लिए ‘आइन-ए-अकबरी’ का मुख्य स्रोत के रूप में विवेचन कीजिए।
उत्तर:
सोलहवीं तथा सत्रहवीं शताब्दी का कृषि इतिहास लिखने के लिए ‘आइन-ए-अकबरी’ एक महत्त्वपूर्ण स्रोत है। ‘आइन-ए-अकबरी’ की रचना मुगल सम्राट अकबर के दरवारी इतिहासकार अबुल फतल ने की थी खेतों की नियमित जुलाई सुनिश्चित करने के लिए राज्य के प्रतिनिधियों द्वारा करों की वसूली करने के लिए तथा राज्य व जमींदारों के बीच के सम्बन्धों के नियमन के लिए राज्य द्वारा किये गए प्रबन्धों का लेखा-जोखा ‘आइन’ में प्रस्तुत किया गया है।

प्रश्न 29.
आइन-ए-अकबरी’ के लेखन का मुख्य उद्देश्य स्पष्ट कीजिये।
उत्तर:
आइन-ए-अकबरी’ का मुख्य उद्देश्य अकबर के साम्राज्य की एक ऐसी रूप-रेखा प्रस्तुत करना था, जहाँ एक सुदृढ़ सत्ताधारी वर्ग सामाजिक मेल-जोल बनाकर रखता था। अबुल फजल के अनुसार मुगल साम्राज्य के विरुद्ध कोई भी विद्रोह या किसी भी प्रकार की स्वायत्त सत्ता की दावेदारी का असफल होना निश्चित था अतः ‘आइन’ से किसानों के बारे में जो कुछ पता चलता है, वह मुगल शासक वर्ग का ही दृष्टिकोण है।

प्रश्न 30.
सोलहवीं सत्रहवीं शताब्दी के कृषि- इतिहास की जानकारी के लिए ‘आइन’ के अतिरिक्त अन्य स्रोतों का वर्णन कीजिए।
उत्तर:
(1) ‘आइन’ के अतिरिक्त सत्रहवीं व अठारहवीं सदियों के गुजरात, महाराष्ट्र और राजस्थान से उपलब्ध होने वाले वे दस्तावेज भी हैं जो सरकार की आय की विस्तृत जानकारी देते हैं।
(2) इसके अतिरिक्त ईस्ट इण्डिया कम्पनी के भी बहुत से दस्तावेज हैं जो पूर्वी भारत में कृषि सम्बन्धों की उपयोगी रूपरेखा प्रस्तुत करते हैं। इन सभी स्रोतों में किसानों, जमींदारों तथा राज्य के बीच होने वाले झगड़ों के विवरण मिलते हैं। इनसे किसानों के राज्य के प्रति दृष्टिकोण की जानकारी मिलती है।

प्रश्न 31.
सोलहवीं तथा सत्रहवीं शताब्दी में किसान की समृद्धि का मापदंड क्या था ?
उत्तर:
सोलहवीं तथा सत्रहवीं शताब्दी में उत्तर भारत के एक औसत किसान के पास शायद ही कभी एक जोड़ी बैल तथा दो हलों से अधिक कुछ होता था। अधिकांश किसानों के पास इससे भी कम था। गुजरात के वे किसान समृद्ध माने जाते थे जिनके पास 6 एकड़ के लगभग जमीन थी दूसरी ओर, बंगाल में एक औसत किसान की जमीन की ऊपरी सीमा 5 एकड़ थी वहाँ 10 एकड़ जमीन वाले किसान को धनी माना जाता था। खेती व्यक्तिगत स्वामित्व के सिद्धान्त पर आधारित थी।

JAC Class 12 History Important Questions Chapter 8 किसान, ज़मींदार और राज्य : कृषि समाज और मुगल साम्राज्य

प्रश्न 32.
‘बाबरनामा’ में बाबर ने कृषि-समाज की विशेषताओं का किस प्रकार वर्णन किया है?
उत्तर:
‘बाबरनामा’ के अनुसार भारत में बस्तियाँ और गाँव, वस्तुत: शहर के शहर, एक क्षण में ही बीरान भी हो जाते थे और बस भी जाते थे। दूसरी ओर, यदि वे किसी पर बसना चाहते थे तो उन्हें पानी के रास्ते खोदने की आवश्यकता नहीं पड़ती थी क्योंकि उनकी समस्त फसलें वर्षा के पानी में उगती थीं भारत की अनगिनत आबादी होने के कारण लोग उमड़ते चले आते थे । यहाँ झोंपड़ियाँ बनाई जाती थीं और अकस्मात एक गाँव या शहर तैयार हो जाता था।

प्रश्न 33.
भारत में सोलहवीं तथा सत्रहवीं शताब्दी में कृषि क्षेत्र में हुए विकास का वर्णन कीजिए।
उत्तर:
जमीन की प्रचुरता, मजदूरों की उपलब्धता तथा किसानों की गतिशीलता के कारण कृषि का निरन्तर विकास हुआ। चूँकि खेती का प्राथमिक उद्देश्य लोगों का पेट भरना था, इसलिए दैनिक भोजन में काम आने वाले खाद्य पदार्थों जैसे चावल, गेहूं, ज्वार इत्यादि फसलें सबसे अधिक उगाई जाती थीं। जिन प्रदेशों में प्रतिवर्ष 40 इंच या उससे अधिक वर्षा होती थी, वहाँ न्यूनाधिक चावल की खेती होती थी कम वर्षा वाले प्रदेशों में गेहूँ तथा ज्वार बाजरे की खेती होती थी।

प्रश्न 34.
भारत में सोलहवीं तथा सत्रहवीं शताब्दी में सिंचाई के साधनों में हुए विकास का वर्णन कीजिए।
उत्तर:
आज की भाँति सोलहवीं सत्रहवीं शताब्दी में भी मानसून को भारतीय कृषि की रीढ़ माना जाता था। परन्तु कुछ फसलों के लिए अतिरिक्त पानी की आवश्यकता थी। इसके लिए सिंचाई के कृत्रिम साधनों का सहारा लेना पड़ा। राज्य की ओर से सिंचाई कार्यों में सहायता दी जाती थी उदाहरणार्थ, उत्तर भारत में राज्य ने कई नई नहरें व नाले खुदवाये तथा कई पुरानी नहरों की मरम्मत करवाई। शाहजहाँ के शासन काल में पंजाब में ‘शाह नहर’ इसका उदाहरण है।

प्रश्न 35.
मुगलकालीन कृषि समाज में महिलाओं की स्थिति बताइए।
उत्तर:
मुगलकालीन कृषि समाज में महिलाएँ व पुरुष मिलजुलकर खेती का कार्य करते थे। पुरुष खेत जोतते थे तथा हल चलाते थे जबकि महिलाएँ बुआई, निराई व कटाई के साथ-साथ पकी हुई फसल से दाना निकालने का कार्य करती थीं। सूत कातना, वर्तन बनाने के लिए मिट्टी को साफ करना व गूंथना तथा कपड़ों पर कढ़ाई जैसे दस्तकारी के कार्य भी महिलाएँ ही करती थीं महिलाओं पर परिवार और समुदाय के पुरुषों का नियन्त्रण बना रहता था।

प्रश्न 36.
सामाजिक कारणों से जंगलवासियों के जीवन में किस प्रकार परिवर्तन आया?
उत्तर:
सामाजिक कारणों से जंगलवासियों के जीवन में परिवर्तन आया। ग्रामीण समुदाय के बड़े आदमियों की तरह जंगली कबीलों के भी सरदार होते थे। कई कबीलों के सरदार धीरे-धीरे जमींदार बन गए। कुछ तो राजा भी बन गए। 16वीं- 17वीं शताब्दी में कुछ राजाओं ने पड़ोसी कवीलों के साथ एक के बाद एक युद्ध किया और उन पर अपना अधिकार स्थापित कर लिया।

JAC Class 12 History Important Questions Chapter 8 किसान, ज़मींदार और राज्य : कृषि समाज और मुगल साम्राज्य

प्रश्न 37.
जंगलवासियों पर कौनसे नए सांस्कृतिक प्रभाव पड़े?
उत्तर:
जंगलवासियों पर नए-नए सांस्कृतिक प्रभाव पड़े कुछ इतिहासकारों की यह मान्यता है कि नए बसे क्षेत्रों के खेतिहर समुदायों ने जिस प्रकार से धीरे-धीरे इस्लाम को अपनाया, उसमें सूफी सन्तों ने एक बड़ी भूमिका निभाई थी।

प्रश्न 38.
सोलहवीं तथा सत्रहवीं शताब्दियों में भारत में खेती के विकास के लिए किसानों द्वारा अपनायी गई तकनीकों का वर्णन कीजिए।
उत्तर:
(1) किसान लकड़ी के ऐसे हल्के हल का प्रयोग करते थे जिसको एक छोर पर लोहे की नुकीली धार या फाल लगा कर सरलता से बनाया जा सकता था।
(2) बैलों के जोड़े के सहारे खींचे जाने वाले बरमे का प्रयोग बीज बोने के लिए किया जाता था परन्तु बीजों को हाथ से छिड़क कर बोने की पद्धति अधिक प्रचलित थी।
(3) मिट्टी की गुड़ाई और निराई के लिए लकड़ी के मूठ वाले लोहे के पतले धार प्रयुक्त किए जाते थे।

प्रश्न 39.
भारत में तम्बाकू का प्रसार किस प्रकार हुआ? जहाँगीर ने तम्बाकू के धूम्रपान पर प्रतिबन्ध क्यों लगा दिया?
उत्तर:
तम्बाकू का पौधा सबसे पहले दक्षिण भारत पहुँचा और वहाँ से सरहवीं शताब्दी के प्रारम्भिक वर्षों में इसे उत्तर भारत लाया गया। अकबर और उसके अमीरों ने 1604 ई. में पहली बार तम्बाकू देखा सम्भवत: इसी समय से भारत में तम्बाकू का धूम्रपान (हुक्के या चिलम में) करने के व्यसन ने जोर पकड़ा। परन्तु जहाँगीर ने तम्बाकू के धूम्रपान पर प्रतिबन्ध लगा दिया। इस प्रतिबन्ध का कोई प्रभाव नहीं पड़ा क्योंकि इसका सत्रहवीं शताब्दी के अन्त तक सम्पूर्ण भारत में प्रचलन था।

प्रश्न 40.
मुगलकाल में मौसम के चक्रों में उगाई जाने वाली विविध फसलों का वर्णन कीजिए।
उत्तर:
मुगल काल में भारत में मौसम के दो मुख्य चक्रों के दौरान खेती की जाती थी एक खरीफ (पतझड़ में) तथा दूसरी रवी (बसन्त में) अधिकतर स्थानों पर वर्ष में कम से कम दो फसलें उगाई जाती थीं जहाँ वर्षा अथवा सिंचाई के अन्य साधन हर समय उपलब्ध थे, वहाँ यो वर्ष में तीन फसलें भी उगाई जाती थीं ‘आइन-ए- अकबरी’ के अनुसार दोनों मौसमों को मिलाकर मुगल- प्रान्त आगरा में 39 तथा दिल्ली प्रान्त में 43 फसलों की उगाई की जाती थी।

प्रश्न 41.
सोलहवीं तथा सत्रहवीं शताब्दियों में भारत ‘में ‘जिन्स-ए-कामिल’ नामक फसलें क्यों उगाई जाती थीं?
उत्तर:
सोलहवीं तथा सत्रहवीं शताब्दियों में भारत में ‘जिन्स-ए-कामिल’ (सर्वोत्तम फसलें) नामक फसलें भी उगाई जाती थीं मुगल राज्य भी किसानों को ‘जिन्स-ए- कामिल’ नामक फसलों की खेती करने के लिए प्रोत्साहन देता था क्योंकि इनसे राज्य को अधिक कर मिलता था। कपास और गने जैसी फसलें श्रेष्ठ ‘जिन्स-ए-कामिल’ श्रीं तिलहन (जैसे सरसों) तथा दलहन भी नकदी फसलों में सम्मिलित थीं।

JAC Class 12 History Important Questions Chapter 8 किसान, ज़मींदार और राज्य : कृषि समाज और मुगल साम्राज्य

प्रश्न 42.
जाति और अन्य जाति जैसे भेदभावों के कारण खेतिहर किसान किन समूहों में बँटे हुए थे?
उत्तर:
खेतों की जुताई करने वालों में एक बड़ी संख्या उन लोगों की थी जो निकृष्ट समझे जाने वाले कामों में लगे थे अथवा फिर खेतों में मजदूरी करते थे कुछ जातियों के लोगों को केवल निकृष्ट समझे जाने वाले काम ही दिए जाते थे। इस प्रकार वे गरीबी का जीवन व्यतीत करने के लिए बाध्य थे। गाँव की आबादी का बहुत बड़ा भाग ऐसे ही समूहों का था। इनके पास संसाधन सबसे कम थे तथा ये जाति व्यवस्था के प्रतिबन्धों से बंधे हुए थे इनकी स्थिति न्यूनाधिक आधुनिक भारत के दलितों जैसी थी।

प्रश्न 43.
“समाज के निम्न वर्गों में जाति, गरीबी तथा सामाजिक हैसियत के बीच सीधा सम्बन्ध था। परन्तु ऐसा बीच के समूहों में नहीं था।” स्पष्ट कीजिए।
उत्तर- सत्रहवीं सदी में मारवाड़ में लिखी गई एक पुस्तक में राजपूतों का उल्लेख किसानों के रूप में किया गया है। इस पुस्तक के अनुसार जाट भी किसान थे परन्तु जाति व्यवस्था में उनका स्थान राजपूतों की तुलना में नीचा था सत्रहवीं सदी में वृन्दावन (उत्तर प्रदेश) के क्षेत्र में रहने वाले गौरव समुदाय ने भी राजपूत होने का दावा किया, यद्यपि वे जमीन की जुताई के काम में लगे थे। अहीर, गुज्जर तथा माली जैसी जातियों के सामाजिक स्तर में भी वृद्धि हुई।

प्रश्न 44.
पंचायत का मुखिया कौन होता था? उसके कार्यों का उल्लेख कीजिए।
उत्तर:
पंचायत का सरदार एक मुखिया होता था, जिसे ‘मुकदम’ चा ‘मंडल’ कहते थे। मुखिया का चुनाव गाँव के बुजुर्गों की आम सहमति से होता था। इस चुनाव के बाद उन्हें इसकी स्वीकृति जमींदार से लेनी पड़ती थी। मुखिया अपने पद पर तभी तक बना रह सकता था, जब तक गाँव के बुजुगों को उस पर भरोसा था। गाँव की आप व खर्चे का हिसाब-किताब अपनी निगरानी में बनवाना मुखिया का प्रमुख कार्य था। इस कार्य में पंचायत का पटवारी उसको सहायता करता था।

प्रश्न 45.
मुगल काल में गाँव के ‘आम खजाने’ से पंचायत के कौनसे खर्चे बलते थे?
उत्तर:
(1) मुगल काल में पंचायत का खर्चा के आम खजाने से चलता था इस खजाने से उन कर अधिकारियों की खातिरदारी का खर्चा भी किया जाता था, जो समय-समय पर गाँव का दौरा किया करते थे।
(2) इस आम खजाने का प्रयोग बाढ़ जैसी प्राकृतिक विपदाओं से निपटने के लिए भी किया जाता था।
(3) इस कोष का प्रयोग ऐसे सामुदायिक कार्यों के लिए भी किया जाता था जो किसान स्वयं नहीं कर सकते थे, जैसे छोटे-मोटे बांध बनाना या नहर खोदना

प्रश्न 46.
मुगलकालीन जाति पंचायतों के कार्यों का वर्णन कीजिए।
उत्तर:

  • राजस्थान में जाति पंचायतें अलग-अलग जातियों के लोगों के बीच दीवानी के झगड़ों का निपटारा करती थीं।
  • वे जमीन से जुड़े दावेदारियों के झगड़े सुलझाती
  • वे यह निश्चित करती थीं कि विवाह जातिगत मानदंडों के अनुसार हो रहे हैं या नहीं।
  • वे यह निश्चित करती थीं कि गाँव के आयोजन में किसको किसके ऊपर प्राथमिकता दी जाएगी। कर्मकाण्डीय वर्चस्व किस क्रम में होगा।

प्रश्न 47
पंचायतों में ग्रामीण समुदाय के निम्न वर्ग के लोग उच्च जातियों तथा राज्य के अधिकारियों के विरुद्ध कौनसी शिकायतें प्रस्तुत करते थे?
उत्तर:
पश्चिमी भारत, विशेषकर राजस्थान और महाराष्ट्र जैसे प्रान्तों से प्राप्त दस्तावेजों में ऐसे कई प्रार्थना-पत्र हैं, जिनमें पंचायत से ‘सी’ जातियों अथवा राज्य के अधिकारियों के विरुद्ध जबरन कर वसूली अथवा बेगार वसूली की शिकायत की गई है। इनमें किसी जाति या सम्प्रदाय विशेष के लोग अभिजात वर्ग के समूहों की उन माँगों के विरुद्ध अपना विरोध दर्शाते थे जिन्हें वे नैतिक दृष्टि से अवैध मानते थे। उनमें एक मांग बहुत अधिक कर की माँग थी।

प्रश्न 48
पंचायतों द्वारा इन शिकायतों का निपटारा किस प्रकार किया जाता था ?
उत्तर:
निचली जाति के किसानों और राज्य के अधिकारियों या स्थानीय जमींदारों के बीच झगड़ों में पंचायत के निर्णय अलग-अलग मामलों में अलग-अलग हो सकते थे। अत्यधिक कर की मांगों में पंचायत प्रायः समझौते का सुझाव देती थीं जहाँ समझौते नहीं हो पाते थे, वहाँ किसान विरोध के अधिक उम्र साधन अपनाते थे, जैसे कि गाँव छोड़ कर भाग जाना।

JAC Class 12 History Important Questions Chapter 8 किसान, ज़मींदार और राज्य : कृषि समाज और मुगल साम्राज्य

प्रश्न 49.
खेतिहर लोग किन ग्रामीण दस्तकारियों में संलग्न रहते थे?
उत्तर:
खेतिहर और उसके परिवार के सदस्य विभिन्न प्रकार की वस्तुओं के उत्पादन में संलग्न रहते थे जैसे-रंगरेजी, कपड़े पर छपाई, मिट्टी के बर्तनों को पकाना, खेती के औजारों को बनाना अथवा उनकी मरम्मत करना। जब किसानों को खेती के काम से अवकाश मिलता था, जैसे-बुआई और सुड़ाई के बीच या सुहाई और कटाई के बीच, उस अवधि में ये खेतिहर दस्तकारी के काम में संलग्न रहते थे।

प्रश्न 50.
ग्रामीण दस्तकार किस रूप में अपनी सेवाएँ गाँव के लोगों को देते थे? इसके बदले गाँव के लोग उन सेवाओं की अदायगी किन तरीकों से करते थे?
उत्तर:
कुम्हार, लोहार, बढ़ई, नाई, यहाँ तक कि सुनार जैसे ग्रामीण दस्तकार अपनी सेवाएँ गाँव के लोगों को देते थे, जिसके बदले गाँव वाले उन्हें विभिन्न तरीकों से उन सेवाओं की अदायगी करते थे। प्रायः वे या तो उन्हें फसल का एक भाग दे देते थे या फिर गाँव की जमीन का एक टुकड़ा अदायगी का तरीका सम्भवतः पंचायत ही तय करती थी कुछ स्थानों पर दस्तकार तथा प्रत्येक खेतिहर परिवार आपसी बातचीत करके अदायगी का तरीका तय कर लेते थे।

प्रश्न 51.
कुछ अंग्रेज अफसरों द्वारा भारतीय गाँवों को ‘छोटे गणराज्य’ क्यों कहा गया?
अथवा
ग्रामीण समुदाय के महत्त्व का विवेचन कीजिए।
अथवा
‘छोटे गणराज्य’ के रूप में भारतीय गाँवों का महत्त्व स्पष्ट कीजिए।
उत्तर:
उन्नीसवीं सदी के कुछ अंग्रेज अधिकारियों ने भारतीय गाँवों को एक ऐसे ‘छोटे गणराज्य’ की संज्ञा दी, जहाँ लोग सामूहिक स्तर पर भाई चारे के साथ संसाधनों तथा श्रम का विभाजन करते थे परन्तु ऐसा नहीं लगता कि गाँव में सामाजिक समानता थी सम्पत्ति पर व्यक्तिगत स्वामित्व होता था। इसके साथ ही जाति और लिंग के नाम पर समाज में गहरी विषमताएँ थीं कुछ शक्तिशाली लोग गाँव की समस्याओं पर निर्णय लेते थे और कमजोर वर्गों के लोगों का शोषण करते थे।

प्रश्न 52.
भूमिहर भद्रजनों में महिलाओं को प्राप्त पैतृक सम्पत्ति के अधिकार का वर्णन कीजिए।
उत्तर:
भूमिहर भद्रजनों में महिलाओं को पैतृक सम्पत्ति का अधिकार प्राप्त था। पंजाब में महिलाएँ (विधवा महिलाएँ भी) पैतृक सम्पत्ति के विक्रेता के रूप में ग्रामीण जमीन के क्रय-विक्रय में सक्रिय भागीदारी रखती थीं हिन्दू और मुसलमान महिलाओं को जमींदारी उत्तराधिकार में मिलती थी, जिसे बेचने अथवा गिरवी रखने के लिए वे स्वतंत्र थीं। बंगाल में भी महिला जमींदार थीं। वहाँ राजशाही की जमींदारी की कर्ता-धर्ता एक स्वी थी।

प्रश्न 53.
‘जंगली’ कौन थे? मुगलकाल में जंगली शब्द का प्रयोग किन लोगों के लिए किया जाता था?
उत्तर:
समसामयिक रचनाएँ जंगल में रहने वालों के लिए ‘जंगली’ शब्द का प्रयोग करती हैं। परन्तु जंगली होने का अर्थ यह नहीं था कि वे असभ्य थे। मुगलकाल में जंगली शब्द का प्रयोग ऐसे लोगों के लिए होता था जिनका निर्वाह जंगल के उत्पादों, शिकार और स्थानान्तरीच खेती से होता था। ये काम मौसम के अनुसार होते थे। उदाहरण के लिए, भील बसन्त के मौसम में जंगल के उत्पाद इकट्ठा करते थे, गर्मियों में मछली पकड़ते थे तथा मानसून के महीनों में खेती करते थे।

JAC Class 12 History Important Questions Chapter 8 किसान, ज़मींदार और राज्य : कृषि समाज और मुगल साम्राज्य

प्रश्न 54.
राज्य की दृष्टि में जंगल किस प्रकार का क्षेत्र था?
उत्तर:
राज्य की दृष्टि में जंगल राज्य की दृष्टि में जंगल उलटफेर वाला इलाका था अपराधियों को शरण देने वाला अड्डा राज्य के अनुसार अपराधी और विद्रोही जंगल में शरण लेते थे अपराध करने के बाद ये लोग जंगल में छिप जाते थे। बाबर के अनुसार जंगल एक ऐसा रक्षाकवच था जिसके पीछे परगना के लोग कड़े विद्रोही हो जाते थे और कर चुकाने से मुकर जाते थे।”

प्रश्न 55.
मुगल काल में बाहरी शक्तियाँ जंगलों में किसलिए प्रवेश करती थीं?
उत्तर:
(1) मुगलकाल में बाहरी शक्तियाँ जंगलों में कई तरह से प्रवेश करती थीं। उदाहरणार्थ, राज्य को सेना के लिए हाथियों को आवश्यकता होती थी इसलिए जंगलवासियों से ली जाने वाली भेंट में प्रायः हाथी भी सम्मिलित होते थे।
(2) शिकार अभियान के नाम पर मुगल सम्राट अपने साम्राज्य के भिन्न-भिन्न भागों का दौरा करते थे और लोगों की समस्याओं और शिकायतों पर उचित ध्यान देते थे।

प्रश्न 56.
मुगल-राजनीतिक विचारधारा में शिकार अभियान का क्या महत्त्व था?
उत्तर:
मुगल राजनीतिक विचारधारा में गरीबों और अमीरों सहित सबको न्याय प्रदान करने के उद्देश्य से ‘शिकार अभियान’ प्रारम्भ किया गया। शिकार अभियान के नाम पर मुगल सम्राट अपने विशाल साम्राज्य के कोने-कोने का दौरा करता था। इस प्रकार वह अलग-अलग प्रदेशों के लोगों की समस्याओं और शिकायतों पर व्यक्तिगत रूप से ध्यान देता था और न्याय प्रदान करता था।

प्रश्न 57.
जंगलवासियों के जीवन पर बाहरी कारक के रूप में वाणिज्यिक खेती का क्या प्रभाव पड़ा?
उत्तर:
जंगलवासियों के जीवन पर वाणिज्यिक खेती का काफी प्रभाव पड़ता था। शहद, मधुमोम तथा लाक जैसे जंगल के उत्पादों की बहुत माँग थी। लाक जैसी कुछ वस्तुएँ तो सत्रहवीं सदी में भारत से समुद्र पार होने वाले निर्यात की मुख्य वस्तुएँ थीं। हाथी भी पकड़े और बेचे जाते थे व्यापार के अन्तर्गत वस्तुओं की अदला-बदली भी होती थी। कुछ कवीले भारत और अफगानिस्तान के बीच होने वाले स्थलीय व्यापार में लगे हुए थे वे पंजाब के गाँवों और शहरों के बीच होने वाले व्यापार में भी भाग लेते थे।

प्रश्न 58.
सामाजिक कारणों से जंगलवासियों के जीवन में क्या परिवर्तन हुए?
उत्तर:
सामाजिक कारणों से जंगलवासियों के जीवन में महत्त्वपूर्ण परिवर्तन हुए। जंगली कबीलों के भी सरदार होते थे। कई जंगली कबीलों के सरदार जमींदार बन गए। कुछ तो राजा भी बन गए। ऐसे में उन्हें सेना का गठन करने की आवश्यकता हुई। अतः उन्होंने अपने ही वंश के लोगों को सेना में भर्ती किया अथवा फिर उन्होंने अपने ही भाई- बन्धुओं से सैन्य-सेच की मांग की। सिन्ध प्रदेश की कमीलाई सेनाओं में 6000 घुड़सवार और 7000 पैदल सैनिक होते थे।

प्रश्न 59.
मुगलकालीन भारत में जमींदारों की शक्ति के स्रोतों का उल्लेख कीजिए।
उत्तर:
(1) जमींदारों को शक्ति इस बात से मिलती धी कि वे प्रायः राज्य की ओर से कर वसूल कर सकते थे। इसके बदले उन्हें वित्तीय मुआवजा मिलता था।
(2) सैनिक संसाधन उनकी शक्ति का एक और स्रोत था अधिकतर जमींदारों के पास अपने किले भी थे और अपनी सैनिक टुकड़ियाँ भी, जिनमें घुड़सवारों, तोपखाने तथा पैदल सैनिकों के जत्थे होते थे।

JAC Class 12 History Important Questions Chapter 8 किसान, ज़मींदार और राज्य : कृषि समाज और मुगल साम्राज्य

प्रश्न 60.
जमींदारों की उत्पत्ति का स्रोत क्या था?
उत्तर:
समसामयिक दस्तावेजों से प्रतीत होता है कि युद्ध में विजय जमींदारों की उत्पत्ति का सम्भावित लोत रहा होगा। प्रायः जमींदारी के प्रसार का एक तरीका था शक्तिशाली सैनिक सरदारों द्वारा कमजोर लोगों को बेदखल करना। परन्तु इसकी सम्भावना कम ही दिखाई देती है कि राज्य किसी जमींदार को इतने आक्रामक रुख अपनाने की अनुमति देता हो, जब तक कि एक राज्यादेश के द्वारा इसकी पुष्टि पहले ही नहीं कर दी गई हो।

प्रश्न 61.
कृषि के विकास में जमींदारों के योगदान का वर्णन कीजिए।
उत्तर:

  • जमींदारों ने कृषि योग्य जमीनों को बसने में नेतृत्व प्रदान किया और खेतिहरों को खेती के उपकरण व उधार देकर उन्हें वहाँ बसने में भी सहायता की।
  • जमींदारी के क्रय-विक्रय से गाँवों के मौद्रीकरण की प्रक्रिया में तेजी आई।
  • जमींदार अपनी मिल्कियत की जमीनों की फसल भी बेचते थे।
  • कुछ साक्ष्य दर्शाते हैं कि जमींदार प्रायः बाजार (हाट) स्थापित करते थे जहाँ किसान भी अपनी फसलें बेचने आते थे।

प्रश्न 62.
“किसानों से जमींदारों के सम्बन्धों में पारस्परिकता, पैतृकवाद तथा संरक्षण का पुट था।” व्याख्या कीजिए।
उत्तर:
यद्यपि जमींदार शोषण करने वाला वर्ग था, परन्तु किसानों से उनके सम्बन्धों में पारस्परिकता, पैतृकवाद तथा संरक्षण का पुट था। भक्ति सन्तों ने बड़ी निर्भीकतापूर्वक जातिगत तथा अन्य प्रकार के अत्याचारों की कटु निन्दा की, परन्तु उन्होंने जमींदारों को किसानों के शोषक या उन पर अत्याचार करने वाले के रूप में नहीं दर्शाया। सत्रहवीं सदी में हुए कृषि विद्रोहों में राज्य के विरुद्ध जमींदारों को प्राय: किसानों का समर्थन मिला।

प्रश्न 63.
मुगलकालीन भू-राजस्व प्रणाली की विवेचना कीजिए।
अथवा
अकबर के शासन में भूमि के वर्गीकरण किये जाने के बारे में अबुल फजल ने अपने ग्रन्थ ‘आइन’ में क्या वर्णन किया है?
अथवा
अकबर के समय में प्रचलित भू-राजस्व व्यवस्था का वर्णन कीजिए।
अथवा
मुगलों की भू-राजस्व व्यवस्था का वर्णन कीजिये ।
अथवा
अकबर की भू-राजस्व व्यवस्था की मुख्य विशेषताओं का उल्लेख कीजिये।
उत्तर:
अकबर ने जमीनों का वर्गीकरण किया और प्रत्येक वर्ग की जमीन के लिए अलग-अलग राजस्व निर्धारित किया। भूमि को चार वर्गों में बाँटा गया-
(i) पोलज
(ii) पोलज और परौती की तीन मध्यम
(iii) चचर
(iv) बंजर। और प्रत्येक वर्ग की तीन किस्में थीं-
(1) अच्छी
(2) किस्म
(3) खराब प्रत्येक की जमीन की उपज को जोड़ा जाता था और उसका तीसरा हिस्सा औसत उपज माना जाता था इसका एक- तिहाई भाग राजकीय शुल्क माना जाता था।

JAC Class 12 History Important Questions Chapter 8 किसान, ज़मींदार और राज्य : कृषि समाज और मुगल साम्राज्य

प्रश्न 64.
अकबर के शासन काल में राजस्व की वसूली के लिए अपनाई गई प्रणालियों का ‘आइन’ में किस प्रकार वर्णन किया गया है?
उत्तर:
अकबर के शासन काल में फसल के रूप में राजस्व वसूली की पहली प्रणाली कुणकुत प्रणाली थी। इसमें फसल को तीन अलग-अलग पुलिन्दों में काटा जाता था दूसरी प्रणाली बटाई या भाओली थी, जिसमें फसल काट कर जमा कर लेते थे और फिर सभी पक्षों की सहमति से बंटवारा किया जाता था। खेत बटाई तीसरी प्रणाली थी जिसमें बीज बोने के बाद खेत बाँट लेते थे। लाँग बटाई चौथी प्रणाली थी जिसमें फसल काटने के बाद उसे आपस में बाँट लेते थे।

प्रश्न 65.
मनसबदारी व्यवस्था’ पर एक संक्षिप्त टिप्पणी लिखिए।
अथवा
मनसबदारी व्यवस्था की मुख्य विशेषताओं का वर्णन कीजिए।
अथवा
अकबर की मनसबदारी व्यवस्था की विवेचना कीजिये।
उत्तर:
मनसबदारी मुगल प्रशासनिक व्यवस्था के शीर्ष पर एक सैनिक नौकरशाही तन्त्र था। इस पर राज्य के सैनिक व नागरिक मामलों की जिम्मेदारी थी कुछ मनसबदारों को नकदी भुगतान किया जाता था परन्तु अधिकांश मनसबदारों को साम्राज्य के भिन्न-भिन्न भागों में राजस्व के आवंटन के द्वारा भुगतान किया जाता था। सम्राट द्वारा समय-समय पर उनका स्थानान्तरण किया जाता था। प्रायः योग्य और अनुभवी व्यक्ति को ही मनसबदार के पद पर नियुक्त किया जाता था।

प्रश्न 66.
सोलहवीं तथा सत्रहवीं सदी में भारत में चाँदी का बहाव क्यों और किस प्रकार हुआ? उत्तर- सोलहवीं और सत्रहवीं सदी में भारत के व्यापार में अत्यधिक उन्नति हुई। इस कारण भारत के समुद्र पार व्यापार में एक ओर भौगोलिक विविधता आई, तो दूसरी ओर अनेक नई वस्तुओं का व्यापार भी शुरू हुआ। निरन्तर बढ़ते व्यापार के साथ, भारत से निर्यात होने वाली वस्तुओं का भुगतान करने के लिए एशिया में भारी मात्रा में चाँदी आई। इस चाँदी का एक बड़ा भाग भारत पहुँचा। यह भारत के लिए लाभप्रद था; क्योंकि यहाँ चाँदी के प्राकृतिक संसाधन नहीं थे।

प्रश्न 67.
सोलहवीं तथा सत्रहवीं शताब्दी में भारत में चाँदी के बहाव के क्या परिणाम हुए?
उत्तर:
(1) सोलहवीं से अठारहवीं सदी के बीच भारत में धातु मुद्रा विशेषकर चाँदी के रुपयों की प्राप्ति में अच्छी स्थिरता बनी रही।
(2) अर्थव्यवस्था में मुद्रा संचार और सिक्कों की तलाई में अभूतपूर्व विस्तार हुआ तथा मुगल राज्य को नकदी कर वसूलने में आसानी हुई।
(3) इटली के यात्री जोवान्नी कारेरी के अनुसार चाँदी समस्त विश्व से होकर भारत पहुँचती थी तथा सत्रहवी शताब्दी में भारत में अत्यधिक मात्रा में नकदी और वस्तुओं का आदान-प्रदान हो रहा था।

प्रश्न 68
आइन-ए-अकबरी’ की त्रुटियों पर प्रकाश डालिए।
उत्तर:
(1) जोड़ करने में कई गलतियाँ पाई गई हैं।
(2) ‘आइन’ के संख्यात्मक आँकड़ों में विषमताएँ हैं।
(3) जहाँ सूबों से लिए गए राजकोषीय आँकड़े बड़े विस्तार से दिए गए हैं, वहीं उन्हीं प्रदेशों से कीमतों और मजदूरी से सम्बन्धित आँकड़े इतने विस्तार के साथ नहीं दिए गए हैं।
(4) मूल्यों और मजदूरी की दरों की सूची मुगल साम्राज्य की राजधानी आगरा या उसके आस-पास के प्रदेशों से ली गई है।

प्रश्न 69.
“कुछ त्रुटियों के होते हुए भी ‘आइन- ए-अकबरी’ अपने समय के लिए एक असाधारण एवं अनूठा दस्तावेज है।” व्याख्या कीजिए।
उत्तर:
‘आइन-ए-अकबरी’ अपने समय के लिए एक असाधारण एवं अनूठा दस्तावेज हैं। मुगल साम्राज्य के गठन और उसकी संरचना की अत्यन्त आकर्षक झलकियाँ दर्शाकर और उसके निवासियों व उत्पादों के सम्बन्ध में सांख्यिकीय आँकड़े प्रस्तुत कर, अबुल फजल मध्यकालीन इतिहासकारों से कहीं आगे निकल गए। ‘आइन’ में भारत के लोगों और मुगल साम्राज्य के बारे में विस्तृत सूचनाएँ दी गई हैं। इस प्रकार यह ग्रन्थ सत्रहवीं शताब्दी के भारत के अध्ययन के लिए संदर्भ-बिन्दु बन गया है।

JAC Class 12 History Important Questions Chapter 8 किसान, ज़मींदार और राज्य : कृषि समाज और मुगल साम्राज्य

प्रश्न 70.
बटाई प्रणाली तथा लाँग बढाई पद्धतियों | का वर्णन कीजिये।
उत्तर:
(1) कटाई प्रणाली में फसल काटकर जमा कर लेते थे और फिर सभी पक्षों की उपस्थिति तथा सहमति से बँटवारा कर लेते थे।
(2) लॉंग बटाई – इस प्रणाली में फसल काटने के बाद उसके ढेर बना लिए जाते थे तथा फिर उन्हें आपस में बाँट लेते थे। इसमें हर पक्ष अपना हिस्सा घर ले जाता था।

प्रश्न 71.
मुगल सम्राट शिकार अभियानों का क्यों आयोजन करते थे?
उत्तर:
मुगल सम्राट गरीबों और अमीरों सहित सबके साथ न्याय करना अपना प्रमुख कर्त्तव्य मानते थे। इसलिए ये शिकार अभियानों का आयोजन करते थे। वे शिकार आयोजन के नाम पर अपने विशाल साम्राज्य के कोने-कोने का दौरा करते थे और विभिन्न क्षेत्रों के लोगों की समस्याओं और शिकायतों को सुनकर उनका निराकरण करते थे।

प्रश्न 72.
कृषि के अतिरिक्त महिलाएँ दस्तकारी के किन कार्यों में संलग्न रहती थीं?
उत्तर:
कृषि के अतिरिक्त महिलाएं सूत कातने वर्तन बनाने के लिए मिट्टी को साफ करने और गूंधने, कपड़ों पर कढ़ाई जैसे दस्तकारी के कार्यों में संलग्न रहती थीं किसी वस्तु का जितना वाणिज्यीकरण होता था, उसके उत्पादन के लिए महिलाओं के श्रम की उतनी ही माँग होती थी। इस प्रकार महिलाएँ न केवल खेतों में कार्य करतीं थीं, बल्कि नियोक्ताओं के घरों पर भी जाती थीं।

प्रश्न 73.
मुगलकाल में कृषि में प्रयोग होने वाली मुख्य तकनीकी कौनसी थी? संक्षेप में लिखिए।
उत्तर:
मुगलकाल में कृषि अपनी उच्च अवस्था में थी क्योंकि उस समय अनेक तकनीकों का प्रयोग किया जाता था। वस्तुत: उस समय प्रमुख कृषि तकनीकी पशु बल पर आधारित थी। इसे हम निम्नलिखित बिन्दुओं के माध्यम से समझ सकते हैं-
(1) लकड़ी का हल एक प्रमुख कृषि उपकरण था। इसे एक छोर पर लोहे की नुकीली धार अथवा फाल लगाकर बनाया जाता था ऐसे हल मिट्टी को अधिक गहरा नहीं खोदते थे। जिसके कारण तेज गर्मी के महीनों में मिट्टी में पर्याप्त नमी बनी रहती थी जो उपज अथवा फसल के लिए अत्यधिक उपयोगी होती थी
(2) मिट्टी की निराई तथा गुड़ाई के लिए लकड़ी के मूठ वाले लोहे के पतले धार वाले उपकरण प्रयोग होते थे।
(3) बैलों के जोड़े के सहारे खींचे जाने वाले बरमे का प्रयोग बीज बोने के लिए किया जाता था, किन्तु बीजों को अधिकतर हाथ से छिड़ककर ही बोया जाता था
(4) रहट, जो उस समय सिंचाई का मुख्य साधन था, में भी बैलों का प्रयोग किया जाता था।

प्रश्न 74.
फ्रांसीसी यात्री ज्यां बैप्टिस्ट तैवर्नियर ने गाँवों में मुद्रा के प्रचलन के सम्बन्ध में अपने यात्रा वृत्तान्त में क्या वर्णन किया है?
उत्तर:
सत्रहवीं सदी में भारत में आए फ्रांसीसी यात्री ज्यां बैप्टिस्ट तैवर्नियर ने भारत के गाँवों में मुद्रा के प्रचलन के सम्बन्ध में अपने यात्रा वृत्तान्त में उल्लेख किया है कि “भारत के गाँव बहुत ही छोटे कहे जाएँगे, जिसमें मुद्रा के फेर-बदल करने वालों को ‘सर्राफ’ कहा जाता है। सर्राफ का कार्य बैंकरों की भाँति होता था, एक बैंक की भाँति सर्राफ हवाला भुगतान करते हैं और अपनी मर्जी के अनुसार पैसे के मुकाबले रुपयों की कीमत और कौड़ियों के मुकाबले पैसे की कीमत बढ़ा देते हैं।”

प्रश्न 75
ग्राम पंचायतों के कार्यों का उल्लेख कीजिये।
उत्तर:
(1) जाति की अवहेलना को रोकने के लिए लोगों के आचरण पर नजर रखना। (2) दोषियों पर जुर्माना लगाना और समुदाय से निष्कासित करना। इसके अन्तर्गत दण्डित व्यक्ति को निर्धारित समय के लिए गाँव छोड़ना पड़ता था। इस दौरान वह अपनी जाति और व्यवसाय से बचित हो जाता था।

JAC Class 12 History Important Questions Chapter 8 किसान, ज़मींदार और राज्य : कृषि समाज और मुगल साम्राज्य

प्रश्न 76.
मध्यकालीन भारत में सिंचाई के कौनसे साधन प्रचलित थे?
उत्तर:
मानसून भारतीय कृषि का रीढ़ था। खेती कृषि पर निर्भर करती थी परन्तु कुछ ऐसी फसलें भी थीं, जिनके लिए अतिरिक्त जल की आवश्यकता थी। अतः कुआँ, नहरों, नदियों, जलाशयों आदि से भी खेती की सिंचाई की जाती थी। बाबर के अनुसार रहट या बाल्टियों से सिंचाई की जाती थी।

निबन्धात्मक प्रश्न

प्रश्न 1.
आप कृषि का इतिहास लिखने के लिए ‘आइन’ का स्रोत के रूप में किस प्रकार उपयोग करेंगे? समझाइये
अथवा
कृषि-इतिहास के स्रोत के रूप में ‘आइन-ए- अकबरी’ का वर्णन कीजिये।
अथवा
सोलहवीं तथा सत्रहवीं शताब्दी के कृषि इतिहास की जानकारी के मुख्य स्रोतों का विवेचन कीजिए।
उत्तर:
सोलहवीं तथा सत्रहवीं शताब्दी के कृषि – इतिहास की जानकारी के मुख्य स्रोत सोलहवीं तथा सत्रहवीं शताब्दी के कृषि इतिहास की जानकारी के मुख्य स्रोतों का विवेचन निम्नलखित बिन्दुओं के अन्तर्गत किया जा सकता है –
(1) मुगल दरबार की निगरानी में लिखे गए ऐतिहासिक ग्रन्थ व दस्तावेज-किसान अपने बारे में स्वयं नहीं लिखा करते थे इसलिए खेतों में काम करने वाले ग्रामीण समाज के क्रिया-कलापों की जानकारी हमें उन लोगों से प्राप्त नहीं होती। परिणामस्वरूप सोलहवीं तथा सत्रहवीं शताब्दियों के कृषि इतिहास की जानकारी देने वाले हमारे मुख्य स्रोत वे ऐतिहासिक ग्रन्थ एवं दस्तावेज हैं, जो मुगल दरबार की निगरानी में लिखे गए थे।

(2) ‘आइन-ए-अकबरी’- सोलहवीं या सत्रहवीं शताब्दियों का कृषि इतिहास लिखने के लिए, ‘आइन- ए-अकबरी’ एक अत्यन्त महत्त्वपूर्ण ग्रन्थ है ‘ आइन-ए- अकबरी’ की रचना मुगल सम्राट अकबर के दरबारी इतिहासकार अबुल फसल ने की थी। ‘आइन-ए-अकबरी’ का मुख्य उद्देश्य अकबर के साम्राज्य की एक ऐसी रूप- रेखा प्रस्तुत करना था, जहाँ एक शक्तिशाली सत्ताधारी वर्ग सामाजिक मेल-जोल बना कर रखता था। ‘आइन’ के तीसरे भाग ‘मुल्क आबादी में उत्तर भारत के कृषि-समाज के बारे में विस्तृत विवरण प्रस्तुत किया गया है। इसमें साम्राज्य व प्रान्तों के वित्तीय पहलुओं तथा | राजस्व की दरों के आंकड़ों की विस्तृत जानकारी दी गई है। इसमें प्रत्येक प्रान्त तथा उसकी अलग-अलग इकाइयों की कुल मापी गई जमीन और निर्धारित राजस्व का विवरण भी प्रस्तुत किया गया है।

(3) अन्य स्रोत- सोलहवीं तथा सत्रहवीं शताब्दियों के कृषि इतिहास की जानकारी के लिए हम निम्नलिखित स्रोतों का भी प्रयोग कर सकते हैं-
(i) मुगलों की राजधानी से दूर के प्रदेशों में लिखे गए ग्रन्थ व दस्तावेज’ आइन’ की जानकारी के साथ- साथ हम उन लोगों का भी प्रयोग कर सकते हैं, जो मुगलों की राजधानी से दूर के प्रदेशों में लिखे गए थे। इनमें सत्रहवीं व अठारहवीं सदियों के गुजरात, महाराष्ट्र और राजस्थान से उपलब्ध होने वाले वे दस्तावेज सम्मिलित हैं. जो सरकार की आय की विस्तृत जानकारी देते हैं।

(ii) ईस्ट इण्डिया कम्पनी के दस्तावेज इसके अतिरिक्त ईस्ट इण्डिया कम्पनी के बहुत से दस्तावेज भी हैं जो पूर्वी भारत में कृषि सम्बन्धों की उपयोगी जानकारी देते हैं। इन सभी स्रोतों में किसानों, जमींदारों तथा राज्य के बीच होने वाले झगड़ों के विवरण मिलते हैं। इन स्रोतों से हमें यह समझने में सहायता मिलती है कि किसानों का राज्य के प्रति क्या दृष्टिकोण था तथा राज्य से उन्हें कैसे न्याय की आशा थी।

प्रश्न 2.
खुद काश्त व पाहि काश्त से आप क्या समझते हैं? मुगलकाल में किसानों की दशा का वर्णन कीजिए।
उत्तर:
किसानों को मुगलकाल में रैयत या मुजरिवान कहा जाता था। किसानों के लिए सत्रहवीं सदी के भारतीय- फारसी स्रोत में इसी शब्द का प्रयोग किया गया है। किसानों के लिए आसामी शब्द का प्रयोग भी किया जाता था। यह शब्द गाँवों में अब भी प्रचलित है। सत्रहवीं शताब्दी के ग्रामीण समाज से सम्बन्धित स्रोतों में दो प्रकार के किसानों का उल्लेख किया गया है; जो इस प्रकार है – खुद काश्त किसन जो उन्हीं गाँवों में निकास करते थे जिनमें उनकी जमीन होती थी अपनी जमीन पर खेती करने वाले किसानों को खुद काश्त किसान कहा जाता था। पाहि काश्त पाहि काश्त उन किसानों को कहा जाता था जो दूसरे गाँवों से आकर ठेके पर खेती लेकर कृषि कार्य करते थे। कभी-कभी लोग दूसरे गाँवों में खेती की अच्छी सम्भावनाओं, जैसेकि ठेके आदि की कम दरें और अच्छी उपज के लालच में पाहि काश्तकारी करते थे।

आर्थिक परेशानी, अकाल या भुखमरी तथा खुद की जमीन न होने पर भी लोग पाहि काश्तकारी करते थे। किसानों के पास उपलब्ध कृषि भूमि किसानों के पास उपलब्ध कृषि भूमि उनकी समृद्धि का मापदण्ड थी। सामान्य तौर पर उत्तर भारत के एक औसत किसान के पास अधिक भूमि न होने के कारण एक जोड़ी बैल और दो हल से अधिक कुछ नहीं होता था। अधिकांश किसानों के पास इससे भी न्यून साधन होते थे किसानों के पास उपलब्ध साधनों के आधार पर हम कह सकते हैं कि भूमि की उपलब्धता कम थी। गुजरात में 6 एकड़ जमीन का स्वामित्व रखने वाले किसान समृद्ध कहे जाते थे। बंगाल में औसत किसान के पास अधिक से अधिक 5 एकड़ जमीन होती थी।

10 एकड़ जमीन रखने वाले किसान अमीर समझे जाते थे। कृषि भूमि की मिल्कियत जिस प्रकार अन्य सम्पत्तियाँ खरीदी और बेची जाती थीं, उसी प्रकार कृषि भूमि भी व्यक्तिगत मिल्कियत के सिद्धान्त पर आधारित थी। व्यक्तिगत मिल्कियत होने के नाते किसान कृषि भूमि तथा क्रय-विक्रय अन्य सम्पत्तियों की भाँति कर सकते हैं। किसानों की मिल्कियत के बारे में एक विवरण उन्नीसवीं सदी के दिल्ली-आगरा प्रदेश के किसानों के सम्बन्ध में प्राप्त होता है। उपर्युक्त विवरण सत्रहवीं सदी के किसानों पर भी इसी भाँति लागू होता है। किसानों की दशा तत्कालीन समय के इतिहासकारों में किसानों की सामाजिक व आर्थिक स्थिति के बारे में मतभेद है।

JAC Class 12 History Important Questions Chapter 8 किसान, ज़मींदार और राज्य : कृषि समाज और मुगल साम्राज्य

प्रश्न 3.
मुगलकाल में प्रचलित सिंचाई साधनों की वर्तमान कृषि में उपयोगिता को समझाइये।
अथवा
‘बाबरनामा’ में वर्णित भारत में सिंचाई के साधनों एवं उपकरणों का विवेचन कीजिए।
उत्तर:
(1) मुगलकाल में प्रचलित सिंचाई साधनों की वर्तमान कृषि में उपयोगिता आज की भाँति मानसून मुगलकाल में भी भारतीय कृषि को रोड़ थी परन्तु कुछ ऐसी फसलें भी थीं जिनके लिए अतिरिक्त पानी की आवश्यकता थी। अतः मुगल काल में वर्षा के अतिरिक्त सिंचाई तालाबों, कुओं, नहरों एवं बांधों के द्वारा होती थी। मुगल काल में प्रचलित सिंचाई साधनों की वर्तमान कृषि में काफी उपयोगिता है। आज भी देश की अधिकांश आबादी गांवों में रहती है। यद्यपि आज धनी लोगों द्वारा ट्यूबवैल आदि आधुनिक साधनों का उपयोग किया जाता है, परन्तु अधिकांशतः खेतों की सिंचाई वर्षा, कुओं, तालाबों, नहरों, बांधों के द्वारा ही की जाती है।

(2) ‘बाबरनामा में वर्णित भारत में सिंचाई के साधन-
(i) बहते पानी के प्रबन्ध का अभाव- भारत का -अधिकतर भाग मैदानी जमीन पर बसा हुआ है। यद्यपि यहाँ शहर और खेती योग्य जमीन की प्रचुरता थी, परन्तु कहीं भी बहते पानी का प्रबन्ध नहीं किया जाता था। इसलिए फसलें उगाने या बागानों के लिए पानी की बिल्कुल आवश्यकता नहीं थी। शरद ऋतु की फसलें वर्षा के पानी से ही पैदा हो जाती थीं यह बड़े आश्चर्य की बात है कि बसन्त ऋतु की फसलें तो वर्षा के अभाव में भी पैदा हो जती थीं।

(ii) रहट द्वारा सिंचाई ‘बाबरनामा’ के अनुसार छोटे पेड़ों तक रहट या बाल्टियों के द्वारा पानी पहुंचाया जाता था लाहौर, दीपालपुर (वर्तमान में पाकिस्तान में) और ऐसे दूसरे स्थानों पर लोग रहट के द्वारा सिंचाई करते थे लोग रस्सी के दो गोलाकार फंदे बनाते थे, जो कुएँ की गहराई के अनुसार लम्बे होते थे। इन फन्दों में थोड़ी-थोड़ी दूरी पर लकड़ी के गुटके लगाए जाते थे और इन गुटकों पर पड़े बांध दिए जाते थे।

लकड़ी के गुटकों और घड़ों से बंधी इन रस्सियों को कुएँ के ऊपर पहियों से लटकाया जाता था। पहिये की धुरी पर एक ओर पहिया होता था। इस अन्तिम पहिए को बैल के द्वारा घुमाया जाता था इस पहिये के दाँत पास के दूसरे पहिए के दांतों को जकड़ लेते थे और इस प्रकार घड़ों वाला पहिया घूमने लगता था। घड़ों से जहाँ पानी गिरता था, वहाँ एक संकरा नाला खोद दिया जाता था और इस विधि से प्रत्येक स्थान पर पानी पहुँचाया जाता था।

(iii) बाल्टियों से सिंचाई ‘बाबरनामा’ के अनुसार वर्तमान उत्तर प्रदेश में स्थित आगरा, चांदवर तथा बवाना में और ऐसे अन्य प्रदेशों में भी लोग बाग बाल्टियों से सिंचाई करते थे। वे कुएँ के किनारे पर लकड़ी के कन्ने गाड़ देते थे। वे इन फन्नों के मध्य बेलन टिकाते थे, एक बड़ी बाल्टी में रस्सी बांधते थे रस्सी को बेलन पर लपेटते थे और इसके दूसरे छोर को बैल से बाँध देते थे। एक व्यक्ति बैल हाँकता था तथा दूसरा बाल्टी से पानी निकालता था।

प्रश्न 4.
भारत में सोलहवीं तथा सत्रहवीं शताब्दियों में ग्रामीण दस्तकारों की स्थिति पर प्रकाश डालिए। उत्तर भारत में सोलहवीं तथा सत्रहवीं शताब्दियों में ग्रामीण दस्तकारों की स्थिति –
(1) गाँवों में बड़ी संख्या में दस्तकारों की उपलब्धता-अंग्रेजी शासन के प्रारम्भिक वर्षों में किए गए गाँवों के सर्वेक्षण तथा मराठाओं के दस्तावेजों से ज्ञात होता है कि गाँवों में दस्तकार काफी बड़ी संख्या में रहते थे। कुछ गाँवों में तो कुल घरों के 25 प्रतिशत घर दस्तकारों के थे।

(2) किसानों और दस्तकारों के बीच सम्बन्ध- कभी-कभी किसानों और दस्तकारों के बीच अन्तर करना कठिन होता था क्योंकि कई ऐसे समूह से जो दोनों प्रकार के काम करते थे। खेतिहर और उसके परिवार के सदस्य अनेक प्रकार की वस्तुओं के उत्पादन में भाग लेते थे। उदाहरणार्थ, वे रंगरेजी, कपड़ों पर छपाई, मिट्टी के बर्तनों को पकाने, खेती के औजार बनाने या उनकी मरम्मत का काम करते थे। बुआई और सुहाई के बीच या सुहाई और के बीच की अवधि में किसानों के पास खेती का काम नहीं होता था, इसलिए इस अवधि में ये खेतिहर दस्तकारी का काम करते थे।

JAC Class 12 History Important Questions Chapter 8 किसान, ज़मींदार और राज्य : कृषि समाज और मुगल साम्राज्य

(3) ग्रामीण दस्तकारों द्वारा गाँव के लोगों को सेवाएँ देना कुम्हार, लोहार, बढ़ई, नाई और सुनार जैसे ग्रामीण दस्तकार अपनी सेवाएं गाँव के लोगों को देते थे। इसके बदले गाँव के लोग उन्हें अलग-अलग तरीकों से उन सेवाओं की अदायगी करते थे। प्रायः वे या तो उन्हें फसल का एक भाग दे दिया करते थे या फिर गाँव की जमीन का एक टुकड़ा यह जमीन का टुकड़ा ऐसा था जो खेती योग्य होने के बावजूद बेकार पड़ा रहता था भुगतान का तरीका सम्भवतः पंचायत ही निश्चित करती थी। महाराष्ट्र में ऐसी जमीन दस्तकारों की ‘मीरास’ या ‘वतन’ बन गई जिस पर दस्तकारों का पैतृक अधिकार होता था।

यही व्यवस्था कभी-कभी थोड़े परिवर्तित रूप में भी पाई जाती थी। इसमें दस्तकार और प्रत्येक खेतिहर परिवार आपसी बातचीत द्वारा सेवाओं के भुगतान की कोई एक व्यवस्था तय कर लेते थे। इस स्थिति में प्रायः वस्तुओं और सेवाओं का विनिमय होता था उदाहरणार्थ, अठारहवीं सदी के स्त्रोतों से ज्ञात होता है कि बंगाल में जमींदार उनकी सेवाओं के बदले लोहारों, बढ़ई और सुनारों तक को “दैनिक भत्ता और खाने के लिए नकदी देते थे। इस व्यवस्था को ‘जजमानी’ कहते थे। इन तथ्यों से पता चलता है कि गाँव के छोटे स्तर पर विनिमय के सम्बन्ध काफी जटिल थे। परन्तु ऐसा नहीं है कि नकद अदायगी का प्रचलन बिल्कुल ही नहीं था।

प्रश्न 5.
भारत में सोलहवीं तथा सत्रहवीं शताब्दियों में कृषि समाज में महिलाओं की भूमिका का विवेचन कीजिए। समाज में उनकी स्थिति पर प्रकाश डालिए । उत्तर- भारत में सोलहवीं तथा सत्रहवीं शताब्दियों में कृषि समाज में महिलाओं की भूमिका-
(1) उत्पादन की प्रक्रिया में पुरुष और महिलाओं की भूमिका उत्पादन की प्रक्रिया में पुरुष और महिलाएँ विशेष प्रकार की भूमिकाएं निभाते हैं। भारत में सोलहवीं तथा सत्रहवीं शताब्दियों में भी महिलाएँ और पुरुष कन्धे से कंधा मिलाकर खेतों में काम करते थे।

पुरुष हल जोतते थे तथा हल चलाते थे और महिलाएँ बुआई, निराई और कटाई के साथ-साथ पकी हुई फसल से दाना निकालने का काम करती थीं। फिर भी महिलाओं की जैव वैज्ञानिक क्रियाओं को लेकर लोगों के मन में पूर्वाग्रह बने रहे। उदाहरण के लिए, पश्चिमी भारत में मासिक धर्म वाली महिलाओं को हल या कुम्हार का चाक छूने की अनुमति नहीं थी। इसी प्रकार बंगाल में अपने मासिक धर्म के समय महिलाएँ पान के बागान में प्रवेश नहीं कर सकती थीं।

(2) दस्तकारी के कामों में महिलाओं का योगदान- सूत कातने वर्तन बनाने के लिए मिट्टी को साफ करने और गूंथने, कपड़ों पर कढ़ाई जैसे दस्तकारी के कार्य महिलाओं के श्रम पर निर्भर थे।

(3) महत्त्वपूर्ण संसाधन के रूप में महिलाएं श्रम प्रधान समाज में बच्चे उत्पन्न करने की अपनी योग्यता के कारण महिलाओं को महत्त्वपूर्ण संसाधन के रूप में देखा
जाता था।

समाज में महिलाओं की स्थिति-
(1) विवाहित महिलाओं की कमी तत्कालीन युग में विवाहित महिलाओं की कमी थी क्योंकि कुपोषण, बार-बार माँ बनने और प्रसव के समय मृत्यु हो जाने के कारण महिलाओं में मृत्यु दर बहुत अधिक थी इससे किसान और दस्तकार समाज में ऐसे सामाजिक रीति- रिवाज उत्पन्न हुए, जो अभिजात वर्ग के लोगों से बहुत अलग थे। कई ग्रामीण सम्प्रदायों में विवाह के लिए ‘दुल्हन की कीमत अदा करनी पड़ती थी, न कि दहेज की। तलाकशुदा महिलाएँ और विधवाएँ दोनों ही कानूनी रूप से विवाह कर सकती थीं।

(2) महिलाओं पर नियन्त्रण रखना प्रचलित रिवाज के अनुसार घर का मुखिया पुरुष होता था। इस प्रकार महिला पर परिवार और समुदाय के पुरुषों का पूर्ण नियन्त्रण बना रहता था दूसरे पुरुषों के साथ अवैध सम्बन्ध रखने के शक पर ही महिलाओं को भयानक दंड दिए जाते थे।

(3) महिलाओं द्वारा अपने पतियों की बेवफाई का विरोध-राजस्थान, गुजरात और महाराष्ट्र आदि पश्चिमी भारत के प्रदेशों से प्राप्त दस्तावेजों में ऐसे प्रार्थना-पत्र मिले हैं, जो महिलाओं ने न्याय और मुआवजे की आशा से ग्राम पंचायतों को भेजे थे। इनमें पत्नियाँ अपने पतियों की बेवफाई का विरोध करती हुई दिखाई देती हैं या फिर गृहस्थी के पुरुष पर पत्नी और बच्चों की अवहेलना करने का आरोप लगाती हुई नजर आती हैं।

JAC Class 12 History Important Questions Chapter 8 किसान, ज़मींदार और राज्य : कृषि समाज और मुगल साम्राज्य

(4) महिलाओं को पैतृक सम्पत्ति का अधिकार- भूमिहर भद्रजनों में महिलाओं को पैतृक सम्पत्ति का अधिकार प्राप्त था।

प्रश्न 6.
मुगलकाल में सिंचाई, कृषि तकनीक व फसलों का वर्णन कीजिए।
उत्तर:
सिंचाई- भारत के किसानों की पारिश्रमिक प्रवृत्ति और कृषि से प्राप्त उनकी गतिशीलता, कृषि योग्य भूमि व श्रम की उपलब्धता के कारण कृषि का निरन्तर विस्तार हुआ खाद्यान्न फसलों, जैसे गेहूं, चावल, ज्वार आदि की खेती प्रचुरता के साथ की जाती थी। कृषि मुख्य रूप से मानसून पर आधारित थी: अधिक वर्षा वाले इलाकों में धान की खेती एवं कम वर्षा वाले इलाकों में गेहूं व बाजरे आदि की खेती की जाती थी कुछ फसलें ऐसी थीं, जिनके लिए अधिक पानी की आवश्यकता पड़ती थी।

ऐसी फसलों के लिए सिंचाई के कृत्रिम साधन विकसित करने पड़े। सिंचाई के कृत्रिम उपायों के विकास में युगल शासकों ने व्यापक रुचि ली। किसानों को व्यक्तिगत रूप से कुएँ तालाब, बाँध आदि बनाने के लिए राज्य से सहायता दी गई। कई-कई नहरों व नालों का निर्माण राज्य द्वारा करवाया गया।

शाहजहाँ के शासन काल में पुरानी नहरों की मरम्मत करवाई गई, जैसेकि पंजाब की शाह नहर कृषि तकनीकें खेती में मानवीय श्रम को कम करने के लिए किसानों ने पशुबल पर आधारित तकनीकों में नए सुधार किए। पहले जुताई के लिए लकड़ी के फाल वाले हल्के हल का प्रयोग किया जाता था।

लकड़ी के फाल वाले हल जमीन की गहरी जुताई कर सकने में सक्षम नहीं थे इसलिए किसानों ने नुकीली धार वाले लोहे के फाल का प्रयोग करना प्रारम्भ कर दिया लोहे की फाल लगा हल आसानी से गहरी जुताई कर सकता था और हल खींचने वाले पशुओं पर जोर भी कम पड़ता था। मिट्टी की गुड़ाई और निराई के लिए लोहे के पतले धार वाले यन्त्र का प्रयोग किया जाने लगा। बीजों की बुआई के लिए बरमे का प्रयोग किया जाता था, जो बैलों द्वारा खींचा जाता था। हाथ से छिड़क कर भी बीज बोए जाते थे।

भरपूर फसलें मौसम के चक्रों पर आधारित स्त्री और खरीफ की फसलें मुख्य थीं खरीफ की फसल पतझड़ में और रबी की फसल वसंत में ली जाती थी। जहाँ पर सिंचाई की पर्याप्त व्यवस्था होती थी, उन क्षेत्रों में तीन फसलें ली जाती थीं। मुख्यतया खाद्यान्न फसलों की खेती पर अधिक जोर दिया जाता था परन्तु कुछ नकदी फसलें भी उगाई जाती थी, जिन्हें ‘जिन्स-ए-कामिल’ म्यानि सर्वोत्तम फसलें कहा जाता था, जैसे- गन्ना, कपास, सरसों, नील आदि ।

प्रश्न 7.
आइन-ए-अकबरी’ मुगलकालीन
शासन व्यवस्था की जानकारी का एक महत्त्वपूर्ण स्त्रोत है।” विवेचना कीजिए।
अथवा
‘आइन-ए-अकबरी’ पर एक संक्षिप्त निबन्ध लिखिए।
अथवा
‘आइन-ए-अकबरी’ के ऐतिहासिक महत्व की विवेचना कीजिए।
उत्तर:
अबुल फजल की ‘आइन-ए-अकबरी’
(1) आइन-ए-अकबरी’ का संक्षिप्त परिचय- मुगल सम्राट अकबर के आदेश पर उसके दरबारी इतिहासकार अबुल फजल ने ‘आइन-ए-अकबरी’ की रचना की थी। 1598 ई. में पाँच संशोधनों के बाद, इस ग्रन्थ को पूरा किया गया। ‘आइन’ इतिहास लेखन की एक ऐसी विशाल परियोजना का भाग थी, जिसकी पहल अकबर ने की थी। ‘आइन” अक्रबरनामा’ की तीसरी जिल्द थी।

(2) ‘आइन’ की विषय-वस्तु’ आइन’ में निम्नलिखित विषयों पर विस्तार से चर्चा की गई है-

  • दरबार, प्रशासन और सेना का संगठन
  • राजस्व के स्रोत और अकबर के साम्राज्य के प्रान्तों का भूगोल ।
  • लोगों के साहित्यिक, सांस्कृतिक एवं धार्मिक रिवाज।
  • अकबर की सरकार के समस्त विभागों एवं प्रान्तों (सूर्वो) के बारे में विस्तार से जानकारी देने के अतिरिक्त, ‘आइन’ इन सूबों के बारे में जटिल और आँकड़ेबद्ध सूचनाएँ बड़ी बारीकी से देती है।

(3) महत्त्व इन सूचनाओं को इकट्ठा करके, उन्हें ‘क्रमबद्ध तरीके से संकलित करना एक महत्त्वपूर्ण शाही प्रक्रिया थी। इसने सम्राट को साम्राज्य के समस्त प्रदेशों में प्रचलित रिवाजें और व्यवसायों की जानकारी दी। इस प्रकार हमारे लिए’ आइन’ अकबर के शासनकाल से सम्बन्धित मुगल साम्राज्य के बारे में सूचनाओं की खान है।

(4) आइन’ के दफ्तर (भाग) आइन पांच भागों (दफ्तरों) का संकलन है। इसके पहले तीन भाग प्रशासन का विवरण देते हैं।
(i) मंजिल आबादी ‘मंजिल आबादी’ के नाम से पहला ग्रन्थ शाही घर-परिवार और उसके रख-रखाव से सम्बन्ध रखता है।

(ii) सिपह- आबादी- दूसरा भाग ‘सिपह आबादी’ सैनिक व नागरिक प्रशासन और नौकरों की व्यवस्था के बारे में है। इस भाग में शाही अधिकारियों (मनसबदारों), विद्वानों, कवियों और कलाकारों की संक्षिप्त जीवनियाँ सम्मिलित हैं।

(iii) मुल्क आबादी- तीसरे भाग ‘मुल्क- आबादी’ में साम्राज्य व प्रान्तों के वित्तीय पहलुओं तथा राजस्व की दरों के आँकड़ों की विस्तृत जानकारी दी गई है। इसके बाद ‘बारह प्रान्तों का वर्णन’ दिया गया है। इस खण्ड में सांख्यिकी सूचनाएँ विस्तार से दी गई हैं। इसमें सूबों (प्रान्तों) और उनकी समस्त प्रशासनिक तथा वित्तीय इकाइयों (सरकार, परगना और महल) के भौगोलिक, स्थलाकृतिक और आर्थिक रेखाचित्र भी सम्मिलित हैं। इसी खण्ड में प्रत्येक प्रान्त और उसकी अलग-अलग इकाइयों की कुल मापी गई जमीन और निर्धारित राजस्व (जमा) की जानकारी भी दी गई है।

JAC Class 12 History Important Questions Chapter 8 किसान, ज़मींदार और राज्य : कृषि समाज और मुगल साम्राज्य

‘सरकार’ सम्बन्धी जानकारी-
सूबा स्तर की विस्तृत जानकारी देने के बाद, ‘आइन’ हमें सूबों के नीचे की इकाई ‘सरकारों’ के बारे में विस्तृत जानकारी देती है। ये सूचनाएँ तालिकाबद्ध तरीके से दी गई हैं। हर तालिका में आठ खाने हैं जो हमें निम्नलिखित सूचनाएं देते हैं-

परगनात / महल
(ii) किला
(iii) अराजी और जमीन-ए- पाईंमूद (मापे गए इलाके)
(iv) नकदी (नकद निर्धारित राजस्व)
(v) सुयूगल (दान में दिया गया राजस्व अनुदान)
(vi) जमींदार
सातवें तथा आठवें खानों में जमींदारों की जातियों और उनके घुड़सवार, पैदल सैनिक (प्यादा) व हाथी (फ्री) सहित उनकी सेना की जानकारी दी गई है। ‘मुल्क- आबादी’ नामक खण्ड उत्तर भारत के कृषि-समाज का विस्तृत, आकर्षक और जटिल चित्र प्रस्तुत करता है।
(iv) और
(v) भाग-ये भाग भारतवासियों के धार्मिक, साहित्यिक तथा सांस्कृतिक रीति-रिवाजों से सम्बन्ध रखते हैं।

प्रश्न 8.
आइन-ए-अकबरी के महत्त्व एवं सीमाओं की विवेचना कीजिए।
अथवा
महत्व की विवेचना ‘आइन-ए-अकबरी’ के कीजिए और इसकी त्रुटियों पर प्रकाश डालिए।
उत्तर:
‘आइन-ए-अकबरी’ का महत्त्व एवं त्रुटियाँ (सीमाएँ ) यद्यपि ‘आइन-ए-अकबरी’ को मुगल सम्राट अकबर के शासन की सुविधा के लिए आधिकारिक तौर पर विस्तृत सूचनाएँ संकलित करने के लिए प्रायोजित किया गया था, मा फिर भी यह ग्रन्थ आधिकारिक दस्तावेजों की नकल नहीं है। अबुलफजल की ग्रन्थ लेखन में बड़ी रुचि थी इसी कारण उसने इसकी पांडुलिपि को पाँच बार संशोधित किया था।

त्रुटियाँ इतिहासकारों के अनुसार ‘आइन’ की प्रमुख त्रुटियाँ (सीमाएँ ) निम्नलिखित हैं-
(1) जोड़ करने में गलतियाँ जोड़ करने में कई गलतियाँ पाई गई हैं। ऐसा माना जाता है कि या तो ये अंकगणित की छोटी-मोटी चूक है अथवा फिर नकल उतारने के दौरान अबुल फजल के सहयोगियों की भूल।
(2) संख्यात्मक आँकड़ों में विषमताएँ आइन’ के संख्यात्मक आँकड़ों में विषमताएँ हैं सभी सूबों (प्रान्तों) से आँकड़े एक ही रूप में एकत्रित नहीं किए गए हैं।
(3) कीमतों और मजदूरी से सम्बन्धित आँकड़ों का विस्तार से वर्णन नहीं करना इसी प्रकार जहाँ सूर्यो से लिए गए राजकोषीय आँकड़े बड़े विस्तार से दिए गए हैं, वहीं उन्हीं प्रदेशों से कीमतों और मजदूरी जैसे इतने ही महत्वपूर्ण आँकड़े इतने विस्तार से साथ नहीं दिए गए हैं।
(4) मूल्यों और मजदूरों की दरों की अपर्याप्त सूची- मूल्यों और मजदूरी की दरों की जो विस्तृत सूची ‘आइन’ में दी भी गई है, वह मुगल साम्राज्य की राजधानी आगरा या उसके आस-पास के प्रदेशों से ली गई है।

निष्कर्ष –
(1) असामान्य एवं अनूठा ग्रन्थ- कुछ त्रुटियों के होते हुए भी, ‘आइन’ अपने समय के लिए एक असामान्य एवं अनूठा दस्तावेज है। मुगल साम्राज्य के गठन और उसकी संरचना की अत्यन्त आकर्षक झलकियाँ दर्शा कर और उसके निवासियों व उत्पादों के सम्बन्ध में सांख्यिकीय आँकड़े देकर, अबुल फसल मध्यकालीन इतिहासकारों से कहीं आगे निकल गए।

(2) भारत के लोगों और उनके व्यवसायों के बारे में विस्तृत जानकारी देना मध्यकालीन भारत में अबुल फसल से पहले के इतिहासकार अधिकतर राजनीतिक घटनाओं के बारे में ही लिखते थे युद्ध-विजय, राजनीतिक प या वंशीय उथल-पुथल से सम्बन्धित विवरण ग्रन्थों में रहते थे। भारत के लोगों और मुगल साम्राज्य के बारे में विस्तृत सूचनाएँ देकर ‘आइन’ ने स्थापित परम्पराओं को पीछे छोड़ दिया और इस प्रकार सत्रहवीं सदी के भारत के अध्ययन के लिए एक संदर्भ बिन्दु बन गया। जहाँ तक कृषि सम्बन्धों का प्रश्न है, ‘आइन’ के सांख्यिकी प्रमाणों के महत्व को चुनौती नहीं दी जा सकती।

JAC Class 12 History Important Questions Chapter 8 किसान, ज़मींदार और राज्य : कृषि समाज और मुगल साम्राज्य

प्रश्न 9.
मुगलों की भू-राजस्व प्रणाली की विवेचना कीजिए।
अथवा
अकबर की भू-राजस्व व्यवस्था का विवेचन कीजिए।
उत्तर:
मुगलों अथवा अकबर की भू-राजस्व व्यवस्था मुगलों अथवा अकबर की भू-राजस्व व्यवस्था की प्रमुख विशेषताओं का विवेचन निम्नलिखित बिन्दुओं के अन्तर्गत किया जा सकता है
(1) भू-राजस्व – मुगल साम्राज्य की आय का प्रमुख साधन-भू-राजस्व मुगल साम्राज्य की आय का प्रमुख साधन था अतः कृषि उत्पादन पर नियंत्रण रखने के लिए सम्पूर्ण साम्राज्य में राजस्व आकलन तथा वसूली के लिए एक प्रशासनिक तन्त्र की स्थापना की गई।
(2) भू-राजस्व के सम्बन्ध में सूचनाएँ एकत्रित करना भू-राजस्व प्रबन्ध के अन्तर्गत दो चरण थे-
(1) कर निर्धारण तथा
(2) वास्तविक वसूली जमा निर्धारित रकम थी तथा हासिल वास्तविक वसूली गई रकम राजस्व निर्धारित करते समय राज्य अपना हिस्सा अधिक से अधिक रखने का प्रयास करता था, परन्तु कभी-कभी वास्तव में इतनी वसूली कर पाना सम्भव नहीं हो पाता था।
(3) भूमि की पैमाइश अकबर ने जुती हुई जमीन तथा खेती योग्य भूमि दोनों की पैमाइश करवाई।
(4) भूमि का वर्गीकरण- कृषि योग्य भूमि को निम्नलिखित चार वर्गों में बांटा गया-

  • पोलज पोलज भूमि सबसे उत्तम भूमि थी जिस पर प्रत्येक वर्ष खेती होती थी।
  • परीती इस भूमि पर भी वर्ष भर खेती होती थी, परन्तु उसे पुनः उर्वरा शक्ति प्रदान करने के लिए एक अथवा दो वर्ष के लिए खाली छोड़ दिया जाता था।
  • चचर ऐसी भूमि जिस पर तीन वर्ष या चार वर्ष खेती नहीं की जाती थी, चचर भूमि कहलाती थी।
  • बंजर इस भूमि पर पाँच अथवा इससे भी अधिक वर्षों तक खेती नहीं की जाती थी।

(5) भूमि कर निर्धारण पोलज तथा परौती भूमि को तीन श्रेणियों में बाँटा गया था
(1) अच्छी
(2) मध्यम
(3) खराब प्रत्येक भूमि की औसत पैदावार का पता लगाया जाता था और इसका तीसरा भाग औसत पैदावार माना जाता था। इस औसत पैदावार का एक तिहाई हिस्सा भूमि-कर के रूप में लिया जाता था।

(6) भूमि कर व्यवस्था की अन्य प्रणालियाँ- अकबर ने आदेश दिया कि भूमि कर केवल नकदी के रूप में नहीं, बल्कि फसलों के रूप में भी वसूल किया जाए। इसके लिए निम्नलिखित प्रणालियाँ अपनाई गई

  • कणकुत प्रणाली- इसके अनुसार खड़ी फसल के अनुमान के आधार पर सरकारी लगान निश्चित किया जात था और फसल कटने पर सरकार अपना भाग ले लेती थी।
  • बटाई (भाओली) प्रणाली- इसमें फसल काट कर जमा कर लेते थे और फिर सभी पक्षों की उपस्थिति में व सहमति से बंटवारा करते थे।
  • खेत बटाई- इसमें बीज बोने के बाद खेत बाँट लिए जाते थे।
  • लाँग बटाई- इसमें फसल काटने के बाद उसके ढेर बना लिए जाते थे और फिर उसे आपस में बाँट लेते थे।

(7) अकबर के बाद मुगलों की भू-राजस्व व्यवस्था-अकबर के बाद के मुगल सम्राटों के शासन- काल में भी भूमि की नपाई के प्रयास जारी रहे। 1665 में औरंगजेब ने अपने राजस्व अधिकारियों को आदेश दिया कि प्रत्येक गाँव में खेतिहरों की संख्या का वार्षिक हिसाब रखा जाए। परन्तु इसके बावजूद सभी प्रदेशों की नपाई सफलतापूर्वक नहीं हुई।

प्रश्न 10.
मुगलकाल में भू-राजस्व निर्धारण की कौनसी प्रणालियाँ प्रचलित थीं?
उत्तर:
मुगलकाल के भू-राजस्व निर्धारण की प्रणालियाँ अकबर ने राजस्व अधिकारियों को आदेश दिया था कि वे लगान नकदी में लेने की आदत न डालें, बल्कि फसल भी लेने के लिए तैयार रहें। अतः अकबर के शासनकाल में भूमि कर निर्धारण के लिए निम्नलिखित प्रणालियाँ अपनाई गई-
(1) कणकुत प्रणाली हिन्दी भाषा में ‘कण’ का अर्थ है-‘ अनाज’ और ‘कुल’ का अर्थ है-‘ अनुमान’ इसके अन्तर्गत फसल को तीन अलग-अलग पुलिन्दों में काटा जाता था

  • अच्छा
  • मध्यम
  • खराब इस प्रणाली के अनुसार खड़ी फसल के अनुमान के आधार पर सरकारी लगान निश्चित किया जाता था और फसल कटने पर सरकार अपना भाग लेती थी। प्रायः अनुमान से किया गया जमीन का आकलन भी पर्याप्त रूप से सही परिणाम देता था।

(2) बटाई या भाओली इस प्रणाली के अन्तर्गत फसल काटकर जमा कर लेते थे और फिर सभी पक्षों की उपस्थिति में व आपसी सहमति से बँटवारा कर लेते थे। इसमें फसल का कुछ भाग सरकार द्वारा से लिया जाता था। परन्तु इसमें कई बुद्धिमान निरीक्षकों की आवश्यकता पढ़ती थी वरना दुष्ट बुद्धि मक्कार और धोखेबाज लोग बेइमानी करते थे तथा किसानों को धोखा देते थे।

(3) खेत बढाई इसमें खेत में बीज बो दिए जाते थे और बीज बोने के बाद खेत बाँट लेते थे। जिस प्रकार की फसल बोई जाती थी, उसी के अनुसार लगान निर्धारित होता था।

(4) लॉंग बटाई इसमें फसल काटने के बाद उसके देर बना लिए जाते थे और फिर उन्हें आपस में बाँट लेते थे प्रत्येक पक्ष फसल का अपना भाग लेकर घर ले जाता था और उससे मुनाफा कमाता था।

JAC Class 12 History Important Questions Chapter 8 किसान, ज़मींदार और राज्य : कृषि समाज और मुगल साम्राज्य

प्रश्न 11.
भारत में सोलहवीं तथा सत्रहवीं सदी में चाँदी के प्रवाह का वर्णन कीजिए। भारत में चाँदी के प्रवाह के बारे में इटली के यात्री जोवान्नी कारेरी ने क्या वर्णन किया है?
उत्तर:
भारत में सोलहवीं तथा सत्रहवीं सदी में चाँदी का प्रवाह सोलहवीं तथा सत्रहवीं सदी में भारत में व्यापार की बहुत अधिक उन्नति हुई खोजी यात्राओं से और ‘नई दुनिया’ की खोज से यूरोप के साथ भारत के व्यापार में अत्यधिक विस्तार हुआ। इसके परिणामस्वरूप भारत के समुद्र पार व्यापार की महत्त्वपूर्ण उन्नति हुई तथा कई नई वस्तुओं का व्यापार भी शुरू हो गया। निरन्तर बढ़ते व्यापार के साथ भारत से निर्यात होने वाली वस्तुओं का भुगतान करने के लिए एशिया में भारी मात्रा में चाँदी का आगमन हुआ। इस चाँदी का एक बड़ा हिस्सा भारत पहुँचा। यह स्थिति भारत के लिए लाभप्रद थी क्योंकि यहाँ चाँदी के प्राकृतिक संसाधन नहीं थे। परिणामस्वरूप, सोलहवीं से अठारहवीं सदी के बीच भारत में धातु मुद्रा – विशेषकर चांदी के रुपयों की प्राप्ति में अच्छी स्थिरता बनी रही। इसके दो लाभ हुए-
(1) अर्थव्यवस्था में मुद्रा- संचार तथा सिक्कों की ढलाई में अभूतपूर्व विस्तार हुआ तथा –
(2) मुगल राज्य को नकदी कर वसूलने में आसानी हुई।

इटली के यात्री जोवान्नी कारेरी का वृत्तांत-
लगभग 1690 ई. में इटली का यात्री जोवान्नी कारेरी भारत से होकर गुजरा था वह यहाँ चाँदी की प्रचुरता देखकर आश्चर्यचकित हो गया। उसने अपने यात्रा वृत्तांत में लिखा कि मुगल साम्राज्य में चाँदी भारी मात्रा में समस्त विश्व में से होकर भारत पहुँचती थी। उसने लिखा है कि समस्त है संसार में विचरण करने वाला समस्त सोना-चाँदी अन्ततः भारत पहुँच जाता है। इसका बहुत बड़ा भाग अमेरिका से आता है और यूरोप के कई राज्यों से होते हुए, इसका कुछ भाग कई प्रकार की वस्तुओं के लिए तुर्की में जाता है और थोड़ा-सा भाग रेशम के लिए फारस पहुँचता है। तुर्की लोग कॉफी से अलग नहीं रह सकते और न ही फारस अरविया तथा तुर्की के लोग भारत की वस्तुओं के बिना रह सकते हैं।

चे मुद्रा की विशाल मात्रा मोचा भेजते हैं। इसी प्रकार वे ये वस्तुएँ बसरा भेजते हैं। बाद में यह समस्त सम्पत्ति जहाजों से भारत भेज दी जाती है भारतीय जहाजों के अतिरिक्त जो डच, अंग्रेजी और पुर्तगाली जहाज प्रतिवर्ष भारत की वस्तुएँ लेकर पेगू, स्याम, श्रीलंका, मालद्वीप, मोजाम्बीक और अन्य स्थानों पर ले जाते हैं, इन्हीं जहाजों को निश्चित तौर पर विपुल सोना-चाँदी इन देशों से लेकर भारत पहुँचना पड़ता है। वह सब कुछ, जो डच लोग जापान की खानों से प्राप्त करते हैं अन्ततः भारत चला जाता है। यहाँ से यूरोप को जाने वाली समस्त वस्तुएँ चाहे वे फ्रांस जाएँ या इंग्लैण्ड या पुर्तगाल, सभी नकद में खरीदी जाती हैं, जो नकद भारत में रह जाता है।

प्रश्न 12.
उन स्त्रोतों का उल्लेख कीजिए जिनका प्रयोग अबुल फजल ने अपने ग्रन्थ ‘आइन-ए-अकबरी’ को पूरा करने के लिए किया।
उत्तर:
अबुल फजल द्वारा ‘अकबरनामा’ को पूरा करने के लिए प्रयुक्त किए गए स्त्रोत
(1) दस्तावेजों तथा प्रमाणों को एकत्रित करना- मुगल सम्राट अकबर ने अबुल फसल को आदेश दिया था कि वह साम्राज्य की शानदार घटनाओं और राज्य क्षेत्र अधीन करने वाली उनकी विजयों का ईमानदारी से वर्णन करे। अबुल फसल ने सम्राट के आदेश का पालन करते हुए अनेक दस्तावेज तथा प्रमाण इकट्ठे किये और राज्य के कर्मचारियों तथा शाही परिवार के सदस्यों से सूचनाएँ माँगीं।

JAC Class 12 History Important Questions Chapter 8 किसान, ज़मींदार और राज्य : कृषि समाज और मुगल साम्राज्य

(2) लोगों के विवरणों को नोट करना अबुल फसल ने सच बोलने वाले बुद्धिमान बुजुर्गों और तीव्र बुद्धिवाले, सत्यकर्मी जवानों दोनों की जाँच-पड़ताल की और उनके विवरणों को लिखित रूप में नोट किया।

(3) पुराने कर्मचारियों द्वारा लिखित संस्मरणों का अवलोकन करना- सूबों (प्रान्तों) को शाही आदेश जारी किया गया कि पुराने कर्मचारी अपने संस्मरण लिखें और उसे दरबार भेज दें। इसके बाद एक और शाही आदेश जारी किया गया कि जो सामग्री जमा की जाए, उसे सम्राट की उपस्थिति में पड़कर सुनाया जाए और बाद में जो कुछ भी

लिखा जाना हो, उसे उस महान ग्रन्थ में परिशिष्ट के रूप में जोड़ दिया जाए। अबुल फजल को आदेश दिया गया कि वह ऐसे विवरण जो उसी समय परिणति तक न लाए जा सकें, को बाद में नोट करे –

(4) घटनाओं का इतिहासवृत्त प्राप्त करना इस शाही आदेश के बाद अबुल फजल ने ऐसे कच्चे प्रारूप लिखने शुरू किये जिसमें शैली या विन्यास का सौन्दर्य नहीं था। उन्होंने इलाही संवत के उन्नीसवें वर्ष से जब बादशाह ने दस्तावेज कार्यालय स्थापित किया था, घटनाओं का इतिहासवृत्त प्राप्त किया और उसके पृष्ठों से कई मामलों का वृत्तान्त प्राप्त किया। उन सभी आदेशों की मूल प्रति वा नकल प्राप्त की गई, जो राज्याभिषेक से लेकर आज तक सूबों को जारी किए गए थे। अबुल फसल ने उनमें से कई रिपोर्टों को भी शामिल किया, जो साम्राज्य के विभिन्न मामलों या दूसरे देशों में घटी घटनाओं के बारे में थीं और जिन्हें उच्च अधिकारियों और मन्त्रियों ने भेजा था।

(5) कच्चे मसौदे तथा ज्ञानपत्र प्राप्त करना अबुल | फसल ने वे कच्चे मसौदे तथा ज्ञानपत्र भी प्राप्त किये जिन्हें जानकार और दूरदर्शी लोगों ने लिखा था।

प्रश्न 13.
मुगल काल में कृषि कीजिए।
की दशा का वर्णन
उत्तर:
मुगल काल में कृषि की दशा मुगल काल में कृषि की दशा का वर्णन निम्नलिखित बिन्दुओं के अन्तर्गत किया जा सकता है-
(1) फसलें मुगलकाल में भी कृषि भारतीयों के जीवन निर्वाह करने का प्रमुख साधन था किसान फसल की पैदावार से सम्बन्धित विविध प्रकार के कार्य करते थे, जैसे कि जमीन की जुताई, बीज बोना तथा फसल पकने पर उसकी कटाई वे उन वस्तुओं के उत्पादन में भी सम्मिलित होते थे जो कृषि आधारित थीं, जैसे कि शक्कर, तेल इत्यादि।

(2) फसलों की भरमार मौसम के दो मुख्य चक्रों के दौरान खेती की जाती थी एक खरीफ (पतझड़ में) तथा दूसरी रबी (बसन्त में)। अधिकांश स्थानों पर वर्ष में कम से कम दो फसलें होती थीं। जहाँ वर्षा या सिंचाई के अन्य साधन हर समय उपलब्ध थे वहाँ तो वर्ष में तीन फसलें भी उगाई जाती थीं। दैनिक आहार की खेती पर अधिक जोर दिया जाता था। कपास और गन्ना जैसी फसलें श्रेष्ठ ‘जिन्स- ए-कामिल थीं। मध्यभारत तथा दक्कनी पठार में कपास उगाई जाती थी। बंगाल गन्ने के उत्पादन के लिए प्रसिद्ध था। तिलहन (सरसों) और दलहन भी नकदी फसलों में गिनी जाती थीं। मक्का भारत में अफ्रीका और स्पेन से पहुँचा टमाटर, आलू व मिर्च जैसी सब्जियाँ नई दुनिया से लाई गई।

(3) सिंचाई के साधन आज की भाँति मानसून मुगल काल में भी भारतीय कृषि की रीढ़ था परन्तु कुछ ऐसी फसलें भी थीं, जिनके लिए अतिरिक्त पानी की आवश्यकता थी। ‘बाबरनामा’ से ज्ञात होता है कि छोटे पेड़ों तक रहट एवं बाल्टियों के द्वारा पानी पहुँचाया जाता था। इसके अतिरिक्त सिंचाई तालाबों, कुओं, नहरों एवं बाँधों के द्वारा भी होती थी। सिंचाई कार्यों को राज्य की ओर से प्रोत्साहन दिया जाता था।

(4) किसानों द्वारा तकनीकों का प्रयोग किसान प्राय: पशुबल पर आधारित तकनीकों का प्रयोग करते थे। वे लकड़ी के उस हल्के हल का प्रयोग करते थे जिसको एक छोर पर लोहे की नुकीली धार या फाल लगाकर आसानी से बनाया जा सकता था। ऐसे हल मिट्टी को बहुत गहरे नहीं खोदते थे जिसके कारण तेज गर्मी के महीनों में नमी बची रहती थी। बैलों के जोड़ों के सहारे खींचे जाने वाले बरमे का प्रयोग बीज बोने के लिए किया जाता था। मिट्टी की गुड़ाई और साथ-साथ निराई के लिए लकड़ी के मूठ वाले लोहे के पतले धार काम में लाए जाते थे।

(5) फलों के बाग मुगल सम्राटों ने फलों के अनेक बाग लगवाये।

(6) कृषि की दशा को सुधारने के लिए मुगल- सम्राटों के प्रयास मुगल सम्राटों ने किसानों को अच्छी फसलें गन्ना, कपास, तिलहन, दलहन आदि का उत्पादन करने के लिए प्रोत्साहित किया।

प्रश्न 14.
सोलहवीं तथा सत्रहवीं शताब्दी में किसानों की दशा का वर्णन कीजिए।
उत्तर:
सोलहवीं तथा सत्रहवीं शताब्दी में किसानों की दशा सोलहवीं तथा सत्रहवीं शताब्दी में किसानों की दशा का वर्णन निम्नलिखित बिन्दुओं के अन्तर्गत किया जा सकता है-
(1) किसानों के प्रकार-सोलहवीं तथा सत्रहवीं शताब्दी में भारतीय फारसी लोत ‘किसान’ के लिए चार शब्दों का प्रयोग करते थे-
(1) रैयत
(2) मुजरियान
(3) किसान और
(4) आसामी सत्रहवीं शताब्दी के स्रोत निम्नलिखित दो प्रकार के किसानों का उल्लेख करते हैं-
(i) खुद काश्त एवं
(ii) पाहि काश्त

(i) खुद काश्तखुद काश्त किसान वे थे जो उन्हीं गाँवों में रहते थे जिनमें उनकी जमीन थी।
(ii) पाहि काश्त पाहि कारण वे खेतिहर थे जो दूसरे गाँवों से ठेके पर खेती करने आते थे लोग अपनी इच्छा से भी पाहि-कारत बनते थे। उदाहरणार्थ-
(1) यदि करों की शर्तें किसी दूसरे गाँव में अधिक अच्छी मिलती थीं तथा
(2) अकाल या भुखमरी के बाद आर्थिक परेशानी के कारण।

(2) किसानों की समृद्धि का मापदंड उत्तर भारत में एक औसत किसान के पास सम्भवतः एक जोड़ी बैल तथा दो हल से अधिक कुछ नहीं होता था अधिकतर किसानों के पास इससे भी कम था। गुजरात में जिन किसानों के पास 6 एकड़ के लगभग जमीन थी, वे समृद्ध माने जाते थे। दूसरी ओर बंगाल में एक औसत किसान की जमीन की ऊपरी सीमा 5 एकड़ भी वहीं 10 एकड़ जमीन वाले आसामी को धनी समझा जाता था।

JAC Class 12 History Important Questions Chapter 8 किसान, ज़मींदार और राज्य : कृषि समाज और मुगल साम्राज्य

(3) व्यक्तिगत मिल्कियत खखेती व्यक्तिगत मिल्कियत के सिद्धान्त पर आधारित थी। किसानों की जमीन अन्य सम्पति मालिकों की भाँति खरीदी और बेची जाती थी। उन्नीसवीं सदी के दिल्ली-आगरा के प्रदेश में किसानों की मिल्कियत का यह विवरण सत्रहवीं सदी पर भी उतना ही लागू होता है- “हल जोतने वाले खेतिहर किसान हर जमीन की सीमाओं पर मिट्टी, ईंट और काँटों से पहचान के लिए निशान लगाते हैं और जमीन के ऐसे हजारों टुकड़े किसी भी गाँव में देखे जा सकते हैं।”

JAC Class 12 Political Science Important Questions Chapter 6 अंतर्राष्ट्रीय संगठन

Jharkhand Board JAC Class 12 Political Science Important Questions Chapter 6 अंतर्राष्ट्रीय संगठन Important Questions and Answers.

JAC Board Class 12 Political Science Important Questions Chapter 6 अंतर्राष्ट्रीय संगठन

बहुचयनात्मक प्रश्न

1. इजराइल ने लेबनान पर हमला किया।
(अ) फरवरी, 2006
(स) अक्टूबर, 2006
(ब) जून, 2006
(द) जनवरी, 2006
उत्तर:
(ब) जून, 2006

2. संयुक्त राष्ट्रसंघ के दूसरे महासचिव थे
(अ) एंटोनियो गुटेरेस
(ब) यू थांट
(स) बान की मून
(द) डेग हैमरशोल्ड
उत्तर:
(द) डेग हैमरशोल्ड

3. अंतर्राष्ट्रीय मुद्रा कोष (IMF) में कितने सदस्य हैं?
(अ) 193
(ब) 191
(स) 190
(द) 189
उत्तर:
(द) 189

4. अमरीकी राष्ट्रपति रूजवेल्ट और ब्रितानी प्रधानमंत्री चर्चिल ने अटलांटिक चार्टर पर हस्ताक्षर किए-
(अ) फरवरी, 1945
(स) अक्टूबर, 1945
(ब) जून, 1941
(द) अगस्त, 1941
उत्तर:
(द) अगस्त, 1941

5. संयुक्त राष्ट्र संघ की वर्तमान सदस्य संख्या कितनी है
(अ) 193
(ब) 190
(स) 189
(द) 191
उत्तर:
(अ) 193

6. संयुक्त राष्ट्र संघ की स्थापना कब की गई-
(अ) 24 अक्टूबर, 1945
(ब) 1 जनवरी, 1945
(स) 1 जून, 1945
(द) 31 दिसम्बर, 1945
उत्तर:
(अ) 24 अक्टूबर, 1945

JAC Class 12 Political Science Important Questions Chapter 6 अंतर्राष्ट्रीय संगठन

7. अन्तर्राष्ट्रीय न्यायालय का मुख्यालय कहाँ है-
(अ) वाशिंगटन में
(ब) न्यूयार्क में
(स) हेग में
(द) जिनेवा में
उत्तर:
(स) हेग में

8. भारत संयुक्त राष्ट्रसंघ में कब शामिल हुआ?
(अ) 24 अक्टूबर, 1995
(ब) 30 अक्टूबर, 1945
(स) 26 जून, 1945
(द) 15 अक्टूबर, 1945
उत्तर:
(ब) 30 अक्टूबर, 1945

9. संयुक्त राष्ट्र संघ दिवस प्रतिवर्ष मनाया जाता है
(अ) 24 अक्टूबर को
(स) 15 अगस्त को
(ब) 20 जनवरी को
(द) 26 जनवरी को
उत्तर:
(अ) 24 अक्टूबर को

10. अंतर्राष्ट्रीय न्यायालय में न्यायाधीशों की संख्या है।
अ) 11
(ब) 9
(स) 15
(द) 16
उत्तर:
(स) 15

रिक्त स्थानों की पूर्ति कीजिए :

1. संयुक्त राष्ट्रसंघ के प्रतीक चिह्न में ……………………….. की पत्तियाँ हैं, जो कि ……………………. का संकेत करती हैं।
उत्तर:
जैतून, विश्व शांति

2. ………………………. का सबसे प्रभावकारी समाधान वैश्विक तापवृद्धि को रोकना है।
उत्तर:
ग्लोबल वार्मिंग

3. अंतराष्ट्रीय मुद्राकोष में समूह -7 के सदस्यों के पास …………………….. प्रतिशत मत है।
उत्तर:
41.29

4. IMF में अमरीका का मताधिकार ……………………. प्रतिशत है।
उत्तर:
16.52

5. अंतर्राष्ट्रीय न्यायालय में न्यायाधीशों का चुनाव ………………………. तथा ………………………… में पूर्ण बहुमत द्वारा होता है।
उत्तर:
आम सभा, सुरक्षा परिषद्

अतिलघूत्तरात्मक प्रश्न

प्रश्न 1.
दूसरा विश्व युद्ध कब समाप्त हुआ?
उत्तर:
दूसरा विश्व युद्ध 1945 में समाप्त हुआ।

प्रश्न 2.
विश्व बैंक की औपचारिक स्थापना कब हुई?
उत्तर:
द्वितीय विश्व युद्ध के दौरान सन् 1944 में विश्व बैंक की औपचारिक स्थापना हुई।

JAC Class 12 Political Science Important Questions Chapter 6 अंतर्राष्ट्रीय संगठन

प्रश्न 3.
संयुक्त राष्ट्रसंघ का प्रतीक चिह्न क्या है?
उत्तर:
संयुक्त राष्ट्रसंघ के प्रतीक चिह्न में दुनिया का नक्शा बना हुआ है और इसके चारों तरफ जैतून की पत्तियाँ हैं।

प्रश्न 4.
समूह -7 के सदस्य देशों के नाम लिखिए।
उत्तर:
समूह – 7 के सदस्य देशों के नाम- अमरीका, जापान, जर्मनी, फ्रांस, ब्रिटेन, इटली और कनाडा है।

प्रश्न 5.
संयुक्त राष्ट्रसंघ की घोषणा पर हस्ताक्षर कब हुआ?
उत्तर:
धुरी शक्तियों के खिलाफ लड़ रहे 26 मित्र राष्ट्र अटलांटिक चार्टर के समर्थन में वाशिंग्टन में मिले ओर दिसंबर, 1943 में संयुक्त राष्ट्रसंघ की घोषणा पर हस्ताक्षर हुए।

प्रश्न 6.
संयुक्त राष्ट्र संघ के महासचिव की नियुक्ति कौन करता है?
उत्तर:
सुरक्षा परिषद् की सिफारिश के आधार पर महासभा महासचिव को नियुक्त करती है।

प्रश्न 7.
संयुक्त राष्ट्र संघ की स्थापना के समय कुल कितने सदस्य थे?
उत्तर:
संयुक्त राष्ट्र संघ की स्थापना के समय कुल 51 सदस्य थे

प्रश्न 8.
संयुक्त राष्ट्रसंघ के नौवें महासचिव कौन हैं?
उत्तर:
संयुक्त राष्ट्रसंघ के नौवें महासचिव एंटोनियो गुटेरेस हैं। इन्होंने महासचिव का पद 1 जनवरी, 2017 को संभाला।

प्रश्न 9.
विश्व की एकमात्र सुपर पॉवर ( महाशक्ति) का नाम बताइये।
उत्तर:
विश्व की एकमात्र महाशक्ति ‘संयुक्त राज्य अमेरिका’ है।

JAC Class 12 Political Science Important Questions Chapter 6 अंतर्राष्ट्रीय संगठन

प्रश्न 10.
आई. एम. एफ. का पूरा नाम लिखें।
उत्तर:
अन्तर्राष्ट्रीय मुद्रा कोष (International Monetary Fund)

प्रश्न 11.
डब्ल्यू. एच. ओ. (W.H.O.) का पूरा नाम लिखें।
उत्तर:
विश्व स्वास्थ्य संगठन (World Health Organization )

प्रश्न 12.
यूनेस्को (UNESCO) का पूरा नाम लिखें।
उत्तर:
संयुक्त राष्ट्र शैक्षणिक, वैज्ञानिक एवं सांस्कृतिक संगठन (United Nations Educational Scientific Cultural Organization)

प्रश्न 13.
इजरायल ने लेबनान पर कब हमला किया? उसने अपने आक्रमण को क्यों जरूरी बताया?
उत्तर:
इजरायल ने लेबनान पर 2006 में हमला किया। उसने उग्रवादी गुट हिजबुल्लाह पर अंकुश लगाने हेतु आक्रमण को आवश्यक बताया।

प्रश्न 14.
इजरायल के लेबनान पर आक्रमण के प्रति संयुक्त राष्ट्र संघ का क्या दृष्टिकोण रहा?
उत्तर:
इजरायल के लेबनान पर 2006 के आक्रमण के प्रति संयुक्त राष्ट्र संघ का यह दृष्टिकोण रहा कि इजरायली सैन्य बल इस क्षेत्र से वापस जाये।

प्रश्न 15.
अन्तर्राष्ट्रीय मुद्रा कोष का प्रमुख कार्य क्या है?
उत्तर:
अन्तर्राष्ट्रीय मुद्रा कोष मांगे जाने पर सदस्य देशों को वित्तीय एवं तकनीकी मदद उपलब्ध कराता है।

प्रश्न 16.
विश्व व्यापार संगठन की स्थापना कब हुई?
उत्तर:
1 जनवरी, 1995 को।

JAC Class 12 Political Science Important Questions Chapter 6 अंतर्राष्ट्रीय संगठन

प्रश्न 17.
विश्व व्यापार संगठन की क्या भूमिका है?
उत्तर:
विश्व व्यापार संगठन की मुख्य भूमिका विश्व के देशों में समान आर्थिक नियमों को लागू करना एवं मुक्त व्यापार को बढ़ावा देना है।

प्रश्न 18.
विश्व व्यापार संगठन की सदस्य संख्या क्या है?
उत्तर:
विश्व व्यापार संगठन की सदस्य संख्या 150 है।

प्रश्न 19.
विश्व व्यापार संगठन में फैसले कैसे लिये जाते हैं?
उत्तर:
विश्व व्यापार संगठन में फैसले आम सहमति से लिये जाते हैं।

प्रश्न 20.
अन्तर्राष्ट्रीय आणविक ऊर्जा एजेन्सी से क्या अभिप्राय है?
उत्तर:
इस संगठन का मुख्य उद्देश्य परमाणु ऊर्जा के शांतिपूर्ण उपयोग को बढ़ावा देना तथा सैन्य उद्देश्यों में इसके प्रयोग को रोकना है।

प्रश्न 21.
अन्तर्राष्ट्रीय आणविक ऊर्जा एजेन्सी (आई.ए.ई.ए.) का स्थापना वर्ष लिखिये
उत्तर:
सन् 1957 ई.

JAC Class 12 Political Science Important Questions Chapter 6 अंतर्राष्ट्रीय संगठन

प्रश्न 22.
अन्तर्राष्ट्रीय आणविक ऊर्जा एजेन्सी क्या प्रयास करता है?
उत्तर:
यह संगठन परमाणु ऊर्जा के शांतिपूर्ण उपयोग को बढ़ावा देने का कार्य करता है।

प्रश्न 23.
संयुक्त राष्ट्र संघ का मुख्य उद्देश्य क्या है?
उत्तर:
संयुक्त राष्ट्र संघ का मुख्य उद्देश्य शांति और सुरक्षा कायम करना है।

प्रश्न 24.
संयुक्त राष्ट्र संघ का मुख्यालय कहाँ स्थित है?
उत्तर:
संयुक्त राष्ट्र संघ का मुख्यालय न्यूयार्क में स्थित है।

प्रश्न 25.
अमरीका, रूस, ब्रिटेन, फ्रांस और चीन को संयुक्त राष्ट्रसंघ का स्थायी सदस्य क्यों चुना गया?
उत्तर:
दूसरे विश्वयुद्ध के तुरंत बाद के समय में ये देश सबसे ज्यादा ताकतवर थे और इस महायुद्ध के विजेता भी रहे, इसलिए इन्हें स्थायी सदस्य के रूप में चुना गया।

प्रश्न 26.
संयुक्त राष्ट्र संघ का सर्वाधिक महत्त्वपूर्ण अंग कौनसा है?
उत्तर:
संयुक्त राष्ट्र संघ का सर्वाधिक महत्त्वपूर्ण अंग ‘सुरक्षा परिषद्’ है।

प्रश्न 27.
संयुक्त राष्ट्र संघ का दूसरा महत्त्वपूर्ण अंग कौनसा है, जिसके सभी सदस्य राष्ट्र सदस्य हैं?
उत्तर:
आम सभा या महासभा।

प्रश्न 28.
संयुक्त राष्ट्र संघ के मुख्य अंग कितने हैं? उनके नाम लिखिये।
उत्तर:
संयुक्त राष्ट्र संघ के प्रमुख 6 अंग हैं। ये हैं।

  1. महासभा
  2. सुरक्षा परिषद्
  3. आर्थिक और सामाजिक परिषद्
  4. न्यास परिषद्
  5. अन्तर्राष्ट्रीय न्यायालय और
  6. सचिवालय।

प्रश्न 29.
अन्तर्राष्ट्रीय न्यायालय के न्यायाधीशों का कार्यकाल कितने वर्ष का होता है?
उत्तर:
अन्तर्राष्ट्रीय न्यायालय के न्यायाधीशों का कार्यकाल 9 वर्ष का होता है । प्रश्न 30. संयुक्त राष्ट्र संघ के किन्हीं दो विशिष्ट अभिकरणों के नाम लिखिये।
उत्तर:
कृषि एवं खाद्य संगठन, विश्व स्वास्थ्य संगठन

प्रश्न 31.
संयुक्त राष्ट्र सुरक्षा परिषद् के अस्थायी सदस्यों की संख्या कितनी है और उनका कार्यकाल कितना है?
उत्तर:
संयुक्त राष्ट्र सुरक्षा परिषद् के अस्थायी सदस्यों की संख्या 10 है तथा उनका कार्यकाल 4 वर्ष का है।

JAC Class 12 Political Science Important Questions Chapter 6 अंतर्राष्ट्रीय संगठन

प्रश्न 32.
संयुक्त राष्ट्र संघ के दो उद्देश्यों का वर्णन कीजिये।
उत्तर:
संयुक्त राष्ट्र संघ के प्रमुख उद्देश्य ये हैं।

  1. सामूहिक व्यवस्था द्वारा अन्तर्राष्ट्रीय शांति एवं सुरक्षा कायम रखना और आक्रामक प्रवृत्तियों को नियंत्रण में रखना।
  2. अन्तर्राष्ट्रीय विवादों का शांतिपूर्ण समाधान करना।

प्रश्न 33.
संयुक्त राष्ट्र के महासचिव के दो कार्य बताइये।
उत्तर:

  1. साधारण सभा की वार्षिक रिपोर्ट तैयार करना
  2. सुरक्षा सम्बन्धी विवादों को महासभा के समक्ष प्रस्तुत करना।

प्रश्न 34.
अन्तर्राष्ट्रीय स्तर पर कार्य कर रहे किन्हीं दो गैर-सरकारी संगठनों के नाम लिखें।
अथवा
मानवाधिकारों की रक्षा में संलग्न दो गैर सरकारी अन्तर्राष्ट्रीय संगठन कौन से हैं?
उत्तर:

  1. एमनेस्टी इन्टरनेशनल
  2. ह्यमन राइट्स वॉच।

प्रश्न 35.
संयुक्त राष्ट्र संघ के कोई दो कार्य बताइये।
उत्तर:

  1. अन्तर्राष्ट्रीय शांति सुरक्षा बनाये रखना।
  2. अन्तर्राष्ट्रीय कानून सम्बन्धी व्याख्या करना।

प्रश्न 36.
संयुक्त राष्ट्र महासभा द्वारा 1950 में ‘शांति के लिए एकता’ प्रस्ताव की दो विशेषताएँ बताइए।
उत्तर:

  1. महासभा का आपातकालीन अधिवेशन बुलाना
  2. शांति निरीक्षण आयोग की स्थापना करना।

प्रश्न 37.
संयुक्त राष्ट्र संघ की महासभा के दो कार्यों का वर्णन कीजिए।
उत्तर:

  1. संयुक्त राष्ट्र संघ का बजट पारित करना।
  2. सुरक्षा परिषद् तथा अन्य संस्थाओं व संगठनों की रिपोर्ट पर विचार करना।

प्रश्न 38.
संयुक्त राष्ट्र की सुरक्षा परिषद् के स्थायी सदस्यों के नाम लिखिये।
उत्तर:
सुरक्षा परिषद् के 5 स्थायी सदस्य हैं। ये हैं।

  1. संयुक्त राज्य अमेरिका
  2.  ब्रिटेन
  3. फ्रांस
  4. रूस व
  5. चीन।

JAC Class 12 Political Science Important Questions Chapter 6 अंतर्राष्ट्रीय संगठन

प्रश्न 39.
सुरक्षा परिषद् के अस्थायी सदस्यों को कौन निर्वाचित करता है? अस्थायी सदस्यों का कार्यकाल कितना होता है?
उत्तर:
सुरक्षा परिषद् के अस्थायी सदस्यों को महासभा निर्वाचित करती है तथा अस्थायी सदस्यों का कार्यकाल चार वर्ष का होता है।

प्रश्न 40.
वर्तमान में सुरक्षा परिषद् के सदस्यों की कुल संख्या कितनी है?
उत्तर:
वर्तमान में सुरक्षा परिषद् के सदस्यों की कुल संख्या 15 है। इनमें 5 सदस्य स्थायी हैं और 10 अस्थायी हैं । प्रश्न 41. संयुक्त राष्ट्र महासभा की प्रतिष्ठा में वृद्धि के कोई दो कारण बताइये।
उत्तर:

  1. महासभा में प्रायः सभी देशों को प्रतिनिधित्व प्राप्त है।
  2. शांति के लिये एकता प्रस्ताव के कारण।

प्रश्न 42.
जी 8 के सदस्य देशों के नाम लिखिये।
उत्तर:
जी 8 के सदस्य देश हैं।

  1. अमरीका
  2. जापान
  3. जर्मनी
  4. फ्रांस
  5. ब्रिटेन
  6. इटली
  7. कनाडा और
  8. रूस।

प्रश्न 43.
संयुक्त राष्ट्र संघ के वार्षिक बजट में सर्वाधिक योगदान किस देश का है और कितना है?
उत्तर:
संयुक्त राज्य अमेरिका का लगभग 22 प्रतिशत।

प्रश्न 44.
एमनेस्टी इण्टरनेशनल का क्या उत्तरदायित्व है?
उत्तर:
इसका उत्तरदायित्व है। मानवाधिकारों से सम्बद्ध रिपोर्ट तैयार करना तथा उन्हें प्रकाशित करना।

प्रश्न 45.
संयुक्त राष्ट्रसंघ के प्रथम महासचिव कौन थे?
उत्तर:
संयुक्त राष्ट्रसंघ के प्रथम महासचिव ट्राइग्व ली थे।

प्रश्न 46.
संयुक्त राष्ट्रसंघ में अस्थायी सदस्य कितने वर्षों के लिए चुने जाते हैं?
उत्तर:
दो वर्षों के लिए।

प्रश्न 47.
मानवाधिकार परिषद् कब से सक्रिय है?
उत्तर:
मानवाधिकार परिषद् 19 जून, 2006 से सक्रिय है।

प्रश्न 48.
विश्व व्यापार संगठन किसके उत्तराधिकारी के रूप में कार्य करता है?
उत्तर:
जनरल एग्रींट ऑन ट्रेड एंड टैरिफ

प्रश्न 49.
विश्व बैंक के मुख्य कार्य क्या हैं?
उत्तर:
कृषि, ग्रामीण, विकास, पर्यावरण सुरक्षा, सुशासन तथा आधारभूत ढाँचे के लिए काम करता है।

JAC Class 12 Political Science Important Questions Chapter 6 अंतर्राष्ट्रीय संगठन

प्रश्न 50.
अंतर्राष्ट्रीय मुद्राकोष के कार्य लिखिए।
उत्तर:
विश्व स्तर पर वित्त व्यवस्था की देखरेख तथा वित्तीय व तकनीकी सहायता देना।

लघूत्तरात्मक प्रश्न

प्रश्न 1.
सुरक्षा परिषद् के अस्थाई सदस्यों का भौगोलिक वितरण किस प्रकार किया गया है?
उत्तर:
सुरक्षा परिषद् के अस्थायी सदस्यों का भौगोलिक वितरण इस प्रकार है। अफ्रीका और एशिया के पाँच सदस्य, पूर्वी यूरोप का एक सदस्य, लातीन अमेरिका के दो सदस्य, पश्चिमी यूरोप तथा अन्य राज्यों के दो सदस्य।

प्रश्न 2.
वीटो किसे कहते हैं?
उत्तर:
वीटो का शब्दार्थ है, “मैं मना करता हूँ।” सुरक्षा परिषद् का कोई स्थायी सदस्य यदि किसी महत्त्वपूर्ण या सारवान प्रश्न पर असहमति प्रकट करता है तो उसे अस्वीकृत कर दिया जाता है। उसके इस अधिकार/ शक्ति को ही वीटो शक्ति कहा जाता है।

प्रश्न 3.
संयुक्त राष्ट्र संघ के पुनर्गठन या लोकतंत्रीकरण से क्या अभिप्राय है?
उत्तर:
संयुक्त राष्ट्र संघ के पुनर्गठन या लोकतन्त्रीकरण से अभिप्राय है— सुरक्षा परिषद् के स्थायी सदस्यों में विस्तार करना तथा उसमें विकासशील देशों को समुचित प्रतिनिधित्व देना।

प्रश्न 4.
संयुक्त राष्ट्र की सुरक्षा परिषद् में स्थायी सदस्यता हेतु भारत की दावेदारी के समर्थन में दो तर्क लिखिये।
उत्तर:

  1. भारत विश्व का सबसे बड़ा प्रजातंत्रीय देश है।
  2. भारत सैनिक, आर्थिक तथा प्रौद्योगिक दृष्टि से एक सशक्त राष्ट्र- है

प्रश्न 5.
संयुक्त राष्ट्र संघ के संगठन एवं प्रक्रिया में सुधार के लिये कोई दो सुझाव दें।
उत्तर:

  1. वीटो की समाप्ति-सुरक्षा परिषद् की कार्यप्रणाली को व्यवस्थित एवं लोकतांत्रिक बनाने के लिए वीटो की शक्ति को समाप्त कर देना चाहिए।
  2. समान सदस्यता- सुरक्षा परिषद् में अस्थाई सदस्यों की धारणा को समाप्त करके सबको समान स्तर की. सदस्यता प्रदान करनी चाहिए।

प्रश्न 6.
‘ग्लोबल वार्मिंग’ की समस्या को स्पष्ट करें।
उत्तर:
‘ग्लोबल वार्मिंग’ से आशय है। वातावरण में ग्रीनहाउस गैसों के बढ़ने से तापमान में वृद्धि इससे समुद्रतल की ऊँचाई बढ़ने का खतरा है। ऐसा होने पर विश्व के समुद्रतटीय इलाके जलमग्न हो सकते हैं। इस समस्या का सबसे प्रभावकारी समाधान वैश्विक तापवृद्धि को रोकना है।

प्रश्न 7.
‘लीग ऑव नेशंस’ की उत्पत्ति क्यों हुई?
उत्तर:
प्रथम विश्वयुद्ध के पश्चात् विश्व को एक ऐसे अंतर्राष्ट्रीय संगठन की जरूरत का आभास हुआ जो अंतर्राष्ट्रीय झगड़ों का निपटारा बातचीत द्वारा कर सके जिससे युद्ध जैसे विनाशकारी स्थिति को टाला जा सके। इसके परिणामस्वरूप ‘लीग ऑव नेशंस’ का जन्म हुआ।

JAC Class 12 Political Science Important Questions Chapter 6 अंतर्राष्ट्रीय संगठन

प्रश्न 8.
एंटोनियो गुटेरेस के बारे में संक्षेप में लिखिये।
उत्तर:
एंटोनियो गुटेरेस संयुक्त राष्ट्रसंघ के नौवें महासचिव हैं। उन्होंने महासचिव का पद 1 जनवरी, 2017 को ग्रहण किया। ये 1995 से 2002 तक पुर्तगाल के प्रधानमंत्री और 2005 से 2015 तक यूनाइटेड नेशंस हाई कमीशनर फॉर रिफ्यूजीज रहे। 1999-2005 तक ये सोशलिस्ट
इंटरनेशनल के अध्यक्ष भी रहे।

प्रश्न 9.
संयुक्त राष्ट्र संघ की चार विशिष्ट एजेन्सियों के नाम लिखो।
उत्तर:
संयुक्त राष्ट्र संघ की चार विशिष्ट एजेन्सियों के नाम इस प्रकार हैं।

  1. अन्तर्राष्ट्रीय श्रम संगठन (I.L.O.)
  2. संयुक्त राष्ट्र शैक्षिक, वैज्ञानिक एवं सांस्कृतिक संगठन (UNESCO)
  3. विश्व स्वास्थ्य संगठन., (W.H.O.)
  4. खाद्य एवं कृषि संगठन।

प्रश्न 10.
संयुक्त राष्ट्र संघ के कोई तीन सिद्धान्त लिखिए।
उत्तर:

  1. संयुक्त राष्ट्र संघ सदस्यों की संप्रभु समानता पर आधारित है।
  2. संयुक्त राष्ट्र के सभी सदस्य ईमानदारी से घोषणा-पत्र के अन्तर्गत निर्धारित दायित्वों का पालन करेंगे।
  3. संयुक्त राष्ट्र के सभी सदस्य अपने विवादों को शांतिपूर्ण ढंग से निपटायेंगे ।0

प्रश्न 11.
संयुक्त राष्ट्रसंघ के द्वितीय महासचिव कौन थे? उनके द्वारा किए गए कार्यों का विवरण कीजिए।
उत्तर:
संयुक्त राष्ट्रसंघ के द्वितीय महासचिव डेग हैमरशोल्ड थे। ये स्वीडन से थे तथा अर्थशास्त्री और वकील थे इन्होंने स्वेज नहर से जुड़े विवादों को सुलझाने तथा अफ्रीका के अनौपनिवेशीकरण में महत्त्वपूर्ण भूमिका निभाई। कांगो- संकट को सुलझाने की दिशा में किए गए प्रयासों के लिए मरणोपरांत इन्हें नोबेल शांति पुरस्कार दिया गया। हालाँकि सोवियत संघ और फ्रांस ने अफ्रीका में इनकी भूमिका की आलोचना की थी।

प्रश्न 12.
सुरक्षा परिषद् में स्थायी सदस्यों को प्राप्त निषेधाधिकार (Veto) शक्ति से क्या आशय है?
उत्तर:
सुरक्षा परिषद् में स्थायी सदस्यों को प्राप्त निषेधाधिकार की शक्ति का अर्थ है कि यदि पांच स्थायी सदस्यों में से कोई भी एक सदस्य संयुक्त राष्ट्र सुरक्षा परिषद् में रखे गये प्रस्ताव के विरोध में वोट डाल दे तो वह प्रस्ताव पास नहीं होगा।

प्रश्न 13.
संयुक्त राष्ट्रसंघ के समक्ष कौन से दो बुनियादी सुधारों का मसला है?
उत्तर:
संयुक्त राष्ट्रसंघ के समक्ष निम्न दो बुनियादी सुधारों का मसला है।

  1. एक तो इस संगठन की बनावट और इसकी प्रक्रियाओं में सुधार किया जाए।
  2. दूसरा, इस संगठन के न्यायाधिकार में आने वाले मुद्दों की समीक्षा की जाए।

प्रश्न 14.
संयुक्त राष्ट्र संघ की स्थापना कब हुई ? आजकल इसके कितने सदस्य हैं?
उत्तर:
संयुक्त राष्ट्र संघ की विधिवत् स्थापना 24 अक्टूबर, 1945 ई. को हुई थी। इस संस्था का मुख्य कार्यालय न्यूयार्क अमेरिका में है। आजकल संसार के छोटे-बड़े लगभग 193 देश इसके सदस्य हैं। करना।

JAC Class 12 Political Science Important Questions Chapter 6 अंतर्राष्ट्रीय संगठन

प्रश्न 15.
संयुक्त राष्ट्र संघ के प्रमुख उद्देश्यों का उल्लेख कीजिये।
उत्तर:
उद्देश्य-संयुक्त राष्ट्र संघ की स्थापना के मुख्य उद्देश्य निम्न हैं।

  1. अन्तर्राष्ट्रीय शांति और सुरक्षा को बनाये रखना।
  2. भिन्न-भिन्न राष्ट्रों के बीच मैत्रीपूर्ण संबंधों को बढ़ाना।
  3. आपसी सहयोग द्वारा आर्थिक, सामाजिक, सांस्कृतिक तथा मानवीय ढंग की अन्तर्राष्ट्रीय समस्याओं को हल

प्रश्न 16.
संयुक्त राष्ट्र की महासभा के संगठन को संक्षिप्त में बताइये।
उत्तर:
हासभा-महासभा संयुक्त राष्ट्र संघ का सबसे बड़ा अंग है। संयुक्त राष्ट्र के सभी सदस्य इसके सदस्य होते हैं और प्रत्येक सदस्य राष्ट्र इसमें पाँच प्रतिनिधि भेजता है परन्तु उनका मत एक ही होता है। प्राय: वर्ष में इसका एक बार अधिवेशन होता है। वर्तमान में इसके सदस्यों की कुल संख्या 193 है।

प्रश्न 17.
सुरक्षा परिषद् के संगठन पर संक्षिप्त टिप्पणी लिखिये।
उत्तर:
सुरक्षा परिषद् संयुक्त राष्ट्र की कार्यपालिका के समान है। इसके 15 सदस्य होते हैं। जिनमें पाँच स्थाई सदस्य हैं। अमेरिका, इंग्लैंड, फ्रांस, चीन और रूस । इसके अलावा 10 अस्थायी सदस्य दो वर्षों के लिये महासभा द्वारा चुने जाते हैं। सुरक्षा परिषद् के पाँच स्थायी सदस्यों को निषेधाधिकार प्राप्त है।

प्रश्न 18.
अन्तर्राष्ट्रीय आणविक ऊर्जा एजेन्सी क्या है?
उत्तर:
अन्तर्राष्ट्रीय आणविक ऊर्जा एजेन्सी परमाणविक ऊर्जा के शांतिपूर्ण उपयोग को बढ़ावा देने और सैन्य उद्देश्यों में इसके प्रयोग को रोकने की कोशिश करता है। इसके अधिकारी विश्व की परमाणविक सुविधाओं की जाँच करते हैं ताकि नागरिक परमाणु संयंत्रों का इस्तेमाल सैन्य उद्देश्यों के लिए न हो।

प्रश्न 19.
एमनेस्टी इंटरनेशनल क्या है?
उत्तर:
एमनेस्टी इन्टरनेशनल
एमनेस्टी इंटरनेशनल एक स्वयंसेवी संगठन है। यह पूरे विश्व में मानवाधिकारों की रक्षा के लिए अभियान चलाता है। यह संगठन मानवाधिकारों से जुड़ी रिपोर्ट तैयार और प्रकाशित करता है। ये रिपोर्टें मानवाधिकारों से संबंधित अनुसंधान और तरफदारी में बड़ी महत्त्वपूर्ण भूमिका निभाती हैं।

प्रश्न 20.
ह्यूमन राइट वाच क्या है?
उत्तर:
ह्यूमन राइट वाच (Human Right Watch):
ह्यमन राइट वाच मानवाधिकारों की वकालत और उनसे संबंधित अनुसंधान करने वाला एक अन्तर्राष्ट्रीय स्वयंसेवी संगठन है। यह अमरीका का सबसे बड़ा अन्तर्राष्ट्रीय मानवाधिकार संगठन है। यह विश्व की मीडिया का ध्यान मानवाधिकारों के उल्लंघन की ओर खींचता है।

प्रश्न 21.
विश्व व्यापार संगठन क्या है?
उत्तर:
विश्व व्यापार संगठन (WTO):
‘विश्व व्यापार संगठन’ (World Trade Organization) एक अन्तर्राष्ट्रीय संगठन है। यह वैश्विक व्यापार के नियमों को तय करता है। इसकी स्थापना सन् 1995 में हुई। इसके सदस्यों की संख्या 164 है।

JAC Class 12 Political Science Important Questions Chapter 6 अंतर्राष्ट्रीय संगठन

प्रश्न 22.
सन् 2018 तक स्थायी सदस्यों द्वारा वीटो पॉवर का इस्तेमाल कितनी-कितनी बार किया गया है? उत्तर-सन् 2018 तक स्थायी सदस्यों द्वारा वीटो पॉवर का इस्तेमाल इस प्रकार किया गया है।

  1. सोवियत संघ / रूस द्वारा 135 बार
  2. अमरीका द्वारा 84 बार
  3. ब्रिटेन द्वारा 32 बार
  4. फ्रांस द्वारा 18 बार
  5. चीन द्वारा 11 बार।

प्रश्न 23.
सन् 1992 में संयुक्त राष्ट्रसंघ की आमसभा में किन तीन शिकायतों का प्रस्ताव स्वीकृत हुआ था?
उत्तर:
सन् 1992 में संयुक्त राष्ट्रसंघ की आम सभा में निम्न तीन शिकायतों का प्रस्ताव स्वीकृत हुआ था।

  1. सुरक्षा परिषद् राजनीतिक वास्तविकताओं की नुमाइंदगी नहीं करती।
  2. इनके फैसलों पर पश्चिमी देशों के मूल्यों और हितों की छाप होती है और इन फैसलों पर चंद देशों का दबदबा होता है।
  3. सुरक्षा परिषद् में बराबर का प्रतिनिधित्व नहीं है।

प्रश्न 24.
सुरक्षा परिषद् के पांच स्थायी सदस्यों को वीटो का अधिकार क्यों दिया गया?
उत्तर:
सुरक्षा परिषद् के पांच स्थायी सदस्यों को वीटो का अधिकार इसलिये दिया गया क्योंकि-

  1. ये देश द्वितीय विश्व युद्ध के विजेता थे।
  2. राजनीतिक मामलों में इनकी सहमति सर्वाधिक महत्त्वपूर्ण थी।
  3. वीटो का अधिकार न देने पर संभवतः ये समस्याओं में अधिक रुचि नहीं लेते।

प्रश्न 25.
मानवाधिकार परिषद् के संस्थापक कौन थे? इनके कुछ प्रमुख उपलब्धियों का वर्णन कीजिए।
उत्तर:
मानवाधिकार परिषद् के आयोग की स्थापना कोफी ए. अन्नान ने की। इनकी कुछ उपलब्धियाँ निम्नलिखित

  1. ये (1997 से 2006 तक) संयुक्त राष्ट्र संघ के महासचिव रहे।
  2. इन्होंने एड्स, टीबी और मलेरिया से लड़ने के लिए एक वैश्विक कोष बनाया।
  3. इन्होंने शान्ति संस्थापक आयोग की स्थापना 2005 में की।
  4. महासचिव के रूप में किए गए कार्यों के कारण 2001 में इनको नोबेल शान्ति पुरस्कार दिया गया।

JAC Class 12 Political Science Important Questions Chapter 6 अंतर्राष्ट्रीय संगठन

प्रश्न 26.
” सोवियत संघ की गैर मौजूदगी में अमरीका एक भाग महाशक्ति है।” इस कथन का सत्यापन
कीजिए।
उत्तर:
सोवियत संघ की गैर मौजूदगी में अमरीका एकमात्र महाशक्ति है। क्योंकि

  1. अमरीका की ताकत पर आसानी से अंकुश नहीं लगाया जा सकता।
  2. अपनी सैन्य और आर्थिक ताकत के सहारे अमरीका संयुक्त राष्ट्रसंघ या किसी अन्य अंतर्राष्ट्रीय संगठन की अनदेखी कर सकता है।
  3. संयुक्त राष्ट्रसंघ के भीतर अमरीका का खास प्रभाव है क्योंकि वह संयुक्त राष्ट्रसंघ के बजट में सबसे ज्यादा योगदान करता है। संयुक्त राष्ट्रसंघ अमरीकी भू-क्षेत्र में स्थित है। संयुक्त राष्ट्रसंघ के कई नौकरशाह अमरीका के नागरिक हैं।

प्रश्न 27.
“अंतर्राष्ट्रीय मुद्रा कोष के हर सदस्य की राय का वजन बराबर नहीं है।” स्पष्ट कीजिए।
उत्तर:
अंतराष्ट्रीय मुद्राकोष में 189 सदस्य हैं लेकिन हर सदस्य की राय का वजन बराबर नहीं है। क्योंकि

  1. समूह – 7 के सदस्य ( अमरीका, जापान, जर्मनी, फ्रांस, ब्रिटेन, इटली और कनाडा) के पास 41.29% मत हैं। अकेले अमरीका के पास 16.52% मताधिकार है।
  2. अन्य अग्रणी सदस्यों में चीन ( 6.09%), भारत (2.64%), रूस (2.59% ), ब्राजील (2.22%) और सऊदी अरब (0.02%) है।

प्रश्न 28.
अंतर्राष्ट्रीय संगठन की आवश्यकता क्यों है?
उत्तर:
अंतर्राष्ट्रीय संगठन की आवश्यकता निम्न कारणों की वजह से है-

  1. समस्याओं का शांतिपूर्ण समाधान हेतु।
  2. ये संगठन सहयोग के उपाय जुटाने मेंसहायक होते हैं।
  3. ये संगठन नियमों तथा नौकरशाही की रूपरेखा तैयार करते हैं।
  4. ये शांति और प्रगति के प्रति मानवता की आशा का प्रतीक़ होते हैं

प्रश्न 29.
संयुक्त राष्ट्र के सिद्धान्तों का वर्णन कीजिये।
उत्तर:
संयुक्त राष्ट्र के सिद्धान्त-संयुक्त राष्ट्र संघ अपने उद्देश्यों की पूर्ति के लिये निम्नलिखित सिद्धान्तों के. अनुसार कार्य करता है।

  1. संयुक्त राष्ट्र · सभी सदस्यों की प्रभुसत्ता और समानता के सिद्धान्त में विश्वास रखता है।
  2. सभी राष्ट्रों की अखण्डता और राजनीतिक स्वाधीनता का आदर करें।
  3. संयुक्त राष्ट्र चार्टर के अनुसार कोई कार्यवाही करें तो सभी सदस्य राष्ट्रों को उसकी सहायता करनी चाहिये।
  4. सभी राष्ट्र अपने विवादों को शांतिपूर्ण तरीकों से निपटायें।
  5. संयुक्त राष्ट्र उन मामलों में हस्तक्षेप नहीं करेगा जो अनिवार्य रूप से किसी राज्य के आंतरिक अधिकार क्षेत्र में आते हैं।

JAC Class 12 Political Science Important Questions Chapter 6 अंतर्राष्ट्रीय संगठन

प्रश्न 30.
क्या संयुक्त राष्ट्र अमरीकी प्रभुत्व के खिलाफ संतुलन के रूप में कार्य कर सकता है?
उत्तर:
संयुक्त राष्ट्रसंघ का अमरीकी प्रभुत्व के खिलाफ संतुलन के रूप में कार्य न कर पाने की निम्न वजहें हैं।

  1. अमरीका अपने वीटो पावर के द्वारा ऐसे किसी भी कदम को रोक सकती है जिससे उसका हित ना सधता हो।
  2. 1991 के बाद से अमरीका एकमात्र महाशक्ति बन कर उभरा है जो अपनी आर्थिक और सैन्य ताकत से किसी भी अंतर्राष्ट्रीय संगठनों की अनदेखी कर सकता है।
  3. अपनी ताकत और निषेधाधिकार के कारण संयुक्त राष्ट्रसंघ के महासचिव के चयन में भी अमरीका की बात बहुत वजन रखती है।

प्रश्न 31.
” भारत संयुक्त राष्ट्र सुरक्षा परिषद् का स्थायी सदस्य बनना चाहता है।” क्या इस कथन से आप सहमत हैं? तर्कों के माध्यम से स्पष्ट कीजिए।
उत्तर:
भारत संयुक्त राष्ट्र सुरक्षा परिषद् का स्थायी सदस्य बनना चाहता है। इस कथन से मैं सहमत हूँ। निम्न कथनों के द्वारा इस कथन को स्पष्ट किया जा सकता है-

  1. भारत विश्व में आबादी वाला दूसरा बड़ा देश है।
  2. भारत में विश्व की कुछ जनसंख्या का 1/5 वाँ हिस्सा निवास करता है।
  3. भारत विश्व का सबसे बड़ा लोकतंत्र है।
  4. भारत ने संयुक्त राष्ट्र की लगभग सभी पहलकदमियों में भाग लिया है।
  5. संयुक्त राष्ट्रसंघ के शांति बहाल करने के प्रयासों में भारत लंबे समय से ठोस भूमिका निभाता आ रहा है।
  6. भारत तेजी से अंतर्राष्ट्रीय फलक पर आर्थिक शक्ति बनकर उभर रहा है।
  7. भारत ने संयुक्त राष्ट्रसंघ के बजट में नियमित रूप से अपना योगदान दिया है कभी भी यह अपने भुगतान से चुका नहीं है।

प्रश्न 32.
संयुक्त राष्ट्र सुरक्षा परिषद् के 5 स्थायी सदस्यों के नाम बताइये तथा इसकी कार्यप्रणाली की कमियाँ बताइये।
उत्तर:

  • संयुक्त राष्ट्र सुरक्षा परिषद् के 5 स्थायी सदस्य हैं।
    1. अमेरिका
    2. फ्रांस
    3. ब्रिटेन
    4. रूस
    5. चीन।
  • संयुक्त राष्ट्र सुरक्षा परिषद् की कार्यप्रणाली की कमियाँ ये हैं।
    1. सुरक्षा परिषद् अब राजनीतिक वास्तविकताओं की नुमाइंदगी नहीं करती।
    2. इसके फैसलों पर पश्चिमी मूल्यों और हितों की छाप तथा चंद देशों का दबदबा होता है।
    3. इसमें बराबर का प्रतिनिधित्व नहीं है।

प्रश्न 33.
एक ध्रुवीय विश्व में संयुक्त राष्ट्रसंघ की प्रासंगिकता को स्पष्ट कीजिए।
उत्तर:

  1. संयुक्त राष्ट्रसंघ क्षेत्रीय या विश्वव्यापी विवाद के मुद्दों पर सभी देशों का बातचीत करके हल निकालने का मंच देता है।
  2. अमरीकी सरकार यह जानती है कि झगड़ों और सामाजिक आर्थिक विकास के मसलों पर संयुक्त राष्ट्रसंघ के माध्यम से 193 राष्ट्रों को एकत्रित किया जा सकता है।
  3. शेष विश्व के लिए संयुक्त राष्ट्रसंघ एक ऐसा मंच है जहाँ अमरीकी रवैये और नीतियों पर अंकुश लगाया जा सकता है।

JAC Class 12 Political Science Important Questions Chapter 6 अंतर्राष्ट्रीय संगठन

प्रश्न 34.
एक ध्रुवीय विश्व में संयुक्त राष्ट्र संघ की क्या भूमिका है? व्याख्या कीजिये।
अथवा
क्या आप इस बात से सहमत हैं कि संयुक्त राष्ट्र संघ जैसा अन्तर्राष्ट्रीय संगठन पूरे विश्व के लिए अपरिहार्य है? अपने उत्तर के पक्ष में कोई चार कारण स्पष्ट कीजिए। एक ध्रुवीय विश्व में संयुक्त राष्ट्र संघ की प्रासंगिकता को स्पष्ट कीजिये।
उत्तर:
संयुक्त राष्ट्र संघ जैसा अन्तर्राष्ट्रीय संगठन पूरे विश्व के लिए अपरिहार्य है। एक ध्रुवीय विश्व में संयुक्त राष्ट्र संघ की प्रासंगिकता या अपरिहार्यता को निम्न प्रकार स्पष्ट किया जा सकता है।

  1. संयुक्त राष्ट्र संघ अमरीका और शेष विश्व के बीच विभिन्न मुद्दों पर बातचीत कायम कर सकता है।
  2. झगड़ों और सामाजिक-आर्थिक विकास के मुद्दे पर संयुक्त राष्ट्र संघ के जरिये 193 देशों को एक साथ किया जा सकता है।
  3. शेष विश्व संयुक्त राष्ट्र संघ के मंच के माध्यम से अमरीकी रवैये और नीतियों पर कुछ न कुछ अंकुश लगा सकता है।
  4. आज विभिन्न समाजों और मुद्दों के बीच आपसी तार जुड़ते जा रहे हैं। आने वाले दिनों में पारस्परिक निर्भरता बढ़ती जायेगी । इसलिये संयुक्त राष्ट्र संघ का महत्त्व भी निरन्तर बढ़ेगा ।

प्रश्न 35.
अन्तर्राष्ट्रीय मुद्राकोष पर संक्षिप्त टिप्पणी लिखिये।
अथवा
अन्तर्राष्ट्रीय मुद्राकोष क्या है? कितने देश इसके सदस्य हैं?
उत्तर:
अन्तर्राष्ट्रीय मुद्रा कोष: यह संगठन वैश्विक स्तर की वित्त व्यवस्था की देखरेख करता है और मांगे जाने. पर वित्तीय तथा तकनीकी सहायता मुहैया करता है। वैश्विक स्तर की वित्त व्यवस्था का आशय अन्तर्राष्ट्रीय स्तर पर काम करने वाली वित्तीय संस्थाओं और लागू होने वाले नियमों से है। वर्तमान में 189 देश अन्तर्राष्ट्रीय मुद्रा कोष के सदस्य हैं लेकिन हर सदस्य की राय का वजन बराबर नहीं है। समूह -7 के सदस्य (अमरीका, जापान, जर्मनी, फ्रांस, ब्रिटेन, इटली और कनाडा) के पास 41.29 प्रतिशत मत है। अन्य अग्रणी सदस्यों में चीन ( 6.09%), भारत (2.64%), रूस (2.59%), ब्राजील. (2.22%) और सऊदी अरब (2.02%) है। अकेले अमरीका के पास 16.52% मताधिकार है।

प्रश्न 36.
विश्व बैंक की स्थापना कब हुई? इसके प्रमुख कार्यों को संक्षेप में बताइये।
उत्तर:
विश्व बैंक – दूसरे विश्व युद्ध के तुरन्त बाद सन् 1945 में विश्व बैंक की औपचारिक स्थापना हुई। विश्व बैंक के कार्य-विश्व बैंक की गतिविधियाँ प्रमुख रूप से विकासशील देशों से संबंधित हैं। यथा

  1. यह बैंक मानवीय विकास ( शिक्षा, स्वास्थ्य), कृषि और ग्रामीण विकास (सिंचाई, ग्रामीण सेवाएँ), पर्यावरण सुरक्षा (प्रदूषण में कमी, नियमों का निर्माण और उन्हें लागू करना), आधारभूत ढाँचा (सड़क, शहरी विकास, बिजली) तथा सुशासन ( कदाचार का विरोध, विधिक संस्थाओं का विकास) के लिए काम करता है ।
  2. यह अपने सदस्य देशों को आसान ऋण और अनुदान देता है । ज्यादा गरीब देशों को ये अनुदान वापिस नहीं चुकाने पड़ते। इस अर्थ में यह संस्था समकालीन वैश्विक अर्थव्यवस्था को भी प्रभावित करती है।

प्रश्न 37.
अन्तर्राष्ट्रीय संगठन की आवश्यकता के कोई दो बिन्दु बताइये।
उत्तर:
अन्तर्राष्ट्रीय संगठन की आवश्यकता – अन्तर्राष्ट्रीय संगठन की आवश्यकता को निम्नलिखित बिन्दुओं के अन्तर्गत स्पष्ट किया गया है।

  1. समस्याओं के शांतिपूर्ण समाधान के लिए आवश्यक – बातचीत के माध्यम से दो या अधिक देशों के मध्य के झगड़ों और विभेदों को बिना युद्ध के हल करने की दृष्टि से अन्तर्राष्ट्रीय संगठनों की भूमिका महत्त्वपूर्ण होती है। अन्तर्राष्ट्रीय संगठन समस्याओं के शांतिपूर्ण समाधान में सदस्य देशों की मदद करते हैं।
  2. चुनौतीपूर्ण समस्याओं को निपटाने में विभिन्न देशों को मिलकर कार्य करने में सहायता करना- अन्तर्राष्ट्रीय संगठन ऐसी चुनौतीपूर्ण समस्याओं को निपटाने के लिए आवश्यक है, जिनसे निपटने के लिए विभिन्न देशों को मिलाकर सहयोग करना आवश्यक होता है।

प्रश्न 38.
1991 के बाद वैश्विक – व्यवस्था में क्या प्रमुख परिवर्तन आये हैं?’
उत्तर:
1991 के बाद विश्व की राजनीतिक और आर्थिक स्थितियों में निम्नलिखित परिवर्तन आए हैं।

  1. 1991 के बाद शीत युद्ध काल की दो महाशक्तियों – सोवियत संघ और संयुक्त राज्य अमेरिका में से एक महाशक्ति सोवियत संघ का पतन हो गया है तथा वह बिखर गया है।
  2. सोवियत संघ के उत्तराधिकारी राज्य रूस और अमरीका के बीच अब संबंध कहीं ज्यादा सहयोगात्मक हैं।
  3. चीन बड़ी तेजी से एक महाशक्ति के रूप में उभर रहा है।
  4. सोवियत संघ से स्वतंत्र हुए अनेक नये देश संयुक्त राष्ट्र संघ में शामिल हुए हैं।
  5. वर्तमान में विश्व के सामने नयी चुनौतियों की एक पूरी कड़ी विद्यमान है। ये चुनौतियाँ हैं। जनसंहार, गृहयुद्ध, जातीय संघर्ष, आतंकवाद, परंमाणविक प्रसार, जलवायु में बदलाव, पर्यावरण की हानि, महामारी आदि।

प्रश्न 39.
संयुक्त राष्ट्र संघ को अधिक प्रासंगिक बनाने के लिए हाल ही में क्या कदम उठाये गये हैं?
उत्तर:
संयुक्त राष्ट्र संघ को अधिक प्रासंगिक बनाने के लिए हाल ही में निम्नलिखित कदम उठाये गये हैं।

  1. शांति संस्थापक आयोग का गठन किया गया है।
  2. यदि कोई राष्ट्र अपने नागरिकों को अत्याचारों से बचाने में असफल हो जाए तो विश्व बिरादरी इसका उत्तरदायित्व ले- इस बात की स्वीकृति।
  3. मानवाधिकार परिषद की स्थापना
  4. सहस्राब्दि विकास लक्ष्य को प्राप्त करने पर सहमति
  5. हर रूप रीति के आतंकवाद की निंदा करना
  6. एक लोकतंत्र कोष का गठन।

JAC Class 12 Political Science Important Questions Chapter 6 अंतर्राष्ट्रीय संगठन

प्रश्न 40.
सुरक्षा परिषद् के स्थायी व अस्थायी सदस्यों के लिए सुझाये गये कोई चार मानदण्ड बताइये।
उत्तर:
सुरक्षा परिषद् के स्थायी व अस्थायी सदस्यों के लिए चार आवश्यक मानदण्ड ये होने चाहिए:

  1. वह देश क्षेत्र तथा जनसंख्या की दृष्टि से बड़ा देश हो।
  2. वह एक लोकतांत्रिक देश हो।
  3. वह आर्थिक तथा सैनिक शक्ति के रूप में उभर रहा हो तथा संयुक्त राष्ट्र संघ के बजट में अपना नियमित योगदान देता आ रहा हो।
  4. वह संयुक्त राष्ट्र संघ के शांति बहाल करने के प्रयासों में निरन्तर अपनी प्रभावी भूमिका निभाता आ रहा हो।

प्रश्न 41.
संयुक्त राष्ट्र संघ के उद्देश्यों का वर्णन कीजिए।
उत्तर:
संयुक्त राष्ट्र संघ के उद्देश्य: संयुक्त राष्ट्र संघ के प्रमुख उद्देश्य निम्नलिखित हैं।

  1. अन्तर्राष्ट्रीय शांति और सुरक्षा बनाए रखना।
  2. सभी राष्ट्रों के बीच लोगों के समान अधिकार एवं स्वतंत्रता के सिद्धान्तों पर आधारित मैत्रीपूर्ण सम्बन्धों पर विकास करना और विश्व शान्ति को सुदृढ़ बनाने के लिए उचित उपाय करना।
  3. सामाजिक, आर्थिक, सांस्कृतिक और मानवीय क्षेत्रों में अन्तर्राष्ट्रीय सहयोग को प्रोत्साहित व मजबूत करना तथा सबके लिए मानवाधिकारों और मौलिक स्वतंत्रताओं के सम्मान को बढ़ाना और प्रोत्साहन देना।
  4. उपर्युक्त सामूहिक उद्देश्यों की प्राप्ति में राष्ट्रों के कार्यों में समन्वय बनाने के लिए संयुक्त राष्ट्र संघ को केन्द्र बनाना।

प्रश्न 42.
एक विश्व संस्था के रूप में संयुक्त राष्ट्रसंघ की असफलताओं पर प्रकाश डालिये।
उत्तर:
एक विश्व संस्था के रूप में संयुक्त राष्ट्रसंघ की असफलताएँ निम्न हैं-

  1. कुछ मसलों पर जैसे निःशस्त्रीकरण, विकसित और विकासशील देशों के बीच आर्थिक असन्तुलन कम करने में आंशिक सफलता ही मिली।
  2. संयुक्त राष्ट्रसंघ अनेक विवादों को सुलझाने तथा महाशक्तियों की मनमानी को रोकने में असफल रहा है।

प्रश्न 43.
‘पारस्परिक निर्भरता’ से आप क्या समझते हैं?
उत्तर:
पारस्परिक निर्भरता – हालाँकि संयुक्त राष्ट्रसंघ में थोड़ी कमियाँ हैं लेकिन इसका अस्तित्व आवश्यक है क्योंकि इसके बिना दुनिया और बदहाल हो जाएगी। संयुक्त राष्ट्रसंघ विश्व के सात अरब लोगों को एक साथ रहने के लिए मददगार है। आज विभिन्न समाजों और मसलों के बीच तार जुड़ते जा रहे हैं, इसे ही ‘पारस्परिक निर्भरता’ का नाम दिया गया है।

प्रश्न 44.
संयुक्त राष्ट्रसंघ की आर्थिक ओर सामाजिक परिषद् के ऊपर संक्षेप में टिप्पणी लिखिए।
उत्तर:
संयुक्त राष्ट्रसंघ की आर्थिक और सामाजिक परिषद् में कुल 54 सदस्य होते हैं जिनका चुनाव महासभा 2/3 बहुमत से तीन वर्ष के लिए करती है। इसके 1/3 सदस्य प्रतिवर्ष अवकाश ग्रहण करते हैं । इस प्रकार यह परिषद् एक स्थायी संस्था है।

प्रश्न 45.
संयुक्त राष्ट्रसंघ की सफलताओं पर प्रकाश डालिए।
उत्तर:
संयुक्त राष्ट्रसंघ की सफलताओं को निम्न बिन्दुओं द्वारा स्पष्ट किया जा सकता है।

  1. आतंकवाद का विरोध करने में।
  2. निःशस्त्रीकरण।
  3. शांति एवं सुरक्षा की स्थापना करने में।
  4. अन्तर्राष्ट्रीय भावना का विकास करने में।
  5. मानवाधिकारों का संरक्षण करने में।

निबन्धात्मक प्रश्न

प्रश्न 1.
हमें अन्तर्राष्ट्रीय संगठन क्यों चाहिए? स्पष्ट कीजिए।
उत्तर:
अन्तर्राष्ट्रीय संगठन की आवश्यकता हमें अन्तर्राष्ट्रीय संगठन की आवश्यकता है। इसके प्रमुख कारण निम्नलिखित हैं।

  1. समस्याओं के शांतिपूर्ण समाधान हेतु: अन्तर्राष्ट्रीय संगठन देशों की समस्याओं के शांतिपूर्ण समाधान में सदस्य देशों की सहायता करते हैं। अन्तर्राष्ट्रीय संगठन का निर्माण विभिन्न राज्य ही करते हैं और यह उनके मामलों के लिए जवाबदेह होता है। एक बार इसका निर्माण हो जाने के बाद ये सदस्य देशों की समस्याओं के शांतिपूर्ण समाधान में सहायता करते हैं।
  2. सहयोग के उपाय जुटाने में सहायक: एक अन्तर्राष्ट्रीय संगठन सहयोग करने के उपाय तथा सूचनाएँ एकत्रित करने में मदद कर सकता है।
  3. नियमों तथा नौकरशाही की रूपरेखा: एक अन्तर्राष्ट्रीय संगठन नियमों और नौकरशाही की एक रूपरेखा दे सकता है ताकि सदस्यों को यह विश्वास हो कि आने वाली लागत में सबकी समुचित साझेदारी होगी, लाभ का बँटवारा न्यायोचित होगा और यदि कोई सदस्य उस समझौते में शामिल हो जाता है तो वह इस समझौते के नियम और शर्तों का पालन करेगा।
  4. शांति और प्रगति के प्रति मानवता की आशा का प्रतीक: एक अन्तर्राष्ट्रीय संगठन शांति और प्रगति प्रति मानवता की आशा का प्रतीक होता है। वर्तमान में संयुक्त राष्ट्र संघ इसका प्रतीक है। इस संगठन को विश्व भर के अधिकांश लोग एक अनिवार्य संगठन मानते हैं।

JAC Class 12 Political Science Important Questions Chapter 6 अंतर्राष्ट्रीय संगठन

प्रश्न 2.
संयुक्त राष्ट्र संघ के विकास क्रम एवं उसकी स्थापना का वर्णन कीजिये।
उत्तर:
संयुक्त राष्ट्र संघ का विकास क्रम एवं स्थापना: संयुक्त राष्ट्र संघ के विकास क्रम को निम्न प्रकार स्पष्ट किया गया है।

  1. अटलांटिक चार्टर ( अगस्त, 1941 ): 14 अगस्त, 1941 को ब्रिटेन के प्रधानमंत्री चर्चिल और अमरीकी राष्ट्रपति रूजवेल्ट ने विश्व शांति की स्थापना के आधारभूत सिद्धान्तों की व्यवस्था की। इस पर दोनों देशों के अध्यक्षों ने हस्ताक्षर किये। इसे अटलांटिक चार्टर के नाम से जाना जाता है।
  2. संयुक्त राष्ट्र घोषणा-पत्र: धुरी शक्तियों के खिलाफ लड़ रहे 26 मित्र राष्ट्र अटलांटिक चार्टर के समर्थन में वाशिंगटन में मिले और दिसम्बर 1943 में संयुक्त राष्ट्र संघ के घोषणा-पत्र पर हस्ताक्षर किये।
  3. याल्टा सम्मेलन ( फरवरी, 1945 ): तीन बड़े नेताओं ( रूजवेल्ट, स्टालिन और चर्चिल) ने याल्टा सम्मेलन में संयुक्त राष्ट्र संघ के गठन, उसकी सदस्यता एवं प्रकृति पर विचार किया तथा एक सम्मेलन करने का निर्णय किया।
  4. सेनफ्रांसिस्को सम्मेलन (अप्रेल-मई 1945): सेन फ्रांसिस्को सम्मेलन में 50 देशों के प्रतिनिधियों ने भाग लिया। इसमें संयुक्त राष्ट्र संघ का चार्टर तैयार किया गया । 26 जून, 1945 को इस घोषणा पत्र (चार्टर) पर 50 देशों के प्रतिनिधियों ने तथा पोलैंड ने 15 अक्टूबर, 1945 को हस्ताक्षर किये। इस तरह संयुक्त राष्ट्र संघ में 51 मूल संस्थापक सदस्य हैं।
  5. संयुक्त राष्ट्र संघ की स्थापना: 24 अक्टूबर, 1945 को संयुक्त राष्ट्र संघ की स्थापना हुई । इसीलिए 24 अक्टूबर को संयुक्त राष्ट्र संघ का स्थापना दिवस मनाया जाता है। 10 फरवरी, 1946 को संयुक्त राष्ट्र संघ के विभिन्न पदाधिकारियों के चुनाव हुए।

प्रश्न 3.
संयुक्त राष्ट्र संघ के संगठन पर एक निबन्ध लिखिये।
उत्तर:
संयुक्त राष्ट्र संघ का संगठन: संयुक्त राष्ट्र संघ के संगठन का विवेचन निम्नलिखित बिन्दुओं के अन्तर्गत किया गया है।

  • संयुक्त राष्ट्र संघ की सदस्यता: वर्तमान में संयुक्त राष्ट्र संघ के सदस्यों की संख्या 193 है।
  • अधिकारिक भाषाएँ तथा मुख्यालय: सामान्य तौर पर संयुक्त राष्ट्र संघ की कार्यवाही की दो भाषाएँ हैं- अंग्रेजी व फ्रेंच। इनके अतिरिक्त चीनी, अरबी, स्पेनिश तथा रशियन भाषाओं को भी मान्यता प्राप्त है। इसका मुख्यालय न्यूयार्क शहर के मैनहट्टन द्वीप में है।
  • संयुक्त राष्ट्र संघ के मुख्य अंग ( निकाय ) संयुक्त राष्ट्र संघ के प्रमुख 6 अंग हैं। यथा
    1. महासभा (आमसभा ) महासभा संयुक्त राष्ट्र का मुख्य विचार-विनिमय निकाय है। इसमें सभी सदस्य देशों के प्रतिनिधि होते हैं।
    2. सुरक्षा परिषद् सुरक्षा परिषद् में कुल 15 सदस्य होते हैं। इसमें 5 स्थायी तथा 10 अस्थायी होते हैं। 5 स्थायी सदस्य देश हैं। अमरीका, चीन, रूस, ब्रिटेन और फ्रांस पाँचों स्थायी सदस्यों को ‘वीटो’ की शक्ति दी गई है।
    3. आर्थिक व सामाजिक परिषद्: आर्थिक-सामाजिक परिषद् के 54 सदस्य हैं, जिनका चुनाव महासभा 2/3 बहुमत से तीन वर्ष के लिए करती है। इसके 1/3 सदस्य प्रतिवर्ष अवकाश ग्रहण करते हैं। इस प्रकार यह परिषद् एक स्थायी संस्था है।
    4. ट्रस्टीशिप परिषद्: इस परिषद् का उद्देश्य 11 ट्रस्ट क्षेत्रों के प्रशासन की देखभाल करना था। 1994 तक ये . सभी ट्रस्ट स्वतंत्र हो चुके हैं।
    5. अन्तर्राष्ट्रीय न्यायालय: अन्तर्राष्ट्रीय न्यायालय में 15 न्यायाधीश होते हैं जिनका चयन महासभा और सुरक्षा परिषद् द्वारा एक साथ ही किया जाता है। न्यायाधीशों का कार्यकाल 9 वर्ष का होता है। इसका मुख्यालय हेग में है।
    6. सचिवालय: सचिवालय में महासचिव तथा संघ की आवश्यकता के अनुसार कर्मचारी रहते हैं।

प्रश्न 4.
संयुक्त राष्ट्र संघ के सुधारों में भारत की भूमिका का वर्णन करें।
उत्तर:
संयुक्त राष्ट्र संघ के सुधारों में भारत की भूमिका: भारत ने संयुक्त राष्ट्र संघ के ढांचे में सुधार के मुद्दे को निम्नलिखित आधारों पर समर्थन दिया है।

  • (1) संयुक्त राष्ट्र संघ की मजबूती पर बल: बदलते हुए विश्व में संयुक्त राष्ट्र संघ की मजबूती और दृढ़ता जरूरी है।
  • (2) विकास के मुद्दे पर बल: संयुक्त राष्ट्र संघ विभिन्न देशों के बीच सहयोग बढ़ाने और विकास को बढ़ावा देने में ज्यादा बड़ी भूमिका निभाए ।
  • (3) सुरक्षा परिषद् की संरचना में सुधार किया जाये:  इस संदर्भ में भारत के प्रमुख तर्क अग्रलिखित हैं।
    1. सुरक्षा परिषद् की संरचना प्रतिनिधित्वमूलक हो:  भारत का तर्क है कि सुरक्षा परिषद् का विस्तार करने पर वह ज्यादा प्रतिनिधिमूलक होगी तथा उसे विश्व – बिरादरी का अधिक समर्थन मिलेगा।
    2. सुरक्षा परिषद् में विकासशील देशों की संख्या बढ़ायी जाये – संयुक्त राष्ट्र संघ की आम सभा में ज्यादातर विकासशील सदस्य देश हैं। इसलिए सुरक्षा परिषद् में उनका यथोचित प्रतिनिधित्व होना चाहिए।
    3. सुरक्षा परिषद् की गतिविधियों का दायरा बढ़ा है: सुरक्षा परिषद् के काम-काज की सफलता विश्व- बिरादरी के समर्थन पर निर्भर है। इस कारण सुरक्षा परिषद् के पुनर्गठन की कोई योजना व्यापक धरातल पर बननी चाहिए।
    4. भारत को सुरक्षा परिषद् का स्थायी सदस्य बनाया जाये:  भारत स्वयं भी पुनर्गठित सुरक्षा परिषद् में एक स्थायी सदस्य बनना चाहता है।

JAC Class 12 Political Science Important Questions Chapter 6 अंतर्राष्ट्रीय संगठन

प्रश्न 5.
संयुक्त राष्ट्रसंघ के महासचिव के पद पर बैठने वाले दो एशियाई व्यक्तियों के नाम लिखिए। उनके कार्यकाल के दौरान उनके कार्यों का उल्लेख कीजिए।
उत्तर:
संयुक्त राष्ट्रसंघ के महासचिव के पद पर बैठने वाले दो एशियाई व्यक्ति हैं।
1. यू थांट-यू थांट 1961 से 1971 तक संयुक्त राष्ट्रसंघ के महासचिव के पद पर बने रहे। ये बर्मा (म्यांमार) से थे। पेशे से ये शिक्षक और राजनयिक थे। अपने महासचिव के कार्यकाल के दौरान इन्होंने क्यूबा के मिसाइल – संकट के समाधान और कांगो के संकट की समाप्ति के लिए प्रयास किए। साइप्रस में संयुक्त राष्ट्रसंघ की सेना बहाल की। वियतनाम युद्ध के दौरान अमरीका की आलोचना की।

2. बान की मून-बान की मून कोरिया गणराज्य से थे और 2007 से 2016 तक संयुक्त राष्ट्रसंघ के महासचिव रहे। ये संयुक्त राष्ट्रसंघ के आठवें महासचिव थे। महासचिव के पद पर बैठने वाले ये दूसरे एशियाई हैं। अपने कार्यकाल के दौरान इन्होंने जलवायु परिवर्तन पर विश्व का ध्यान खींचा। इनका ध्यान सहस्राब्दि विकास लक्ष्य और सतत विकास लक्ष्य की तरफ रहा। इन्होंने यूएन वीमेन के निर्माण में महत्त्वपूर्ण भूमिका निभाई । युद्ध वियोजन और परमाणु निरस्त्रीकरण पर जोर दिया।

प्रश्न 6.
1997 के बाद के सालों में सुरक्षा परिषद् की स्थायी और अस्थायी सदस्यता हेतु नए मानदंड सुझाए · गए, इनकी संक्षेप में व्याख्या कीजिए।
उत्तर:
1997 के बाद के सालों में सुरक्षा परिषद् की स्थायी और अस्थायी सदस्यता हेतु नये मानदंड सुझाए गए। इस सुझावों के अनुसार नए सदस्य को-

  1. बड़ी आर्थिक ताकत होना चाहिए।
  2. बड़ी सैन्य ताकत होना चाहिए।
  3. संयुक्त राष्ट्रसंघ के बजट में ऐसे देशों का योगदान ज्यादा होना चाहिए।
  4. आबादी के लिहाज से वह राष्ट्र बड़ा होना चाहिए।
  5. वह देश लोकतंत्र और मानवाधिकार का सम्मान करता हो।
  6. वह देश ऐसा हो जो अपने भूगोल, अर्थव्यवस्था और संस्कृति के लिहाज से विश्व की विविधता में नुमाइंदगी करता हो।

प्रश्न 7.
एक विश्व संस्था के रूप में संयुक्त राष्ट्र संघ की सफलताओं और असफलताओं पर एक लेख लिखिए।
उत्तर:
एक विश्व संस्था के रूप में संयुक्त राष्ट्र संघ की प्रमुख सफलताएँ व असफलताएँ निम्नलिखित हैं- संयुक्त राष्ट्र संघ की सफलताएँ

  1. शांति एवं सुरक्षा की स्थापना: संयुक्त राष्ट्र संघ ने अनेक अन्तर्राष्ट्रीय विवादों, मतभेदों व तनावों को अनेक बार युद्ध में परिणत होने से बचाया है।
  2. आतंकवाद का विरोध: संयुक्त राष्ट्र महासभा ने सब प्रकार के आतंकवाद के विरुद्ध एक प्रस्ताव पारित कर विश्व – -समुदाय से आतंकवाद की चुनौती को मिलकर सामना करने का आग्रह किया है।
  3. निःशस्त्रीकरण: संयुक्त राष्ट्र संघ ने निःशस्त्रीकरण के लिए अनेक प्रयास किये हैं।
  4. साम्राज्यवाद और उपनिवेशवाद का विरोध: संयुक्त राष्ट्र संघ ने साम्राज्यवाद और उपनिवेशवाद को समाप्त करने में बहुत सहायता दी है।
  5. आर्थिक एवं सामाजिक क्षेत्र में सफलताएँ – आर्थिक व सामाजिक असन्तोष, भूख, दरिद्रता, निरक्षरता आदि को दूर करने में संयुक्त राष्ट्र संघ ने निरन्तर प्रयत्न किये हैं।
  6. अन्तर्राष्ट्रीय भावना का विकास-संयुक्त राष्ट्र संघ निरन्तर अन्तर्राष्ट्रीय सहयोग, सद्भावना और सहअस्तित्व की भावना का विकास कर रहा है।
  7. मानवाधिकारों का संरक्षक: मानवाधिकार आयोग के माध्यम से संयुक्त राष्ट्र संघ विश्व में मानवाधिकारों के संरक्षक की भूमिका निभा रहा है।

संयुक्त राष्ट्र संघ की असफलताएँ

  1. आंशिक सफलताएँ: संयुक्त राष्ट्र संघ को निःशस्त्रीकरण, हिन्द महासागर को शांति क्षेत्र बनाने, विकसित और विकासशील देशों के बीच आर्थिक असन्तुलन को कम करने, समुद्री सम्पदा के उचित दोहन आदि मामलों में आंशिक सफलता ही मिली है।
  2. पूर्णतः असफलताएँ: संयुक्त राष्ट्र संघ अनेक विवादों को सुलझाने तथा महाशक्तियों की मनमानी रोकने में असफल रहा है।

प्रश्न 8.
एक ध्रुवीय विश्व में संयुक्त राष्ट्र संघ क्या अमरीका की मनमानी को रोक सकता है? यदि नहीं तो क्यों? एक ध्रुवीय विश्व में संयुक्त राष्ट्र संघ की प्रासंगिकता को स्पष्ट कीजिये।
उत्तर:
एक – ध्रुवीय विश्व में संयुक्त राष्ट्र संघ एक-ध्रुवीय विश्व में संयुक्त राष्ट्र अमरीका की मनमानी को नहीं रोक सकता क्योंकि

  1. अपनी सैन्य और आर्थिक ताकत के बल पर अमरीका संयुक्त राष्ट्र संघ या किसी अन्य अन्तर्राष्ट्रीय संगठन की अनदेखी कर सकता है।
  2. संयुक्त राष्ट्र संघ के भीतर उसके बजट, अमरीकी भू क्षेत्र में संयुक्त राष्ट्र संघ की स्थिति आदि के कारण अमरीका का अत्यधिक प्रभाव है।
  3. सुरक्षा परिषद् का वह स्थायी सदस्य है तथा उसके पास वीटो की शक्ति है।
  4. अमरीका अपनी ताकत और निषेधाधिकार के कारण संयुक्त राष्ट्र संघ के महासचिव के चयन में भी दखल रखता है।
  5. अमरीका अपनी सैन्य और आर्थिक शक्ति के बल पर विश्व बिरादरी में फूट डाल सकता है।

एक – ध्रुवीय विश्व में संयुक्त राष्ट्र की प्रासंगिकता: एक – ध्रुवीय विश्व में संयुक्त राष्ट्र संघ की प्रासंगिकता को निम्न प्रकार बताया जा सकता है।

  1. संयुक्त राष्ट्र संघ क्षेत्रीय या विश्वव्यापी विवाद के मुद्दों पर सभी देशों के बीच बातचीत का एक मंच प्रस्तुत करता है।
  2. अमरीकी नेता समझते हैं कि झगड़ों और सामाजिक-आर्थिक विकास के मुद्दे पर संयुक्त राष्ट्र संघ के जरिये 193 राष्ट्रों को एक साथ किया जा सकता है।
  3. शेष विश्व के लिए संयुक्त राष्ट्र संघ एक ऐसा मंच है जहाँ अमरीकी रवैये और नीतियों पर कुछ अंकुश लगाया जा सकता है।

JAC Class 12 Political Science Important Questions Chapter 6 अंतर्राष्ट्रीय संगठन

प्रश्न 9.
‘विश्व व्यापार संगठन’ पर टिप्पणी लिखिए।
उत्तर:
विश्व व्यापार संगठन (WTO):
यह एक अंतर्राष्ट्रीय संगठन है जो कि वैश्विक व्यापार के नियमों को तय करता है। इस संगठन की स्थापना 1995 में हुई थी। यह संगठन ‘जनरल एग्रीमेंट ऑन ट्रेड एंड टैरिफ के उत्तराधिकारी के रूप में काम करता है जो कि दूसरे विश्वयुद्ध के बाद अस्तित्व में आया था । 29 जुलाई, 2016 तक इसके सदस्यों की संख्या 164 थी।

इस संगठन में हर फैसला सभी सदस्यों की सहमति से किया जाता है लेकिन अमरीका, यूरोपीय संघ तथा जापान जैसी बड़ी आर्थिक शक्तियाँ विश्व व्यापार संगठन में व्यापार के नियमों को इस तरह बना दिया है जिससे उनका हित सधता हो। विकासशील देशों की अक्सर शिकायत रहती है इस संगठन की कार्यविधि पारदर्शी नहीं है और बड़ी आर्थिक ताकतें इन देशों को पीछे ढकेलती है।’

प्रश्न 10.
संयुक्त राष्ट्रसंघ के सुरक्षा परिषद् की संरचना का वर्णन कीजिए। इसके स्थायी और अस्थायी सदस्यों को दिए गए विशेषाधिकारों में क्या अंतर है।
उत्तर:
सुरक्षा परिषद् में पाँच स्थायी और दस अस्थायी सदस्य हैं।

  1. स्थायी सदस्य: दूसरे विश्वयुद्ध के बाद दुनिया में स्थिरता कायम करने के लिए संयुक्त राष्ट्रसंघ के घोषणा पत्र में पाँच स्थायी सदस्यों को विशेष हैसियत दी गई। पाँच स्थायी सदस्यों की सदस्यता स्थायी होगी और उन्हें ‘वीटो ‘ का अधिकार होगा।
  2. अस्थायी सदस्य: अस्थायी सदस्य दो वर्षों के लिए चुने जाते हैं और इस अवधि के बाद उनकी जगह नए सदस्यों का चयन होता है। दो साल की अवधि तक अस्थायी सदस्य रहने के तत्काल बाद किसी देश को फिर से इस पद के लिए नहीं चुना जा सकता अस्थायी सदस्यों का निर्वाचन इस प्रकार होता है कि विश्व के सभी महादेशों का प्रतिनिधित्व हो सके अस्थायी देशों को वीटो का अधिकार नहीं है।

प्रश्न 11.
कुछ देश संयुक्त राष्ट्र सुरक्षा परिषद् के स्थायी सदस्य के रूप में भारत को शामिल करने के मुद्दे पर सवाल क्यों उठाते हैं? व्याख्या कीजिए।
अथवा
भारत या किसी और देश के लिए भविष्य में संयुक्त राष्ट्रसंघ की सुरक्षा परिषद् का स्थायी सदस्य बन पाना मुश्किल लगता है। क्यों?
उत्तर:
यद्यपि भारत चाहता है कि वह संयुक्त राष्ट्रसंघ की सुरक्षा परिषद् का स्थायी सदस्य बने तथा उसे भी निषेधाधिकार प्राप्त हो लेकिन कुछ देश सुरक्षा परिषद् में भारत की स्थायी सदस्यता का विरोध करते हैं। यथा।

  1. सिर्फ पड़ोसी पाकिस्तान ही नहीं, जिनके साथ भारत के संबंध दिक्कततलब रहे हैं, बल्कि कुछ और देश भी चाहते हैं कि भारत को सुरक्षा परिषद् में वीटोधारी सदस्य के रूप में ना शामिल किया जाए।
  2. कुछ देश भारत के परमाणु हथियारों को लेकर चिंतित हैं।
  3. कुछ देशों का मानना है कि पाकिस्तान के साथ संबंधों में कठिनाई के कारण भारत स्थायी सदस्य के रूप में प्रभावी नहीं रहेगा।
  4. कुछ अन्य देशों की राय है कि उभरती हुई ताकत के रूप में अन्य देशों जैसे कि -ब्राजील, जर्मनी, जापान और शायद दक्षिण अफ्रीका को भी शामिल करना पड़ेगा जिसका ये देश विरोध करते हैं।
  5. कुछ देशों का मत है कि यदि सुरक्षा परिषद् में किसी तरह का विस्तार होता है तो अफ्रीका और दक्षिणी अमरीका को जरूर प्रतिनिधित्व मिलना चाहिए क्योंकि मौजूदा सुरक्षा परिषद् में इन्हीं महादेशों की नुमाइंदगी नहीं है। उपर्युक्त वजहों को देखते हुए यह अनुमान लगाया जा सकता है कि भारत या किसी और देश के लिए निकट भविष्य में संयुक्त राष्ट्रसंघ की सुरक्षा परिषद् का स्थायी सदस्य बन पाना मुश्किल है।

प्रश्न 12.
“संयुक्त राष्ट्रसंघ एक अपरिहार्य संगठन है।” स्पष्ट कीजिए।
उत्तर:
संयुक्त राष्ट्रसंघ एक अपरिहार्य संगठन है। इस कथन को निम्न तर्कों से स्पष्ट किया जा सकता है।

  1. संयुक्त राष्ट्रसंघ ही वो माध्यम है जिसके द्वारा अमरीका जो महाशक्ति है और बाकी विभिन्न देशों को विभिन्न मुद्दों पर चर्चा करने के लिए एकत्रित किया जा सकता है।
  2. संयुक्त राष्ट्रसंघ ने संघर्ष और आर्थिक विकास से निपटने के लिए 190 से अधिक देशों को एक साथ लाने का कार्य किया है।
  3. संयुक्त राष्ट्रसंघ अमरीका के अलावा बाकी देशों को बातचीत करने का मंच प्रदान करवाता है जहाँ अमरीकी दृष्टिकोण और नीतियों को संशोधित करना संभव हो। हालाँकि अमरीका के खिलाफ देशों को शायद ही एकजुट किया जाता है। संयुक्त राष्ट्र एक ऐसा मंच है जहाँ अमरीकी दृष्टिकोण तथा नीतियों के खिलाफ बहस सुनी जाती है और उसके अनुरूप बदलाव किया जा सकता है।
  4. वैश्वीकरण को इस आधुनिक दुनिया में संयुक्त राष्ट्रसंघ में कमियों के बावजूद इसका अस्तित्व आवश्यक है। आज़ विभिन्न समाजों और मसलों के बीच तार जुड़ते जा रहे हैं। इसे ‘पारस्परिक निर्भरता’ का नाम दिया गया है। प्रौद्योगिकी यह सिद्ध कर रही है कि आने वाले समय में विश्व में पारस्परिक निर्भरता बढ़ती जाएगी। ऐसी स्थिति में संयुक्त राष्ट्रसंघ उन तरीकों को खोजने में मददगार होगा जो अंतर्राष्ट्रीय समुदाय के हितों के अनुरूप होगा।

JAC Class 12 Political Science Important Questions Chapter 5 समकालीन दक्षिण एशिया

Jharkhand Board JAC Class 12 Political Science Important Questions Chapter 5 समकालीन दक्षिण एशिया Important Questions and Answers.

JAC Board Class 12 Political Science Important Questions Chapter 5 समकालीन दक्षिण एशिया

बहुचयनात्मक प्रश्न

1. दक्षिण एशिया में सबसे बड़ा देश है।
(अ) भारत
(ब) पाकिस्तान
(स) श्रीलंका
(द) बांग्लादेश
उत्तर:
(अ) भारत

2. दक्षिण एशियाई सहयोग संगठन (सार्क) का गठन किया गया।
(अ) 1985 में
(ब) 1986 में
(स) 1990 में
(द) 1991 में
उत्तर:
(अ) 1985 में

3. कश्मीर को लेकर भारत और पाकिस्तान के बीच लड़ाई कब हुई ?
(अ) 1954
(ब) 1947
(स) 1966
(द) 1955
उत्तर:
(ब) 1947

4. ‘साफ्टा’ सम्बन्धित है।
(अ) आसियान से
(ब) सार्क से
(स) हिमतेक्ष से
(द) ओपेक से
उत्तर:
(ब) सार्क से

5. आजादी के बाद से श्रीलंका की राजनीति पर किस समुदाय का दबदबा रहा?
(अ) तमिल
(ब) हिन्दू
(स) मुस्लिम
(द) सिंहली
उत्तर:
(द) सिंहली

JAC Class 12 Political Science Important Questions Chapter 5 समकालीन दक्षिण एशिया

6. भारत और पाकिस्तान के बीच 1948 में हुए युद्ध के फलस्वरूप किस प्रांत के दो हिस्से हुए?
(अ) बांग्लादेश
(ब) बंगाल
(स) कश्मीर
(द) असम
उत्तर:
(स) कश्मीर

7. 1998 में पाकिस्तान ने कहाँ पर परमाणु परीक्षण किया?
(अ) कश्मीर
(ब) पश्चिमी पाकिस्तान
(स) चगाई पहाड़ी
(द) पोखरण
उत्तर:
(स) चगाई पहाड़ी

8. बांग्लादेश एक स्वतन्त्र सम्प्रभु राष्ट्र बना।
(अ) 1947 में
(ब) 1948 में
(स) 1949 में
(द) 1971 में
उत्तर:
(द) 1971 में

रिक्त स्थानों की पूर्ति कीजिए

1. 1998 में भारत ने …………………………. में परमाणु परीक्षण किया।
उत्तर:
पोखरण

2. …………………… के दशक में भारत तथा पाकिस्तान ने परमाणु हथियार और ऐसे हथियारों को एक-दूसरे पर दागने ………………………की क्षमता वाले मिसाइल हासिल कर लिए।
उत्तर:
1990

3. 1960 में विश्व बैंक की मदद से भारत और पाकिस्तान ने …………………… पर दस्तखत किए।
उत्तर:
सिंधु जल संधि

4. भूटान ………………………. में संवैधानिक राजतंत्र बना।
उत्तर:
2008

5. मालदीव ………………………. में गणतंत्र बना और यहाँ शासन की ……………………… प्रणाली अपनायी गयी।
उत्तर:
1968, अध्यक्षात्मक

6. …………………………. से ………………………………. तक बांग्लादेश पाकिस्तान का अंग था।
उत्तर:
1947, 1971

अतिलघूत्तरात्मक प्रश्न

प्रश्न 1.
स्वतंत्रता के बाद दक्षिण एशिया के किन दो देशों में आज तक लोकतांत्रिक व्यवस्था कायम है?
उत्तर:
भारत और श्रीलंका में।

प्रश्न 2.
भारत में स्वतंत्रता के बाद किस प्रकार की शासन प्रणाली अपनाई गई?
उत्तर:
लोकतांत्रिक शासन प्रणाली।

प्रश्न 3.
भारत और पाकिस्तान के बीच हुए 1971 के युद्ध का क्या परिणाम निकला?
उत्तर:
1971 में भारत और पाकिस्तान के बीच हुए युद्ध से पूर्वी पाकिस्तान ‘बांग्लादेश’ के रूप में नया स्वतंत्र देश

प्रश्न 4.
भारत के पड़ौसी देश कौन-कौन से हैं?
उत्तर:
भारत के पड़ौसी देश हैं।

  1. पाकिस्तान,
  2. नेपाल,
  3. भूटान,
  4. बांग्लादेश,
  5. श्रीलंका,
  6. चीन आदि।

प्रश्न 5.
भारत और पाकिस्तान के बीच शिमला समझौता कब हुआ?
उत्तर:
जुलाई, 1972 में भारत-पाक के बीच शिमला समझौता हुआ।

JAC Class 12 Political Science Important Questions Chapter 5 समकालीन दक्षिण एशिया

प्रश्न 6.
श्रीलंका को आजादी के बाद से ही किस कठिन चुनौती का सामना करना पड़ रहा है?
उत्तर:
जातीय संघर्ष का।

प्रश्न 7.
दक्षिण एशियायी देशों द्वारा बहुस्तरीय साधनों से आपस में सहयोग के लिए उठाया गया कदम कौनसा है?’ – यह बड़ा कदम है। दक्षेस (सार्क) की स्थापना का।
उत्तर:

प्रश्न 8.
12वें दक्षेस सम्मेलन में किस संधि पर हस्ताक्षर किये गये?
उत्तर:
मुक्त व्यापार संधि पर।

प्रश्न 9.
दक्षेस के देशों ने साफ्टा (SAFTA) पर कब हस्ताक्षर किये?
उत्तर:
दक्षेस के देशों ने सन् 2004 में साफ्टा पर हस्ताक्षर किये।

प्रश्न 10.
साफ्टा में क्या वायदा किया गया है?
उत्तर:
‘साफ्टा’ में पूरे दक्षिण एशिया के लिए मुक्त व्यापार क्षेत्र बनाने का वायदा है।

प्रश्न 11.
सार्क का पूरा नाम लिखिये।
उत्तर:
सार्क का पूरा नाम है। साउथ एशियन एसोसियेशन फॉर रीजनल कोऑपरेशन (दक्षिण एशियाई क्षेत्रीय सहयोग संघ)।

प्रश्न 12.
सार्क में कितने देश हैं?
उत्तर:
सार्क में कुल आठ देश हैं। ये हैं- भारत, पाकिस्तान, नेपाल, बांग्लादेश, श्रीलंका, भूटान, मालदीव और अफगानिस्तान।

JAC Class 12 Political Science Important Questions Chapter 5 समकालीन दक्षिण एशिया

प्रश्न 13.
दक्षिण एशिया का इलाका एक विशिष्ट प्राकृतिक क्षेत्र के रूप में नजर आता है। क्यों?
उत्तर:
उत्तर की विशाल हिमालय पर्वत श्रृंखला, दक्षिण का हिन्द महासागर, पश्चिम का अरब सागर और पूरब में मौजूद बंगाल की खाड़ी से यह इलाका एक विशिष्ट प्राकृतिक क्षेत्र के रूप में नजर आता है। उत्तर- ‘साफ्टा’ का पूरा नाम है।’ दक्षिण एशियाई मुक्त व्यापार क्षेत्र समझौता’ (साउथ एशियन फ्री ट्रेड एरिया)।

प्रश्न 14.
दक्षेस को अधिक प्रभावी बनाने हेतु कोई दो सुझाव दीजिये।
उत्तर:

  1. महाशक्तियों को इस क्षेत्र से दूर रखा जाये।
  2. अन्तर्राष्ट्रीय मंचों पर दक्षेस देशों द्वारा सर्वसम्मत दृष्टिकोण अपनाया जाये।

प्रश्न 15.
ताशकंद समझौता कब और किसके मध्य हुआ?
उत्तर:
सन् 1966 में भारत और पाकिस्तान के मध्य ताशकंद समझौता हुआ।

प्रश्न 16.
दो समझौतों के नाम लिखिये जिन्हें भारत ने पाकिस्तान के साथ किये।
उत्तर:

  1. 1965 में ताशकंद समझौता
  2. 1971 में शिमला समझौता।

प्रश्न 17.
शिमला समझौते के दो प्रावधान लिखिये।
उत्तर:

  1. दोनों देश अपने मतभेदों को द्विपक्षीय वार्ता द्वारा शांतिपूर्ण ढंग से हल करने का प्रयास करेंगे।
  2. दोनों देश एक-दूसरे के विरुद्ध बल का प्रयोग नहीं करेंगे।

प्रश्न 18.
भारत और बांग्लादेश के बीच तनाव के दो मुख्य बिन्दु बताइये।
उत्तर:

  1. कमा शरणार्थियों की समस्या।
  2. गंगा – जल के वितरण की समस्या।

प्रश्न 19.
भारत-पाक मैत्री के मार्ग में आने वाली किन्हीं दो बाधाओं का उल्लेख कीजिये।
उत्तर:
ये दो बाधायें हैं।

  1. चीन का पाकिस्तान को सामरिक सहयोग तथा
  2. भारतीय आतंकवादियों को प्रशिक्षण व सहायता देना।

JAC Class 12 Political Science Important Questions Chapter 5 समकालीन दक्षिण एशिया

प्रश्न 20.
भारत को किन दो कठिनाइयों का सामना पड़ौसियों के साथ अच्छे सम्बन्ध बनाने में करना पड़ता है?
उत्तर:

  1. पड़ौसी देशों द्वारा भारतीय आतंकवादियों को संरक्षण, प्रशिक्षण व सहयोग देना।
  2. सीमा विवाद तथा शरणार्थियों की समस्या।

प्रश्न 21.
राजनीतिक प्रणाली की दृष्टि से भारत और श्रीलंका की किसी एक समानता को बताइये।
उत्तर:
भारत और श्रीलंका में ब्रिटेन से आजाद होने के बाद, लोकतांत्रिक व्यवस्था सफलतापूर्वक कायम है। एक राष्ट्र के रूप में भारत और श्रीलंका हमेशा लोकतांत्रिक रहे हैं।

प्रश्न 22.
पाकिस्तान में शीत युद्ध के बाद के सालों में कौनसी दो लोकतांत्रिक सरकारें बनीं?
उत्तर:
पाकिस्तान में शीत युद्ध के बाद के सालों में लगातार ये दो लोकतांत्रिक सरकारें बनीं

  1. बेनजीर भुट्टो के नेतृत्व में बनी सरकार और
  2. नवाज शरीफ के नेतृत्व में बनी सरकार।

प्रश्न 23.
दक्षिण एशिया के दो सबसे छोटे देशों के नाम लिखिये।
उत्तर:
दक्षिण एशिया के दो सबसे छोटे देश हैं। मालदीव और भूटान।

प्रश्न 24.
मालदीव में 1968 के बाद कौनसी शासन प्रणाली अपनाई गई?
अथवा
मालदीव में कैसी शासन प्रणाली है?
उत्तर:
1968 से मालदीव में लोकतन्त्र की अध्यक्षात्मक शासन प्रणाली अपनाई गई है।

प्रश्न 25.
दक्षिण एशियायी देशों की जनता किस प्रकार के शासन को वरीयता देती है?
उत्तर:
दक्षिण एशियायी देशों के लोग लोकतंत्र को वरीयता देते हैं।

प्रश्न 26.
दक्षेस का कार्यालय कहाँ स्थित है?
उत्तर:
दक्षेस का कार्यालय नेपाल की राजधानी काठमाण्डू में स्थित है।

प्रश्न 27.
भारत के उत्तर में किन्हीं दो सदस्य देशों का उल्लेख कीजिये जो सार्क में सम्मिलित हैं।
उत्तर:
भारत के उत्तर में स्थित नेपाल और भूटान सार्क के सदस्य देश हैं।

प्रश्न 28.
भारत के दक्षिण में सार्क के किन्हीं दो सदस्य देशों का उल्लेख कीजिये।
उत्तर:
श्रीलंका और मालदीव

JAC Class 12 Political Science Important Questions Chapter 5 समकालीन दक्षिण एशिया

प्रश्न 29.
आर्थिक वैश्वीकरण का दक्षिण एशिया पर क्या सकारात्मक प्रभाव पड़ा?
उत्तर:
आर्थिक वैश्वीकरण से दक्षिण एशिया में मुक्त व्यापार को बढ़ावा मिला तथा आर्थिक सुविधाओं में गुणात्मक सुधार हुआ।

प्रश्न 30.
नेपाल को धर्मनिरपेक्ष राज्य में कब परिवर्तित किया गया?
उत्तर:
नेपाल को 17 मई, 2006 को एक धर्मनिरपेक्ष राज्य में परिवर्तित किया गया।

प्रश्न 31.
श्रीलंका में जातीय संघर्ष के कोई दो कारण बताइये।
उत्तर:

  1. श्रीलंका में सिंहली जाति बहुसंख्या में है।
  2. उसमें बहुसंख्यकवाद की भावना है जो अल्पसंख्यक तमिलों की उपेक्षा करती है।

प्रश्न 32
भारत द्वारा शांति सेना कब और किस देश में भेजी गई?
उत्तर:
1987 में श्रीलंका में।

प्रश्न 33.
श्रीलंका के जातीय समुदायों के नाम लिखिये।
उत्तर:
श्रीलंका के जातीय समुदाय ये हैं

  1. सिंहली
  2. श्रीलंकाई तमिल
  3. भारतवंशी तमिल तथा
  4. मुसलमान।

JAC Class 12 Political Science Important Questions Chapter 5 समकालीन दक्षिण एशिया

प्रश्न 34.
सियाचीन की समस्या किन देशों के बीच मतभेद का प्रमुख कारण है?
उत्तर:
भारत और पाकिस्तान के बीच।

प्रश्न 35.
फरक्का संधि पर हस्ताक्षर कब और किनके बीच हुआ था?
उत्तर:
फरक्का संधि पर हस्ताक्षर दिसंबर, 1996 में भारत और बांग्लादेश के बीच हुआ था।

प्रश्न 36.
1975 में बांग्लादेश के संविधान में क्या संशोधन किया गया?
उत्तर:
1975 में शेख मुजीबुर्रहमान ने संविधान में संशोधन कराया और संसदीय प्रणाली की जगह अध्यक्षात्मक शासन प्रणाली को मान्यता मिली।

प्रश्न 37.
श्रीलंका में पैदा हुए जान्ति संघर्ष का निवारण करने हेतु किन दो स्कैंडिनेवियाई देशों ने मध्यस्थ की भूमिका निभाई?
उत्तर:
नार्वे और आइसलैंड।

लघूत्तरात्मक प्रश्न

प्रश्न 1.
दक्षिण एशिया से क्या आशय है?
उत्तर:
सामान्यतः दक्षिण एशिया प्रदेश का प्रयोग सात देशों

  1. बांग्लादेश,
  2. भूटान,
  3. भारत,
  4. मालदीव,
  5. नेपाल,
  6. पाकिस्तान और
  7. श्रीलंका के लिए किया जाता है। इसमें जब-तब अफगानिस्तान और म्यांमार को भी शामिल किया जाता है।

प्रश्न 2.
पाकिस्तान में लोकतंत्र के स्थायी न बन पाने के क्या कारण हैं?
उत्तर:
पाकिस्तान में लोकतंत्र के स्थायी न बन पाने के प्रमुख कारण ये हैं।

  1. यहाँ सेना, धर्मगुरु और भू-स्वामी अभिजनों का सामाजिक दबदबा है।
  2. भारत के साथ निरन्तरं तनातनी रहने के कारण सेना समर्थक समूह अधिक मजबूत है।
  3. अमरीका तथा अन्य पश्चिमी देशों ने अपने स्वार्थ पूर्ति हेतु पाकिस्तान में सैनिक शासन को बढ़ावा दिया।

प्रश्न 3.
पूर्वी पाकिस्तान के लोग मूलतः पश्चिमी पाकिस्तान के विरोधी क्यों थे?
उत्तर:
पूर्वी पाकिस्तान क्षेत्र के लोग पश्चिमी पाकिस्तान के दबदबे और अपने ऊपर उर्दू भाषा को लादने के खिलाफ थे। पाकिस्तान के निर्माण के तुरंत बाद ही यहाँ के लोगों ने बंगाली संस्कृति और भाषा के साथ किये जा रहे दुर्व्यवहार के खिलाफ विरोध जताना शुरू कर दिया था ।

प्रश्न 4.
किन कारणों की वजह से अमरीका तथा अन्य पश्चिमी देश पाकिस्तान को ‘पश्चिम’ तथा दक्षिण एशिया में पश्चिमी हितों को रखवाला मानते हैं?
उत्तर:
पाकिस्तान में सेना, धर्मगुरु और भूस्वामी अभिजनों का सामाजिक दबदबा है। परिणामतः कई बार निर्वाचित सरकारों को गिराकर सैनिक शासन कायम हुआ । यद्यपि पाकिस्तान में लोकतंत्र का जज्बा मजबूती के साथ कायम रहा है तथापि लोकतंत्र पूरी तरह से सफल नहीं हो पाया। लोकतांत्रिक शासन के लिए पाकिस्तान को कोई खास अंतर्राष्ट्रीय समर्थन नहीं मिलता। इस वजह से भी सेना को अपना प्रभुत्व कायम करने के लिए बढ़ावा दिया।

अमरीका तथा अन्य पश्चिमी देशों ने अपने-अपने स्वार्थों की वजह से भी पाकिस्तान में सैनिक शासन को प्रोत्साहन दिया। इन देशों को विश्वव्यापी इस्लामी आतंकवाद से भय लगता है कि पाकिस्तान के परमाण्विक हथियार आतंकवादी समूहों के हाथ न लग जाएँ। उपरोक्त बातों के मद्देनजर पाकिस्तान को ये देश ‘पश्चिम’ तथा दक्षिण एशिया में पश्चिमी हितों का रखवाला मानते हैं।

JAC Class 12 Political Science Important Questions Chapter 5 समकालीन दक्षिण एशिया

प्रश्न 5.
दक्षेस से आप क्या समझते हैं?
उत्तर:
दक्षेस से आशय है। दक्षिण एशिया क्षेत्रीय सहयोग संगठन। यह दक्षिण एशिया के आठ देशों:

  1. भारत,
  2. पाकिस्तान,
  3. बांग्लादेश,
  4. श्रीलंका,
  5. नेपाल,
  6. भूटान,
  7. मालदीव,
  8. अफगानिस्तान- का एक क्षेत्रीय आर्थिक संगठन है जिसकी स्थापना इन देशों ने आपसी सहयोग बढ़ाने के उद्देश्य से की है।

प्रश्न 6.
लिट्टे क्या है?
उत्तर:
लिट्टे का पूरा नाम है- लिबरेशन टाइगर्स ऑफ तमिल ईलम । यह 1983 के बाद से श्रीलंका में सक्रिय एक उग्र तमिल संगठन है जो श्रीलंका के तमिलों के लिए एक अलग देश की मांग को लेकर श्रीलंकाई सेना के साथ सशस्त्र संघर्ष कर रहा है।

प्रश्न 7.
श्रीलंका की प्रमुख सफलताएँ क्या हैं?
उत्तर:
श्रीलंका की प्रमुख सफलताएँ निम्न हैं।

  1. श्रीलंका ने अच्छी आर्थिक वृद्धि और विकास के उच्च स्तर को हासिल किया है।
  2. इसने जनसंख्या की वृद्धि दर पर सफलतापूर्वक नियंत्रण स्थापित किया है।
  3. दक्षिण एशियायी देशों में सबसे पहले श्रीलंका ने ही आर्थिक उदारीकरण किया।
  4. श्रीलंका में निरन्तर लोकतांत्रिक व्यवस्था कायम रही है।

प्रश्न 8.
भारत-श्रीलंका की दो समान विशेषताएँ लिखिये।
उत्तर:

  1. भारत तथा श्रीलंका दोनों ही ब्रिटेन के अधीन रहे तथा कुछ माह के अन्तराल से इन्हें स्वतंत्रता मिली – भारत 1947 में तथा श्रीलंका 1948 में स्वतंत्र हुआ।
  2. स्वतंत्रता के बाद से दोनों ही देशों में लोकतांत्रिक शासन चला आ रहा है।

प्रश्न 9.
‘बांग्लादेश’ के निर्माण पर संक्षिप्त टिप्पणी लिखिए।
उत्तर:
जनरल याहिया खान के सैनिक शासन में पश्चिमी पाकिस्तानी सेना ने पूर्वी पाकिस्तान के बंगाली जनता के आंदोलन को कुचलने का प्रयत्न किया। हजारों लोगों को मौत के घाट उतार दिया गया। इस वजह से पूर्वी पाकिस्तान से बड़ी संख्या में लोग भारत पलायन कर गए। परिणामस्वरूप इन शरणार्थियों को संभालने की समस्या भारत के सामने खड़ी हो गई। अतः भारतीय सरकार ने पूर्वी पाकिस्तान की आजादी का समर्थन किया। इस कारण 1971 में भारत और पाकिस्तान के बीच युद्ध आरंभ हो गया। युद्ध की समाप्ति पूर्वी पाकिस्तान में पाकिस्तनी सेना के आत्मसमर्पण तथा एक स्वतंत्र राष्ट्र ‘बांग्लादेश’ के निर्माण के साथ हुई।

प्रश्न 10.
दक्षिण एशिया के देशों में मुख्य रूप से कौन-कौन सी समस्यायें हैं?
उत्तर:
दक्षिण एशिया के देशों में प्रमुख समस्यायें ये हैं।

  1. निर्धनता और बेरोजगारी की समस्या
  2. अन्तर्राष्ट्रीय आतंकवाद की समस्या
  3. जातीय संघर्ष
  4. संसाधनों के बंटवारे में होने वाले झगड़े
  5. सीमा एवं नदी जल बंटवारे से संबंधित विवाद

प्रश्न 11.
दक्षिण एशिया क्षेत्र की विशेषताओं को स्पष्ट कीजिये।
उत्तर:

  1. दक्षिण एशिया एक ऐसा क्षेत्र है, जहाँ सद्भाव और शत्रुता, आशा और निराशा तथा पारस्परिक शंका और विश्वास साथ-साथ बसते हैं।
  2. सामान्यतः बांग्लादेश, भूटान, भारत, मालदीव, नेपाल, पाकिस्तान और श्रीलंका को इंगित करने के लिए ‘दक्षिण एशिया’ पद का व्यवहार किया जाता है। इस क्षेत्र में जब-तब अफगानिस्तान और म्यांमार को भी शामिल किया जाता है।
  3. उत्तर की विशाल हिमालय पर्वत श्रृंखला, दक्षिण का हिंद महासागर, पश्चिम का अरब सागर और पूरब में मौजूद बंगाल की खाड़ी से यह क्षेत्र एक विशिष्ट प्राकृतिक क्षेत्र के रूप में नजर आता है 1
  4. दक्षिण एशिया विविधताओं से भरा-पूरा इलाका है फिर भी भू-राजनैतिक धरातल पर यह एक क्षेत्र है।

प्रश्न 12.
दक्षिण एशिया के देशों की राजनैतिक प्रणाली किस प्रकार की है?
उत्तर:
दक्षिण एशिया के विभिन्न देशों में एक सी राजनैतिक प्रणाली नहीं है। यथा।

  1. इस क्षेत्र के दो देशों, भारत और श्रीलंका में ब्रिटेन से आजाद होने के बाद, सफलतापूर्वक कायम है। लोकतांत्रिक व्यवस्था
  2. पाकिस्तान और बांग्लादेश में लोकतांत्रिक और सैनिक दोनों तरह की शासन व्यवस्थाएँ बदलती रही हैं। वर्तमान में दोनों में लोकतांत्रिक शासन व्यवस्था है।
  3. नेपाल में 2006 तक संवैधानिक राजतंत्र था। अप्रेल, 2006 से वहाँ लोकतंत्र की स्थापना हुई है।
  4. भूटान में अब भी राजतंत्र है लेकिन यहाँ के राजा ने भूटान में लोकतंत्र स्थापित करने की योजना की शुरुआत . कर दी है।
  5. मालदीव में 1968 तक राजतंत्र (सल्तनत) शासन था। 1968 में यह एक गणतंत्र बना।

JAC Class 12 Political Science Important Questions Chapter 5 समकालीन दक्षिण एशिया

प्रश्न 13.
” दक्षिण एशियायी देशों की जनता लोकतंत्र की आकांक्षाओं में सहभागी है।” इस कथन की व्याख्या कीजिये।
उत्तर:
दक्षिण एशिया में लोकतंत्र का रिकार्ड मिला-जुला रहा है। इसके बावजूद इस क्षेत्र के देशों की जनता लोकतंत्र की आकांक्षाओं में सहभागी है अर्थात् वह लोकतंत्र को अन्य शासन प्रणालियों से अच्छा समझती है। इस क्षेत्र के पांच बड़े देशों – बांग्लादेश, नेपाल, भारत, पाकिस्तान और श्रीलंका में हाल ही में एक सर्वेक्षण किया गया था जिसमें यह बात स्पष्ट हुई कि इन पांच देशों में लोकतंत्र को व्यापक जन-समर्थन हासिल है। इन देशों में हर वर्ग और धर्म के आम नागरिक लोकतंत्र को अच्छा मानते हैं और प्रतिनिधिमूलक लोकतंत्र की संस्थाओं का समर्थन करते हैं। इन देशों के लोग शासन की किसी और प्रणाली की अपेक्षा लोकतंत्र को वरीयता देते हैं और मानते हैं कि उनके देश के लिए लोकतंत्र ही ठीक है।

प्रश्न 14.
बांग्लादेश में बहुदलीय चुनावों पर आधारित प्रतिनिधि मूलक लोकतंत्र कब से कायम है?
उत्तर:
शेख मुजीब की हत्या के बाद जियाउर्रहमान ने बांग्लादेश में सैनिक शासन की पुनर्स्थापना की तथा बांग्लादेश नेशनल पार्टी बनायी। जियाउर्रहमान की हत्या के बाद लेफ्टिनेंट जनरल एच एम इरशाद ने सैन्य शासन की बागडोर संभाली। परन्तु बांलादेश की जनता लोकतंत्र की माँग करने लगी। इस आंदोलन के फलस्वरूप जनरल इरशाद को राजनीतिक गतिविधियों में छूट देनी पड़ी। इसके बाद जनरल इरशाद पाँच सालों के लिए राष्ट्रपति निर्वाचित हुए। परन्तु 1990 में जनता के विरोध के आगे जनरल इरशाद को राष्ट्रपति पद छोड़ना पड़ा। और 1991 में पुन: चुनाव हुए। इसके बाद से बांगलादेश में बहुदलीय चुनावों पर आधारित प्रतिनिधि मूलक लोकतंत्र कायम है।

प्रश्न 15.
नेपाल में लोकतंत्र की राह कैसे अवरुद्ध हुई?
उत्तर:
नेपाल अतीत में एक हिन्दू-राज्य था फिर आधुनिक काल में कई सालों तक संवैधानिक राजतंत्र रहा। संवैधानिक राजतंत्र के समय में नेपाल की राजनीतिक पार्टियाँ और आम जनता ज्यादा खुले और उत्तरदायी शासन की आवाज उठाते रहे । परन्तु राजा ने सेना की सहायता से शासन पर पूरा नियंत्रण कर लिया और इस प्रकार नेपाल में लोकतंत्र की राह अवरुद्ध हो गई।

प्रश्न 16.
बांग्लादेश का निर्माण क्यों और कैसे हुआ?
उत्तर:
1947 से 1971 तक बांग्लादेश पाकिस्तान का एक अंग था, जिसे ‘पूर्वी पाकिस्तान’ नाम से जाना जाता था। इस क्षेत्र के लोग पश्चिमी पाकिस्तानी दबदबे और अपने ऊपर उर्दू भाषा को लादने के खिलाफ थे।

  1. शेख मुजीब की गिरफ्तारी:
    जब 1970 के चुनावों में शेख मुजीब के नेतृत्व वाली अवामी लीग को संपूर्ण पाकिस्तान के लिए प्रस्तावित संविधान सभा में बहुमत हासिल हो गया तो सरकार ने सभा को आहूत करने से इन्कार कर दिया। शेख मुजीब को गिरफ्तार कर लिया गया। इसकी प्रतिक्रिया में पूर्वी पाकिस्तान क्षेत्र में शासन के विरुद्ध जन- आंदोलन फैल गया।
  2. भारत-पाक युद्ध और बांग्लादेश का निर्माण: जन-आंदोलन के दौरान पूर्वी पाकिस्तान से बड़ी संख्या में लोग भारत शरणार्थी के रूप में आये भारत सरकार ने लोगों के जन-आंदोलन का समर्थन किया परिणामस्वरूप 1971 में भारत-पाक युद्ध हुआ जिसकी समाप्ति पूर्वी पाकिस्तान के एक स्वतंत्र राष्ट्र ‘बांग्लादेश’ के निर्माण के साथ हुई।

प्रश्न 17.
1975 में बांग्लादेश में सेना ने शासन के खिलाफ बगावत क्यों की?
उत्तर;
स्वतंत्रता के बाद बांग्लादेश ने अपना संविधान बनाकर उसमें अपने को एक धर्मनिरपेक्ष, लोकतांत्रिक तथा समाजवादी देश घोषित किया। लेकिन, 1975 में शेख मुजीबुर्रहमान ने संविधान में संशोधन कराया और संसदीय प्रणाली की जगह अध्यक्षात्मक शासन प्रणाली को मान्यता दी गई। शेख मुजीब ने अपनी पार्टी अवामी लीग को छोड़कर अन्य सभी पार्टियों को समाप्त कर दिया। इससे तनाव और संघर्ष की स्थिति पैदा हुई। अगस्त, 1975 में सेना ने शेख मुजीब के शासन के खिलाफ बगावत कर दी।

प्रश्न 18.
श्रीलंका की जातीय समस्या पर एक टिप्पणी लिखिये।
उत्तर:
श्रीलंका की जातीय समस्या: श्रीलंका की राजनीति पर बहुसंख्यक सिंहली समुदाय के हितों की नुमाइंदगी करने वालों का दबदबा रहा है। ये लोग भारत से आकर श्रीलंका में बसे तमिलों के खिलाफ हैं। सिंहली राष्ट्रवादियों का मानना है कि श्रीलंका सिर्फ सिंहली लोगों का है। श्रीलंका में तमिलों के साथ कोई रियायत नहीं बरती जानी चाहिए। सिंहलियों के तमिलों के प्रति इस उपेक्षा भरे व्यवहार से श्रीलंका में एक उग्र तमिल राष्ट्रवाद की आवाज बुलंद हुई। 1983 के बाद से उग्र तमिल संगठन ‘लिबरेशन टाइगर्स ऑव तमिल ईलम’ (लिट्टे) श्रीलंका सेना के साथ सशस्त्र संघर्ष कर रहा है। इसने ‘तमिल ईलम’ यानी श्रीलंका के तमिलों के लिए एक अलग देश की माँग की है। इस प्रकार श्रीलंका को जातीय संघर्ष का सामना करना पड़ रहा है जिसकी मांग है कि श्रीलंका के एक क्षेत्र को अलग राष्ट्र बनाया जाये।

प्रश्न 19.
भारत सरकार श्रीलंका के तमिल मुद्दे पर प्रत्यक्ष रूप से कब और किस रूप में शामिल हुई?
उत्तर:
भारत की तमिल जनता का भारतीय सरकार पर भारी दबाव रहा कि वह श्रीलंकाई तमिलों के हितों की रक्षा करे। भारत सरकार समय-समय पर इस संदर्भ में श्रीलंका सरकार से बातचीत करने की कोशिश करती रही। लेकिन 1987 में भारतीय सरकार श्रीलंका के तमिल मुद्दे पर प्रत्यक्ष रूप से शामिल हुई। सन् 1987 में भारत की सरकार ने श्रीलंका सरकार से तमिल मुद्दे को सुलझाने के लिए एक समझौता किया तथा श्रीलंका सरकार और तमिलों के बीच रिश्ते सामान्य करने के लिए भारत ने शांति सेना को भेजा। लेकिन शांति सेना अन्ततः लिट्टे के साथ संघर्ष में फंस गई। भारतीय सेना की उपस्थिति को श्रीलंका की जनता ने श्रीलंका के अंदरूनी . मामलों में भारत की दखलंदाजी समझा । परिणामस्वरूप ‘शांति सेना’ अपना लक्ष्य हासिल किए बिना वापस बुला ली गई।

प्रश्न 20.
श्रीलंका की आर्थिक व्यवस्था पर एक टिप्पणी लिखिए।
उत्तर:
श्रीलंका ने अच्छी आर्थिक वृद्धि और विकास के उच्च स्तर को हासिल किया है। उसने जनसंख्या की वृद्धि दर पर सफलतापूर्वक नियंत्रण किया है। दक्षिण एशिया के देशों में सबसे पहले श्रीलंका ने ही अपनी अर्थव्यवस्था का उदारीकरण किया । जातीय संघर्ष से गुजरने के बावजूद कई सालों से इस देश का प्रति व्यक्ति सकल घरेलू उत्पाद दक्षिण एशिया में सबसे ज्यादा है।

प्रश्न 21.
सार्क की स्थापना के क्या उद्देश्य थे?
उत्तर:
सार्क की स्थापना के मुख्य उद्देश्य निम्नलिखित थे।

  1. दक्षिण एशिया के लोगों के कल्याण करने की कामना तथा उनके जीवन स्तर को सुधारना।
  2. दक्षिण एशिया क्षेत्र में आर्थिक विकास, सामाजिक प्रगति तथा सांस्कृतिक उन्नति को प्राप्त करना तथा इस क्षेत्र के सभी व्यक्तियों के लिए प्रतिष्ठा के अवसर प्रदान करना।
  3. दक्षिण एशिया के देशों में सामूहिक आत्मविश्वास और आत्मनिर्भरता पैदा करना तथा उसको बढ़ावा देना।
  4. एक-दूसरे की समस्याओं को पारस्परिक विश्वास, सूझ-बूझ तथा अभिमूल्यन की दृष्टि से देखना।

JAC Class 12 Political Science Important Questions Chapter 5 समकालीन दक्षिण एशिया

प्रश्न 22.
क्षेत्रीय सहयोग के साधन के रूप में सार्क की प्रमुख उपलब्धियों का वर्णन कीजिये।
उत्तर:
दक्षिण एशिया क्षेत्र में सहयोग के साधन के रूप में सार्क की प्रमुख उपलब्धियाँ निम्नलिखित हैं।

  1. इस क्षेत्र में सातों देश एक-दूसरे के काफी नजदीक आए हैं तथा इससे उनमें दिखाई देने वाला तनाव कम हुआ है।
  2. दक्षेस (सार्क) के कारण इस क्षेत्र के देशों के छोटे-मोटे मतभेद अपने-आप आसानी से सुलझ रहे हैं और इन देशों में अपनापन विकसित हुआ है।
  3. सार्क के माध्यम से इस क्षेत्र में विदेशी शक्तियों का इस क्षेत्र में प्रभाव कम हुआ है और ये देश अपने को अधिक स्वतंत्र महसूस करने लगे हैं।
  4. सार्क ने एक संरक्षित अन्न भंडार की स्थापना की है जो इस क्षेत्र के देशों की आत्मनिर्भरता की भावना के प्रबल होने का सूचक है।

प्रश्न 23.
भारत और बांग्लादेश के बीच मतभेद के कारणों पर एक टिप्पणी लिखिए।
उत्तर:
भारत और बांग्लादेश के बीच मतभेद के मुद्दे: भारत और बांग्लादेश के बीच मतभेद के प्रमुख मुद्दे इस प्रकार हैं।
(अ) भारत के बांग्लादेश से अप्रसन्नता के कारण – भारतीय सरकारों के बांग्लादेश से नाखुश होने के कारण हैं।

  1. भारत में अवैध आप्रवास पर ढाका द्वारा खंडन करना।
  2. बांग्लादेश सरकार द्वारा भारत विरोधी इस्लामी कट्टरपंथी जमातों को समर्थन देना।
  3. भारतीय सेना को पूर्वोत्तर भारत में जाने के लिए अपने इलाके से रास्ता देने से बांग्लादेश का इन्कार करना।
  4. म्यांमार को बांग्लादेशी इलाके से भारत को प्राकृतिक गैस निर्यात न करने देना।

(ब) बांग्लादेश भारत पर निम्न कारणों से अप्रसन्न है।

  1. भारतीय सरकार गंगा और ब्रह्मपुत्र नदी – जल में हिस्सेदारी के प्रश्न पर इलाके के दादा की तरह बरताव करती है।
  2. भारत की सरकार चटगाँव पर्वतीय क्षेत्र में विद्रोह को हवा दे रही है।
  3. भारत उसकी प्राकृतिक गैस में सेंधमारी कर रहा है तथा व्यापार में बेईमानी बरत रहा है।

प्रश्न 24.
भारत और बांग्लादेश के सहयोग के मुद्दों को स्पष्ट कीजिये।
उत्तर:
भारत और बांग्लादेश के बीच सहयोग के मुद्दे भारत और बांग्लादेश निम्नलिखित मुद्दों पर आपस में सहयोग करते हैं।

  1.  पिछले बीस वर्षों के दौरान दोनों देशों के बीच आर्थिक सम्बन्धं ज्यादा बेहतर हुए हैं।
  2. बांग्लादेश भारत के ‘लुक ईस्ट’ और 2014 से ‘एक्ट ईस्ट’ की नीति का हिस्सा है। इस नीति के अन्तर्गत स्यांमार के जरिए दक्षिण-पूर्व एशिया से सम्पर्क साधने की बात है ।
  3. आपदा प्रबंधन और पर्यावरण के मसले पर भी दोनों देशों ने निरन्तर सहयोग किया है।
  4. इस बात के भी प्रयास किये जा रहे हैं कि साझे खतरों को पहचान कर तथा एक-दूसरे की जरूरतों के प्रति ज्यादा संवेदनशीलता बरत कर सहयोग के दायरे को बढ़ाया जाये।

प्रश्न 25.
भारत-नेपाल के सम्बन्धों के बीच कड़वाहट के मुद्दों पर एक टिप्पणी लिखिये । भारत-नेपाल के सम्बन्धों के बीच तनाव के मुद्दे
उत्तर:
भारत-नेपाल के मधुर सम्बन्धों के बीच निम्नलिखित मुद्दे मनमुटाव पैदा करते रहे हैं।

  1. नेपाल की चीन के साथ मित्रता को लेकर भारत सरकार ने अक्सर अपनी अप्रसन्नता जतायी है।
  2. नेपाल सरकार भारत विरोधी तत्त्वों के विरुद्ध आवश्यक कदम नहीं उठाती है। इससे भी भारत नाखुश है।
  3. भारत की सुरक्षा एजेंसियां नेपाल में चल रहे माओवादी आंदोलन को अपनी सुरक्षा के लिए खतरा मानती हैं।
  4. नेपाल के लोगों की यह सोच है कि भारत की सरकार नेपाल के अंदरूनी मामलों में दखल दे रही है और उसके नदी जल तथा पन बिजली पर आँख गड़ाए हुए है।
  5. नेपाल को यह भी लगता है कि भारत उसको अपने भू-क्षेत्र से होकर समुद्र तक पहुँचने से रोकता है।

प्रश्न 26.
” भारत-नेपाल के बीच मधुर संबंध हैं ।” स्पष्ट कीजिए।
उत्तर:
भारत और नेपाल के बीच मधुर संबंध हैं, इसे हम निम्न उदाहरणों से स्पष्ट कर सकते हैं।
1. भारत और नेपाल दोनों देशों के बीच पारगमन संधि है। इस संधि के तहत दोनों देशों के नागरिक एक-दूसरे के देश में बिना पासपोर्ट (पारपत्र) और वीजा के आ-जा सकते हैं और काम कर सकते हैं।
2. दोनों देश व्यापार, वैज्ञानिक सहयोग, साझे प्राकृतिक संसाधन, बिजली उत्पादन और जल प्रबंधन ग्रिड के मसले पर एक साथ हैं।
3. नेपाल में लोकतंत्र की बहाली से दोनों देशों के बीच संबंधों के और मजबूत होने की संभावना है। प्रश्न 27. भारत-श्रीलंका के सम्बन्धों पर एक टिप्पणी लिखिये।
उत्तर:
भारत और श्रीलंका के सम्बन्ध

  1. यद्यपि श्रीलंका और भारत की सरकारों के संबंधों में तनाव इस द्वीप में जारी जातीय संघर्ष को लेकर है तथापि 1987 के सैन्य हस्तक्षेप के बाद से भारतीय सरकार श्रीलंका के अंदरूनी मामलों में असंलग्नता की नीति अपनाये हुए है।
  2. दोनों देशों की सरकारों ने परस्पर एक मुक्त व्यापार समझौते पर हस्ताक्षर कर अपने सम्बन्धों को और मजबूत किया है।
  3. श्रीलंका में ‘सुनामी’ से हुई तबाही के बाद के पुनर्निर्माण कार्यों में भारतीय मदद से भी दोनों देश एक-दूसरे के नजदीक आये हैं।

JAC Class 12 Political Science Important Questions Chapter 5 समकालीन दक्षिण एशिया

प्रश्न 28.
भूटान और मालदीव के साथ भारत के सम्बन्धों पर एक टिप्पणी लिखिये।
उत्तर:

  • भारत-भूटान सम्बन्ध – भारत के भूटान के साथ बहुत अच्छे सम्बन्ध हैं तथा भूटानी सरकार के साथ भारत का कोई बड़ा झगड़ा नहीं है।
    1. भूटान से अपने काम का संचालन कर रहे पूर्वोत्तर भारत के उग्रवादियों और गुरिल्लों को भूटान ने अपने क्षेत्र से खदेड़ भगाया। भूटान के इस कदम से भारत को बड़ी सहायता मिली है।
    2. भारत भूटान में पन बिजली की बड़ी परियोजनाओं में हाथ बँटा रहा है
    3. भूटान के विकास कार्यों के लिए सबसे ज्यादा अनुदान भारत से हासिल होता है।
  • भारत और मालदीव सम्बन्ध:
    1. मालदीव के साथ भारत के संबंध सौहार्द्रपूर्ण तथा गर्मजोशी से भरे हैं।
    2. भारत ने मालदीव के आर्थिक विकास, पर्यटन और मत्स्य उद्योग में भी मदद की है।

प्रश्न 29.
‘साफ्टा’ क्या है?
उत्तर:
साफ्टा (SAFTA ) साफ्टा’ का पूरा नाम है। दक्षिण एशिया मुक्त व्यापार क्षेत्र (South Asia Free Trade Area)! दक्षेस के सदस्य देशों ने सन् 2002 में ‘दक्षिण एशियाई मुक्त व्यापार क्षेत्र समझौते’ पर दस्तखत करने का वायदा किया। इसमें पूरे दक्षिण एशिया के लिए मुक्त व्यापार क्षेत्र बनाने का वायदा है। इस समझौते पर 2004 में हस्ताक्षर हुए और यह समझौता 1 जनवरी, 2006 से प्रभावी हो गया है। इस समझौते का लक्ष्य है कि इन देशों के बीच आपसी व्यापार में लगने वाले सीमा शुल्क को 2007 तक 20 प्रतिशत कम कर दिया जाये।

प्रश्न 30.
भारत-पाकिस्तान के बीच सियाचिन की समस्या को संक्षेप में बताइये।
उत्तर:
भारतीय नक्शे में शीर्ष पर स्थित सियाचिन हिमनाद को लेकर भारत पाकिस्तान के बीच विवाद बना हुआ है। सियाचिन सैनिक तथा सामरिक दृष्टि महत्त्वपूर्ण है क्योंकि यहाँ से पाक अधिकृत कश्मीर, चीन के सिक्यांग प्रान्त, रूसी राष्ट्रकुल के देशों तथा अफगानिस्तान पर पैनी दृष्टि रखी जा सकती है। इसलिए इसे भारत और पाकिस्तान दोनों अपने-अपने अधिकार में लेना चाहते हैं। इसे लेकर दोनों देशों में अनेक मुठभेड़ें हो चुकी हैं। वर्तमान में दोनों ही देश इस समस्या के स्थायी हल के लिए वार्ताएँ कर रहे हैं।

प्रश्न 31.
“श्रीलंका की समस्या भारतवंशी लोगों से जुड़ी है।” इस कथन को स्पष्ट कीजिए।
उत्तर:
भारत की तमिल जनता का भारतीय सरकार पर भारतीय सरकार पर भारी दबाव है कि वह श्रीलंकाई तमिलों के हितों की रक्षा करे भारतीय सरकार ने समय-समय पर तमिलों के सवाल पर श्रीलंका की सरकार से बातचीत की कोशिश की है। 1987 में भारतीय सरकार श्रीलंका के तमिल मसले में प्रत्यक्ष रूप से शामिल हुई। भारतीय सरकार ने श्रीलंका से समझौता किया तथा श्रीलंका और तमिलों के बीच रिश्ता सामान्य करने के लिए भारतीय सेना भेजा अतः हम कह सकते हैं कि श्रीलंका की समस्या भारतवंशी लोगों से जुड़ी है।

प्रश्न 32.
1989 में भारत को अपनी ‘शांति सेना’ वापस बुलानी पड़ी।
उत्तर:
भारतीय सरकार ने श्रीलंका से समझौता किया और इसके फलस्वरूप भारतीय सरकार ने श्रीलंका सरकार और तमिलों के बीच रिश्ते सामान्य करने के लिए भारतीय सेना भेजी सेना लिट्टे के साथ संघर्ष में फँस गई। भारतीय सेना की उपस्थिति का श्रीलंका की जनता ने विरोध किया। वहाँ की जनता ने यह समझा कि भारतीय सेना उनके अंदरूनी मामलों में हस्तक्षेप कर रही है। फलस्वरूप 1989 में भारत ने अपनी ‘शांति सेना’ लक्ष्य हासिल किए बिना वापस बुला ली।

JAC Class 12 Political Science Important Questions Chapter 5 समकालीन दक्षिण एशिया

प्रश्न 33.
कश्मीर के दो हिस्सों में विभाजित होने के कारणों को स्पष्ट कीजिए।
उत्तर:
कश्मीर के दो हिस्सों में विभाजित होने के कारण निम्नलिखित हैं।

  1. दक्षिण एशिया में स्थित देशों के आपसी संघर्षों में सबसे प्रमुख और सर्वग्रासी संघर्षों में सबसे प्रमुख और सर्वग्रासी संघर्ष भारत और पाकिस्तान के बीच का संघर्ष है।
  2. भारत और पाकिस्तान के विभाजन के तुरंत बाद ही दोनों देश कश्मीर के मसले पर लड़ पड़े।
  3. पाकिस्तान सरकार कश्मीर पर अपना दावा कर रही थी।
  4. पाकिस्तान सरकार कश्मीर पर अपना दावा कर रही थी।
  5. 1948 के युद्ध के फलस्वरूप कश्मीर के दो हिस्से हो गए।

प्रश्न 34.
“दक्षिण एशिया के सारे झगड़े सिर्फ भारत और उसके पड़ोसी देशों के बीच ही नहीं है।” इस कथन को स्पष्ट कीजिए।
उत्तर:
दक्षिण एशिया के सारे झगड़े सिर्फ भारत और उसके पड़ोसी देशों के बीच ही नहीं है क्योंकि।

  1. नेपाल – भूटान तथा बांग्लादेश – म्यांमार के बीच जातीय मूल के नेपालियों के भूटान आप्रवास तथा रोहिंग्या लोगों म्यांमार में आप्रवास के मामलों पर मतभेद रहे हैं।
  2. बांग्लादेश और नेपाल के बीच हिमालयी नदियों के जल की हिस्सेदारी को लेकर खटपट है।

प्रश्न 35.
‘साफ्टा’ के उद्देश्य पर भारत और अन्य देश एकमत नहीं है। इस कथन की संक्षेप में व्याख्या कीजिए।
उत्तर:
दक्षिण एशिया के सभी देशों के बीच 2004 में ‘साफ्टा’ पर हस्ताक्षर हुआ और यह समझौता 1 जनवरी, 2006 से प्रभावी हो गया। यद्यपि इस समझौते का मकसद इन क्षेत्रों के सीमा रेखा मुक्त व्यापार पर सहमत होना था। परंतु कुछ देशों का मानना था कि ‘साफ्टा’ की ओट लेकर भारत उनके बाजार में सेंध मारना चाहता है ओर व्यावसायिक उद्यम तथा व्यावसायिक मौजूदगी जरिये उनके समाज ओर राजनीति पर असर डालना चाहता है।

जबकि भारत सोचता है कि ‘साफ्टा’ के जरिये इस क्षेत्र के हर देश का फायदा होगा और क्षेत्र में मुक्तव्यापार बढ़ने से राजनीतिक मसलों पर सहयोग बेहतर होगा। भारत में कुछ लोगों का मानना है कि भारत को साफ्टा पर ज्यादा ध्यान नहीं देना चाहिए क्योंकि भारत भूटान, नेपाल और श्रीलंका से पहले ही द्विपक्षीय समझौता कर चुका है।

प्रश्न 36.
दक्षिण एशिया के क्षेत्रों की सुरक्षा और शांति के भविष्य से अमरीका के हित बंधे हुए हैं। इस कथन की पुष्टि कीजिए।
उत्तर:
शीतयुद्ध के बाद से दक्षिण एशिया में अमरीकी प्रभाव तेजी से बढ़ा है। अमरीका ने शीतयुद्ध के बाद भारत और पाकिस्तान दोनों देशों से अपने संबंध सुधारे हैं। आवश्यकता पड़ने पर अमरीका ने भारत-पाक के बीच मध्यस्थता की भूमिका भी निभाई है। दोनों देशों में आर्थिक सुधार हुए हैं और उदार नीतियाँ अपनाई गई हैं। इस वजह से दक्षिण एशिया में अमरीकी भागीदारी ज्यादा गहरी हुई है। अमरीका में दक्षिण एशियाई मूल के लोगों की जनसंख्या अच्छी-खासी है। दक्षिण एशिया की जनसंख्या और बाजार भी भारी भरकम है। इस कारण इस क्षेत्र की सुरक्षा और शांति के भविष्य से अमरीका के हित भी बंधे हुए हैं।

प्रश्न 37.
नेपाल में लोकतंत्र की बहाली किस प्रकार हुई ? संक्षेप में लिखिए।
उत्तर:
नेपाल में अप्रेल, 2006 में लोकतंत्र के लिए देशव्यापी आंदोलन हुए। लोकतंत्र के समर्थन में जगह-जगह प्रदर्शन किए गए। संघर्षरत लोकतंत्र – समर्थकों को जीत हासिल हुई और नेपाल के राजा को बाध्य होकर संसद को चलने की अनुमति दे दी इस प्रकार नेपाल में लोकतंत्र की बहाली हुई।

निबन्धात्मक प्रश्न

प्रश्न 1.
दक्षिण एशिया से आप क्या समझते हैं? शीत युद्ध के बाद के काल में दक्षिण एशिया के देशों में राजनीतिक प्रणाली की मुख्य प्रवृत्ति क्या रही है?
उत्तर:
दक्षिण एशिया से आशय सामान्य रूप से बांग्लादेश, भूटान, भारत, मालदीव, नेपाल, पाकिस्तान और श्रीलंका इन सात देशों को इंगित करने के लिए दक्षिण एशिया पद का इस्तेमाल किया जाता है। इस क्षेत्र की चर्चा में जब-तब अफगानिस्तान और म्यांमार को भी शामिल किया जाता है। दक्षिण एशिया के देशों में राजनीतिक प्रणाली शीत युद्ध के बाद के काल में इस क्षेत्र के देशों में तथा यहाँ के लोगों में मुख्य प्रवृत्ति लोकतांत्रिक राजनीतिक प्रणाली को अपनाने की रही है। यथा

  1. भारत, श्रीलंका और बांग्लादेश: भारत और श्रीलंका में ब्रिटेन से स्वतंत्र होने के बाद से आज तक लोकतांत्रिक व्यवस्था सफलतापूर्वक कायम है तथा शीत युद्ध के बाद से बांग्लादेश में भी लोकतांत्रिक सरकारें कायम हैं।
  2. पाकिस्तान: पाकिस्तान में शीत युद्ध के बाद के सालों में लगातार दो लोकतांत्रिक सरकारें बनीं। 1999 से 2007 तक सैनिक तख्ता पलट के कारण पाकिस्तान में गैर लोकतांत्रिक सरकार रही लेकिन 2008 से आज तक वहां लोकतांत्रिक सरकार कार्यरत है।
  3. नेपाल: अप्रेल, 2006 तक नेपाल में संवैधानिक राजतंत्र था, लेकिन अप्रेल 2006 से आज तक नेपाल में भी लोकतांत्रिक राजनीतिक व्यवस्था कायम है।
  4. भूटान: भूटान में 2007 तक राजतंत्र शासन रहा। लेकिन मार्च, 2008 से यहाँ भी लोकतांत्रिक व्यवस्था कायम है।
  5.  मालदीव: मालदीव में 1968 तक राजतंत्र शासन व्यवस्था रही। 1968 से यहाँ भी लोकतांत्रिक व्यवस्था कायम है। स्पष्ट है कि दक्षिण एशिया के देशों में लोकतंत्र को अपनाने की मुख्य प्रवृत्ति रही है।

JAC Class 12 Political Science Important Questions Chapter 5 समकालीन दक्षिण एशिया

प्रश्न 2.
पाकिस्तान में लोकतंत्र और उसके सैनिक तंत्र में बदलाव तथा पुनः लोकतंत्र की स्थापना की प्रक्रिया पर एक निबंध लिखिये।
उत्तर:
14 अगस्त, 1947 को भारत का विभाजन करके पाकिस्तान की स्थापना हुई। पाकिस्तान की राजनीतिक व्यवस्था मुस्लिम लीग के अध्यक्ष मुहम्मद अली जिन्ना पाकिस्तान के प्रथम गवर्नर जनरल बने और लियाकत अली खां ने प्रधानमंत्री का पद संभाला। लेकिन 1948 में जिन्ना की मृत्यु हो गई और पाकिस्तान के शासन की बागडोर लियाकत अली खां के हाथों में पूरी तरह से आ गई। अक्टूबर, 1951 में लियाकत अली खां की हत्या कर दी गई। इस घटना के बाद ख्वाजा निजामुद्दीन को प्रधानमंत्री तथा गुलाम मोहम्मद को गवर्नर जनरल के पद पर नियुक्त किया गया। 7 अप्रैल, 1953 को गवर्नर जनरल गुलाम मोहम्मद ने निजामुद्दीन मंत्रिमंडल को भंग कर दिया।

अमरीकी प्रभाव में अब मुहम्मद अली को प्रधानमंत्री बनाया गया। सैनिक तानाशाही: 7 अक्टूबर, 1958 को प्रधान सेनापति जनरल अयूब खां के नेतृत्व में सेना ने सरकार के विरुद्ध विद्रोह कर दिया तथा सत्ता प्राप्त कर ली। तब से लेकर 1971 तक तथा पुनः 5 जुलाई, 1977 से 1988 तक पाकिस्तान में सैनिक शासन रहा।

लोकतंत्र: 1971 में बांग्लादेश के निर्माण के बाद पाकिस्तान में जुल्फिकार अली भुट्टो के नेतृत्व में लोकतांत्रिक सरकार बनी जो 1977 तक कायम रही। इसके बाद 1989 से 1999 तक पाकिस्तान में पुनः लोकतान्त्रिक सरकारों ने काम किया।

  1. पुनः सैनिक शासन ( 1999 से 2007 तक ): सन् 1999 में सेना प्रमुख जनरल परवेज मुशर्रफ ने प्रधानमंत्री नवाज शरीफ को पदच्युत करके अपने आपको वहां का शासक बना लिया जो 2007 तक सत्ता पर काबिज रहे।
  2. पुनः लोकतंत्र की वापसी (2008 से वर्तमान तक ): जन आंदोलनों और जनमत के दबाव में 2008 में पाकिस्तान में चुनाव हुए और तब से वर्तमान तक वहाँ लोकतांत्रिक सरकारें चली आ रही हैं।

प्रश्न 3.
बांग्लादेश का निर्माण क्यों और किस प्रकार हुआ?
उत्तर-
बांग्लादेश का निर्माण क्यों और कैसे?
1947 से 1971 तक बांग्लादेश पाकिस्तान का अंग था, जिसे पूर्वी पाकिस्तान के नाम से जाना जाता था।

  1. बंगाली संस्कृति की उपेक्षा: पूर्वी पाकिस्तान के लोग पश्चिमी पाकिस्तान के दबदबे और अपने ऊपर उर्दू भाषा को लादने के खिलाफ थे। पाकिस्तान के निर्माण के बाद से ही यहां के लोगों ने बंगाली संस्कृति और भाषा के साथ किये जा रहे दुर्व्यवहार के खिलाफ विरोध जताना शुरू कर दिया था।
  2. प्रशासन तथा राजनीतिक सत्ता में भागीदारी की माँग- इस क्षेत्र की जनता ने प्रशासन में अपने न्यायोचित प्रतिनिधित्व तथा राजनीतिक सत्ता में समुचित हिस्सेदारी तथा पूर्वी क्षेत्र के लिए स्वायत्तता की मांग उठायी।
  3. 1970 के आम चुनाव और शेख मुजीब की अवामी लीग को बहुमत मिलना:1970 के आम चुनाव में शेख मुजीबुर्रहमान के नेतृत्व वाली अवामी लीग को सम्पूर्ण पाकिस्तान के लिए प्रस्तावित संविधान सभा में बहुमत हासिल हो गया। लेकिन सरकार पर पश्चिमी पाकिस्तान के नेताओं का दबदबा था और सरकार ने इस सभा को आहूत करने से इन्कार कर दिया। शेख मुजीब को गिरफ्तार कर लिया गया।
  4. भारत-पाक युद्ध 1971: पाकिस्तानी सेना ने पूर्वी पाकिस्तान की जनता के आंदोलन को कुचलने हेतु हजारों लोगों को मौत के मुँह सुला दिया। वहाँ के लोगों ने भारत में पलायन किया। इससे भारत में शरणार्थी की समस्या खड़ी हो गई। फलतः भारत-पाक युद्ध 1971 हुआ ।
  5. बांग्लादेश का निर्माण: युद्ध समाप्त होने से पहले ही पूर्वी पाकिस्तान में पाकिस्तानी सेना ने आत्मसमर्पण कर दिया और अन्त में अप्रेल, 1971 में बांग्लादेश का निर्माण हुआ।

JAC Class 12 Political Science Important Questions Chapter 5 समकालीन दक्षिण एशिया

प्रश्न 4.
बांग्लादेश में लोकतंत्र के समक्ष चुनौतियों की व्याख्या कीजिए। (कोई चार )
अथवा
बांग्लादेश में लोकतंत्र की स्थापना की चर्चा कीजिए।
अथवा
बांग्लादेश में लोकतंत्र की स्थापना की प्रक्रिया को संक्षेप में बताइये।
उत्तर:
बांग्लादेश में लोकतंत्रीय शासन स्थापना की प्रक्रिया: बांग्लादेश में लोकतंत्र की स्थापना निम्न प्रकार हुई।

1. संसदीय लोकतंत्र की स्थापना:
12 अप्रेल, 1971 को स्वतंत्र बांग्लादेश सरकार का गठन हुआ। बांग्लादेश ने अपना एक नवीन संविधान बनाया जिसमें इसे संसदात्मक, धर्मनिरपेक्ष, लोकतांत्रिक तथा समाजवादी देश घोषित किया गया।

2. संसदीय लोकतंत्र के स्थान पर अध्यक्षीय लोकतंत्र:
19075 में शेख मुजीब ने संसदीय लोकतंत्र के स्थान पर अध्यक्षात्मक प्रणाली को मान्यता दी। इसके विरोध में सेना ने बगावत कर दी तथा शेख मुजीब की हत्या कर दी।

3. सैनिक शासन:
शेख मुजीब की हत्या के बाद 1975 में सैनिक शासके जियाउर्रहमान ने बांग्लादेश नेशनल पार्टी बनाई तथा 1979 के चुनाव में विजयी रही। लेकिन जियाउर्रहमान की हत्या हुई। अब लेफ्टिनेंट जनरल एम. एम. इरशाद के नेतृत्व में एक और सैनिक शासन ने बागडोर संभाली तथा 1990 तक सत्ता में बने रहे।

4. पुनः लोकतंत्र की स्थापना: जनता के व्यापक विरोध के समक्ष झुकते हुए ले. जनरल इरशाद को राष्ट्रपति का पद 1990 में छोड़ना पड़ा और 1991 में चुनाव हुए। इसके बाद से बांग्लादेश में बहुदलीय लोकतांत्रिक व्यवस्था कायम है।

प्रश्न 5.
नेपाल में लोकतंत्रीकरण की प्रक्रिया की एक समीक्षा प्रस्तुत कीजिये।
उत्तर:
नेपाल में संवैधानिक राजतंत्र: 1947 से 1960 तक नेपाल में संवैधानिक राजतंत्र रहा। संवैधानिक राजतंत्र के दौर में नेपाल की राजनीतिक पार्टियाँ और आम जनता खुले और उत्तरदायी शासन की माँग उठाते रहे। विवश होकर राजा ने 1990 में नए लोकतान्त्रिक संविधान की माँग मान ली। इस प्रकार नेपाल में 1990 से लोकतांत्रिक सरकार का गठन हुआ। लेकिन 1990 के दशक में ही 5-6 जिलों में माओवादियों की समानान्तर शासन व्यवस्था को नियंत्रित करने की प्रक्रिया में शाही सेना व माओवादियों के बीच हिंसक लड़ाई छिड़ गई।

कुछ समय तक राजा की सेना, लोकतंत्र – समर्थकों और माओवादियों के बीच त्रिकोणीय संघर्ष हुआ। अन्त में सन् 2002 में राजा ने संसद को भंग कर दिया और सरकार को गिरा दिया। लोकतंत्र की बहाली ( 2006 ) – अप्रेल, 2006 में नेपाल में देशव्यापी लोकतंत्र समर्थक प्रदर्शन हुए तथा 24 अप्रेल, 2006 को राजा ज्ञानेन्द्र ने बाध्य होकर संसद को बहाल किया।

प्रश्न 6.
भारत के नेपाल व श्रीलंका के साथ सम्बन्धों की विवेचना कीजिये।
अथवा
भारत-श्रीलंका सम्बन्धों का विस्तार से वर्णन कीजिए।
उत्तर:
भारत-नेपाल सम्बन्ध: भारत के नेपाल के साथ सम्बन्धों को निम्न प्रकार स्पष्ट किया गया है।

  1. दोनों देशों के बीच बहुत नजदीकी सांस्कृतिक सम्बन्ध रहे हैं।
  2. 1950 से नेपाल और भारत के बीच पारगमन और व्यापार संधि रही है।
  3. नेपाल के राजनीतिक विकास और लोकतंत्रीकरण में भारत की महत्त्वपूर्ण भूमिका रही है।
  4. भारत ने नेपाल में शिक्षण संस्थाओं के विकास, हवाई सेवा के विकास, भारतीय सशस्त्र बलों से सेवानिवृत्त नेपाली लोगों के कल्याण आदि में नेपाल को सहयोग दिया है।
  5. दोनों देश व्यापार, वैज्ञानिक सहयोग, साझे प्राकृतिक संसाधन, बिजली उत्पादन और जल प्रबंधन ग्रिड के मुद्दों पर एक साथ हैं।
  6. दोनों देशों के बीच पारगमन संधि पर नेपाल के संदेह, नेपाल के चीन से बढ़ते रिश्ते, कालापानी के मुद्दे, जल- विवाद आदि के सम्बन्ध में मतभेद उभरते रहे हैं। उक्त विवादों के होते हुए दोनों देशों के सम्बन्ध मैत्रीपूर्ण बने हुए हैं।

भारत-श्रीलंका सम्बन्ध: भारत और श्रीलंका दोनों पड़ौसी देश हैं। दोनों के बीच घनिष्ठ मैत्री सम्बन्ध हैं। यथा।

  1. दक्षिण एशिया में भारत और श्रीलंका दोनों ही स्वतंत्रता के बाद से लोकतांत्रिक देश रहे हैं।
  2. भारत और श्रीलंका के बीच केवल समुद्र है और दोनों देशों के बीच समुद्री सीमा विषयक समझौता है।
  3. भारत ने समय-समय पर आर्थिक सहयोग दिया है। दोनों गुटनिरपेक्ष देश हैं। दोनों के बीच मुक्त व्यापार समझौता है। यथा।
  4. दोनों देशों के बीच श्रीलंका तमिल समस्या विवाद का विषय रही। उक्त विवेचन से स्पष्ट है कि भारत-श्रीलंका के बीच परस्पर अच्छे पड़ौसी के सम्बन्ध कायम हैं।

प्रश्न 7.
भारत और नेपाल के आपसी सम्बन्धों में विवाद और सहयोग के मुख्य मुद्दों की विवेचना कीजिये भारत और नेपाल सम्बन्ध
उत्तर:
भारत व नेपाल के बीच सम्बन्धों में विवाद और सहयोग दोनों का सुगुंफन है। यथा।
I. भारत और नेपाल के सम्बन्धों में सहयोग के मुद्दे – भारत-नेपाल के बीच सहयोग के प्रमुख मुद्दे निम्नलिखित

  1. दोनों देशों के बीच बहुत नजदीकी सांस्कृतिक सम्बन्ध रहे हैं।
  2. 1950 से नेपाल और भारत के बीच पारगमन और व्यापार संधि रही है।
  3. नेपाल के राजनीतिक विकास और लोकतंत्रीकरण में भारत की महत्त्वपूर्ण भूमिका रही है।
  4. भारत ने नेपाल में शिक्षा के क्षेत्र में भी अनेक संस्थाओं और संगठनों के विकास में योगदान दिया है।
  5. दोनों देशों के बीच हवाई सेवा सम्बन्धी समझौता है।

II. भारत-नेपाल के बीच विवाद के मुद्दे – भारत – नेपाल के बीच अनेक मुद्दों पर विवाद उभरता रहा है।

  1. नेपाल सरकार भारत-विरोधी तत्त्वों के खिलाफ कदम नहीं उठाती।
  2. भारत की सुरक्षा एजेंसियाँ नेपाल में चल रहे माओवादी आंदोलन को अपनी सुरक्षा के लिए खतरा मानती है क्योंकि वे भारत विरोधी गतिविधियों में शामिल हैं।
  3. नेपाल के चीन से बढ़ते रिश्तों के कारण भारत की चिंता बढ़ी है।
  4. नेपाल के बहुत से लोग सोचते हैं कि भारत सरकार नेपाल के अंदरूनी मामलों में दखल दे रही है और उसके नदी जल तथा पन बिजली पर आँख गड़ाए हुए है।
  5. नेपाल को यह संशय है कि भारत उसको अपने भूक्षेत्र से होकर समुद्र तक पहुँचने से रोकता है।

JAC Class 12 Political Science Important Questions Chapter 5 समकालीन दक्षिण एशिया

प्रश्न 8.
भारत-पाक के बीच प्रमुख विवादग्रस्त मुद्दे क्या हैं? प्रकाश डालिये।
अथवा
भारत-पाकिस्तान के बीच तनाव के प्रमुख मुद्दों का विवेचन कीजिये । भारत – पाकिस्तान के बीच तनाव के प्रमुख मुद्दे
उत्तर;
भारत-पाकिस्तान के बीच तनाव के प्रमुख मुद्दे निम्नलिखित हैं।’
1. कश्मीर का मुद्दा: विभाजन के तुरन्त बाद दोनों देश कश्मीर के मुद्दे पर लड़ पड़े। पाकिस्तान की सरकार का दावा था कि कश्मीर पाकिस्तान का है जबकि भारत का कहना है कि कश्मीर भारत का अंग है। दोनों देशों के अपने-अपने तर्क हैं। इस मुद्दे को लेकर भारत पाकिस्तान के बीच 1947-48 तथा 1965 का युद्ध हो चुका है, लेकिन इन युद्धों से इस मसले का समाधान नहीं हो पाया है।

2. सियाचिन ग्लेशियर पर नियंत्रण का मुद्दा:
हिमालय में भारत – पाक-चीन सीमा पर स्थित सियाचिन ग्लेशियर उचित सीमा निर्धारण नहीं किये जा सकने के कारण भारत-पाक के बीच विवाद का मुद्दा बना हुआ है। सामरिक दृष्टि से इस क्षेत्र का अत्यधिक महत्त्व होने के कारण दोनों देश इस पर अपना अधिकार स्थापित करना चाहते हैं।

3. हथियारों की होड़ का मुद्दा: हथियारों की होड़ को लेकर भी भारत और पाकिस्तान के बीच तनातनी रहती है। 1998 में दोनों ने परमाणु परीक्षण किये तथा दोनों परमाणु अस्त्रों से लैस हैं।

4. एक-दूसरे पर संदेह तथा आरोप-प्रत्यारोप: दोनों देशों की सरकारें लगातार एक-दूसरे को संदेह की नजर से देखती हैं। उग्रवाद, आतंकवाद, जासूसी आदि के लिए एक-दूसरे पर आरोप प्रत्यारोप करती रहती है।

5. नदी – जल बँटवारे पर विवाद: भारत और पाकिस्तान के बीच सिंधु जल संधि की व्याख्या और नदी – जल के इस्तेमाल को लेकर विवाद बना हुआ है।

6. सरक्रीक की समस्या: कच्छ के रन में सरक्रीक की सीमा रेखा को लेकर दोनों देशों के बीच मतभेद हैं।

प्रश्न 9.
पाकिस्तान के साथ भारत के सम्बन्धों को किस प्रकार सुधारा जा सकता है?
अथवा
भारत-पाक सम्बन्धों को सुधारने हेतु सुझाव दीजिये।
उत्तर:
भारत-पाक सम्बन्धों को सुधार हेतु सुझाव: भारत और पाकिस्तान के बीच सम्बन्धों को सुधारने के लिए निम्नलिखित सुझाव दिये जा सकते हैं।

  1. राजनीतिक स्तर पर बातचीत एवं विश्वास बहाली के प्रयास: भारत और पाकिस्तान दोनों राजनीतिक स्तर पर प्रयास करके आपसी विवादों को बातचीत और समझौतों के द्वारा दूर कर सकते हैं।
  2. आर्थिक स्तर पर प्रयास: दोनों देशों को आपसी सम्बन्ध सुधारने के लिए आर्थिक स्तर पर ‘मुक्त व्यापार संधि’ तथा एक-दूसरे की आर्थिक जरूरतों को पूरा करके सम्बन्धों में सुधार के प्रयत्न करने चाहिए।
  3. सांस्कृतिक स्तर पर प्रयास: सांस्कृतिक स्तर पर दोनों देशों को साहित्य, कला और खेल गतिविधियों के आदान-प्रदान, वीजा सुविधा तथा सिनेमा के द्वारा सहयोग बढ़ाना चाहिए।
  4. सामाजिक स्तर पर प्रयास: भारत और पाकिस्तान को अपने सम्बन्ध सुधारने के लिए समय पर इन लोगों को आपस में मिलने की सुविधा प्रदान करें।समय
  5. तकनीकी तथा चिकित्सा सेवा का आदान-प्रदान- दोनों देश तकनीकी ज्ञान तथा चिकित्सा के क्षेत्र में भी साथ काम करके आपसी सम्बन्ध सुधार सकते हैं। गया है।
  6. शिमला समझौते का पालन- दोनों देशों को शिमला समझौते की शर्तों का पालन करना चाहिए।

JAC Class 12 Political Science Important Questions Chapter 5 समकालीन दक्षिण एशिया

प्रश्न 10.
भारत और बांग्लादेश के मध्य तनाव तथा सहयोग के मुद्दों का विवेचन कीजिये।
उत्तर:
भारत और बांग्लादेश के मध्य तनाव और सहयोग के मुद्दे भारत और बांग्लादेश के बीच तनाव तथा सहयोग के मुद्दों का विवेचन निम्नलिखित बिन्दुओं के अन्तर्गत किया दोनों देशों के मध्य तनाव के मुद्दे भारत और बांग्लादेश के मध्य तनाव के अग्रलिखित प्रमुख मुद्दे हैं।
(अ) भारत सरकार निम्न कारणों से बांग्लादेश से नाराज है।

  1. भारत में अवैध आप्रवास (बांग्लादेशी शरणार्थियों की भारत में घुसपैठ ) पर ढाका द्वारा खंडन करना।
  2. बांग्लादेश सरकार का भारत विरोधी इस्लामी कट्टरपंथी जमातों को समर्थन देना।
  3. भारतीय सेना को पूर्वोत्तर भारत में जाने के लिए अपने इलाके से रास्ता देने से बांग्लादेश का इन्कार करना।
  4. ढाका का भारत को प्राकृतिक गैस निर्यात न करने का फैसला।
  5. म्यांमार को बांग्लादेशी इलाके से होकर भारत को प्राकृतिक गैस निर्यात नहीं करने देना।

(ब) बांग्लादेश सरकार निम्न कारणों से भारत सरकार से नाखुश है।

  • बांग्लादेश सरकार नदी – जल में हिस्सेदारी के सवाल पर भारत सरकार से नाखुश है।
  • भारत की सरकार पर चटगांव पर्वतीय क्षेत्र में विद्रोह को हवा देने, बांग्लादेश के प्राकृतिक गैस में सेंधमारी करने के भी आरोप हैं।
    1. पिछले दस वर्षों के भारत और बांग्लादेश के मध्य सहयोग के मुद्दे. दौरान दोनों देशों के बीच आर्थिक सम्बन्ध ज्यादा बेहतर हुए हैं।
    2. आपदा प्रबंधन और पर्यावरण के मसले पर भी दोनों देशों ने निरंतर सहयोग किया है।
    3.  इस बात के भी प्रयास किये जा रहें हैं कि साझे खतरों को पहचान कर तथा एक-दूसरे की जरूरतों के प्रति ज्यादा संवेदनशीलता बरतकर सहयोग के दायरे को बढ़ाया जाये ।

प्रश्न 11.
दक्षिण एशियायी क्षेत्रीय सहयोग के क्षेत्र में सार्क की स्थापना के उद्देश्यों को स्पष्ट कीजिये तथा उसके संगठन पर प्रकाश डालिये।
उत्तर:
‘सार्क सार्क की स्थापना: दक्षिण एशियाई क्षेत्रीय सहयोग संगठन (सार्क) की स्थापना दिसम्बर, 1985 में हुई । वर्तमान में, सार्क में आठ सदस्य राष्ट्र हैं। सार्क के उद्देश्य – सार्क के प्रमुख उद्देश्य निम्नलिखित हैं।

  1. दक्षिण एशिया के लोगों के कल्याण में वृद्धि तथा उनके जीवन स्तर में उन्नति लाना।
  2. इस क्षेत्र में आर्थिक विकास, सामाजिक प्रगति तथा सांस्कृतिक विकास लाना।
  3. दक्षिण एशिया के देशों के बीच सामूहिक आत्मविश्वास को विकसित करने का प्रयास करना।
  4. एक-दूसरे की समस्याओं को समझने, सुलझाने तथा परस्पर विश्वास को लाने में योगदान करना।
  5. आर्थिक, सामाजिक, सांस्कृतिक, तकनीकी तथा वैज्ञानिक क्षेत्रों में परस्पर सहयोग करना।
  6. दूसरे विकासशील देशों के साथ पारस्परिक सहयोग में वद्धि करना।
  7. समान हितों के मामलों में अन्तर्राष्ट्रीय आधारों पर परस्पर सहयोग में वृद्धि करना।
  8. समान उद्देश्यों वाले क्षेत्रीय और अन्तर्राष्ट्रीय संगठनों के साथ सहयोग करना।

सार्क का संगठन: सार्क की प्रमुख संस्थाएँ निम्नलिखित हैं।

  1. शिखर सम्मेलन: सार्क देशों का प्रतिवर्ष एक शिखर सम्मेलन आयोजित किया जाता है जिसमें सदस्य देशों के शासनाध्यक्ष भाग लेते हैं।
  2. मंत्रिपरिषद्: सार्क के सभी राष्ट्रों के विदेशमंत्रियों से मिलकर एक मंत्रिपरिषद् का निर्माण किया गया है जो नीतियों का निर्माण करती है ।
  3. स्थायी समिति: सार्क की एक स्थायी समिति है जो परिषद् की योजनाओं की स्वीकृति देती है तथा उनका वित्तीय प्रबन्ध करती है।
  4. तकनीकी समिति: सार्क की तकनीकी समिति क्षेत्रीय सहयोग के विस्तार, योजनाओं का निर्माण व उनके कार्यान्वयन का मूल्यांकन आदि कार्य करती है।
  5. सचिवालय: सार्क का एक सचिवालय है। इसका एक महासचिव होता है जिसका कार्यकाल 2 वर्ष रखा गया है।
  6. वित्तीय व्यवस्था: सार्क के चार्टर के अनुच्छेद 9 में वित्तीय व्यवस्थाओं का प्रावधान किया गया है।

प्रश्न 12.
दक्षिण एशिया में शांति एवं सहयोग हेतु किये गये प्रयासों की विवेचना कीजिए।
अथवा
दक्षिण (सार्क) क्या है? दक्षिण एशिया की शांति व सहयोग में इसका क्या योगदान है।?
उत्तर:
सार्क (दक्षेस) से आशय: सार्क (दक्षेस) दक्षिण एशिया के देशों का इस क्षेत्र में शांति एवं सहयोग की स्थापना हेतु स्थापित किया गया एक क्षेत्रीय संगठन है। इसकी स्थापना 1985 में हुई। वर्तमान में इसके सदस्य देश हैं- भारत, बांग्लादेश, नेपाल, श्रीलंका, पाकिस्तान, भूटान, मालदीव और अफगानिस्तान। दक्षिण एशिया के देशों में शांति एवं सहयोग के कदम दक्षिण एशिया के देशों ने दक्षिण एशिया में शांति एवं सहयोग की स्थापना की दिशा में निम्न प्रमुख कदम उठाये हैं

(1) सार्क (दक्षेस) की स्थापना तथा उसके कार्य:
सार्क के विभिन्न शिखर सम्मेलनों में दक्षिण एशिया के देशों में नशीले पदार्थों की तस्करी रोकने, पर्यटन के विकास, आपदा प्रबन्ध, खाद्य सुरक्षा भण्डार की स्थापना, पर्यावरण की सुरक्षा, संयुक्त उद्यम स्थापित करने, क्षेत्रीय परियोजनाओं हेतु सामूहिक कोष गठित करने आदि के निर्णय लिये गये

(2) दक्षिण एशिया मुक्त व्यापार क्षेत्र (South Asia Free Trade Area)-SAFTA ): इन देशों ने साफ्टा (SAFTA) लागू करना स्वीकार कर लिया। सन् 2004 में दक्षिण एशिया मुक्त व्यापार क्षेत्र के समझौते पर सार्क के देशों ने हस्ताक्षर किये तथा यह समझौता 2006 से प्रभावी हो गया। इसके अतिरिक्त इन सम्मेलनों में एक गरीबी उन्मूलन कोष गठित करने; दोहरे करारोपण से बचाव, वीसा नियमों में उदारता तथा दक्षेस संचार विषयों पर सहमति हुई है।

(3) भारत-पाकिस्तान सहयोग के प्रयास: भारत और पाकिस्तान के बीच तनाव को कम करने और शांति बहाल करने के लिए सार्क सम्मेलनों में लगातार प्रयास हुए हैं।

प्रश्न 13
श्रीलंका के जातीय संघर्ष का वर्णन कीजिए।
उत्तर: श्रीलंका का जातीय संघर्ष श्रीलंका 1948 में स्वतंत्र हुआ और स्वतंत्रता के बाद से अब तक वहाँ लोकतंत्र कायम है। लेकिन स्वतंत्रता के बाद से ही श्रीलंका को जातीय संघर्ष का सामना करना पड़ रहा है।

  1. सिंहली समुदाय व उसका दृष्टिकोण: श्रीलंका की राजनीति में सिंहली समुदाय बहुसंख्यक है। तथा सिंहली समुदाय का ही वहाँ दबदबा रहा है। सिंहलियों का मानना है कि श्रीलंका सिर्फ सिंहली लोगों का है। अतः तमिलों को कोई रियायत नहीं दी जाए।
  2. तमिल समुदाय: श्रीलंका के तमिल समुदाय की माँग रही है कि श्रीलंका के एक क्षेत्र को तमिलों के लिए एक अलग राष्ट्र बनाया जाये। लेकिन सिंहलियों के तमिल अल्पसंख्यकों के उक्त उपेक्षा भरे व्यवहार के कारण वहाँ उग्र- तमिल राष्ट्रवाद की आवाज बुलंद हुई।
  3. जातीय संघर्ष: 1983 के बाद श्रीलंका के एक उग्रवादी तमिल संगठन लिट्टे ने श्रीलंका की सेना के साथ एक पृथक तमिल राष्ट्र की स्थापना के लिए सशस्त्र संघर्ष प्रारंभ कर दिया। श्रीलंका का यह हिंसक संघर्ष लगातार चलता आ रहा है। अन्तर्राष्ट्रीय मध्यस्थ के रूप में नार्वे और आइसलैंड जैसे देश युद्धरत दोनों पक्षों को बातचीत करने के लिए राजी कर रहे हैं। लेकिन यह संभव नहीं हो पाया है। अंदरूनी संघर्ष के झंझावतों को झेलकर भी श्रीलंका ने लोकतांत्रिक राज व्यवस्था कायम रखी है।

प्रश्न 14.
पूर्वी पाकिस्तान से बड़ी संख्या में लोग पलायन करके भारत आ गए। कारण सहित स्पष्ट कीजिए।
उत्तर:
अंग्रेजी शासन के समय बंगाल और असम के विभाजित हिस्सों से पूर्वी पाकिस्तान बना था। भारत और पाकिस्तान के विभाजन के बाद से ही पूर्वी पाकिस्तान के लोगों ने बंगाली संस्कृति और भाषा के साथ किए जा रहे दुर्व्यवहार के खिलाफ विरोध का प्रदर्शन करने लगे। इस क्षेत्र की जनता ने प्रशासन में अपने न्यायोचित प्रतिनिधित्व तथा राजनीतिक सत्ता में हिस्सेदारी की माँग की। पश्चिमी पाकिस्तान के प्रभुत्व के खिलाफ लोगों ने आंदोलन किया तथा पूर्वी क्षेत्र के लिए स्वायत्तता की माँग की।

परिणामस्वरूप शेख मुजीबुर्रहमान के नेतृत्व में आवामी लीग को 1970 में पूर्वी पाकिस्तान की सारी सीटों पर विजय मिली। लेकिन सरकार पर पश्चिमी पाकिस्तान के नेताओं का दबदबा था। जनरल याहिया खान के सैनिक शासन में पाकिस्तानी सेना ने बंगाली जनता के आंदोलन को कुचलने की कोशिश की। हजारों लोग पाकिस्तानी सेना के हाथों मारे गए। इस वजह से पूर्वी पाकिस्तान से बड़ी संख्या में लोग भारत पलायन कर गए।